Master Series 7 Exam Practice Questions

Réussis tes devoirs et examens dès maintenant avec Quizwiz!

Moody's Investment Grade (MIG) ratings are applied to

municipal notes.

Who has the final responsibility for debt service on an industrial revenue bond?

The corporation leasing the facility

Automatic exercise will occur for equity options at expiration that are in the money by at least

$0.01

ABC Corporation has net income of $10,000,000 and 5,000,000 common shares outstanding. ABC Corporation pays out $1,000,000 in dividends annually. ABC Corporation pays an annual dividend per share of:

$0.20 Explanation: To determine the dividend being paid per share, divide the $1,000,000 in dividends by the 5,000,000 shares of common stock outstanding. $1,000,000 dividends divided by 5,000,000 shares equals $0.20.

Mr. Blue's margin account has a market value of $20,000 and a debit balance of $9,000. If he purchases $2,000 of options, he will need to deposit:

$1,000 Explanation: The margin requirement when purchasing options is 100% of the purchase price (premium). Since the purchase price of the options is $2,000, Mr. Blue may use the $1,000 SMA and will be required to deposit an additional $1,000. The SMA is found by subtracting the required equity, $10,000 ($20,000 x 50%) from the current equity in the account ($11,000).

The purchase price of all of the following can be negotiated:

-Commercial paper -CD -Banker's acceptance **NOT VARIABLE ANNUITY**

Which statements are TRUEa about federal taxation of contributions to 529 plans?

-Contributions are not tax deductible to donor -A 1-time gift up to 5x the gift tax exclusion can be given that will not be subject to gift tax

Which of the following are defined as PASSIVE income?

-Distributive share of income from a real estate limited partnership investment

When is a foreign broker-dealer permitted to solicit U.S. based clients?

-If the foreign broker-dealer establishes an SEC-registered U.S. subsidiary -If the foreign-broker dealer only deals with major institutional investors

What would be considered when evaluating the credit risk of municipal revenue bond?

-Management experience -The effect of competing facilities -Coverage ratios NOT COLLECTION RATIOS

A customer writes two ABC Jul 15 puts at 2 when ABC is 14. If the contracts are closed at a premium of 4 when ABC is 13, the customer has

$400 loss. The investor receives $400 in premiums (2 × $200) and pays $800 to close out the options (2 × $400), resulting in a net loss of $400 ($800 − $400).

Super Display Book system:

-Orders are routed directly to NYSE DMM -Only round lots are permitted; odd lots cannot -Member firms use it b/c efficient and cheaper -Executed trades are directly reported to firm

Emily and her sister Lucy have the following accounts at a brokerage firm. - Emily has a cash account with $420,000 of securities. - Lucy has a margin account with $665,000 of securities and a debit balance of $365,000. - A cash account for Emily and Lucy as JTWROS with $290,000 in securities. If the brokerage firm were to go bankrupt, SIPC would provide a maximum of:

$1,010,000 coverage for all the accounts combined Explanation: SIPC provides protection of $500,000 for each customer (different account title). Since each account has a different title, each would receive coverage of $500,000 of securities. Emily's cash account would receive $420,000 and the joint account would receive $290,000. SIPC will cover Lucy's current equity in a margin account of $300,000. The total coverage is $1,010,000 ($420,000 + $300,000 + $290,000).

A bond convertible at $40 is selling in the market for 120. If the stock has a current market price of $50, the parity price for the bond is:

$1,250 Explanation: It is necessary to find the conversion ratio to solve this problem. The bond is convertible at $40. $1,000 divided by $40 equals the conversion ratio of 25 shares of stock to one bond, or 25 to 1. To find the parity price of the bond, multiply the market price of the stock of $50 by the conversion ratio of 25 ($50 x 25 = $1,250). This means that the bond must sell for $1,250 to be equal in value to the stock when the stock has a market value of $50 per share.

A customer's margin account has a market value of $15,000, a debit balance of $8,000, and SMA of $1,000. If the customer sold $1,000 of securities, what is the maximum amount the customer is permitted to withdraw after the sale?

$1,500 Explanation: This account is restricted since the equity ($7,000) is less than the Reg T requirement of the account's market value ($15,000 x 50% = $7,500). When stock is sold in a restricted account, 100% of the sale proceeds will be used by the brokerage firm to reduce the customer's debit balance. The broker-dealer will also credit the customer's SMA with an amount equal to the sale proceeds multiplied by the Reg T requirement of 50%. In this question, the sale of $1,000 worth of stock will result in a $500 credit to the customer's current SMA ($1,000). The customer is then at liberty to borrow the total SMA of $1,500.

Mordecai is a 73-year-old retired machine lathe operator. He earns $35,000 in retirement benefits. Last year he earned $1,650 as a pitching instructor for the Altoona Miners and received $800 in dividend income. What is the maximum contribution he may make to his Roth IRA?

$1,650 Explanation: Roth IRAs do not have an age limitation placed on contributions or withdrawals. The annual contribution is limited to 100% of the individual's earned income, not to exceed $6,500 ($5,500 + $1,000 catch-up provision) per year. Mordecai may contribute up to $1,650. Earnings received as retirement benefits and investment income are not eligible for the calculation of Roth IRA contributions.

An investor purchases $40,000 of a mutual fund when the price of the fund is $18.50. In the same year, the investor receives a $700 dividend distribution and a capital gain distribution of $1,100. Both distributions are reinvested in additional shares at a price of $17.90. If the fund has a current value of is $22.80 and the investor sells $9,000 worth of the fund, what is the investor's capital gain using the average cost method?

$1,710 Explanation: Using the average cost method, the gain is found by subtracting the cost basis from the sales proceeds. To calculate the cost basis using the average cost method, divide the sum of all investments by the number of shares owned by the investor. The investor purchased $40,000 of the fund at a price of $18.50, The total number of shares purchased is 2,162.16. The investor also received a total of $1,800 in distributions, all reinvested in additional shares when the price is $17.90. The total number of shares purchased is 100.59. The total amount invested is $41,800. The total number of shares owned is 2,262.75. Therefore, the average cost is $18.47. The number of shares being sold is 394.74 ($9,000 / 22.80). If we subtract the cost basis of $7,290 (394.74 x $18.47) from $9,000, this equals a capital gain of $1,710.

A customer opens a new margin account and buys 100 shares of XYZ Corporation at $40 per share. She then writes a call option against the position and receives a $2 premium. The customer must deposit cash in the account of:

$1,800 Explanation: The purchase of $4,000 worth of stock would require a $2,000 deposit (50% of $4,000 = $2,000). Since the call is covered, there is no margin requirement. The customer received $200 in premiums. This would be deducted from the $2,000 margin call, requiring a cash deposit of $1,800.

A company's common stock is selling in the market at a "multiple" of 15. If the market price of the common stock is currently $15, what is the earnings per share?

$1.00 When a stock is selling at a "multiple" of 15, this means that the market price is 15 times the current earnings per share. Since the market price is at $15, and the P/E ratio is 15, earnings per share is $1.00.

A Treasury bond has increased in value from 98.4 to 98.8. The bond has increased by:

$1.25 per $1,000 par value Explanation: Treasury bonds are quoted in 32nds of a point, and are then calculated as a percentage of the par value ($1,000). The difference between 98.4 and 98.8 is 4/32. One point equals $10, so 4/32 or 1/8 of a point equals $1.25.

In 2017, a customer earns $400,000 as a self-employed doctor, and contributes the maximum permitted amount to a Keogh plan. The doctor has a full-time nurse earning $40,000 per year. The contribution to be made for the nurse is: A. $0 B. $2,500 C. $3,000 D. $10,000

$10,000 If an employer earns $270,000 or more and contributes the maximum amount of $54,000 to a Keogh for 2017, then 25% of "after Keogh earnings" is used to compute the percentage to be contributed for employees. $270,000 - $54,000 = $216,000 $54,000/$216,000 = 25%, thus $40,000 x 25% = $10,000

The anti-fraud provisions of the Securities Exchange Act of 1934 apply to?

-Persons trading exempt securities in the secondary market -Persons trading non-exempt securities in the secondary market **1934 applies to Secondary Market ONLY**

An investor owns 1,000 shares of an open-end investment company. The bid price is $11.00 and the offer price is $11.58. The investment company charges a 1/2% redemption fee. If the investor redeems his 1,000 shares, how much will he receive?

$10,945 Explanation: When redeeming shares of an open-end investment company (mutual fund), an investor receives the NAV (bid price) minus any redemption fee. The investor would receive $11,000 (1,000 shares x $11.00 NAV) minus the redemption fee of $55 ($11,000 x 1/2%), which equals $10,945.

A broker-dealer wishes to give a gift to a registered representative of another firm. Industry regulations consider the gift to be of material value if it exceeded a value of:

$100 Explanation: According to the industry regulations, a gift of more than $100 would be considered substantial or of material value.

A customer purchased on margin 100 shares of ABC stock at 120 and sold short 100 shares of XYZ stock at 100. The customer also wrote an ABC 120 call @ 3 and an XYZ 100 put @ 2. What is the margin requirement for the combined transactions?

$11,000 Explanation: The FRB margin requirement for the purchase or short sale of stock is 50%. Therefore, the margin requirement for the stock purchase is $6,000 (50% of $12,000) and for the short sale is $5,000 (50% of $10,000). The call is covered since the customer owns the underlying stock and the put is covered since the customer is short the underlying stock. Since there is no margin requirement for a covered call or put, the total margin requirement is $11,000. If the question had asked for the cash deposit, subtract the total premiums received ($500) from the margin requirement of $11,000.

A customer has a long margin account with the following securities in the account. (Assume a 50% FRB initial margin requirement.) Stock Market Price Market Value Debit Balance 100 A Co. $30 $3,000 $8,400 100 B Co. $25 $2,500 200 C Co. $15 $3,000 100 D Co. $35 $3,500 ----------- $12,000 How far could the market value of the securities in the account decline, before a maintenance call will be sent?

$11,200 Explanation: To determine how far the securities worth $12,000 in the account can decline before the customer receives a maintenance call, multiply the debit balance of $8,400 by 4/3. $8,400 x 4 = $33,600 divided by 3 = $11,200. Another method that can be used is to take 1/3 of the debit balance, which is $2,800, and add it to the debit balance of $8,400. The result would be 4/3rds of the debit balance and would equal $11,200. ($8,400 + $2,800 = $11,200.)

A convertible preferred stock is convertible at $10, pays a 4% annual dividend, is callable at $110, and is trading at a current market price of $116. Based on these details, what is the parity price of the common stock?

$11.60 Explanation: The first step in determining the parity price for a convertible security is to find the conversion ratio (i.e., the number of common shares to be received if the preferred stock is converted). The conversion ratio is calculated by dividing the par value of the preferred stock ($100) by the conversion price ($10). As a result, the stock is convertible into 10 shares of common stock ($100 ÷ $10). To find the parity price of the common stock, the current market price of the preferred stock ($116) is divided by the conversion ratio (10 shares). Therefore, the parity price is $11.60 per share.

On March 1, an individual, age 40, wants to open and fund a Roth IRA at the maximum permitted level. She earns less than the adjusted gross income level that would limit her contribution. What is the maximum amount that she may place in a new Roth IRA?

$12,000

Listed below is part of the balance sheet of the MEM Corporation. ASSETS LIABILITIES Cash $2,000,000 Notes Payable $ 100,000 AccRec 3,000,000 AccPay 1,400,000 Inventories 10,000,000 Taxes Payable 1,500,000 Goodwill 20,000,000 Notes due 2021 8,000,000 Land 30,000,000 Debentures 20,000,000 The net working capital is:

$12,000,000 Explanation: Net working capital is the difference between the current assets and the current liabilities. In this example, the current assets are: Cash $ 2,000,000 Accounts Receivable 3,000,000 Inventories 10,000,000 Total Current Assets $ 15,000,000 The current liabilities are: Notes Payable $ 100,000 Accounts Payable 1,400,000 Taxes Payable 1,500,000 Total Current Liabilities $ 3,000,000 The net working capital is, therefore, $15,000,000 minus $3,000,000, which equals $12,000,000.

A customer is long 100 shares of ABC @ 32 and short 1 ABC July 30 call @ 4. What is his deposit requirement?

$1200 $1600 (50% of stock price for Reg T) - $400 (Premium received)

An investor purchases $25,000 of a mutual fund when the price of the fund is $13.20. In the same year, the investor receives a $400 dividend distribution and a capital gain distribution of $700. Both distributions are reinvested in additional shares at a price of $12.80. If the fund has a current value of $14.50, what is the investor's cost basis using the average cost method?

$13.18 Explanation: To calculate the cost basis using the average cost method, divide the total sum of all investments by the shares owned by the investor. The investor purchased $25,000 of the fund at a price of $13.20. The total number of shares purchased was 1,893.94. He also received a total of $1,100 in distributions, all reinvested in additional shares when the price was $12.80. The number of shares purchased is 85.94. The total amount invested is $26,100. The total number of shares owned is 1,979.88. Therefore, the average cost is $13.18. The current value of the fund is not relevant.

An individual purchases an Australian dollar June 65 put at 2.34 that was sold at 3.68. If the contract size is 10,000 Australian dollars, what is the individual's total profit?

$134 Explanation: Premiums for Australian dollar options are quoted in cents per unit. To express the premium in dollar terms, the decimal must be moved two places to the left. The total cost is the contract size (10,000) times the premium expressed in dollars (decimal moved two places to the left, $.0234), which equals $234. Since the contract was sold at 3.68 ($368), the profit is $134.

A customer purchases an ABC 6.5% convertible preferred stock at $80. The conversion price is $20. If the common stock is trading two points below parity, the price of ABC common is

$14 ($100 par/ $20 = 5) ($80/ $5 = $16-2= $14)

A customer's margin account has a credit balance of $20,000 and a debit balance of $15,000. On what amount will the customer be charged interest?

$15,000 Explanation: Customers are charged interest on the average daily amount of the debit balance in their account. Generally, they are not charged interest in a short account.

A convertible preferred stock with a par value of $100 is currently trading at $125 per share. The conversion ratio is 5:1. If the common stock is trading at $30 per share, what must the preferred stock's price be to be at parity?

$150 (30 x 5)

An investor purchases $40,000 of a mutual fund when the price of the fund is $18.50. In the same year, the investor receives a $700 dividend distribution and a capital gain distribution of $1,100. Both distributions are reinvested in additional shares at a price of $17.90. If the fund has a current value of is $22.80, what is the investor's cost basis using the average cost method?

$18.47 Explanation: To calculate the cost basis using the average cost method, divide the sum of all investments by the total shares owned by the investor. The investor purchased $40,000 of the fund at a price of $18.50. The total number of shares purchased was 2,162.16. The investor also received a total of $1,800 in distributions, all reinvested in additional shares when the price was $17.90. The total number of shares purchased is 100.56. The total amount invested is $41,800. The total number of shares owned is 2,262.72. Therefore, the average cost is $18.47. The current value of the fund is not relevant.

Mr. Green, a new client, decides to short 100 shares of JRF at $18 per share. What is the initial margin requirement for this trade?

$2,000 Explanation: If the initial transaction in a margin account is a short sale, industry rules require a minimum equity deposit of $2,000 or the required Reg. T deposit, whichever is greater. Since $2,000 is greater than $1,800, the required deposit is $2,000. For a purchase, the minimum equity requirement is the lesser of $2,000 or 100% of the purchase price.

An individual's home has a resale value of $500,000 and an assessed value of $200,000. If the tax rate is 10 mills, the property tax is:

$2,000 Explanation: Property tax is computed by multiplying the assessed value by the millage rate. A mill equals 0.001 or $1 per $1,000 assessed value. The tax is $2,000 ($200,000 x .001 x 10 mills)

An investor purchased $100,000 face value of a 12% municipal bond that matures December 1, 2041. The transaction settles on August 1. The investor owes accrued interest of:

$2,000 Explanation: The bonds purchased by the investor will generate yearly interest of $12,000 ($100,000 par multiplied by 12%). The fact that the bonds mature on December 1, 2041 signifies that interest payments are made every December 1 and June 1. The investor will, therefore, owe 60 days of accrued interest (from June 1, the last coupon, up to but not including the settlement date of August 1). Since the yearly interest is $12,000, accrued interest would be $2,000 (60/360 x $12,000).

An investor writes an uncovered RST May 25 put for a premium of 4. The maximum loss the investor could sustain is:

$2,100 Explanation: If RST Corporation's market price declines to pennies per share, the owner of the put could buy the RST stock for pennies and put it to the writer for $25 per share, or $2,500. This is the price that the writer would be required to pay for the stock. However, since the writer received $400 in premium, the maximum loss he could have will be $2,100 ($2,500 loss - $400 premium = $2,100 loss).

A client has a margin account with the following positions: short 2,000 shares of EXA at $22 and long 2,000 shares of EXA at $24. The client's maintenance requirement is: Series 7 (21-21) - Short Against The Box

$2,400 Explanation: If a client is long and short an equal number of shares of the same security, the maintenance requirement is equal to 5% of the long position. 5% of $48,000 (2,000 shares x $24) equals $2,400.

The initial FRB margin requirement is 50%. A customer purchased 100 XRX at $100 per share depositing the required margin. If Xerox increased in value to $150 per share, how much SMA would the customer have in the account?

$2,500 Explanation: First, determine the amount of the debit balance. If the customer purchased $10,000 worth of stock at a 50% margin requirement and deposited $5,000, the debit balance is $5,000. ($10,000 market value - $5,000 margin requirement = $5,000 debit balance). XRX increased to $150 per share, making the market value $15,000. The equity then increases to $10,000. The SMA (excess equity) is found by subtracting the FRB required equity of the current market value in the account (50% x $15,000 = $7,500) from the actual equity in the account ($10,000). The SMA is, therefore, $2,500.

A 60-year-old individual has invested $30,000 in a nonqualified variable annuity. The annuity's value is currently $40,000. If the individual withdraws $20,000 and is in a 28% tax bracket, his tax liability will be:

$2,800 Explanation: The amount invested in a nonqualified variable annuity may not be deducted from income. All earnings accrue tax-deferred. A withdrawal will be taxed on a LIFO method, meaning the earnings (last in) will be considered the first to be withdrawn. Earnings are taxed as ordinary income. Withdrawal of the invested amount is considered return of capital and is not taxed. The annuity has earnings of $10,000 and, therefore, $10,000 of the $20,000 withdrawn is taxable and the remaining $10,000 is considered return of capital. The tax liability is $2,800 ($10,000 taxable amount x 28% tax bracket). Had the individual been under 59 1/2 years of age when the withdrawal was made, the distribution also would have been subject to a 10% penalty on the taxable portion.

A GNMA pass-through is quoted 98.10 to 98.18. This quote represents a spread per $1,000 face value of:

$2.50 Explanation: GNMA pass-through certificates (as with T-notes and T-bonds) are quoted in 32nds. The spread of .08 represents 8/32 or 1/4 (.25) and has a value of $2.50 per $1,000.

An investor sells ten 5% bonds and buys another 10 bonds with a 5 1/4% coupon rate. The investor's yearly cash flow from the bonds will have increased by:

$2.50 per bond Explanation: The investor's yearly return will have increased by $2.50 per bond. The increase is 1/4% (5% to 5 1/4%), which is 1/4 of 1% of the par value of $1,000, or $2.50.

A corporation has $20 million net income after taxes, 7 million common shares outstanding, and $15 million of 6% preferred stock ($25 par). What is the corporation's earnings per share (EPS)?

$2.73 dividends (15 mill x 6%= 900,000) (20 mill - 900k = 19.1 mill earnings available) (19.1 mill / 7 mill = $2.73)

A convertible preferred stock issue (par value $100) is selling at $125 and is convertible into five shares of common stock. The conversion price of the common stock is

$20 (100/5)

A customer has a long margin account with the following securities in the account. (Assume a 50% FRB initial margin requirement.) Stock Market Price Market Value Debit Balance 100 A Co. $30 $3,000 $8,400 100 B Co. $25 $2,500 200 C Co. $15 $3,000 100 D Co. $35 $3,500 ----------- $12,000 The minimum maintenance requirement for this account is:

$3,000 Explanation: The minimum maintenance requirement states that the equity must equal at least 25% of the market value of the securities in the account. This equals $3,000. (25% of $12,000 = $3,000.)

A customer has the following accounts with a brokerage firm. Cash Account $20,000 securities (market value) $10,000 cash Long Margin Account $60,000 securities (market value) $30,000 debit balance $10,000 SMA Short Margin Account $40,000 securities (market value) $60,000 credit balance The total amount of cash that may be withdrawn from all the accounts is:

$20,000 Explanation: The $10,000 in cash may be withdrawn from the cash account. The $10,000 SMA in the long margin position may also be withdrawn for a total of $20,000. The short margin position does not have SMA. Therefore, in this example, nothing can be withdrawn from that position.

A municipal finance professional (MFP) and her spouse make a political contribution of $400 from a joint account and they both sign the check. According to the MSRB political contribution rules, the contribution is viewed as a:

$200 contribution from each party Explanation: When both an MFP and her spouse sign a contribution check, the contribution is considered to have been made by both and is split equally between the contributors. On the other hand, if the MFP is the only person who signs the check, then the entire amount of the contribution is allocated to the MFP. If that were the case, the underwriting ban would have been triggered since the contribution amount exceeds $250. There is no limit if the MFP's spouse writes a check from his personal account, rather than from a joint checking account.

An investor writes an uncovered ABC March 50 call for a premium of 4. At expiration, ABC is at $56 per share and the call option is exercised. If the stock is purchased by the writer at the current market price for delivery, what is the writer's profit or loss?

$200 loss Explanation: When the stock is called away from the writer, he is obligated to sell the stock at the strike price of 50, receiving $5,000. The writer also received the premium of $400. Since the stock cost $5,600 to buy in the market, the writer incurs a loss of $200 ($5,600 cost - $5,400 received).

On Monday, June 15, an investor purchases for regular-way settlement, $20,000 face value of 8% municipal bonds that mature on November 1, 2035. What is the dollar amount of accrued interest that the investor is required to pay?

$208.88 Explanation: Accrued Interest Formula = (Principal x Rate x Days of Interest) / 360 = ($20,000 x 8% x 47) / 360 = $208.88

An investor writes an uncovered RST May 25 put for a premium of 4. At what market price will the investor break even?

$21 Explanation: The writer calculates his breakeven point by deducting the $4 premium from the $25 exercise price. The writer will, therefore, break even at $21.

The following closed-end funds are listed in The Wall Street Journal. Net Asset Value Market Price American Fund 23.75 24.25 Bunker Hill Fund 21.85 21.50 A customer purchasing the Bunker Hill Fund at the current market price will pay:

$21.50 + a commission Explanation: The customer will pay $21.50 plus a commission. The Bunker Hill Fund is a closed-end investment company, which sells at its current market value ($21.50) plus a commission. A client will purchase the common stock of a corporation in the same manner. Open-end investment companies (mutual funds) sell at their offering price, which is the net asset value plus a sales charge (when applicable).

A closed-end fund trading on the NYSE has a current bid price of $21.50 and an offer price of $21.70. A customer purchasing the fund would pay:

$21.70 plus a commission Explanation: The customer would pay $21.70 plus a commission. A closed-end fund is purchased and sold like any other stock traded on the NYSE. The customer would pay the offer price plus a commission or receive the bid price less a commission when selling the security. The term sales charge refers to the built-in compensation charged by an open-end (mutual fund) company when a customer buys shares of the fund.

If ABC stock is currently trading at 35.25 and the October 35 put option has a premium of 2.25, what is the time value of this option?

$225 Explanation: Time value is calculated by taking the difference between an option's premium and its intrinsic value. Since the market price of the stock is greater than the strike price of the put option, this option is out-of-the money and has no intrinsic value. The entire premium of this option, $225, is considered the time value.

The minimum equity requirement for a pattern day trader is:

$25,000, which must be deposited before the client may continue day trading Explanation: The minimum equity requirement for a pattern day trader is $25,000. This amount must be deposited in the account before the customer may continue day trading and must be maintained in the customer's account at all times. Day-trading buying power is limited to four times the trader's maintenance margin excess, determined as of the close of the previous day.

A customer buys an IBM call option and pays a 2.50 point premium. The aggregate dollar amount paid is:

$250.00 Explanation: Each option contract is based on 100 shares of common stock. The dollar amount paid is $250 ($2.50 x 100 shares).

A customer owns a total of 750 shares of a mutual fund and has invested $22,000 over the last three years. If the fund is currently valued at $31.20, what is the customer's cost basis using the average cost method?

$29.33 Explanation: To calculate the cost basis using the average cost method, divide the sum of all investments by the total shares owned by the investor. The investor owns 750 shares and the total amount invested is $22,000. Therefore, the average cost is $29.33. The current value of the fund is not relevant.

An investor has taken the following gains and losses during the tax year: a $19,000 capital gain on stock positions and a $24,000 loss on option positions. What amount of ordinary income may the investor offset this year?

$3,000 Explanation: Capital gains may be offset against capital losses regardless of whether they are from stocks or options. The maximum capital loss an investor may write off against ordinary income in one tax year is $3,000. The balance of the $2,000 capital loss must be carried forward to the next year.

An individual purchases 10 ABC June 90 calls @ 4 and writes 10 ABC June 95 calls @ 2. The individual's maximum profit is:

$3,000 Explanation: The investor has established a debit spread and will profit if the spread (difference in premium) widens. The widest the spread can get is the difference in strike prices (95 - 90 = 5). The maximum profit is the difference in the strike prices ($5,000) minus the net debit ($2,000) or $3,000.

Corporate bonds are usually :

-Term Bonds -Quoted on a percentage of par basis

Mrs. Jones owns stock from which she received $3,000 in cash dividends. Mr. Jones owns stock from which he received $400 in cash dividends. How much of the cash dividends received are Mr. and Mrs. Jones liable for when filing their joint return?

$3,400 Explanation: Cash dividends received by individuals are fully taxable and, therefore, the entire $3,400 total of dividends is liable for taxes.

An investor sells short 1,000 shares of JonCo stock at 3.50. The customer must deposit:

$3,500 Explanation: The required equity for a short sale where the stock is less than $5 per share is the greater of $2.50 per share or 100% of the market value. An investor selling 1,000 shares of JonCo stock short must deposit $3,500 because the market value (1,000 shares x $3.50) is greater than $2.50 per share (1,000 shares x $2.50).

An investor purchases 1 XYZ October 40 put when the market price of XYZ is $41 per share, and pays a premium of $3. What is the maximum profit the investor can have?

$3,700 Explanation: XYZ shares could possibly become worthless. The investor can then buy 100 shares for pennies and put (sell) it to the writer for the $40 per share strike price. This equals $4,000 ($40 x 100 shares). The investor's profit is $4,000 minus the $300 premium paid for the put, which equals $3,700. The $3,700 is the maximum profit the investor can have since the share's price cannot go lower than zero.

A customer is long 1 July 30 Call @ 3. How much cash must he deposit?

$300 100% of the premium

A customer in the highest tax bracket has $1,500 in long-term capital gains from stock transactions at the end of the year. The customer will need to pay taxes of:

$300 Explanation: Long-term capital gains are gains on securities held in excess of 12 months and are taxed at a maximum rate of 20%. Although the investor is in the highest tax bracket, the investor will be taxed at a rate of 20%. Therefore, the customer will need to pay taxes of $300 ($1,500 x 20% = $300).

Mr. Smith sells an ABC Corporation April 30 put for $5 and an April 30 call for $3. ABC Corporation is selling in the market at $28. ABC Corporation subsequently declines to $25 per share. The call option expires and the put side of the straddle is exercised. Mr. Smith then sells the 100 shares of ABC Corporation put to him, at the current market price of $25. The overall profit or loss for Mr. Smith is a:

$300 profit Explanation: Mr. Smith received $800 in premiums. The call option expires. The put side of the straddle is exercised. Mr. Smith must buy 100 shares of ABC Corporation that is put to him at the exercise price of $30. He then sells the shares purchased for $30 at the current market price of $25, realizing a loss of $5. However, he has received $8 in premiums. Therefore, he will have an overall $300 profit ($8 premium received for the straddle minus the $5 loss on the sale of 100 shares of ABC Corporation equals a $3 profit).

XYZ Corporation is selling 10,000,000 shares of common stock through an underwriter, at $15 per share. The underwriting spread is as follows. The manager's fee is 20 cents, the underwriting risk 20 cents, and the selling concession 60 cents. Selling group members have been allocated 500,000 shares. If the selling group members sell their entire allocation, their compensation will be:

$300,000 Explanation: Selling group members are broker-dealers who participate in the sale of the issue on a best-efforts basis (i.e., assuming no risk). They receive a selling concession (compensation) that is less than that received by syndicate members, who do assume risk. The selling concession is $.60. This is part of the $1.00 underwriting spread. If the selling group members sell their entire allocation, they will receive $300,000 (500,000 shares x $0.60 per share). Syndicate members receive the selling concession and the underwriting risk per share sold, or $.80.

The Class A shares of the GEMCO Balanced Fund carry a sales charge of 4.5%. If the next computed net asset value per share is $32.74, purchase orders will be filled at a price of

$34.28 per share. = NAV / (100 - sales charge)

A client purchases $900,000 of stock in a margin account and deposits the Regulation T margin requirement. If the current value of the stock is $800,000 and the broker-dealer declares bankruptcy, SIPC would cover:

$350,000 Explanation: When the customer met the original Reg. T call, he deposited $450,000 (50% of $900,000) and borrowed $450,000, which is the debit balance. Now the market value (MV) has fallen from $900,000 to $800,000, so the new equity level is $350,000 ($800,000 MV minus the debit balance of $450,000 equals $350,000). SIPC will cover the customer's current equity in the margin account of $350,000. SIPC does not cover a decline in the market value of securities.

A customer's margin account has a market value of $750,000 and a debit balance of $400,000. She also has a commodities account that has equity of $150,000. If the firm goes bankrupt, SIPC will provide coverage to this customer for:

$350,000 in the margin account and nothing for the commodities account Explanation: SIPC will cover the customer's equity in the margin account ($350,000). SIPC does not provide coverage for commodities accounts.

A customer has purchased 10 ABC January 50 calls, paying a $2 premium and 10 ABC January 50 puts, paying a $2 premium. The market price of ABC stock is $50 per share. The buyer of these 10 straddles will need to deposit

$4,000 Explanation: When buying options, 100% of the purchase price (the premium) must be deposited. The customer paid a $2 ($200) premium for the call and a $2 ($200) premium for the put (a $4 premium for one straddle). The customer purchased 10 straddles and paid $400 per straddle for a total of $4,000. (10 straddles x $400 = $4,000.)

An investor purchases an ABC Corporation October 50 put and pays a premium of $7. The underlying security declines to $40 per share. For tax purposes, the proceeds of the sale are:

$4,300 Explanation: The proceeds of the sale for tax purposes are $4,300 ($5,000 strike price minus the $700 premium paid for the option equals the proceeds of the sale). The cost basis of the stock purchased is $4,000. The customer's profit is then $300.

A town's ad valorem tax rate is 20 mills on 60% of the assessed value. Your client owns a property with a market value of $500,000, and the town has assessed it at $400,000. The taxes due on this property are

$4,800 (400,00 x 60%) (240,000 x 0.02)

An investor owns 280 shares of XYZ Corporation. XYZ Corporation pays a 15-cent quarterly dividend. XYZ Corporation announces a 5-for-4 split. The dividend per share is adjusted to reflect the split. How much will the investor receive in dividends each quarter after the split?

$42.00 Explanation: After the split, the investor would own 350 shares (280 x 5/4 = (280 x 5) / 4 = 350) and would receive $42.00 each quarter (350 shares x $0.12 = $42.00) in dividends. To find the adjusted dividend per share, multiply the inverse of the split by the original dividend of $0.15 ([$0.15 x 4] / 5 = $0.12). Since the dividend is adjusted for the split, the investor would receive the same total dividends after the split as before (280 shares x $0.15 per share = $42).

With no other securities position, a customer sells short 100 shares of ABC at $40 and sells 1 ABC October 40 put for $500. The customer will break even when the price of the stock is at:

$45 Explanation: An individual who sells short risks a loss if the price of the stock rises. If the price rises to $50 and the stock is bought in the open market to cover, the loss will be $1,000 minus the premium, for a net loss of $500. If the market price rises to 45, the loss of $500 is exactly matched by the premium income of $500 and the investor breaks even. The breakeven point for a short seller who writes a put is the market price of the short sale plus the premium.

An investor buys 100 shares of XYZ at $50 per share and, at the same time, writes an XYZ May 50 call option for a $5 premium. Excluding commissions and dividends, at what price would XYZ need to be selling for the writer to break even?

$45 Explanation: The breakeven point for the writer of a covered call is the original cost of the stock minus the premium received on the option (50 - 5 = 45). If the market price were at 45 at expiration, the call would expire and the writer would keep the $500 premium. However, the stock purchased at $50 would be worth only $45, which is equal to the investor's cost.

A company has a noncumulative preferred stock outstanding that pays a $5 dividend per year. If dividends on the preferred stock were not paid last year, but will be paid this year, how much will the preferred stockholder receive?

$5 Explanation: The preferred stock is noncumulative, which means that if the dividend is not paid, it does not accumulate to the next year. Therefore, the preferred stockholder will receive only $5 for this year.

A new issue of municipal bonds has an aggregate par value of $10,000,000. The syndicate received $5,000,000 in designated orders, $5,000,000 in group orders, and $5,000,000 in member orders. How will the issue be allocated?

$5 MM group and $5 MM designated Explanation: When allocating bonds in a new municipal issue, presale orders normally have first priority. This is followed by group net, designated, and then member orders. The 5 MM in group orders and 5 MM in designated orders will be allocated. There are no bonds left for member orders.

A customer purchases $10,000 of stock on margin. Before depositing the required amount, the stock rises to a market value of $12,000. How much will the customer be required to deposit?

$5,000 Explanation: Regulation T requires 50% of the purchase price to be deposited by the customer within two business days after the settlement date of the transaction (five business days from the trade date). Any market price change during this period will not affect the amount of the deposit. The requirement will be $5,000 (50% of the purchase price of $10,000).

On March 9, an investor purchased 1,000 shares of ABC at $20 and then on July 20, the investor purchased an additional 1,000 shares of ABC at $12. On May 11 of the following year, the investor sold 1,000 shares of ABC at $25. For tax purposes, he must report a:

$5,000 long-term capital gain Explanation: In this question, the investor has two positions in ABC stock and each position was purchased at different times and at different prices. When an investor sells a portion of his holdings, unless his sell order ticket identifies the specific shares that he is selling, the IRS will assume that first-in, first-out (FIFO) will be the method to be used. Since the investor did not identify the shares to be sold, it is the first shares that were purchased in March at $20 that were sold. Therefore, since the shares were held for more than one year, the investor will report a $5,000 long-term capital gain.

A customer buys 5M of 3 1/2% Treasury Bonds at 101-16 The customer will pay how much for the bonds?

$5,075 The purchase price is 101-16 = 101 16/32nds = 101.5% of $5,000 = $5,075

A customer is short a DMF 50 call for which he received a premium of 4. Seven months later, the call was exercised when the current market for DMF was 56. Under the Internal Revenue Code, what were the proceeds of his sale?

$5,400 50+4= 54

An accountant earns $200,000 and wishes to make the maximum IRA contribution for himself and his nonworking spouse. He can contribute a maximum of:

$5,500 in her account plus $5,500 in his account Explanation: An individual with earned income and a nonworking spouse may contribute a total of $11,000 for himself and his wife. However, the contribution must be made in two separate accounts, each housing $5,500.

A customer has a restricted margin account with a debit balance of $7,500. The account is credited with $1,600 in cash dividends and debited with interest charges of $50. The debit balance after the adjustments is:

$5,950 Explanation: The debit balance is reduced from $7,500 to $5,900 when the cash dividends of $1,600 are credited to the account ($7,500 - $1,600 = $5,900). Adding interest charges of $50 to the debit balance results in a final debit balance after adjustments of $5,950 ($5,900 + $50 interest charges = $5,950).

A customer buys bonds with a $50,000 par value at 85 1/2. The bonds are callable at 110. If the customer holds the bonds to maturity, he will receive:

$50,000 Explanation: At maturity, the holder of the bonds will receive the par value, which in this example is $50,000. The call price and market value are not relevant.

On June 5, 2013, an investor purchased 100 shares of ABC at 20. On November 10, 2013, he purchased an additional 100 shares of ABC at 12. On January 20, 2014, he sold 100 shares of ABC at 15. For tax purposes, he would have reported a:

$500 capital loss in 2014 Explanation: In this question, the investor has two positions in ABC stock. Each was purchased at different times and at different prices. When selling a portion of his holdings, unless the investor identifies (on the order ticket) the specific shares he is selling, the Internal Revenue Code requires the use of the FIFO method. Since the investor did not identify the shares sold, it is assumed that the first shares purchased (at 20, in June) were the shares sold. Therefore, the investor would have reported a loss of $500 in 2014.

An investor writes an uncovered ABC March 45 put for a premium of 4. At expiration, ABC is at $36 per share and the put option is exercised. If the stock is immediately sold by the writer at the current market price, what is the writer's profit or loss?

$500 loss Explanation: When the stock is put to the writer, he must buy the stock for $4,500. His cost basis for tax purposes is $4,100 ($4,500 strike price - $400 premium received). Since he sold the stock for $3,600, he has a net $500 loss ($4,100 - $3,600).

In May, a customer sells an STC July 40 listed call for a $6 premium and buys an STC July 30 listed call for $10. Near expiration, STC is selling at $39. The 40 call expires and the customer closes out the 30 call at its intrinsic value. The net result is a:

$500 profit Explanation: When the market price of STC is at $39, the July 30 call has an intrinsic value of 9 points. Since the investor paid a debit of $400, this will result in a profit of $500 ($900 intrinsic value - $400 debit).

What is the SRO maintenance requirement on a $1 million purchase of a 2x Long Gold Index ETF?

$500,000 Explanation: Leveraged ETFs have maintenance requirements in excess of the typical SRO thresholds of 25% on long positions and 30% on short positions. The margin requirement on these securities can be computed by multiplying the portfolio leverage factor by the standard SRO maintenance requirement. In this case, the standard long requirement is 25% multiplied by a factor of 2, so the client must maintain a 50% margin. $1,000,000 x 25% = $250,000. $250,000 x 2 = $500,000.

A customer has a cash account that has securities valued at $320,000 and $180,000 in cash. The customer and a spouse also have a joint account with securities valued at $120,000 and $270,000 in cash. If the member firm were to become bankrupt, the coverage under SIPC would be:

$500,000 for the individual account and $370,000 for the joint account Explanation: Both the individual account and the joint account are considered separate customers and will each receive independent coverage of $500,000, of which no more than $250,000 may be for cash. In the individual account, full coverage will be provided of $500,000 ($320,000 of securities and $180,000 in cash). In the joint account, the full value of the securities is covered. However, only $250,000 of the cash in the account is covered. The total coverage for the joint account would be $370,000 ($120,000 + $250,000). For the balance of $20,000 cash, the customer will become a general creditor of the broker-dealer.

A Keogh Plan is a type of retirement plan that allows self-employed individuals to contribute 20% of their income with a maximum contribution of:

$53,000 Explanation: A Keogh Plan allows self-employed individuals to contribute 20% of their income with a maximum deductible contribution of $53,000. (For 2014, the limit was $52,000.)

XYZ Technology Fund permits rights of accumulation. A shareholder has invested $9,000 and signed a letter of intent for a $15,000 investment. If his reinvested dividends during the 13 months total $720, how much money must he contribute to fulfill the letter of intent?

$6,000

The closing prices of two mutual funds on Monday, July 17, are: Bid Offer Change WORLD FUND 18.30 20.00 +.10 OCEAN FUND 5.25 5.50 +.02 An investor who bought 300 shares of WORLD Fund on Monday, July 17, will pay:

$6,000 Explanation: When an investor buys shares of a mutual fund (open-end investment company), the investor pays the offering price, which includes the sales charge. When buying the WORLD Fund, the investor would pay the offering price of $20.00. Three hundred shares purchased at $20.00 equals, $6,000 ($20.00 x 300 shares).

An investor owns 4,000 shares of common stock that pays a quarterly dividend of 35 cents. If the investor purchases 500 additional shares prior to the first ex-dividend date of the year, what is the investor's expected annual income from the investment?

$6,300 Explanation: The annual income from the common stock is determined by multiplying the annual dividend by the number of shares owned by the client. Since the additional 500 shares were purchased prior to the first ex-dividend date of the year, the investor is entitled to four quarterly dividends on an ownership level of 4,500 shares. If the annual dividend is $1.40 (35 cents x 4), the annual income would be $6,300 (4,500 x $1.40).

A municipality is issuing 40,000 bonds at a public offering price of $1,000. The manager of the underwriting syndicate receives $1.50 per bond. The total takedown is $6.50 per bond and the selling concession is $4.00 per bond. When the issue is completely sold, the managing underwriter's fee will total:

$60,000 Explanation: The syndicate manager receives $1.50 for every bond. The manager will receive, in total, $60,000 (40,000 bonds x $1.50 per bond).

A customer buys 100 shares of ABC stock at $58 and buys 1 ABC Jul 55 Put @ 2.50 on the same day. The breakeven point is:

$60.50 Long Stock/Long Put Breakeven = Stock Cost + Premium

A customer buys 10M of Allied Corporation 12% debentures, M '29, at 90 on Monday, Oct 8th. The interest payment dates are Feb 1 and Aug 1. The trade settled on Tuesday, Oct 11th. The amount of the next interest payment will be:

$600 10M stands for 10 - $1,000 bonds (M is latin for $1,000) = $10,000 face amount of bonds. The bonds pay 12% interest annually. 12% of the $10,000 is $1,200. However, the question asks how much will be received in the next payment. Since bonds pay interest semi-annually, the amount received per payment is $600.

An investor buys an STC May 30 call @ 8 and sells an STC May 40 call @ 2. The investor's maximum potential loss is:

$600 Explanation: This is a debit spread since the investor paid a net premium of $600. The maximum potential loss for a debit spread is the net premium ($600).

Wilsons Chemicals bonds have a nominal yield of 6.6%. They closed the previous day at 91 7/8. An owner of 10 bonds will receive a yearly interest payment of:

$660 Explanation: A nominal yield of 6.6% for a corporate bond with a $1,000 par value equals $66 in interest payments. If an investor owns 10 bonds, he will receive an annual interest payment of $660.

In November, a customer sells 1 ABC Jan 60 Call @ 7 when the market price of ABC is $61. The breakeven point for the position is: A. $63 B. $64 C. $67 D. $68

$67 Short call breakeven = Strike Price + Premium

ABC Jan 50 call contracts are trading in the market at 3.40. What is the dollar price that a customer would pay for 2 contracts at this price?

$680 The premium of 3.40 x 2 = 680

XYZ corporation has income before taxes of $2 million and received $100,000 in preferred dividends from a company in which it owns 25% of the outstanding shares. If XYZ corporation is in the 34% tax bracket, it will pay taxes of:

$686,800 Explanation: Since XYZ corporation owns 25% of the outstanding shares, it is exempt from paying taxes on 80% of dividends received from the stock. The corporation would need to pay taxes on only $20,000 of the dividends received (20% of the $100,000 in preferred dividends) plus the $2,000,000 of income the corporation earned. Since the corporation is in the 34% tax bracket, the tax would be $686,800. (34% of $2,020,000 = $686,800.)

A customer's margin account has a market value of $15,000, a debit balance of $8,000, and SMA of $1,000. The equity in the account is:

$7,000 Explanation: The equity in a long margin account equals market value minus the debit balance. The equity equals $7,000 ($15,000 - $8,000). SMA does not enter into the calculation of equity.

Ms. Green buys 300 shares of RSW at 15. She then writes 3 RSW July 20 calls at 1 and writes 3 RSW July 10 puts at 50 cents. Ms. Green's maximum potential loss is:

$7,050 Explanation: Ms. Green has written 3 covered calls and 3 uncovered puts. In both cases, the maximum loss occurs if the underlying stock (RSW) becomes worthless. If the market price of RSW is zero, the 3 covered calls would result in a $4,200 loss (300 shares x $15 purchase price - $300 premium received). At zero, the 3 uncovered puts are exercised for a net loss of $2,850 (3 contracts x $10 strike price - the premium received of $150). Thus, the total loss is $7,050 ($4,200 + $2,850).

A corporate bond has increased in value by 3/4 of a point. The bond has increased by:

$7.50 per $1,000 par value Explanation: One point equals $10. An increase of 3/4 of a point in a corporate bond is $7.50 per $1,000 of par value.

A customer buys a real estate limited partnership interest by contributing $20,000 and signing a nonrecourse note for $50,000. The customer's beginning basis is

$70,000.

A customer sells 1 ABC Jan 50 Call @4 and sells 1 ABC Jan 50 Put @3 when the market price of ABC = $51. The maximum potential gain is:

$700 The customer has sold a put and a call on the same stock with the same price and same expiration. This is a short straddle. If the market does not move from $50, both the call and put expire "at the money" and the $700 premium is paid.

On September 1, an investor sold 100 shares of KLP Corporation common stock for a loss of $1 per share. On September 15, he purchased a KLP convertible bond with a conversion price of $40. How much of the original loss may he now declare for tax purposes?

$75 (1000 / 40= 25) (100-25= 75)

A bond purchased at 104 eight years ago has an adjusted cost basis of 102 1/2. If the bond was sold at 110, the tax consequence is a:

$75 gain per bond Explanation: When a bond is purchased at a premium, the premium must be amortized over the life of the bond. If the bond is sold prior to maturity, the loss or gain is determined from its adjusted cost basis. The adjusted cost basis is given as 102 1/2, so no calculation is necessary. There is a gain of $75 per bond ($1,100 sale price minus $1,025 adjusted cost basis).

A customer purchases 10 M Dade Co. Florida 7.50% G.O. bonds at a 9.50 basis. How much interest will she collect each year?

$750 Explanation: 10 M equal's $10,000 par value of bonds (the symbol M refers to thousands). The coupon rate is 7.50%. Therefore, the annual interest is $750 ($10,000 x 7.50%).

A client is long and short 1,000 shares of the same security. If the current market value is $80,000, the client is permitted to borrow up to:

$76,000 Explanation: If a client is long and short an equal number of shares of the same security, the maintenance requirement is equal to 5% of the long position. The maintenance requirement is equal to $4,000 (5% of $80,000). Therefore, the client is permitted to borrow 95% of $80,000, or $76,000.

If a customer opens a spread on Canadian dollars (10,000 units) by purchasing 1 Dec 74 call for 2.30 and selling 1 Dec 77 call for 1.50, what is the total cost of this debit spread?

$80 The customer pays a premium of 2.30 ($230) and receives a premium of 1.50 ($150) for a net debit of .80, or $80.

A customer has the following accounts with a brokerage firm. Cash Account $20,000 securities (market value) $10,000 cash Long Margin Account $60,000 securities (market value) $30,000 debit balance $10,000 SMA Short Margin Account $40,000 securities (market value) $60,000 credit balance The Federal Reserve Board margin requirement is 50%. The total equity in all the accounts is:

$80,000 Explanation: The equity in the cash account equals $20,000 market value of the securities plus $10,000 in cash, for a total of $30,000. The equity in the long margin account is the market value of the securities ($60,000) minus the debit balance ($30,000). This equals $30,000. The $10,000 SMA is not taken into account when computing equity. The equity in a short margin account is computed by subtracting the current market value of the securities ($40,000) from the credit balance ($60,000). This equals $20,000. Adding the equity in all of the accounts, the total equity is equal to $80,000. ($30,000 equity in the cash account + $30,000 equity in the long margin account + $20,000 equity in the short margin account = $80,000.)

An individual invested $30,000 in an oil and gas balanced program as a limited partner. His portion of a recourse loan is $50,000. Assuming sufficient passive income, the maximum passive losses that a limited partner may claim is:

$80,000 Explanation: The maximum amount of losses that may be deducted by a limited partner is the extent of his basis (in this question, $80,000). Assuming sufficient passive income, the limited partner may deduct $80,000.

A customer sells short 100 shares of XYZ Corporation at $78 per share. The support and resistance levels for XYZ are at $70 and $80, respectively. If he wishes to protect his position, which of the following is the best place to put in a buy stop order?

$80.10 The client will want to place a buy stop a little above the resistance level to protect himself against an upside breakout. Entering a buy stop order too close to the purchase price (78.10) would not afford the client an opportunity for gain.

A customer sells 1 ABC Jul 45 Put at $9 when the market price of ABC is $38. The market falls to $28 and the customer is assigned. The customer then sells the stock in the market. The loss is:

$800 When exercised, the writer must buy the stock for $45. He then sells the stock at $28 for a loss of 17 points. Since 9 points was collected as premiums, the net loss is 8 or $800.

An investor purchases 100 shares of XYZ at 60 and also writes an XYZ 65 call @ 3. If the call is exercised when the market price of XYZ is 70, what is the investor's profit?

$800 Explanation: If the call is exercised, the investor will be required to sell his stock at the strike price of 65 (not the market price of 70). The proceeds of the sale will be $6,800 ($6,500 strike price plus $300 premium received). Since his original cost is $6,000, he will have a profit of $800.

On December 16, a Mr. Smith purchased 2 listed XYZ May 70 calls and paid a $4 premium for each call when the current market price of XYZ Corporation was $69 per share. If, in May, the market price of XYZ Corporation is $67 and the calls expire, Mr. Smith loses:

$800 Explanation: Mr. Smith will not exercise the call options. At expiration, the market price of XYZ is $67, which is less than the exercise price. Therefore, the options expire worthless. Mr. Smith loses $800 ($400 per contract times 2), the entire amount of the premium paid.

A customer buys an EK October 50 call paying a $4 premium and an EK October 50 put for a $4 premium. The price of EK increases to $66 per share. The put option expires unexercised but the customer closes out the call option at its intrinsic value. The customer has a net:

$800 profit Explanation: The customer paid $800 in premiums ($400 for the call and $400 for the put). The call is liquidated for its intrinsic value of $1,600 (the in-the-money amount). After deducting the $800 paid in premiums from the $1,600 proceeds, the customer has a net $800 profit.

If an investor sold two BCD Feb 40 calls at 4 on August 4, 2018, and the call expired unexercised, what were the tax consequences?

$800 short-term capital gain for tax year 2019 Uncovered options writers always have short-term gains or losses. LO 10.i

An investor places $100,000 into an oil and gas limited partnership program. To comply with FINRA rules, what is the minimum amount of the investment that must be received by the business?

$85,000 - 15% is expenses and the rest is put into the program

Disregarding commissions, and investor purchasing $10,000 face amount of Treasury notes at a price of 98.12 would expect to pay

$9,837.50 98.12 = 98 12/32,

XYZ corporation has 7,000,000 shares of common stock ($1 par value) authorized, of which 5,000,000 shares have been issued. There are 500,000 shares of treasury stock. The current market price of XYZ is 20. The market capitalization of the outstanding common stock is:

$90,000,000 Explanation: Outstanding shares are issued shares minus treasury stock (shares repurchased by the company). There are 4,500,000 shares outstanding with a market value of $20.00 per share. Therefore, the market capitalization is $90,000,000.

What is the SRO maintenance requirement on a $1 million short position of a 3x Inverse Gold Index ETF?

$900,000 Explanation: Leveraged ETFs have maintenance requirements in excess of the typical SRO thresholds of 25% on long positions and 30% on short positions. The margin requirement on these securities can be computed by multiplying the portfolio leverage factor by the standard SRO maintenance requirement. In this case, the standard short requirement is 30% multiplied by a factor of 3, so the client must maintain a 90% margin. $1,000,000 x 30% = $300,000. $300,000 x 3 = $900,000.

If a municipal bond is issued at par and later purchased for 97 plus accrued interest of $32, what is the purchaser's cost basis?

$970 purchase price of 97 (970) is the cost basis

A 68 year old client has made $12,000 in contributions over the last 10 years into a non-tax qualified variable annuity and is now retiring. The value of the annuity now is $20,000 and the client elects to take a lump-sum settlement. What amount of the payout will be subject to ordinary income tax? (A) $ 8,000 (B) $ 12,000 (C) $20,000 (D) Zero

(A) $ 8,000

An investor buys 100 shares of ABC at 50 in May. He also buys an August 80 put for 3 on an index with a multiplier of 100. In August, the ABC shares decline to 45 and the index declines to 70. If the investor closes out his stock position and exercises his put, what is his profit or loss? (A) $200 profit (B) $500 profit (C) $200 loss (D) $500 loss

(A) $200 profit

A constant dollar investment plan requires that: (A) A fixed amount in dollars be invested at regular intervals. (B) A fixed percentage of a portfolio's assets be kept in equities regardless of stock prices. (C) A fixed amount of dollars of a portfolio's assets be kept in equities regardless of stock prices. (D) A fixed number of shares be invested at regular intervals.

(A) A fixed amount in dollars be invested at regular intervals.

All of the following entities are permitted to issue municipal bonds that are exempt from taxation at the federal level EXCEPT: (A) A utility company that is privately owned (B) An authority created to build and manage an airport (C) A hospital created as a non-profit entity (D) A district created to build and manage infrastructure such as dams and levies

(A) A utility company that is privately owned

Which of the following is an activity that is NOT performed by a broker's broker in the municipal securities industry? (A) Broker's brokers provide direct bid and ask pricing to investors at the retail level. (B) Broker's brokers provide anonymity to municipal securities dealers who do not wish to be identified in bond trades. (C) Broker's brokers normally only charge a fee if a bond trade actually takes place. (D) Broker's brokers act in an agency capacity with regards to the transactions that they facilitate.

(A) Broker's brokers provide direct bid and ask pricing to investors at the retail level.

A customer wants to sell a block of municipal bonds that they own. MSRB rules require the dealer, when acting as agent, to do which of the following? (A) The dealer must consult with a minimum of three dealers to obtain bids. (B) A reasonable effort by the dealer must be made to obtain fair pricing on the bonds. (C) The dealer must consult with a minimum of three broker's brokers to obtain bids. (D) The dealer must seek competitive bids for the bonds.

(B) A reasonable effort by the dealer must be made to obtain fair pricing on the bonds.

Dividend Information Company Period Amt. Payable Date Record Date Charles River Inc. Q .68 4/15/09 3/26/09 Thursday Charles River Inc. $8 Pref. Q 1.40 4/1/09 3/16/09 Monday Survey Says Inc. Q 62.50 3/1/09 2/18/09 Tuesday An investor purchased 100 Charles River Inc. common regular way on March 25. Which of the following is correct? (A) He is not entitled to the dividend. (B) He is entitled to the dividend. (C) He is entitled to the dividend if the dividend claim is made by the settlement date. (D) He may or may not be entitled to the dividend since it depends on when the seller delivers the certificate.

(A) He is not entitled to the dividend.

An investor holds an equity REIT. Of the following, which TWO would have the most detrimental effect on the value of this investor's product? I. Builders have flooded the market with new structures. II. The prime rate has seen a notable increase. III. Property values have increased over 10%. IV. Retail and rental properties are almost entirely occupied. (A) I and II (B) I and III (C) II and III (D) II and IV

(A) I and II

Of the following, which TWO constitute a "spread"? I. Buy an ABC July 50 Call, Sell an ABC July 60 Call II. Buy an ABC June 50 Call, Sell an ABC July 50 Call III. Buy an ABC Jan 50 Call, Buy an ABC Jan 60 Call IV. Sell an ABC Apr 50 Call, Sell an ABC Jan 50 Call (A) I and II (B) I and IV (C) II and III (D) III and IV

(A) I and II

Which of the following would be illustrated when viewing a chart showing the business cycle? I. Downturns typically result in a trough before a recovery takes place. II. Upward trends typically result in a peak before a recession takes place. III. Inflation numbers are clearly demonstrated on a business cycle. IV. The yield curve can be easily determined from a chart showing the business cycle. (A) I and II (B) I and III (C) II and IV (D) III and IV

(A) I and II

A "head and shoulders" top formation means what when viewed from a technical analysis perspective? I. Taking bearish positions will benefit investors. II. Taking bullish positions will benefit investors. III. The previous upward trend is now reversing. IV. The previous downward trend is now reversing. (A) I and III (B) I and IV (C) II and III (D) II and IV

(A) I and III

Of the following statements, which TWO are TRUE of interest rate risk? I. Long term maturities and low coupon bonds are most susceptible to this risk. II. Short term maturities and high coupon bonds are most susceptible to this risk. III. Rising interest rates will cause outstanding bond prices to fall. IV. Lower interest rates will cause outstanding bond prices to fall. (A) I and III (B) I and IV (C) II and III (D) II and IV

(A) I and III

When a firm "position trades", it: I. makes a market in securities. II. trades for the firm's account. III. sells short in all transactions. IV. executes agency trades for customers. (A) I, II (B) II, III (C) I, II, IV (D) I, II, III, IV

(A) I, II

Which of the following are characteristics of a Hedge Fund? I. Low liquidity II. High risk III. Low initial investment IV. Heavily regulated due to the types of investments in the portfolio (A) I, II (B) I, II, III (C) IV (D) III, IV

(A) I, II

Specific regulations pertain to advertisements related to equity options. Which of the following is prohibited in advertising for equity options? (A) Information pertaining to specific equity options and performance (B) Company logos and slogans (C) The name of the exchange on which the options trade (D) The use of the name of the OCC (Options Clearing Corporation)

(A) Information pertaining to specific equity options and performance

The interest rate in a collateralized mortgage obligation: (A) Is paid at the coupon rate over the life of a bond (B) Varies depending on the federal funds rate (C) Is based on variable coupon rates until stated maturity (D) Varies depending on a differential from Treasury bond rates

(A) Is paid at the coupon rate over the life of a bond

How often is interest paid on a U.S. Treasury Receipt? (A) It is paid when the security comes due at maturity. (B) It is paid on an annual basis. (C) It is paid on a semi-annual basis. (D) It is paid on a quarterly basis or four times annually.

(A) It is paid when the security comes due at maturity.

All of the following comments would NOT apply to holders of preferred stock EXCEPT: (A) Preferred shareholders typically see lower levels of price volatility than common shareholders. (B) Dividends that are in arrears are always paid to preferred stockholders, even when the preferred stock does not have cumulative features. (C) Dividend payments on preferred stock are guaranteed. (D) In the event of the dissolution of a corporation, bondholders and preferred stockholders have equal claim over corporate assets.

(A) Preferred shareholders typically see lower levels of price volatility than common shareholders.

Susan Smith has a large portfolio of blue chip common stocks and is expecting the market value of her stocks to remain about the same or decline modestly in price and would like to improve the rate of return on her portfolio. Which of the following option positions would be best for her? (A) Short Calls (B) Long Calls (C) Short Puts (D) Long Puts

(A) Short Calls

What determines the tax liability to an individual who surrenders their holding of a variable annuity that is labeled as "non-qualified"? (A) Tax liability is determined by subtracting the amount which was invested in the annuity from the net proceeds upon surrender. (B) Tax liability is determined by the capital gains that an investor accumulates in the separate account. (C) Tax liability is determined by income which is accumulated in the separate account. (D) Tax liability is determined by the holding period of securities in the separate account.

(A) Tax liability is determined by subtracting the amount which was invested in the annuity from the net proceeds upon surrender.

A municipal bond's market price exceeds the par value of the bond. Which of the following best corresponds to this scenario? (A) The bond is selling at a premium. (B) The bond is selling at a discount. (C) The difference between the two prices is commonly referred to as the spread in the securities industry. (D) The difference between the two prices can be attributed to the fees levied by the municipal dealer selling the bonds.

(A) The bond is selling at a premium.

A financial summary of a company's revenues, expenses and profits or losses for a certain accounting period is called: (A) A balance sheet (B) An income statement (C) A statement of retained earnings (D) A statement of cash flow

(B) An income statement

At the beginning of the trading day, a customer calls in and wishes to buy 1,000 shares of ABC Incorporated at a limit price of $40.00 per share. The customer wishes to leave the order outstanding through the close of trading on that day. The following purchases took place in the customer's account over the trading day: - Buy 275 shares at $39.75 - Buy 200 shares at $39.95 - Buy 300 shares at $39.90 At the close of the day, the remaining order for 225 shares has not been filled. Which of the following is TRUE of this scenario? (A) The customer's remaining order for 225 shares is cancelled at the close of trading and the customer must accept the 775 shares that were purchased over the trading day. (B) Due to the fact that there was partial execution of the order, the customer is permitted to demand completion of the order as long as the average price of all shares purchased is less than $40 per share. (C) Because the order was only partially executed, this customer reserves the right to refuse the partial fill and cancel the entire order. (D) Because the order was only partially executed, the broker/dealer firm is permitted to fill the remaining order for 225 shares when trading commences the following business day.

(A) The customer's remaining order for 225 shares is cancelled at the close of trading and the customer must accept the 775 shares that were purchased over the trading day.

The state in which you operate has contracted out your firm to be the program manager for its 529 program. The 529 college savings plan is not gathering the attention that it used to, so the state has asked that your firm's marketing department produce an advertisement which includes the total return data for the plan. The advertisement is completed in late April, but is not scheduled to run until July. If the advertisement goes online during July as scheduled, with no modifications to include the most current total return data, your firm is responsible for providing a toll-free number where prospective investors viewing the advertisement can obtain return information for which of the following? (A) The most current data to the past month's end (B) The past six months of 529 operation (C) The past 2 years of 529 operation (D) The most current data through the past year's end

(A) The most current data to the past month's end

What is the obligation of a put writer upon exercise of the put? (A) The writer must purchase the underlying security at the strike price. (B) The writer must purchase the underlying security at the current market value. (C) The writer must sell the underlying security at the strike price. (D) The writer must sell the underlying security at the current market price.

(A) The writer must purchase the underlying security at the strike price.

Under the regulations of the security industry, can unlisted securities be sold short by an OTC trader? (A) Yes, there is no restriction on such transactions. (B) No, because they are not marginable. (C) No, because it is against Federal Regulations. (D) Yes, but only on an uptick or zero plus tick.

(A) Yes, there is no restriction on such transactions

An investor's portfolio has decreased in value from $250,000 to $200,000 in this tax year. No security positions have been sold. The investor's write off against ordinary income on this year's tax return is equal to which of the following? (A) Zero (B) $3,000 (C) $25,000 (D) $50,000

(A) Zero

A customer enters an order to Buy 100 shares of ABC @49 stop. This order will be activated by which of the following? (A) a trade executed at $49 or higher (B) a trade executed at $49 or lower (C) Buy stop orders are not allowed (D) Buy stop orders are executed as soon as they reach the floor of the exchange

(A) a trade executed at $49 or higher

Which of the following would NOT be the responsibility of the FINRA? (A) establish minimum or maximum sentences for criminal violations (B) set conduct rules for broker/dealers (C) handle arbitration disputes (D) set uniform practice rules for securities transactions

(A) establish minimum or maximum sentences for criminal violations

All of the following transactions are exempt from the SEC penny stock disclosure rules EXCEPT those: (A) recommended by a registered broker-dealer (B) where the customer is an institutional accredited investor (C) which meet the private placement requirements of Regulation D (D) in which the customer owns 5% or more of the stock

(A) recommended by a registered broker-dealer

Mr. Smith has been bearish about ABC common stock and decides to sell short 1,000 shares @40 in his margin account. The stock later declines to $35. What is Mr. Smith's equity in his margin account after the decline in the market value of the stock? (A) $20,000 (B) $25,000 (C) $35,000 (D) $40,000

(B) $25,000

A customer sells 1 WL September 100 call for 10 and buys 100 shares of WL stock at 105. The customer will break even at what market price? (A) 90 (B) 95 (C) 110 (D) 115

(B) * 95

One of your clients purchases a callable corporate bond. The bond is callable at 106.55. What will the customer receive if the corporation calls the bonds and the customer tenders them back to the corporation? (A) The customer will receive $1,065.50 for the bond and accrued interest up to the call date will be subtracted from that figure. (B) The customer will receive $1,065.50 for the bond and accrued interest up to the call date will be added to that figure. (C) The customer will receive par value for the bond of $1,000 and will receive $65.50 in accrued interest. (D) The customer will receive par value for the bond of $1,000 and will receive $106.55 in accrued interest.

(B) * The customer will receive $1,065.50 for the bond and accrued interest up to the call date will be added to that figure.

An individual owns 1,000 shares of common stock in a foreign corporation. The corporation declares a dividend on their common of $0.25 per share. Prior to paying the individual the dividend, the corporation withheld 15%. For taxation purposes under U.S. law, what should this individual report? (A) The individual does not have to report any dividend due to exemptions. (B) * The individual must report the whole dividend and may file for a foreign tax credit of $37.50. (C) The individual should report only the amount received after the 15% is withheld, or $212.50. (D) The individual should report the entire dividend of $250.

(B) * The individual must report the whole dividend and may file for a foreign tax credit of $37.50.

A signed "options agreement" must be obtained from a new options customer: (A) Before the first options order (B) Within 15 days after the account has been approved for options transactions (C) No later than the settlement date of the first options trade in the account (D) Not later than 30 days after the settlement date of the first options trade in the account

(B) * Within 15 days after the account has been approved for options transactions

An investor with a margin account is looking to calculate the amount of margin in their account and figure the loan value in the account. Which of the following does NOT have loan value for purposes of this calculation? (A) Shares of a closed-end investment company (B) Equity option contracts, including both put contracts and call contracts (C) Equity securities traded on NASDAQ (D) Municipal bonds issued by state and local governments

(B) Equity option contracts, including both put contracts and call contracts

Mr. Jones regularly buys and sells corporate debt securities in his account. In the process of buying 1 XYZ Corp bond at a market price of 98, he pays accrued interest of $250. Later on, he sells the 1 XYZ Corp bond at a market price of 99 and receives $100 in accrued interest. In reporting these transactions for taxation purposes, Mr. Jones has which of the following? (A) He will have a capital loss of $140. (B) He will have a capital gain of $10. (C) He will have a capital loss of $150. (D) He will have a capital gain of $1.

(B) He will have a capital gain of $10.

Which of the following would not be considered to be an investment advisor according to the Investment Advisors Act of 1940? I. A registered representative who gives advice, but does not charge clients for the advice. II. An accountant who gives specific advice and charges clients for the advice. III. An accountant who gives advice which is incidental to the practice of their profession and does not charge. IV. A registered representative who handles managed accounts and charges clients a management fee on the account. (A) I and II (B) I and III (C) II and III (D) II and IV

(B) I and III

A client opens an account and does NOT sign any form of discretionary authority for the registered representative handling the account. If no authorization is given by the customer in writing, which of the following would be CORRECT in terms of discretion? I. The RR can exercise discretion as to the price of the security on a particular trading day. II. The RR can exercise discretion as to the type of security purchased on a particular trading day. III. The RR can exercise discretion as to the number of shares purchased on a particular trading day. IV. The RR can exercise discretion as to the time at which the order is entered on a particular trading day. (A) I and III (B) I and IV (C) II and III (D) III and IV

(B) I and IV

When establishing a discretionary account that will trade options which of the following would be true? I. Approval must be received from the Exchange on which the options trade II. Written authorization must be received from the customer opening the account III. Approval of the account must be obtained from a registered options principal (ROP) IV. Trades must be approved by the Branch Manager before they are entered (A) I & IV (B) II & III (C) II & IV (D) I & III

(B) II & III

In reference to Interest Rate or Yield Based options with U.S. Treasury Bonds as the underlying security, it may be said that buyers of: I. Puts expect interest rates to increase. II. Puts expect interest rates to decline. III. Calls expect interest rates to increase. IV. Calls expect interest rates to decline. (A) I and II (B) II and III (C) II and IV (D) III and IV

(B) II and III

One of your clients purchased 500 shares of ABC Fund, an equity mutual fund, several months ago, and now wants to purchase another 500 shares of the same fund. Which two of the following should be explained to the client? I. They should consider Closed-end funds for diversification. II. Sign Letter of Intent, allowing them to combine both purchases new purchase may qualify for a reduced sales load. III. That the fund also offers Rights of Accumulation and that if the investor continues to invest in the same fund, they will continue to qualify for more discounts on new purchases in the future. IV. That the portfolio of securities is fixed therefore the investor's return will also be fixed. (A) I and II (B) II and III (C) II and IV (D) I and III

(B) II and III

In an account where the customer has given a registered representative limited discretionary authority while they are away on vacation, it would be permissible for the RR to do which of the following? I. Transfer funds to outside parties. II. Enter a stop order to sell a particular security held in the customer's account. III. Move funds and securities from one account to other accounts. IV. Use the customer's investment objectives to enter appropriate orders in the account. (A) I and III (B) II and IV (C) II, III, and IV (D) I, II, III, and IV

(B) II and IV

Pre-refunding an issue of municipal securities which has call protection usually results in an: (A) Automatic cancellation of the call protection. (B) Increase in the marketability and quality of the bonds. (C) Increase in the current yield of the bonds. (D) Immediate request by the issuer for all bondholders to tender their bonds

(B) Increase in the marketability and quality of the bonds.

Which of the following is true of cumulative voting? (A) It is advantageous to majority shareholders. (B) Minority shareholders have a better chance of electing a director of their choice. (C) Majority shareholders can always elect the entire board of directors. (D) A stockholder of 400 shares with 5 persons to be elected to the board could only vote 400 shares for each director of his choice.

(B) Minority shareholders have a better chance of electing a director of their choice

A investor at your firm has approximately $750,000 invested mostly in equity securities. The investor executed over 200 trades last year. The average trade cost to the investor was $200 including commissions and fees. The client comes into the branch with their statements and wants to talk to the RR about lowering costs, which totaled over $40,000 last year. An account is available from the firm which charges 1.5% of all equities. With these factors in mind, what is BEST for the client? (A) The client would be best served by a separately managed account. (B) The client would be best served by a fee-based account that is not managed. (C) The client would be best served by the traditional style of account which charges commissions per transaction. (D) The client would be best served by a traditional style of account which charges commissions per equity transaction, but which offers reduced fees for additional services.

(B) The client would be best served by a fee-based account that is not managed.

Of the following, which could cause a reduction of quality or rating in relation to a specific issuance of municipal bonds? (A) The issuer makes regular scheduled contributions to a sinking fund. (B) The issuer offers another issue of bonds that hold equal claim to the original issue. (C) The issuer deposits funds into an escrow account which will be used to pay the bonds off at maturity. (D) The issuer performs a pre-refunding of part of the original bond issue.

(B) The issuer offers another issue of bonds that hold equal claim to the original issue.

Which of the following consists of a list of new offerings expected to come to market over the 30 calendar days following? (A) The list described would be the Bond Buyer Index. (B) The list described would be the Visible Supply. (C) The list described would be the Bloomberg Offering List. (D) The list described would be the Commodities and Futures Market List.

(B) The list described would be the Visible Supply

Your branch does a somewhat sizeable business in deferred variable annuities. During the course of day-to-day business, a principal at an OSJ (Office of Supervisory Jurisdiction) gets an application that comes across her desk for a 1035 exchange. Assuming the paperwork is complete and there are no discrepancies, review by this principal for approval or rejection must take place with regards to the exchange recommendation within what time period? (A) The principal must review the application within 10 business days. (B) The principal must review the application within 7 business days. (C) The principal must review the application within 5 business days. (D) The principal must review the application promptly.

(B) The principal must review the application within 7 business days.

One of your customers has a sizeable amount of holdings in a particular mutual fund. The customer is looking over a statement and calls into the office to discuss the fact that there has been a decrease in the fund's NAV (net asset value) on a per share basis. Which of the following would cause this type of decrease? (A) This type of decrease would be caused by others who own fund shares and their choice to redeem those shares back to the fund. (B) This type of decrease would be caused by a cash dividend payment made to those holding mutual fund shares. (C) This type of decrease would be caused by others buying fund shares and new shares being issued. (D) This type of decrease would be caused by the mutual fund selling securities held by the fund's portfolio, whether the securities appreciated or depreciated in value.

(B) This type of decrease would be caused by a cash dividend payment made to those holding mutual fund shares.

One of your clients is looking into investing in direct participation programs. He is examining several different types of programs. One of the programs that he is evaluating promises a relatively high tax write-off in the immediate future with low costs for land and rights associated with the land and minerals. Of the following, which BEST describes programs with these features? (A) This would be a developmental drilling program. (B) This would be an exploratory drilling program. (C) This would be a balanced drilling program. (D) This would be an oil and gas income program.

(B) This would be an exploratory drilling program.

Hedge Funds are normally established in the form of: (A) an investment company (B) a limited partnership (C) a variable annuity (D) a CMO

(B) a limited partnership

An investor is short 3 ABC July 50 call options and 3 ABC July 45 put options. Currently, ABC stock is selling for $47.50 and the investor has a small profit. The investor should consider closing the options position if ABC stock is likely to: (A) remain between $45 and $50 (B) be the target of a takeover bid (C) continue to exhibit low volatility (D) have unchanged earnings per share

(B) be the target of a takeover bid

The safekeeping of securities held by a mutual fund is done by the: (A) transfer agent (B) custodian (C) underwriter (D) syndicate manager

(B) custodian

Self-regulatory organizations (SROs) are required by federal securities laws to do all of the following EXCEPT: (A) established rules of fair practice (B) determine fair market prices (C) establish arbitration procedures (D) establish standards of professional qualification

(B) determine fair market prices

When analyzing a municipal bond the yield to call computation is used when (A) the bond is not redeemable (B) the bond is callable and trading at a premium (C) the bond is callable and trading at a discount (D) the bond is held to maturity

(B) the bond is callable and trading at a premium

The regular way settlement date on U.S. Treasury Bonds is: (A) the same day as the trade date (B) the next business day after the trade date (C) three business days after trade date (D) five business days after the trade date

(B) the next business day after the trade date

A customer's margin account is as follows: $20,000 Mkt SMA $1,000 16,000 DB $ 4,000 EQ The NYSE minimum maintenance call would be met with a deposit of which of the following? (A) $10,000 (B) $6,000 (C) $1,000 (D) 0

(C) $1,000

You are an RR and one of your fellow RRs decides to open an online trading account with an unaffiliated firm because the transaction fees are lower with their online services. Of the following, which are correct regarding NYSE regulations on this type of situation? I. Only margin accounts must abide by NYSE regulations. II. Regardless of whether it is a margin account or a cash account, the NYSE regulations must be followed. III. The employing firm must receive duplicate statements and confirmations. IV. The employing firm must consent to the opening of the account and must do so in writing. (A) I only (B) I, II, and III only (C) II, III, and IV only (D) I, II, III, and IV

(C) * II, III, and IV only

Variable annuities are investment vehicles that should be used for clients with certain investment objectives in mind. Of the following, which client would BENEFIT MOST from investment in a variable annuity? (A) A female client in her early 60s who currently falls in the 33% tax bracket and wishes to have a portion of returns on her portfolio guaranteed. (B) A male client in his early 50s who currently falls in the 10% tax bracket and wishes to take on very low levels of risk with short-term investing. (C) A male client in his mid 40s who currently falls in the highest tax bracket and who is looking for an alternative investment vehicle since he has currently maximized contributions to his company-sponsored retirement account. (D) A female client in her mid 30s who currently falls in the 28% tax bracket and wishes to have liquidity and access to funds as she seeks to purchase a home.

(C) A male client in his mid 40s who currently falls in the highest tax bracket and who is looking for an alternative investment vehicle since he has currently maximized contributions to his company-sponsored retirement account.

An investor would have the greatest upside risk potential with which of the following option positions? (A) A call spread performed at a credit (B) A put spread performed at a debit (C) A short straddle (D) A long straddle

(C) A short straddle

An investor has $120,000 in savings. The man goes to four different companies that offer mutual funds and invests $30,000 in the moderate yield funds of each. When looking at this strategy from an asset allocation perspective, which best describes this scenario? (A) Because all of the investor's funds are tied up in mutual funds, the investor has not diversified. (B) Because the investor went to four different companies to invest, the investor has diversified. (C) Because all of the funds will likely contain the same asset classes, the investor has not diversified. (D) Because the investor has shares in four different funds, the investor has diversified.

(C) Because all of the funds will likely contain the same asset classes, the investor has not diversified

Liquidity is a term indicating how easy assets may be: (A) Transferred to another owner. (B) Purchased on credit. (C) Converted into cash. (D) Used as collateral.

(C) Converted into cash.

Which of the following would NOT be true about a Variable Annuity? (A) The investment objectives may be changed after a vote by those holding the variable annuity. (B) Management of such portfolios is performed by a professional in the industry. (C) Holders are insured against all loss pertaining to their investment. (D) Mutual fund shares can be a part of the investments of the annuity.

(C) Holders are insured against all loss pertaining to their investment.

A client at your firm has given their registered representative limited trading authority over her account. If this is the case, which of the following are ACCURATE of this scenario? I. Discretionary stop orders may not be entered by the RR for this account. II. No checks or withdrawals may be issued from the account in the RR's name. III. The account and orders made in the account are subject to the review of a principal of the firm on a frequent basis. IV. Account statements and all confirmations are directed toward the RR for the account, rather than the customer. (A) I and II (B) I and III (C) II and III (D) II and IV

(C) II and III

During a discussion with a client, the client indicates that he feels that interest rates are going to be going down soon. If the client wishes to invest according to this assumption, which of the following recommendations should the registered representative give to the client? I. The client should begin a pattern of swapping his existing bonds for bonds selling at a premium. II. The client should attempt to increase the overall level of call protection on his bond portfolio. III. The client should attempt to extend the overall maturity on his bond portfolio. IV. The client should attempt to shorten the overall maturity on his bond portfolio. (A) I and III (B) I and IV (C) II and III (D) II and IV

(C) II and III

A control relationship exists between a broker/dealer firm and a municipality. In a discretionary account for a customer at this broker/dealer, which of the following is ACCURATE when a purchase is taking place? (A) If the transaction that is taking place is occurring in the secondary market, there is no need for disclosure in relation to the control relationship. (B) If the broker/dealer is acting in a principal capacity, this is the only scenario where the broker/dealer needs to make a disclosure in relation to the control relationship. (C) In the event that such a purchase is to take place in a customer's account, the broker/dealer firm must obtain specific authorization from the customer in order to execute the trade. (D) Under no circumstances can a broker/dealer with such a control relationship execute any trades in the customer's discretionary account.

(C) In the event that such a purchase is to take place in a customer's account, the broker/dealer firm must obtain specific authorization from the customer in order to execute the trade.

Which of the following statements regarding "inflation" and "interest rates" would be true? (A) Interest rates will remain unchanged during periods of inflation. (B) Interest rates will decline in periods of inflation. (C) Interest rates will increase in periods of inflation. (D) Interest rates and inflation have no correlation.

(C) Interest rates will increase in periods of inflation

LMN Incorporated is listed on the American Stock Exchange. Jane sits on the board of directors for LMN and wishes to buy some LMN stock. She executes a trade on the floor of the AMEX and purchases the shares. How do the provisions of Rule 144 apply to this scenario? (A) Jane is only permitted to sell the stock if she holds it for a minimum of 3 years. (B) Jane must keep the stock indefinitely and is not permitted to sell. (C) Jane is permitted to sell the stock purchased on AMEX immediately, but certain restrictions may apply. (D) Jane is permitted to sell the stock purchased on AMEX immediately and there are no restrictions with regards to the sale.

(C) Jane is permitted to sell the stock purchased on AMEX immediately, but certain restrictions may apply

When determining position limits an investor that is short XYZ calls would be affected by which of the following? (A) Short XYZ puts (B) Long XYZ calls (C) Long XYZ puts (D) Long XYZ shares

(C) Long XYZ puts

Principal payments of a collateralized mortgage obligation (CMO) are: (A) Paid in full as a lump sum at maturity (B) Paid in equal installments over the life of the CMO (C) Paid in varying amounts over the life of the CMO (D) Paid in full when the CMO is refunded.

(C) Paid in varying amounts over the life of the CMO

A convertible preferred stock is trading at $140 per share with a conversion price of $25, while the common stock is at $35 per share. This situation would be best described as: (A) Premium (B) Discount (C) Parity (D) Disparity

(C) Parity

A wealthy client has a trip planned where they intend to do business in China for a few weeks and then follow the business side of the trip with a vacation in the region. The client asks the registered representative to have their mail held by the broker/dealer firm and submits the request in writing. Which of the following is TRUE of this situation? (A) Regulations do not permit the broker/dealer to hold the clients mail under any circumstances. (B) Regulations permit the broker/dealer to hold the client's mail for up to two months. (C) Regulations permit the broker/dealer to hold the client's mail for up to three months. (D) Regulations permit the broker/dealer to hold the client's mail for up to one year.

(C) Regulations permit the broker/dealer to hold the client's mail for up to three months.

Repurchase Agreements are characterized by all of the following EXCEPT: (A) Repo rates are negotiated by the parties involved. (B) Banks and thrifts use Repos to raise temporary capital funds (C) The FRB uses Repos as instruments of monetary policy (D) Their fixed maturities make them unsuitable for use by corporations

(D) Their fixed maturities make them unsuitable for use by corporations

Which of the following would NOT be a primary concern for a municipal bond trader who focuses on general obligation bonds? (A) Risks associated with overall market fluctuations and bond market fluctuations (B) Risks associated with the call of bonds held in a portfolio (C) Risks associated with the flow of funds dictated by the bond's indenture (D) Risks associated with the credit rating and dependability of the issuer

(C) Risks associated with the flow of funds dictated by the bond's indenture

Which of the following is allowed by SEC Rule 144A? (A) Rule 144A allows foreign corporations to buy registered securities. (B) Rule 144A allows qualified institutional buyers to buy registered securities. (C) Rule 144A allows qualified institutional buyers to buy unregistered securities. (D) Rule 144A allows affiliates of the issuer to purchase registered securities.

(C) Rule 144A allows qualified institutional buyers to buy unregistered securities.

One of your clients has large holdings of a few blue-chip corporations. The investor wishes to hold onto the stock for multiple reasons, but she is uncertain about price fluctuations in the short-term future. How can she use options to provide the MOST downside protection? (A) She can purchase call options on the securities in her portfolio. (B) She can sell call options on the securities in her portfolio. (C) She can purchase put options on the securities in her portfolio. (D) She can sell put options on the securities in her portfolio.

(C) She can purchase put options on the securities in her portfolio.

What happens to the NAV per share (net asset value) of a mutual fund on an ex-dividend date? (A) The dividend does not affect the NAV per share. (B) The NAV per share will increase in direct relation to the amount paid in the dividend. (C) The NAV per share will decrease in direct relation to the amount paid in the dividend. (D) The NAV per share will fluctuate depending on supply and demand.

(C) The NAV per share will decrease in direct relation to the amount paid in the dividend.

A large corporation has a substantial amount of convertible bonds outstanding. The bonds are callable at a slight premium over par ($102) and the conversion price of the bonds is $25. Common stock for the corporation is currently trading at $28 per share. The corporation wishes to reduce the amount of debt on its books and has decided to force a conversion of the bonds. How does the corporation go about doing so in this scenario? (A) The corporation should go into default on interest payments for the bonds. (B) The corporation should perform a primary offering of enough common stock to cover all conversions, which would force the conversion. (C) The corporation should simply call in the outstanding bonds. (D) The corporation should raise the conversion price from $25 to $50.

(C) The corporation should simply call in the outstanding bonds.

One of your clients owns an individual account and does not want any outside influence over the account. The client would like to ensure that probate is avoided in the event that they unexpectedly pass away and has named an immediate family member as the beneficiary of their account. This customer's needs will be best met by which of the following? (A) The customer should set up a Joint Tenants with Rights of Survivorship Account. (B) The customer should set up a Uniform Gifts to Minors Account. (C) The customer should set up a Transfer on Death agreement for securities in the account. (D) The customer should set up a Tenants in Common Joint Account.

(C) The customer should set up a Transfer on Death agreement for securities in the account

If a client of yours places an order to buy 100 shares of BCD at a market price of $41 per share and also places an order to sell 1 BCD June 35 call for 7, at what market price will the customer break even? (A) The customer will break even at a market price of $48 per share. (B) The customer will break even at a market price of $42 per share. (C) The customer will break even at a market price of $34 per share. (D) The customer will break even at a market price of $28 per share.

(C) The customer will break even at a market price of $34 per share.

All of the following could be applied to fixed UITs EXCEPT: (A) The term "diversified" (B) The fact that they are registered (C) The fact that they are managed (D) The fact that they are regulated

(C) The fact that they are managed

Which would be a counter-cyclical industry? (A) The computer software industry (B) The pharmaceutical industry (C) The gold mining industry (D) The automobile industry

(C) The gold mining industry

Which of the following investment strategies is BEST suited for an investor who believes that the market will have large amounts of volatility in the coming months, but that despite the volatility, the market will remain stable or only go down slightly? (A) The investor should sell short the SPY ETF. (B) The investor should purchase the SPY ETF. (C) The investor should purchase call options on VIX. (D) The investor should purchase an index fund for the NYSE.

(C) The investor should purchase call options on VIX.

You are evaluating a number of preferred stocks with varying features. One of the preferred stocks that you are evaluating shows a pattern of increased market value in relation to increases in the market value of the common stock of the company. Of the following preferred stock features, which is the MOST LIKELY cause of this pattern? (A) The preferred stock likely carries a participating feature. (B) The preferred stock likely carries a cumulative feature. (C) The preferred stock likely carries a conversion feature. (D) The preferred stock likely carries a call feature.

(C) The preferred stock likely carries a conversion feature.

One of your clients is somewhat new to options trading with multiple positions. They understand how options work, but sometimes ask questions in relation to specific positions. They put on the following options position: - Purchase 1 MNO July 50 put for 4 - Sell 1 MNO July 60 put for 11. The client calls in and asks their RR to explain a scenario where this position could lose money. Which of the following describes a scenario where the client will lose money on this position? (A) The put options both expire and no exercise is made on either contract. (B) The spread between the two contracts narrows and becomes less than 7 points. (C) The spread between the two contracts widens and becomes greater than 7 points. (D) The customer sells the MNO July 50 put for 6 and buys the MNO July 60 put for 10.

(C) The spread between the two contracts widens and becomes greater than 7 points.

Specialist/DMM on the NYSE can perform all of the following functions:

-Act as market maker -Act as broker's broker -Handle odd lot transactions **CANNOT ACT AS AN UNDERWRITER**

U.S. Government Bond auctions:

-Bids are accepted on a yield basis -Non-competitive bids are always filled

Gross Domestic Product (GDP) differs from the Gross National Product (GNP) in that the GDP measures: (A) The value of goods and services produced inside and outside the U.S. by Americans. (B) The value of goods and services imported into the U.S. (C) The value of goods and services produced within the boundaries of the United States. (D) The value of goods and services produced exported by the United States.

(C) The value of goods and services produced within the boundaries of the United States.

A municipal bond is purchased at a discount. Over the course of the life of the bond, the book value is increased while the customer holds the security. This is performed using which of the following accounting methods? (A) This is referred to as marking up the security. (B) This occurs when we see the security appreciate in value. (C) This is referred to as accretion of the security. (D) This occurs when we amortize of the security.

(C) This is referred to as accretion of the security.

The "third market" refers to: (A) The original issuance market. (B) The exchanges & over the counter markets. (C) Trading exchange listed securities over the counter. (D) Trading large blocks of securities between institutional investor.

(C) Trading exchange listed securities over the counter.

Under which of the following circumstances is it UNNECESSARY for a municipal securities dealer to follow special procedures when opening a municipal securities account? (A) When the client is married to an individual engaged in trading municipal securities with another municipal securities dealer. (B) When the account is for a minor who is the child of an individual who oversees municipal securities trading at another municipal securities dealer. (C) When the client holds stock issued by another municipal securities dealer. (D) When the client files paperwork and performs other clerical services while being employed by another municipal securities dealer.

(C) When the client holds stock issued by another municipal securities dealer.

A single 28 year old professional has income of $350,000 per year and a net worth that exceeds $1,000,000. Her investment objective is capital appreciation. Which of the following would be the least attractive investment choice for this investor? (A) growth funds (B) small cap equity stocks (C) high yield preferred stocks (D) equities in emerging markets

(C) high yield preferred stocks

If ABC 10% bond closed at 101 3/8 down 3/4 for the day and would provide a current yield of 9.87%. The current yield if the bond had been purchased on the previous trading day would be: (A) 9.87% (B) higher than 9.87% (C) lower than 9.87% (D) 10%

(C) lower than 9.87%

The compensation on municipal bonds sold as part of a new issue is called a "takedown" when the bonds are being bought by: (A) institutional investors (B) bond dealers for their investors (C) members of the underwriting syndicate to be sold to the public (D) non-members of the underwriting syndicate

(C) members of the underwriting syndicate to be sold to the public

One of your friends is excited to inform you that they have decided to open up an IRA account. They are doing so in late April and starting the account with a deposit of $1,750. Consideration for this deposit to the IRA account MUST be for (A) only the previous year. (B) the past year or the present year, depending upon your friend's wishes. (C) only the current year. (D) the previous year, given that your friend has filed the proper extensions with the IRS to do so.

(C) only the current year.

One of your clients just purchased 5 corporate bonds at a discount. The bonds had $150 in accrued interest and were priced at 97. The customer calls and wants to know their cost basis on the total purchase. What is it? (A) $820 (B) $940 (C) $4,700 (D) $4,850

(D) $4,850

After using reasonable diligence to develop a customer profile, a broker recommends a suitable investment to a customer. The FINRA suitability rule requires all of the following of this recommendation EXCEPT that it: (A) Be in the best interest of the customer (B) Apply to customers only, not potential customers (C) Be documented if it is complex and/or a potentially risky investments (D) Be in the best interest of the Broker-dealer.

(D) Be in the best interest of the Broker-dealer.

ABC has traded consistently between $40 and $50 for several months. An investor who wants to buy ABC if it breaks through its resistance level, should place which of the following orders? (A) Buy at the market (B) Buy limit at 49.50 (C) Buy limit at 50.13 (D) Buy stop at 50.13

(D) Buy stop at 50.13

All of the following securities may be purchased on margin in a customer's margin account EXCEPT? (A) Municipal bonds (B) U.S. Treasury Bonds (C) Corporate bonds (D) Call options on equities

(D) Call options on equities

Which of the following is probably a public utility? Co. A Co. B Co. C Co. D Earnings Per Share $1.50 $2.00 $3.50 $4.00 Dividends Per Share .30 .80 1.62 3.00 Dividend Payout Ratio 20% 40% 46% 75% Retained Earnings % 80% 60% 54% 25% (A) Company A (B) Company B (C) Company C (D) Company D

(D) Company D

Which of the following instruments/asset classes would generally be considered to be the least liquid? (A) Over the counter stock (B) Listed stock (C) General obligation municipal bonds (D) Direct participation program units

(D) Direct participation program units

The ratio at which one country's currency can be converted into another country's currency is known as the: (A) Federal funds rate (B) Deficit rate (C) European currency rate (D) Exchange rate

(D) Exchange rate

Factors that affect the amount of an underwriting spread include which of the following? I. The size of the issue II. The type of security involved III. The business history of the issuer (A) I only (B) II only (C) III only (D) I, II and III

(D) I, II and III

You work at ABC Broker/Dealer. A corporation approaches you and wants to open a margin account with your firm. Which of the following documents would be required by your new accounts department to open such an account? I. Corporate resolution II. Customer's margin agreement III. Loan Consent Agreement IV. Certified copy of corporate by-laws (A) I and IV (B) II and III (C) I, II and III (D) I, II, III and IV

(D) I, II, III and IV

Which of the following positions would cover a short call in a margin account? I. Convertible Bond II. Long the Underlying Stock III. Long a call with an equal or lower exercise price IV. Escrow Receipt (A) I, II, and IV (B) II and IV (C) II, III, and IV (D) I, II, III, and IV

(D) I, II, III, and IV

Which of the following statements are true concerning Alpha and Beta? I. Market risk associated with a security is measured by Beta. II. Non-market risk associated with a security is measured by Alpha. III. A security with Beta of 1.4 is expected to fluctuate more than the general market IV. A security with an Alpha of 1.3 is projected to rise in price by 30% regardless of the performance of the market as a whole, based on favorable factors relating to the individual security. (A) I and III (B) II and IV (C) I, III, and IV (D) I, II, III, and IV

(D) I, II, III, and IV

Which of the following would be expected to increase during an expansionary period in a business cycle? I. Profits II. Production III. Employment IV. Demand for goods (A) I and III only (B) II and IV only (C) III and IV only (D) I, II, III, and IV

(D) I, II, III, and IV

Which of the following procedures, specified in the "order protection rules" of Reg SHO, must be followed by trading centers? I. Centers are permitted to have locked markets or crossed markets. II. The execution of trades must take place at a price that is at or better than the best bids and offers found displayed nationally. III. Different trading centers are allowed to develop and establish their own discretionary fees. IV. The access to quotes must be immediate and automatic. (A) I and II (B) I and III (C) II and III (D) II and IV

(D) II and IV

Which two of the following are generally true when using the price-earnings ratio? I. It is used to compare a particular stock to other stock yields. II. It is used to compare a particular stock to other stocks, frequently within the same industry. III. The higher the P/E, the greater the value is to investors. IV. The lower the P/E, the greater the value is to investors. (A) I and III (B) I and IV (C) II and III (D) II and IV

(D) II and IV

Which of the following are characteristics of T-Bills? I. T-Bills normally will trade at a premium to par. II. For taxation purposes, all proceeds received upon maturity that are greater than the price paid are considered capital gains. III. Several maturities are available for T-Bills including 1 year down to 13 weeks. IV. Interest rates are not "stated" on T-Bills. (A) I and II (B) II and III (C) II and IV (D) III and IV

(D) III and IV

Of the following assets, which are most likely to be included in the investment portfolio of a real estate investment trusts: (A) Farmland and parks (B) Airports and stadiums (C) Single family housing and raw land (D) New apartment houses and shopping centers

(D) New apartment houses and shopping centers

When discussing rights offerings, which of the following statements is ACCURATE? (A) Rights offerings allow a third party to vote an investor's shares in the event that the shareholder cannot be reached. (B) Rights offerings cause an increase in market price to the debt securities of the company offering the rights. (C) Rights offerings are factored into the participation feature of preferred stock to ensure that preferred stockholders can achieve higher levels of profits. (D) Rights offerings allow existing shareholders to maintain their proportionate share of ownership when new shares are being issued.

(D) Rights offerings allow existing shareholders to maintain their proportionate share of ownership when new shares are being issued.

What happens when an RR FAILS to complete the regulatory element of continuing education? (A) The RR will only be cut off from receiving commissions. (B) The RR only must stop prospecting and soliciting new clients. (C) The RR will automatically be terminated from their position. (D) The RR must immediately stop performing all activities which require the RR to be registered.

(D) The RR must immediately stop performing all activities which require the RR to be registered.

Treasury STRIPS have all of the following characteristics EXCEPT: (A) They are sold at a discount (B) Their price is volatile (C) They lock in a rate for a predetermined period (D) The accrued interest is taxable at maturity

(D) The accrued interest is taxable at maturity

A customer performs the following transactions: The customer buys 100 shares of ABC at $55 per share. The customer also sells 1 ABC Jan 60 call for a premium of 4. The customer receives an exercise notice on the call when the price of the underlying security is $63 per share. What is the resulting gross profit or loss? (A) The customer will realize a gross loss of $100 (B) The customer will realize a gross loss of $400. (C) The customer will realize a gross profit of $1,200. (D) The customer will realize a gross profit of $900.

(D) The customer will realize a gross profit of $900.

If the manager of a portfolio is attempting to create a portfolio which mimics the S&P 500's performance, how would this strategy be BEST described? (A) The manager is attempting a form of technical analysis. (B) The manager is attempting a form of fundamental analysis. (C) The manager will most likely use an active management strategy on the portfolio. (D) The manager will most likely use a passive management strategy on the portfolio.

(D) The manager will most likely use a passive management strategy on the portfolio.

Which of the following statements pertaining to fiduciary accounts is FALSE? (A) Generally, "Prudent Investor" rules apply to such accounts. (B) Speculation is typically prohibited in such accounts. (C) Unless the documents which establish the account specifically permit it, purchasing securities on margin in such accounts is prohibited. (D) The profits of such an account may be shared with the fiduciary in charge of managing the account.

(D) The profits of such an account may be shared with the fiduciary in charge of managing the account.

An existing client comes into the office to discuss her portfolio. The client's risk tolerance is currently mid-level and she discusses her desire to add some diversification to her portfolio during the meeting. Her primary investment objective is total return with as much tax advantage as possible. Which of the following would be the MOST appropriate recommendation by the registered representative in this case, considering the information given? (A) The rep should advise the client to split the allocation of the portfolio 50/50 between corporate bond issues and municipal bond funds that are specific to the client's state of residence. (B) The rep should advise the client to split the allocation of the portfolio 50/50 between equity mutual funds and corporate bond mutual funds. (C) The rep should advise the client to use an asset mix of 50% corporate bond mutual funds, 25% cash and cash equivalents, 7.5% municipal bonds, 7.5% equity securities, 10% sector ETFs. (D) The rep should advise the client to use an asset mix of 50% municipal bond mutual funds, 45% equities and the remainder in cash and cash equivalents.

(D) The rep should advise the client to use an asset mix of 50% municipal bond mutual funds, 45% equities and the remainder in cash and cash equivalents.

Which of the following statements regarding warrants is not true? (A) They allow the owner to purchase additional shares of common stock. (B) They can be used to increase the attractiveness of a new issue. (C) They are generally long-term. (D) They are an obligation which must be issued by the corporation.

(D) They are an obligation which must be issued by the corporation.

When a customer opens a margin account at a broker/dealer, they are required to sign certain documents. Which of the following best describes the agreement between the customer and the broker/dealer related to the terms and conditions of financing involved in purchasing on margin? (A) This is covered in the customer's new account form. (B) This is covered in a consent by the client to the loan from the broker/dealer. (C) This is covered in the customer's hypothecation agreement. (D) This is covered in the agreement for a line of credit with the customer. .

(D) This is covered in the agreement for a line of credit with the customer.

Which of the following is the term used to describe the institutional arbitrage of various securities, including stocks and options, as well as indices? (A) This is known as the allocation of assets. (B) This is known as overall portfolio analysis. (C) This is known as systematic investment. (D) This is known as program trading.

(D) This is known as program trading.

The "wash sale rule" would apply to which of the following scenarios? (A) A customer buys a bond at 95 and one week later, sells the bond at 100. (B) A customer buys an identical bond to one sold at a loss 6 months earlier. (C) A customer buys 100 shares of ABC at $25 per share in late January and sells the shares at $20 per share in mid-February. (D) Two weeks prior to selling a stock for a loss, a customer buys a call option on the same underlying security.

(D) Two weeks prior to selling a stock for a loss, a customer buys a call option on the same underlying security.

One of your clients owns 100 shares of stock in MNO Corporation. MNO comes out and announces at 1:2 reverse split. MNO closed at $16 per share on the day prior to the split. Your client calls and wants to know what their position will look like after this reverse split. Which of the following is CORRECT? (A) You should inform the client that after the split, the value of the stock will be $16 per share and the client will own 200 shares. (B) You should inform the client that after the split, the value of the stock will be $16 per share and the client will own 100 shares. (C) You should inform the client that after the split, the value of the stock will be $16 per share and the client will own 50 shares. (D) You should inform the client that after the split, the value of the stock will be $32 per share and the client will own 50 shares.

(D) You should inform the client that after the split, the value of the stock will be $32 per share and the client will own 50 shares.

When a portfolio is established which consists of 90% equities and 10% money market instruments would be most appropriate for which of the following investors? (A) an investor wanting preservation of capital and current income (B) a married couple with three children in or nearing college (C) a couple in their retirement years (D) a single 28 year old with sizable income and assets

(D) a single 28 year old with sizable income and assets

The power authority of a municipality might issue tax-exempt commercial paper for all of the following reasons EXCEPT: (A) working capital (B) extraordinary expenses (C) partial payment of construction costs (D) advance refunding of outstanding bonds

(D) advance refunding of outstanding bonds

Which of the following projects would LEAST likely be financed by revenue bonds? (A) toll roads and bridges (B) airport expansion (C) sewer system expansions (D) construction of a new high school

(D) construction of a new high school

A securities analyst or broker-dealer issuing a research report on a certain company may not provide any of the following information to the company prior to distribution of the reports EXCEPT: (A) the stock price target (B) the rating and recommendation (C) the research summary (D) factual information for verification

(D) factual information for verification

The one type of oil and gas program that does not involve drilling is: (A) exploratory (B) developmental (C) balanced (D) income

(D) income

Mr. Jones purchases a B rated corporate bond maturing in 20 years would be least concerned with which of the following risks? (A) default risk (B) purchasing power risk (C) interest rate risk (D) liquidity risk

(D) liquidity risk

The following are duties of the transfer agent EXCEPT: (A) to issue new stock certificates (B) to cancel old stock certificates (C) to resolve problems arising from lost certificates (D) to prevent the over-issuing of shares of stock

(D) to prevent the over-issuing of shares of stock

U.S. government securities that are deposited with a trustee against which certificates are sold representing principal payments only on the securities are

(STRIPS) stripped bonds. subject to annual taxation on the per-year accreted amount.

An individual invested $100,000 in a real estate limited partnership. The individual's portion of the income and expenses are as follows. Gross revenue $ 220,000 Operating expenses $ 150,000 Interest on mortgage $ 35,000 Depreciation $ 50,000 The cash flow of the real estate program is:

+$35,000 Explanation: The income or loss on the project would be calculated as follows. Gross revenue $ 220,000 - Operating expenses - 150,000 - Interest on mortgage - 35,000 - Depreciation - 50,000 Loss ($15,000) After reducing the gross revenue by all of the expenses listed above you determine the net loss which is $15,000. From the loss you would add back any depreciation expense to determine the cash flow. Depreciation is a non-cash expense and it is added to cash flow. So the $15,000 loss plus the $50,000 in depreciation expense would equal $35,000. Cash flow = Net Income or Loss + Depreciation Expense.

An investor buys 100 shares of ABC stock at $60 in January of 2017. In March of 2018, the stock is worth $100 per share and the investor donates it to charity. The tax consequences are:

-A $10,000 charitable deduction -No tax is due on the appreciation The investor gets to deduct the fair market value of the donation. If it was held for 1 year or less, tax is due on the appreciation.

Municipal bonds:

-AMBCAC -FGIC -MBIA

Conflicts of interest in a limited partnership:

-Accepting loans from the partnership -Competing with the partnership -Selling assets to the partnership

A corporation has issued 8% AA rated sinking fund debentures at par. Three years later, similar issues are being offered in the primary market at 7%. What is TRUE about the outstanding 8% issue?

-The current yield will be lower than the nominal yield -The dollar price of the bond will be at a premium to par

What statement is TRUE under FINRA rules?

-The maximum annual 12b-1 fee is .75% =If a fund charges a 12b-1 fee, the maximum upfront sales charge is limited to 7.25%

What is true about the preliminary prospectus?

-The preliminary prospectus may be sent to a potential customer prior to that customer expressing an indication of interest -The preliminary prospectus does not constitute an offer to sell the issue

Which of the following are TRUE statements regarding short term negotiable CDs?

-Trading of negotiable CDs occur in the secondary market -Short term negotiable CDs are non-callable

If an issue on the NYSE stops trading, which of the following statements are TRUE regarding the option on that particular stock?

-Trading the option is halted -The option CAN BE exercised

Which of the following statements are TRUE regarding warrants?

-Warrants are typically issued with an exercise price that is higher than the stock's current market price -Warrants would be exercised when the stock's market price is above the warrant strike price

Variable annuity:

-contributions are not tax deductible -distributions are taxable

Accretion of a bond discount will:

-increase the bond's cost basis -increase annual reported interest income

Publicly traded fund shares are:

-negotiable -managed

Services offered by prime brokers include all of the following

-processing transactions -supplying clearing services -providing back office support

The amount of accretion to be reported for tax purposes on an original issue discount bond requires the use of all of the following:

-purchase price -redemption price -maturity **NOT MARKET VALUE**

Structured products are typically comprised of two components including:

A fixed-income note and a derivative product Explanation: A structured product is typically built around a fixed-income instrument and a derivative product. The note pays a specified rate of interest to the investor at defined intervals. The derivative component establishes the amount of payment at maturity.

What is the maximum allowable percentage that may be sold above the original size of the offering through a Green Shoe option?

15% Explanation: The overallotment provision of an underwriting agreement may contain a Green Shoe clause that allows the syndicate to increase the number of shares sold by 15% over the original number of shares in the offering.

A new municipal bond issue has a total par value of $80,000,000. A member of the underwriting syndicate has sold its entire commitment of $10,000,000. If the syndicate is organized as a divided (western) account and there is an unsold balance of $2,000,000, what is the member's remaining liability?

0 Explanation: In a divided account, a member is responsible for selling only its participation. The member's responsibility ends once the firm has sold its $10,000,000 commitment. In an undivided (eastern) account, a syndicate member retains liability for unsold bonds. Regardless of the amount of bonds sold, the member is still liable for an amount of bonds equal to its percentage participation.

An investor buys a 5% municipal bond at 102 1/2. The bond has a yield to maturity of 4 1/2%. If the investor holds the bond to maturity, he will have a loss for tax purposes of:

0 Explanation: The IRS requires that a premium paid for a municipal bond be amortized over the life of the bond. At maturity, the investor will have an adjusted cost (after amortization) of par ($1,000). Since this is the amount received at maturity, there is no loss for tax purposes.

A customer has a long margin account with a market value of $30,000 and a debit balance of $20,000. His short margin account has a $7,000 market value and a $10,000 credit balance. The FRB margin requirement is 50%. How much cash may the customer withdraw from the account?

0 Explanation: The long account is restricted because the equity of $10,000 is less than the initial FRB requirement ($30,000 market value times 50% FRB requirement equals $15,000 required equity). There is no excess equity in the short account since the equity of $3,000 ($10,000 credit balance minus $7,000 market value) is less than the FRB requirement of $3,500 (50% of $7,000 market value).

Seventy-five basis points are equal to which of the following?

0.75% $7.50

A foreign currency investor is long 40,000 Swiss francs at $0.81. If the investor buys 4 Jul 80 SF puts at 1.25 to hedge, the breakeven point is

0.8225 (0.81 + 0.125)

In a limited partnership, a general partner's minimum participation in profits and losses is:

1% Explanation: According to tax law, a general partner must have at least a 1% participation in profits and losses for a business to maintain limited partnership status.

100 basis points equals:

1% One basis point equals .01%, so 100 basis points equals 1%

Company AZX has announced a partial tender offer for Company BHQ. A shareholder of Company BHQ is long 1,000 shares of stock and long 2 BHQ puts. For the purpose of tendering shares, the stockholder may tender:

1,000 shares Explanation: An investor who holds stock in a company that is the subject of a tender offer may tender only stock that he holds long. Short tendering is not permitted. The long puts do not affect the client's net long position. If a shareholder had written call options positions against the long stock, the options positions will reduce his net long holdings in the stock.

A corporation has $125,000,000 of convertible bonds outstanding. The conversion price is $50. The corporation refunds $75,000,000 of the bonds for nonconvertible bonds. How many additional shares of common stock will be outstanding if the remaining bonds are converted?

1,000,000 shares Explanation: After the refunding, $50,000,000 of convertible bonds will remain outstanding. If these bonds are converted, there will be an additional 1,000,000 shares of common stock outstanding ($50,000,000 of bonds / the conversion price of $50 = 1,000,000 shares of common stock).

A customer sells short 1,000 shares of stock. A few weeks later the company declares a 5% stock dividend. When the customer covers the short sale, the customer will be required to deliver:

1,050 shares Explanation: When a customer sells short, the brokerage firm borrows stock to deliver it to the buyer. All cash and stock dividends declared are the responsibility of the customer who sold the stock short. In this example, the company declared a 5% stock dividend. Therefore, a customer who sold short 1,000 shares would be required to deliver 1,050 shares (1,000 shares x 5% = 50 additional shares) when covering the short sale.

A designated market maker places a GTC order in his book to buy 1,000 shares of XYZ at $30. XYZ declares a 50% stock dividend. The designated market maker should adjust the order when the stock sells ex-dividend to:

1,500 shares at $20 Explanation: The order must be adjusted to reflect the change in XYZ stock. The number of shares will be increased to reflect the dividend and will now be 1,500 shares (1,000 shares plus 50% of 1,000). The price of ABC will be adjusted downward to $20. The total value of the order before the dividend (1,000 shares at $30 = $30,000) must equal the value after the dividend (1,500 shares at $20 = $30,000).

A municipal revenue bond is secured by the revenues of a toll road system showing the following information. Annual Debt Service $3,000,000 Annual Gross Revenues $6,000,000 Annual Operating and Maintenance Expenses $2,000,000 Based on this information, the annual debt service coverage ratio is:

1.33 to 1 Explanation: The debt service coverage ratio for the municipal revenue bond is 1.33 to 1. The formula for the debt service coverage ratio is net revenues divided by the annual debt service. The information listed in the question is as follows. Annual Debt Service $3,000,000 Annual Gross Revenues $6,000,000 Annual Operating and Maintenance Expenses $2,000,000 Step 1: Calculate the net revenue for the municipal revenue bond. Annual Gross Revenues $6,000,000 - Annual O/M Expenses $2,000,000 Net Revenue $4,000,000 Step 2: Divide net revenue of $4,000,000 by the debt service of $3,000,000 to calculate the annual debt service coverage ratio which is 1.33 to 1.

Completed trades of NASDAQ stocks must be reported within:

10 seconds

ABC Corporation has net income of $10,000,000 and 5,000,000 common shares outstanding. ABC Corporation pays out $1,000,000 in dividends annually. ABC Corporation's dividend payout ratio is:

10% Explanation: The dividend payout ratio is the percentage of earnings per share that is being paid in the form of dividends. The EPS is $2.00 ($10,000,000 divided by 5,000,000 shares). ABC pays a $0.20 dividend per share ($1,000,0000 dividends divided by 5,000,000 shares). The $0.20 dividend divided by the $2.00 EPS equals a 10% dividend payout ratio.

The maximum underwriting compensation for selling limited partnerships in public offerings is:

10% Explanation: The maximum underwriting compensation for selling partnership units in a public offering is 10%. This is based on the gross dollar amount of the units sold. The 10% limit applies to all compensation, regardless of the source, if it is in connection with the offering.

The prospectus for a limited partnership states that the subscription price for each unit is $20,000. According to industry rules, the maximum allowable underwriting compensation for this public offering is:

10% of the gross proceeds of the offering Explanation: Industry rules allow a maximum total compensation of 10% of the gross proceeds of the offering in a limited partnership. This includes all items of compensation including trailing commissions

Public orders on a designated market maker's book show an inside market comprised of Broker A bidding for 100 shares of ABC Corporation at 42.25. Broker B is offering to sell 300 shares of ABC at 42.63. The size of the market would be:

100 by 300 Explanation: The amount of shares at the highest bid and the lowest offer on the designated market maker's book is called the size of the market. The size of the market is 100 shares bid for at 42.25 and 300 shares offered at 42.63 or 100 by 300.

How much margin must the purchaser of one RFQ Feb 60 call for a $3 premium deposit?

100% Explanation: Options may not be purchased on margin. According to Regulation T, the full purchase price (the premium) must be deposited.

What is the maximum amount a customer may withdraw from a Special Memorandum Account?

100% of the SMA Explanation: The full amount or 100% of the SMA may be withdrawn. The buying power is 2 times the SMA.

To meet a Fed call, a customer must deposit which of the following?

100% of the call in cash 200% of the call in marginable securities

An investor purchases a zero-coupon municipal bond maturing in 15 years that is callable in five years at 102. If the bond is called, the investor will receive:

102% of the compound accreted value Explanation: The investor would receive 102% of the compound accreted value since the security is a zero-coupon bond or original issue discount (OID) bond. The compound accreted value is equal to the original value of the bond plus the annual accretion as of the call date. If the bond was not an OID bond and was called, the investor would receive 102% of par or $1,020.

A customer sells 100 shares of GM short. GM pays a 5% stock dividend. When the customer covers the short position, the customer will need to deliver:

105 shares of GM Explanation: When a customer sells short, the brokerage firm borrows stock to deliver it to the buyer. All cash and stock dividends paid are the responsibility of the customer who sold the stock short. In this example, GM paid a 5% stock dividend. Therefore, a customer who sold 100 shares of GM short would need to deliver 105 shares (100 shares x 5% = 5 additional shares) to cover the short sale.

In early September, a customer buys 100 shares of QRS stock for $83 per share and simultaneously writes 1 QRS Mar 90 call for $4 per share. If the QRS Mar 90 call was exercised and the QRS stock delivered, what would be the customer's per-share profit?

11 he breakeven point is 79 (83 − 4), and the stock was sold (delivered) at 90 for an 11-point gain.

An individual in the 28% tax bracket can purchase an 8 1/2% municipal bond at par. What taxable yield would be required to equal the yield on the municipal bond?

11.8% Explanation: The taxable equivalent yield of a municipal bond equals the municipal yield divided by the complement of the tax bracket (100% minus the tax bracket). In this example, the municipal yield (8 1/2%) divided by the complement of the tax bracket (72% or 0.72) equals 11.8%.

In a new margin account, a customer buys 300 shares of ABC at $40 per share, 100 shares of the Ajax Mutual Fund at $24, and 10 PDQ Aug 30 calls at 4. The customer will receive a margin call for

12,400 6000 (50% of ABC)+ 2400 (full price of mf) + prem 4x10x10= 12400 The customer must pay 50% of the value of the stock, and 100% of the value of the mutual fund shares and the options because these securities are nonmarginable (NMS) and must be paid for in full (50% of $12,000 = $6,000). To calculate the total payment required, add $6,000 (stock) plus $2,400 (Ajax) plus $4,000 (PDQ calls) which equals $12,400.

The maximum amount of a customers securities that can be rehypothecated by a broker is:

140% of the debit balance

Which of the following exemption provisions of the Act of 1933 may not be used for an initial offering of securities?

144

When may a new issue become marginable?

30 days from the effective date Explanation: When approved for margin trading by the FRB, a new issue becomes marginable 30 days from the effective date of the offering.

A customer sells short 100 shares of DEF stock at $82 per share. The stock falls to $71, at which point the customer writes 1 DEF Sep 70 Put @4. The stock falls to $62 and the put is exercised. The customer has a gain per share of:

16 points. The customer sold the stock short at $82 per share (sale proceeds). Later, the customer sold a Sept 70 Put @ 4 on this stock. If the short put is exercised, the customer is obligated to buy the stock at $70 per share. Since the customer received $4 in premiums when the put was sold, the net cost to the customer is $66 per share for the stock (this is the cost basis in the stock for tax purposes). The stock that has been purchased is delivered to cover the short sale, closing the transaction. The customer's gain is: $82 sale proceeds - $66 cost basis = 16 points.

Contributions to a Coverdell Education Savings Account must cease when the beneficiary reaches the age of :

18

A corporation calls for the redemption of 1,000,000 shares of convertible preferred stock. The corporation announces that the convertible preferred will be redeemed at a price of $20 plus an accumulated dividend of 12 cents. Each share of preferred can be converted into 1/2 share of common. The preferred stock is selling at $19. There are 2,000,000 shares of common outstanding. Earnings for the common stock are $2.50 per share. The common stock is selling at 35.75. If all shares are converted, how many shares of common stock will be outstanding?

2,500,000 Explanation: If all of the preferred stock were converted into common stock, there will be an additional 500,000 shares of common stock outstanding, (1/2 of 1,000,000 = 500,000.) This, added to the 2,000,000 shares outstanding, equals 2,500,000 shares of common stock.

XYZ Corporation has 2,000,000 shares of common stock authorized. The company has issued 1,000,000 common shares of which 200,000 shares are treasury stock. The company has earnings of $2.00 per share. The XYZ Corporation has repurchased:

200,000 shares Explanation: XYZ corporation has repurchased 200,000 shares. This is known as treasury stock. Treasury stock is previously outstanding stock that has been repurchased by a corporation.

Your firm is a syndicate member and an underwriter of an initial public offering (IPO). How many days must the firm's research analyst wait before issuing a research report in this IPO?

25 days Explanation: If a firm is involved in an underwriting of an initial public offering and is the manager or comanager, it must maintain a quiet period of 40 days following an IPO or 10 days following a secondary offering. During this time, the firm may not issue research reports on its investment banking clients' stock. If the firm was a syndicate member or selling group member, the firm would need to wait 25 days.

A registered representative of a FINRA member firm has been found guilty of a trade practice violation. If desired, the Code of Procedure requires the individual to file an appeal within

25 days after receiving the decision.

Use the following quote to answer this question. ABC 25.13 + .25 B 25 A 25.25 Excluding any markups, what price will a customer pay to purchase the security?

25.25 Explanation: When purchasing stock, a customer will pay the ask (offer) price. A customer selling stock will receive the bid price.

A convertible bond is convertible at $25 and is selling in the market at 108. At what price should the common stock be selling for it to be at parity with the bond?

27 Explanation: To find the number of shares the bond is convertible into (i.e., the conversion ratio), divide the conversion price into the par value ($1,000 divided by 25 = 40 shares). To be at parity, the 40 shares must be equal to the value of the bond which is 108 or $1,080. Dividing 40 shares into $1,080 gives the parity price of the stock of $27.

Under FINRA rules to find the best available market in an OTC security, over-the-counter trades are obligated to obtain at least:

3 quotes

A shareholder is long 5,000 shares of XAM stock and has written 20 XAM call options. If XAM makes a tender offer to purchase shares of its stock, the shareholder may tender:

3,000 shares Explanation: An investor who holds stock in a company that is the subject of a tender offer may tender only stock that he holds long. If a shareholder has written call options positions against the long stock, the options positions will reduce his net long holdings in the stock. 20 call options are equal to 2,000 shares (20 calls x 100 shares). This customer is net long 3,000 shares (5,000 - 2,000).

The Jefferson County Water Works revenue bond is being underwritten by a syndicate led by ABC Securities, Inc. The bond has serial maturities going out up to 25 years with a balloon at 30. The coupons range from 3.2% to 4.1%, and all the bonds are offered at par. The terms of the syndicate agreement call for a total takedown of ¾ of a point with a selling concession of ½ point. A syndicate member who sells 500 of the bonds will earn

3,750 ¾ of a point ($7.50) per bond. The computation is 500 bonds sold × $7.50 per bond = $3,750 underwriting profit.

Ashton purchased 100 shares of XYZ common stock in January 2003, at a price of $25 per share. XYZ pays a quarterly dividend of $.25 per share. Today, XYZ closed at $30 per share. What is the dividend yield of XYZ common stock?

3.33% Explanation: The dividend yield for a stock is equal to the annualized dividend divided by the current market price. Since dividends are paid quarterly, the annual dividend is $1 per share ($.25 x 4). The annualized dividend of $1 divided by the current market price of $30 per share results in a dividend yield of 3.33%.

A new municipal bond issue is dated January 1 and pays interest each April 1 and Oct. 1. An investor purchased bonds from the issuer with a Thursday, January 31 settlement date. How many days of accrued interest does the investor owe?

30 Explanation: Accrued interest on a new municipal issue is calculated from the dated date up to, but not including the settlement date. Since the investor's settlement date was January 31, he owes from January 1 to January 30 (30 days).

On February 22, an investor sells ABC stock at $31 for a 3-point loss. On March 10, the investor purchases ABC stock at a price of $27. For tax purposes, the investor's cost basis for the stock purchased on March 10 is:

30 Explanation: When the wash sale rule is activated, the investor must add the loss to the new cost of the stock regardless of whether the stock is repurchased at a price that is higher or lower than the original cost. In this example, the investor's cost basis for tax purposes is found by adding the 3-point loss to the new cost of $27.

A customer entered a GTC sell stop order for GM at $35. GM was selling at $38 when the order was entered. GM sells ex-dividend by the amount of the dividend which is $1.60. The customer's order will appear on the designated market maker's book after the stock goes ex-dividend as:

33.40 Explanation: All GTC orders that are entered below the current market on the designated market maker's (DMM) book (buy limit, sell stop, and sell stop-limit orders) will be reduced by the amount of the dividend when the stock sells ex-dividend. The stock will always be reduced by an amount to cover the dividend entirely. The dividend is $1.60, so the order will be reduced 1.60, which will reduce the stop price on the order to 33.40.

If a customer buys 1 OEX Feb 350 call at 5, then sells 1 OEX Feb 335 call at 16 when the underlying index is at 344, the breakeven point is

346 premium paid (5) and the premium received (16), or 11 (335 + 11 = a breakeven point of 346).

Which of following investments would be the BEST recommendation for a customer with a medium or moderate risk tolerance? a. 10% large-cap equity funds, 5% international equity mutual funds, 55% bond funds, and 30% cash b. 15% large-cap equity funds, 5% small-cap equity funds, 10% international equity funds, 45% bond funds, and 25% cash c. 35% large-cap equity funds, 15% small-cap equity funds, 15% international equity funds, 30% bond funds, and 5% cash d. 50% large-cap equity funds, 20% small-cap equity funds, 20% international equity funds, 5% bond funds, and 5% cash

35% large-cap equity funds, 15% small-cap equity funds, 15% international equity funds, 30% bond funds, and 5% cash Explanation: An investment risk tolerance in which the customer is willing to accept some risk to her initial principal, with some volatility and a possible loss of the funds invested in exchange for higher returns, is best defined as moderate. Choices (a) and (b) offer too small of an allocation in equities, and choice (d) is too heavily weighted in equities.

According to suitability rules, 1035 exchanges should occur not more frequently than once every:

36 months Explanation: 1035 exchanges which occur within 36 months of previous exchanges may be considered churning and unsuitable. In order for an exchange of one variable annuity for another to be suitable, an adequate analysis of the investor's situation should include: Age Annual income Financial situation and needs Investment experience and objectives Intended use of the deferred variable annuity Investment time horizon (not intended for senior citizens) Existing assets Liquidity needs Liquid net worth Risk tolerance Tax status

An investor purchases 200 shares of STC at $35 and subsequently purchases 2 STC Jan 35 puts at 2. At what market price must STC trade for the investor to have a profit?

38 Explanation: If an investor is long stock and long a put, he will have a profit if the market price exceeds the cost of his stock plus the premium for the option. The stock must trade above 37 (35 cost + 2 premium).

An individual wishes to sell stock according to Rule 144 requirements. There are 3,500,000 shares outstanding. The individual decides to sell on January 30. The trading volume for the stock is as follows. Week ending January 1 20,000 shares Week ending January 8 48,000 shares Week ending January 15 30,000 shares Week ending January 22 36,000 shares Week ending January 29 40,000 shares How many shares may be sold?

38,500 Explanation: The individual may sell the greater of 1% of the outstanding shares or the average weekly trading volume for the preceding four weeks. The average is 38,500 [(40,000 + 36,000 + 30,000 + 48,000) divided by 4]. This is greater than 1% of the outstanding shares (1% of 3,500,000 equals 35,000).

The net borrowing cost to a municipal issuer of a Direct Pay Build America Bond (BAB) with a 7% interest rate is:

4.55% Explanation: The Treasury will reimburse 35% of the interest payment, which results in a net borrowing cost of 4.55% (7.00% x [100% - 35%]). These bonds may be suitable for taxable, fixed-income investors. BABs allow a municipality to issue a bond with a higher interest rate, but pay an equivalent tax-free rate.

Your firm is the managing underwriter of an initial public offering. How many days must the firm's research analyst wait before issuing a research report on this IPO?

40 days Explanation: If a firm is involved in an underwriting of an initial public offering and is the manager or comanager, it must maintain a quiet period of 40 days following an IPO or 10 days following a secondary offering. During this time, the firm may not issue research reports on its investment banking clients' stock. If the firm was a syndicate member or selling group member, the firm would need to wait 25 days.

Based on yesterday's closing price of $60 per share, Blech Sheet Metal, Inc., has a current P/E ratio of 12:1. If the current quarterly dividend payment is $0.50 per share, the dividend payout ratio is

40% ($60/ 12:1 = 5) ($0.50 x 4 = 2) (2/5)

XYZ Corporation has $20 million of convertible bonds outstanding. Each bond is convertible into 20 shares of common stock. If all the bonds were converted into common stock, how many additional shares of common stock would be outstanding?

400,000 Explanation: $20 million par value ($1,000) bonds, which equals 20,000 bonds ($20,000,000 divided by $1,000 equals 20,000), multiplied by the 20-to-1 conversion ratio results in 400,000 shares of additional common stock outstanding (20,000 bonds x 20 = 400,000).

Public orders on a designated market maker's book show an inside market comprised of Broker A bidding for 100 shares of ABC Corporation at 42.25. Broker B is offering to sell 300 shares of ABC at 42.63. If the designated market maker wanted to bid for his own account, what is the lowest price he may bid for the stock?

42.26 Explanation: The designated market maker must buy and sell for his own account (acting as a dealer) to make the market fair and orderly. He must be a buyer when there are no buyers and be a seller when there are no sellers. By so doing, he narrows the spread between sales. The DMM may not compete with an order on his book (a public order) at the same price and must always bid higher and offer lower than orders on his book. There is a bid of 42.25 on his book. He must bid at least one cent higher than 42.25, which is 42.26.

Public orders on a designated market maker's book show an inside market comprised of Broker A bidding for 100 shares of ABC Corporation at 42.25. Broker B is offering to sell 300 shares of ABC at 42.63. If the designated market maker wanted to offer stock, what is the highest price at which he may offer the stock?

42.62 Explanation: On the offer side, the DMM must be willing to sell for at least one cent lower than the offer on his book. There is an offer of 42.63 on his book. He must offer at least one cent lower, which is 42.62.

A customer sells short 400 shares and the company declares a 10% stock dividend. When the customer covers the short position, the customer will be required to deliver:

440 shares Explanation: When a customer sells short, the brokerage firm borrows stock to deliver it to the buyer. All cash and stock dividends declared are the responsibility of the customer who sold the stock short. In this example, the company declares a 10% stock dividend. Therefore, a customer who sold short 400 shares will be required to deliver 440 shares (400 shares x 10% = 40 additional shares) when he covers the short sale.

An individual purchases one XYZ 40 call for 4 and one XYZ 50 call for 2. The market price of XYZ stock is currently 43. The individual's breakeven price is:

46 Explanation: Buying two calls with different strike prices is a bullish strategy. In this example, since one of the strike prices is higher and out-of-the-money, it is less expensive than buying two calls with the same strike price. The total cost of the XYZ options is 6. The 40 call would be exercised first, resulting in a total cost of 46 (40 + 6). This is the amount at which the individual would need to sell XYZ stock to break even. If the market price of the stock is trading at any other value, the client would either have a profit or loss. For example, if the market price at expiration is $52, the client will have a $1,200 gain on the 40 call (52 - 40) and a $200 gain on the 50 call (52 - 50). The total gain of $1,400 less the combined $600 premium equals an $800 profit.

On Monday, June 15, an investor purchases for regular-way settlement, $20,000 face value of 8% municipal bonds that mature on November 1, 2035. How many days of accrued interest is the investor required to pay?

47 Explanation: Since the bonds mature on Nov. 1, we know the semiannual interest payments are made on Nov. 1 and May 1. The bonds were purchased in June, so accrued interest must be calculated from the last interest payment date, (May 1, up to but not including settlement.) Since the transaction will settle on June 18, we count 17 days in June. So the total number of days of accrued interest is 30 days for May (remember, in calculating accrued interest for municipal bonds, a 30-day month and 360-day year are used) and 17 days for June. Accrued interest of 47 days is owed to the seller.

An individual has an IRA account. He is 75 years old, but has not withdrawn funds from the plan. What is the penalty that the individual will be subject to for not withdrawing funds from the plan?

50% penalty on the actuarial amount Explanation: The individual will be subject to a penalty of 50% of the actuarial amount (the amount specified by the IRS that should have been withdrawn). This is the penalty for not making withdrawals from the plan. Distributions must start after the planholder reaches age 70 1/2.

A company has 1,000,000 shares outstanding. It plans to issue 2,000,000 additional common shares to raise funds to build a new manufacturing facility. Your client owns 50,000 shares of this company. How many rights will the customer receive?

50,000 The customer receives 1 right per shares, so he or she receives 50,000 rights.

ABC Company has issued $20,000,000 of convertible bonds with a coupon of 5% and a current market value of 120. The conversion price is $40. If all the bonds are converted, how many additional shares of common stock will ABC have outstanding?

500,000 (1000/ 40= 25) (20,000,000/ 1000= 20,000 shares issued) (20,000 x 25= 500,000 shares)

ABC Corporation is paying a $5 yearly dividend on its preferred stock. The market price of the preferred stock is $80. The current yield is:

6.25% Explanation: The current yield on common or preferred stock is found by dividing the yearly dividend by the market price of the stock. In this example, the market price of the preferred stock is $80 and the yearly dividend is $5. This equals a current yield of 6.25% ($5 divided by $80 equals 6.25%).

A customer purchased a municipal bond with a 6.50% coupon rate that was priced at a 6.95 basis. If the bond is currently trading at $945, the current yield is:

6.88% Explanation: The current yield is found by dividing the yearly interest payment of $65 by the market price of $945. This equals 6.88%. The fact that the bond was purchased at a 6.95 basis is not relevant.

An investor owns a $1,000,000 diversified portfolio of stocks with a beta of 1.5. He wishes to hedge his portfolio by buying S&P 100 Index options. He can accomplish this by buying:

60 S&P 100 puts with a strike price of 250 Explanation: Beta is a measure of a stock's (or portfolio of stock) volatility in relation to the market as a whole. The market as a whole (represented in this question by the S&P 100 Index) is assigned a beta of 1. The portfolio's beta of 1.5 means the portfolio's price will change 1 1/2 times as much as the market. Buying puts provides a hedge since a decrease in the value of the portfolio can be offset by a profit in the puts. A strike price of 250 represents an overall value of $25,000 (250 strike price x $100 multiplier). Therefore, 40 puts represent a total value of $1,000,000 (the value of the portfolio). Since the portfolio will change by $1.50 for each $1 change in the market (beta = 1.5), 60 puts (1 1/2 x 40) will be necessary to effectively hedge the portfolio.

A customer is currently short 100 shares of ABC common stock at 57.50 and, for protection, has entered a buy stop order at 60. Round lot trades that took place after these orders were entered were: 58, 59.50, 60.10, 60.50, 60, 59.85 The round-lot trade that activated the order was:

60.10 Explanation: By entering the buy stop order, the client is attempting to limit any loss to about $250 (2 1/2 points on 100 shares). The buy stop is entered above the current market price. The stop will activate on the first round-lot trade, which occurs at or through (above, since this is a buy stop) the stop price. That trade occurs at 60.10. That is not the same price as the executing price

Your client has entered a limit order to buy 600 shares of DMF at $50 per share. DMF declares a 10% stock dividend. How would this order be adjusted on the ex-date?

600 shares at $45.45 $50 ÷ (1 + 0.10) = $45.45

Which of the following choices is NOT considered of material value by FINRA if given by a mutual fund underwriter to a registered representative? a. A gift of $100 b. A gift of $200 c. Overrides in excess of commissions as stated in the prospectus d. A gift of $500

A gift of $100 Explanation: According to industry rules, a gift of more than $100 is considered substantial or of material value.

An insider owning 500,000 shares of unregistered ABC stock has filed a Form 144 Notice of Offering. The weekly volume of trading for ABC on all exchanges was: June 30 61,000 June 23 62,000 June 16 64,000 June 9 65,000 June 2 40,000 ABC has 6,500,000 shares of stock outstanding. On July 3, the insider would like to sell a portion of his unregistered stock. What is the maximum amount of shares he may sell under Rule 144?

65,000 Explanation: On July 3, the insider wants to sell unregistered ABC stock under Rule 144. The trading volume for the previous four weeks was: June 30 61,000 June 23 62,000 June 16 64,000 June 9 65,000 Total four-week volume 252,000 The average volume is 63,000 shares (252,000 divided by four weeks equals 63,000). Rule 144 states that the insider may sell an amount equal to the average weekly volume of the previous four weeks, or 1% of the outstanding shares, whichever is greater. One percent of the 6,500,000 outstanding shares equals 65,000. Therefore, the investor may sell 65,000 shares of the security.

In a new margin account, if a customer buys 300 shares of XYZ for 48 and simultaneously writes 3 XYZ Jan 50 calls at 1, the margin deposit required will be for

6900 The Regulation T requirement is $7,200 (50% × $14,400). There is no Regulation T requirement for writing covered calls. The requirement to establish both positions is $7,200. However, the question asked for the margin deposit. The requirement of $7,200 is reduced by the premium income received ($300). By depositing $6,900, the customer will have $7,200 in the account—the difference being the premium income credited to the account on settlement date.

An investor in the 28% tax bracket can buy a 5.10% tax-free municipal bond at par. What yield would the investor need in a taxable corporate bond to receive the same after-tax yield in the municipal bond?

7.08% Explanation: If an investor in a particular tax bracket would like to compare the benefit of tax-free interest income to after-tax income of a corporate bond, it is necessary to find the equivalent taxable yield. The formula is: Municipal Bond Yield = Equivalent Taxable Yield divided by 100% - investor's tax bracket The customer is in the 28% tax bracket. The municipal bond has a 5.10% coupon rate and since it is purchased at par, the yield is also 5.10%. 5.10% (municipal bond yield) divided by 72% (100% - 28%) = 7.08% (Equivalent Taxable Yield). If the individual was to buy the 7.08% corporate bond, the tax liability for the interest received would be $19.82 ($70.80 interest income x 28% tax bracket = $19.82 tax liability). This would give the investor an after-tax return of $50.98 which is the same return he could receive if he purchased the 5.10% ($51.00) municipal bond.

A municipal bond with a 6% coupon is priced at a 7.20 basis. If the bond's yield to maturity increases by 40 basis points, the yield to maturity is:

7.60% Explanation: The term priced at a 7.20 basis refers to a serial bond that is priced to yield 7.20 or a YTM of 7.20%. If the bond's basis increased by 40 basis points, the new yield to maturity is 7.60%. The 6% coupon rate is relevant if the question asked about whether the bond was trading at a discount or a premium. Since the YTM is greater than 6%, the bond is trading at a discount.

At what age must IRA withdrawals begin?

70 1/2 Explanation: IRA withdrawals may begin at any age, but a penalty may be assessed if withdrawals begin prior to age 59 1/2. Withdrawals in traditional IRAs must begin by age 70 1/2. There is no required minimum distribution (RMD) for Roth IRAs. (Note: If the question does not specifically use the term Roth, students must assume it covers the traditional IRA.)

On September 22, a customer purchases 2 listed XYZ May 70 calls and pays a $4 premium for each call. The current market price of XYZ Corporation is $69 per share. What is the customer's breakeven point?

74 Explanation: The strike price plus the premium equals the breakeven point for the buyer of a call. The breakeven point is $74 (the $70 strike price + the $4 premium = $74). The fact that the investor bought multiple contracts is not relevant since the breakeven point is a price per share.

A registered representative is provided with the following financial information concerning a company: Debt of $225 million, par value of the common stock $40 million, paid-in capital of $70 million, and retained earnings of $750 million. The common stock ratio is:

79% Explanation: The common stock ratio is found by dividing total shareholder equity by a company's total capital. Shareholder equity is equal to the par value of the common stock + paid-in capital + retained earnings, and the total capital is found by adding the debt to shareholder equity. The common stock ratio is 79% [par value of the common stock is $40 million + paid-in capital of $70 million + retained earnings of $750 million = $860 million / $1,085 million ($225 million + $860 million)]. The common stock ratio is used to analyze the capital structure of a company.

A corporation has $7,000,000 in income after paying preferred dividends of $500,000. The company has 1,000,000 shares of common stock outstanding. The market price of the stock is $56. What is the price-earnings ratio?

8 times Explanation: The price-earnings ratio is the market price ($56) of the stock divided by the earnings per share ($7), which equals 8 times. The earnings per share of $7.00 is found by dividing the $7,000,000 of available income to the common stockholders by the 1,000,000 shares of common stock outstanding.

An investor has a federal tax rate of 35% and a state tax rate of 6% and is offered a 4.90% in- state municipal bond. What yield would the investor need in a taxable bond to receive the same after-tax yield as the municipal bond?

8.31% Explanation: The major advantage of municipal bonds for most investors is that the interest received from the bond is exempt from federal taxes. In addition, most states also exempt interest from bonds issued within their state from a resident's state and local income taxes. However, if a state resident earns interest from an out-of-state municipal security, that interest is usually subject to state and local taxation. If an investor in a particular tax bracket would like to compare the benefit of tax-free interest income to after-tax income of a taxable bond, it is necessary to find the equivalent taxable yield. Since the investor is purchasing an in-state bond, we use the combined rate of 41%. The formula is: Municipal Bond Yield / (100% - Investor's Tax Bracket) = Equivalent Taxable Yield The customer is in the 41% tax bracket. The municipal bond has a yield of 4.90%. 4.90% (Municipal Bond Yield) / 59% (100% - 41%) = 8.31% Equivalent Taxable Yield

A provision permitting the syndicate members participating in a firm commitment offering to engage in short selling on an IPO is

A green shoe clause, negotiated with and agreed to by the issuer, allows the syndicate to sell up to 15% more shares than initially registered within 30 days of the IPO beginning to trade. Because the underwriters do not own those additional shares, they are, in reality, selling them short.

A customer purchases a municipal bond for settlement on Tuesday, October 10. The bond pays interest on January 15 and July 15. The number of days of accrued interest the buyer owes to the seller is:

85 days Explanation: Interest is figured from the last interest payment date, July 15, up to but not including the settlement date (which is given as October 10). Therefore, accrued interest is figured up to and including October 9. The customer buying the bonds needs to pay accrued interest for 85 days. Corporate and municipal bond interest is computed on the basis of a 30-day month and a 360-day year. If interest is paid on the first of the month, there will be 30 days of accrued interest to calculate. If interest is paid on the fifteenth of the month, there will be 16 days of accrued interest to calculate for that particular month. July 16 days August 30 days September 30 days October 9 days 85 days

Ms. Jones, a shareholder of XYZ Corporation, reads in the newspaper that XYZ Corporation intends to issue new shares through a rights offering. The terms of the rights offering are as follows: 1. 10 rights plus $10.50 are required to subscribe to one new share of stock 2. Fractional shares become whole shares 3. The record date is Friday, October 17 4. JPMorgan Chase and Bank of America are the transfer agents 5. Goldman Sachs and Morgan Stanley are the standby underwriters Ms. Jones owns 87 shares of the XYZ Corporation. How many shares can she subscribe to and how much will it cost her?

9 shares plus $94.50 Explanation: Ms. Jones can subscribe to nine shares at a cost of $94.50. The terms of the rights offering indicate that 10 rights plus $10.50 are needed to subscribe to one new share of stock. Fractional shares become whole shares. She will receive 87 rights. It takes 10 rights to get one new share of stock. 10 rights divided into 87 rights equals 8.7 shares. Since fractional shares become whole shares, Ms. Jones can subscribe to nine shares at a cost of $10.50 a share for a total of $94.50 (9 x $10.50 = $94.50)

RSR Corporation has earned $4 per share and has paid a 75 cent dividend per share. If the stock is selling at $38 a share, what is its price/earnings ratio?

9.5 Explanation: The price/earnings ratio is found by dividing the market price of $38 by the earnings per share of $4. This equals a price/earnings ratio of 9.5 ($38 / $4). The amount of the dividend is not relevant in calculating the price/earnings ratio.

There are 2,600,000 shares of XYZ Corporation outstanding, which are listed on the NYSE. Mr. Smith owns 300,000 shares of restricted securities, which he has held for more than six months. He is not an affiliate of XYZ. Mr. Smith would like to sell some of his securities under Rule 144. The weekly trading volume for the last six weeks is: 1 week ago 25,000 shares 2 weeks ago 26,000 shares 3 weeks ago 27,000 shares 4 weeks ago 28,000 shares 5 weeks ago 27,000 shares 6 weeks ago 27,000 shares According to Rule 144, Mr. Smith would need to file with the SEC a notice of intent to sell which is valid for:

90 days Explanation: The notice of offering for a Rule 144 sale is valid for 90 days.

The Barge Towing Corporation has announced in a tombstone ad that it will issue $500,000,000 of 6 1/2% convertible subordinated debenture bonds convertible into common stock at $10.50. The bonds will mature in November 2040 and are being issued at a $1,000 par value. The conversion ratio of the bonds is approximately:

95 to 1 Explanation: The conversion price is given as $10.50. To find the conversion ratio, divide the par value ($1,000) of the bond by the conversion price of $10.50. This equals a conversion ratio of 95 to 1 ($1,000 divided by $10.50 equals 95).

A customer owns 50 shares of ABC Corporation. ABC Corporation is engaging in a rights offering. Each existing share receives one right. The terms of the offering are that 10 rights plus $35 is required to buy one new share of stock. If the customer wanted to subscribe to the rights offering, how many additional rights would she need to buy 100 new shares of stock?

950 Explanation: The terms of the rights offering are that 10 rights are required to subscribe to one new share of stock. If an investor wanted to subscribe to 100 shares of stock, the investor would need 1,000 rights. (10 rights x 100 shares = 1,000 rights.) The investor owns 50 shares of stock and will receive 50 rights from the corporation (one right for each share owned). If the customer wanted to subscribe to 100 shares through the rights offering, the investor would need to purchase an additional 950 rights.

The regular hours of operation of the NASDAQ system are:

9:30 AM - 4:00 PM ET

A customer sells XYZ short at $40 and sells one XYZ October 40 put at 5. What will the customer's profit or loss per share be if the put is exercised when the market value of the stock is $35, and the stock received pursuant to that exercise is used to cover the short stock position?

A $5 profit Explanation: If the put is exercised when the market price is less than $40, the stock acquired by the writer will be used to close out the short position. There is no profit or loss on the short sale (sell short at $40 and cover at $40). The entire profit is the premium income of $5. The seller of a put who sells short can never profit by more than the premium when the short sale price and the strike price are the same.

A customer buys an ABC July 50 call, paying a $3 premium. Seven months later, the customer exercises the call when the market price of ABC stock is $60 per share. The customer immediately sells the stock for $6,000. If the customer had sold the option at $8 instead of exercising the option, the profit would have been taxable as:

A $500 capital gain Explanation: If the customer had sold the option at $8 instead of exercising it, the $5 profit per share ($8 sale minus $3 cost equals $5 profit) would be taxable as a capital gain.

Accrued interest for municipal bonds is computed on:

A 30-day month and a 360-day year Explanation: Accrued interest for municipal bonds is computed in the same manner as for corporate bonds, which is based on a 30-day month and a 360-day year. Accrued interest for U.S. government bonds is figured on a 365-day year counting actual days elapsed. Accrued interest on all bonds is calculated from the last interest payment, up to but not including the settlement date.

The Bond Buyer Municipal Bond Index is based on:

A 40-Bond Index Explanation: The Bond Buyer Municipal Bond Index represents the average of the prices of 40 long-term municipal bonds adjusted to a yield of 6%.

An investor is in the 28% tax bracket. Which of the following investments will afford him the BEST after-tax yield?

A 5% municipal bond Explanation: The 5% municipal bond will offer the best after-tax yield because the interest income is completely free from federal income taxes. The other investments are types of corporate debt subject to federal income taxes and 28% of the income received will be taxable. The taxable equivalent yield of the 5% municipal bond is 6.94%. This is calculated by dividing the 5% municipal yield by the complement of the tax bracket which is 72%. The result is greater than the other choices.

To which of the following customers would a registered representative be LEAST likely to recommend an exchange-traded note (ETN)? a. A customer seeking to benefit if an index increases b. A customer seeking to benefit if an index decreases c. A customer seeking income on a regular basis d. A customer seeking capital appreciation

A customer seeking income on a regular basis Explanation: Exchange-traded notes (ETNs) are a type of unsecured debt security. ETNs carry issuer risk that is tied to the creditworthiness of the financial institution backing the note. These securities are not like traditional fixed-income securities since they typically do not make interest payments to investors. An investor seeking income on a regular basis would not be a suitable candidate for an ETN. The returns are linked to the performance of an index, currency, or commodity and would be suitable for investors who want to speculate on the value of an index. Most ETNs are traded on a national exchange (e.g., NYSE) which has the feature of liquidity. Therefore, an investor seeking capital appreciation has the ablity to sell when advantageous.

A customer has a federal tax rate of 35% and a state tax rate of 5%. Which of the following investments would afford him the BEST after-tax yield?

A 7.05% out-of-state municipal bond Explanation: The major advantage of municipal bonds for most investors is that the interest received from the bond is exempt from federal taxes. In addition, most states also exempt interest from bonds issued within their state from a resident's state and local income taxes. However, if a state resident earns interest from an out-of-state municipal security, that interest is usually subject to state and local taxation. If an investor in a particular tax bracket would like to compare the benefit of tax-free interest income to after-tax income of a taxable bond, it is necessary to find the equivalent taxable yield. The investment-grade corporate bond and REIT are fully taxable. Since the investor can purchase an in-state municipal bond and out-of-state municipal bond, we use the combined rate of 40% for the in-state bond and the federal rate of 35% for the out-of- state bond. The formula is: Municipal Bond Yield / (100% - Investor's Tax Bracket) = Equivalent Taxable Yield The customer is in the 40% combined tax rate. The municipal bond has a yield of 6.40%. 6.40% (Municipal Bond Yield) / 60% (100% - 40%) = 10.67% Equivalent Taxable Yield The out-of-state municipal bond has a yield of 7.05% and the equivalent taxable yield is 10.85% (7.05% / 65%). The out-of-state municipal bond has the best or highest after-tax yield.

If an investor buys a LEAPS contract on issuance and allows it to expire unexercised, what is the investor's tax consequence at expiration?

A LEAPS contract has an expiration of more than one year. Upon expiration, the buyer incurs a long-term capital loss equal to the amount of the premium paid.

Which of the following are types of underwriting commitments? A. Best Efforts B. All or None C. Not Held D. Immediate or cancel

A and B Best efforts and all-or-none are types of underwriting commitments where the underwriter acts as an agent, taking no liability. "not held" is a type of order where a floor trader can exercise discretion as to price and time of execution. "immediate or cancel" is a type of order to be filled immediately in full or in part, with any unfilled balance cancelled.

Which of the following are types of Fiduciary accounts? A. custodian account B. partnership account C. joint account D. trust account

A and B only

In August, an investor sells an uncovered listed option and receives a $1,100 premium. The following February, the customer makes a closing purchase transaction at 3. The result of the transaction is:

A capital gain of $800 Explanation: The investor made an $800 profit on the closing transaction (sale at $1,100 and purchase at $300). The profit is treated as a capital gain in the year the transaction is closed out.

A municipal bond issue is called due to an event beyond the control of the issuer that affects the use of property (e.g., earthquakes, hurricanes, condemnation of property). This is known as:

A catastrophe call Explanation: A catastrophe call allows an issuer to call an entire issue in situations that are beyond its control, such as a condemnation.

Which of the following statements is TRUE concerning the disclosure requirements in CMO correspondence? a. A comparison between a CMO and an highly rated corporate bond is permitted b. A comparison between a CMO and a municipal bond is permitted if the client is in a high tax bracket c. A comparison between a CMO and a bank certificate of deposit is permitted if the bank is FDIC-insured d. A comparison between a CMO and a bank certificate of deposit is not permitted under any circumstances

A comparison between a CMO and a bank certificate of deposit is not permitted under any circumstances Explanation: Due to their unique characteristics, CMOs may not be compared to any other types of investment, including a certificate of deposit. This prohibition applies to any communications with the public about CMOs, which includes retail communications and correspondence.

A registered representative for ABC Brokerage has just been elected president of the Harper Valley School Board. The school district is going to float a new serial issue of general obligation bonds backed by ad valorem taxes. ABC Brokerage's relationship to the school district can be described as:

A control relationship Explanation: A control relationship exists when a brokerage firm or an employee of a firm is in a position to control or influence the issuance of securities by an issuer.

An investor writes an ABC June 70 put at 4. If the option is exercised, the investor will have:

A cost basis of $6,600 for the stock acquired in the exercise Explanation: When a put is exercised, the premium received by the writer is treated as a reduction in the cost of the underlying stock. The strike price of the put (70), minus the premium received by the writer (4), equals the writer's cost basis in the underlying stock (66). The writer will have a gain or loss depending on the stock's price when it is sold.

In order to sell variable annuities to clients, a person must hold which of the following?

A life insurance license and securities registration Explanation: Variable annuities are considered both insurance products and securities. As a result, an individual must be properly registered (Series 6 or 7) and hold a life insurance license.

All of the following choices are requirements for a stock to be listed on the NYSE, EXCEPT:

A minimum 25% dividend payout ratio Explanation: To be listed on the NYSE, a corporation must have a minimum number of round-lot shareholders, a minimum number of publicly held shares, a minimum aggregate market value of publicly held shares, a positive earnings history, national interest in the corporation, and agreement to solicit proxies. Dividend payout ratios are not a listing requirement.

Advertising for municipal fund securities investments must be approved prior to its official use by:

A principal of the firm who is selling the program Explanation: A 529 College Savings Plan is a type of municipal fund security. All advertising regarding municipal securities and municipal fund securities must be approved by a municipal securities principal of the firm prior to its initial use.

A customer sells an XYZ April 30 put for $5 and an XYZ April 30 call for $3. If the put is repurchased at $4 and the call is repurchased for $1, the customer will have:

A profit of $300 Explanation: The customer sold a straddle and received $500 for the put and $300 for the call or a total of $800 for the straddle. The put was repurchased for $400 and the call for $100, or a total cost of $500. The difference between the amount of premiums received from the sale of the straddle ($800) and the cost of repurchasing the straddle ($500) is a profit of $300 for the customer.

A call option is covered by all of the following choices, EXCEPT: a. An escrow receipt b. A short position in the underlying stock c. A bond convertible into 100 shares of the underlying stock d. 100 shares of the underlying stock

A short position in the underlying stock Explanation: For a call option writer to be considered covered, the writer could own 100 shares of the underlying stock, have an escrow receipt, or own bonds convertible into at least 100 shares of the underlying stock. Since the writer's obligation is to deliver stock if the option is exercised, a short position would not cover a call.

If an investor practices value investing, which of the following stock types is he least likely to purchase?

A stock with an above-average price-to-earnings (P/E) ratio

Which of the following circumstances is NOT a reason for rejecting a municipal bond delivery? a. The lack of a legal opinion b. A missing coupon c. A sudden change in market value d. A mutilated certificate

A sudden change in market value Explanation: A municipal bond can be rejected if it is missing a legal opinion, has missing or mutilated coupons, or the certificate is mutilated. It may not be rejected because of a sudden change in market price.

Which investment company does NOT charge a management fee?

A unit investment trust Explanation: A unit investment trust does not charge a management fee. The portfolio is fixed and there is no investment adviser since unit investment trusts are supervised, not managed.

An investor wants to invest $20,000 but anticipates needing those funds in five years for a business investment. Currently, with inflation rising, the government is expected to take action to push interest rates up to reduce the money supply. Given these conditions, which of the following securities would be the least suitable for this investor who needs a specific amount of money in five years?

A zero-tranche CMO is subject to interest rate risk as well as extension risk when interest rates rise, and therefore, it would not be suitable for a customer that needs her investment back at a specific point in the future.

A customer bought a bond that yields 6-½% with a 5% coupon. If the bond matures at this point, the customer will receive: A) $1,025. B) $1,050. C) $1,000 plus a call premium. D) $1,065.

A) $1,025. Upon redemption of a bond, whatever current interest rates may be, the investor receives par ($1,000) plus the final semiannual interest payment ($25 in this case), for a total of $1,025.

A customer in the 28% tax bracket owns a 9% ABC Corporation 20-year bond that currently is yielding 8.7%. He is considering buying tax-exempt securities. What is the comparable yield for a municipal bond? A) 0.0626. B) 0.125. C) 0.0648. D) 0.1208.

A) 0.0626. When comparing the yield of a taxable corporate bond to a tax-free municipal bond, use the formula: interest on corporate bond × (100% − tax bracket). In this case, 8.7% × .72 = 6.264%. Remember to use the yield to maturity, not the coupon rate. The bond is currently priced to yield 8.7%. In this case, a tax-exempt bond yielding more than 6.264% will provide a higher after-tax return.

An investor has an established margin account with a short market value of $8,000 and a credit balance of $13,000, with Regulation T at 50%. A maintenance call would be triggered if the short market value increased above: A) 10000. B) 13000. C) 9000. D) 8000.

A) 10000. To find short market value at maintenance, divide the credit balance of $13,000 by 1.3 ($10,000).

The Three Contact Rule does NOT apply to the purchase or sale of a non-Nasdaq security provided there is at least: A) 2 priced quotations available electronically . B) 3 priced quotations available electronically . C) 4 priced quotations available electronically . D) 1 priced quotation available electronically.

A) 2 priced quotations available electronically . Provided there are at least 2 priced quotations available electronically, the Three Contact Rule does not apply.

If a customer is short 100 XYZ shares at 54 and long 1 XYZ 55 call at 2, what is the maximum potential loss? A) 300. B) 200. C) Unlimited. D) 100.

A) 300. The customer has protected his short stock position from a market advance by purchasing the call. If the market rises, the call is exercised, allowing the customer to buy stock at the options strike price of 55 to cover the short position. Therefore, the most the customer can lose is $100 on the stock position (the difference between the option strike price and short sale price), plus the premium paid for the option ($100 + $200 = $300).

If a customer fails to meet a Regulation T margin call of $2,500, securities may be sold out of the account with a value of: A) 5000. B) 3333. C) 8000. D) 2500.

A) 5000. Securities valued at twice the Regulation T cash call must be sold out if a customer fails to meet a Regulation T margin call ($2,500 × 2 = $5,000).

In an existing margin account with no SMA, if a customer buys 300 ABC at 40 and simultaneously buys 3 ABC OCT 40 puts at 2.50, the customer must deposit: A) 6750. B) 6375. C) 5250. D) 6100.

A) 6750. Buying 300 shares at 40 ($12,000) requires a deposit of $6,000. In addition, the customer is purchasing 3 puts with a total premium of $750 (3 × 2½). Most options have no loan value and must be paid in full. Adding $6,000 and $750 results in a deposit of $6,750.

The current yield on a bond with a coupon rate of 7.5% currently selling at 105-½ is approximately: A) 7.1%. B) 6.5%. C) 7.5%. D) 8%.

A) 7.1%. A bond with a coupon rate of 7.5% pays $75 of interest annually. Current yield equals annual interest amount divided by bond market price, or $75 / $1,055 = 7.109%, or approximately 7.1%.

A customer buys a real estate limited partnership interest by contributing $20,000 and signing a nonrecourse note for $50,000. The customer's beginning basis is: A) 70,000. B) 50,000. C) 20,000. D) 30,000.

A) 70,000. Generally, nonrecourse debt does not add to basis because the limited partner is not responsible (at risk) for the repayment of the debt. However, in real estate partnerships, the at-risk rules do not apply, and therefore, add to basis in this type of partnership.

A commercial bank purchasing qualified GO bonds may deduct what percentage of the interest cost necessary to fund the purchase? A) 80%. B) 100%. C) 50%. D) 20%.

A) 80%. A bank-qualified municipal bond issue is a small issue, generally a GO issue of $10 million or less. If a bank were to purchase any part of a qualified issue, 80% of the annual costs necessary to fund the purchase would be tax deductible to the bank.

An investor is looking for an investment that will generate deductions but also provide the potential for future cash flow. Which of the following is NOT an appropriate investment? A) A raw land program B) An existing properties real estate program C) An oil and gas drilling program D) An oil and gas exploratory program

A) A raw land program All of the choices provide potential for future cash flow and deductions except for a raw land program. In a raw land program, deductions are negligible and the profit potential comes in the form of capital appreciation, not cash flow.

A registered representative (RR) has just explained to a customer that to purchase a particular security the customer would pay the asking price plus a commission, not a sales charge. Which of the following is the RR is speaking of? A) Any closed-end fund B) All open-end funds C) All management company offerings D) Mutual funds

A) Any closed-end fund Closed-end funds are purchased on an exchange or over the counter where buyers pay the ask price plus a commission. The RR could not be speaking of all open-end funds because mutual funds, one classification of open-end funds, are purchased at the POP, which includes a sales charge. Management company offerings include both open-end and closed-end funds.

Your customer is asking if either exchange-traded funds (ETFs) or exchange-traded notes (ETNs) might be suitable investments for his portfolio. The customer makes several statements regarding his understanding of the products but only one of them is accurate. Which is it? A) Exchange-traded notes (ETNs) are issued by financial institutions and therefore I should be concerned about the credit worthiness of the issuer. B) If I want to sell my shares of an exchange-traded fund (ETF) I have to wait until the next price is calculated to value the portfolio of securities. C) Exchange-traded funds (ETFs) have a fixed coupon rate that I should expect to realize when they mature. D) Exchange-traded notes (ETNs) are equity securities because they trade on exchanges.

A) Exchange-traded notes (ETNs) are issued by financial institutions and therefore I should be concerned about the credit worthiness of the issuer. The only accurate statement is the one expressing that (ETNs) are issued by financial institutions and therefore the credit worthiness of the issuer should be a concerning factor. ETNs are debt instruments not equity instruments. ETNs have a final payment at maturity based on the return of a single stock, a basket of stocks or an equity index. While ETF prices fluctuate based on the value of the securities within the fund portfolio throughout the trading day they are priced by supply and demand like all exchange traded products. They are not forward priced like open-end mutual fund shares are.

Which of the following govern the sale of a publicly offered direct participation program? I) FINRA II) Securities Act of 1933 III) The Investment Company Act of 1940 IV) The Internal Revenue Service A) I and II B) III and IV C) II and III D) II and IV

A) I and II The sale of a publicly registered DPP, like any other newly issued nonexempt security, is governed by the Securities Act of 1933, FINRA, and any applicable blue-sky (state) securities laws. While IRS tax code is applicable to DPPs, the IRS does not govern the sale of the securities.

Which of the following would be found on a when-, as-, and if-issued confirmation? I. Trade date. II. Settlement date. III. Price. IV. Accrued interest. A) I and III. B) II and IV. C) I and II. D) III and IV.

A) I and III. Information that does not appear on a when-issued confirmation can easily be remembered as SAT (settlement date, accrued interest, and total amount due). The trade date and price per bond are included on the when-issued confirmation.

If a customer places an order to sell 500 ABC at 46 stop limit, which of the following statements are TRUE? The order will be elected at 46 or lower. The order will be elected at 46 or higher. The order can be executed at 46 or higher. The order can be executed at 46 or lower. A) I and III. B) II and III. C) I and IV. D) II and IV

A) I and III. Sell stop limits are placed below the current market and will be elected when the stock trades at or through (lower than) the stop price. Once elected, the order becomes a limit order to sell at 46 or better (higher).

Depletion allowances in oil and gas programs are based on the amount of oil: A) sold. B) in reserve. C) extracted. D) lost to shrinkage.

A) sold. Depletion allowances are allowed to compensate for a mineral resource, which is considered accomplished when it is sold.

If a client has a margin account with $23,000 in securities and a debit of $12,000, and Regulation T is 50%, which of the following statements are TRUE? I. The account is restricted. II. The client will receive a margin call for $500. III. The client may withdraw securities if he deposits 50% of the securities' value. IV. The account has excess equity of $5,250. A) I and III. B) II and III. C) I and II. D) III and IV.

A) I and III. The account is restricted by $500 because the equity of $11,000 is less than the Regulation T requirement of 50% ($11,500). However, the client will not receive a margin call for the $500 because Regulation T applies only to the initial purchase. Because the account is restricted, any withdrawal of securities requires a cash deposit of 50% or a deposit of securities with a loan value of 50% of the value of the securities withdrawn. The account is $5,250 above the required minimum maintenance margin, but this amount is not considered excess equity.

Which of the following may be done only with the approval of the shareholders of an investment company? I. A change from diversified to nondiversified status. II. The purchase of particular bonds on the open market. III. Personnel changes in the transfer agent's organization. IV. A change in the fund's objectives. A) I and IV. B) I and III. C) II and IV. D) II and III.

A) I and IV. Any substantive change in an investment company's form, structure, investment objective, or business operation must be approved by a majority vote of the outstanding shares. Bond purchases are left to the fund's portfolio manager, and the transfer agent is trusted with its organization's personnel changes.

Assuming ABC is subject to a 60,000 contract position limit, which of the following customer accounts are in violation of the exchange's position limits? I. Long 35,000 ABC January calls; long 30,000 ABC January 08 LEAP calls. II. Long 35,000 ABC March calls; long 30,000 ABC March puts. III. Long 35,000 ABC March calls; short 30,000 ABC January 08 LEAP calls. IV. Long 35,000 ABC March calls; short 30,000 ABC March puts. A) I and IV. B) III and IV. C) II and IV. D) I and II.

A) I and IV. The maximum limit for ABC is 60,000 contracts on the same side of the market. The upside is long calls and short puts; the down side is long puts and short calls. LEAPs are included in the calculation.

The issuance of a debenture by a company would have an immediate effect on which of the following balance sheet items? I. The total assets. II. The total liabilities. III. The working capital. IV. The shareholders' equity. A) I, II and III. B) II, III and IV. C) I, II and IV. D) I, III and IV.

A) I, II and III. The cash received from the sale of the bonds is a current asset of the company and as such would increase assets and working capital on the balance sheet. The debentures are debts of the company and would increase the liabilities of the company. Shareholders' equity is only affected by gains, losses, new invested capital, and cash distributions (dividends) to shareholders.

Which of the following is true regarding exchange traded funds (ETFs)? I. The SEC has classified them as mutual funds. II. The SEC has classified them as a type of open-end fund. III. They have operating costs and expenses that are higher than most mutual funds. IV. They have operating costs and expenses that are lower than most mutual funds. A) II and IV B) I and IV C) II and III D) I and III

A) II and IV The SEC has classified exchange traded funds as a type of open-end fund but not a mutual fund. ETFs traditionally have operating costs and expenses that are lower than most mutual funds because they do not have to purchase and sell holdings within the portfolio to accommodate investors purchasing shares or redeeming shares, as is the case with mutual funds.

Which of the following statements regarding a shelf offering are TRUE? I. It can be used to distribute an initial public offering only. II. It can be used to distribute an additional offering only. III. Its maximum duration is 90 days. IV. Its maximum duration is 3 years. A) II and IV. B) I and III. C) II and III. D) I and IV

A) II and IV. Shelf offerings are used by publicly traded companies to issue additional equity or debt securities. The issuer must sell the securities within 3 years after the registration is declared effective.

Which of the following oil and gas programs would be associated with the least risk? A) Income. B) Exploratory. C) Developmental. D) Raw land.

A) Income. For oil and gas programs, ranking from least to most risk would be as follows: Income, Developmental and Exploratory. Raw land is a type of real estate program.

Which of the following describes a quote on Nasdaq Level I? A) Inside market. B) Nominal. C) Lowest bid/highest offer. D) Available to traders only.

A) Inside market. Nasdaq Level 1 quotes represent the highest bid and lowest asking prices of all dealers. This is known as the inside market.

Under MSRB rules, which of the following statements regarding a fidelity bond is TRUE? A) It insures against loss due to theft. B) It will insure against the price decline of a security. C) It insures brokerage firm customers in the event the brokerage firm must liquidate. D) It is contained in the trust indenture.

A) It insures against loss due to theft. MSRB members are required to have a fidelity bond which protects against loss due to employee theft.

Which of the following best describes an intangible drilling cost? A) Labor, fuel, or drilling rig rental. B) Proven reserve of oil or gas. C) Tax liability. D) Exploratory well drilling.

A) Labor, fuel, or drilling rig rental. Intangible drilling costs are the noncapital costs of putting in a well. They are currently deductible expenses, like fuel, wages, and rent. An intangible drilling cost is one which, after expenditure, has no salvage value.

A technical analyst is concerned with all of the following trends EXCEPT: A) PE ratios. B) support levels. C) changes in the DJIA. D) reversals.

A) PE ratios. Technical analysts are more interested in forecasting market trends and securities prices than in studying individual corporations. Therefore, they are concerned with market prices, trading volumes, changes in the Dow Jones Industrial Average, reversals, support and resistance levels, advance/decline lines, short interest, and many other factors that might help them time buying and selling decisions. Fundamental analysts, on the other hand, concentrate on a stock's intrinsic quality and are concerned with PE ratios and earnings per share.

Which of the following exemption provisions of the Act of 1933 may NOT be used for an initial offering of securities? A) Rule 144. B) Rule 147. C) Regulation D. D) Regulation A.

A) Rule 144. Rule 144 does not pertain to primary offerings; it affects secondary market transactions in restricted or control securities.

All of the following are required by limited partnerships EXCEPT: A) SEC approval. B) partnership agreement. C) subscription agreement. D) certificate of limited partnership.

A) SEC approval. The SEC does not approve limited partnerships or any other securities. In public offerings of limited partnerships (as opposed to private placements), federal registration and a prospectus are required.

The determination of a broker/dealer's financial failure is made under the provisions of the A) Securities Investor Protection Act of 1970 B) 1939 Trust Indenture Act C) Securities Act of 1933 D) Bank Secrecy Act

A) Securities Investor Protection Act of 1970 Determination of financial failure is made under the Securities Investor Protection Act of 1970

A certificate in the name of Smith & Company may be signed: A) Smith & Company, "Smith & Co.," or "Smith and Company". B) Smith & Company, aka SmithCo. C) Smith & Company. D) Smith & Company or "Smith & Co.".

A) Smith & Company, "Smith & Co.," or "Smith and Company". Corporate signers are the exception to the general rule that endorsement of a certificate must match exactly the name on the front. The word "and "may be substituted with "&" and the word "company" may be abbreviated.

A 6% bond is selling at a 6.25% basis. The bond will mature in 25 years and has 3 call dates. Which of the following bonds will give the investor the best return? A) The bond is called after 10 years at 103 B) The bond is called after 15 years at 102 C) The bond is called after 20 years at 101 D) The bond is held to maturity

A) The bond is called after 10 years at 103 The bond is selling at a discount. The first call in 10 years at 103 will give the investor the best return. The investor receives the highest call price in the shortest number of years.

A customer buys a municipal bond regular way on Tuesday, December 23. The transaction will settle on the following: A) Tuesday. B) Friday. C) Monday. D) Thursday

A) Tuesday. Municipal bonds, like corporate bonds, settle 3 business days after the trade date. December 25 (Christmas) is not a business day.

Which of the following would be the least appropriate investment in a traditional IRA for a 67-year-old client? A) Variable annuities. B) Common stock. C) Corporate bonds. D) Treasury notes.

A) Variable annuities. Why buy a tax-deferred product in a tax-deferred account? A variable annuity will provide no additional tax savings and will likely increase the expense of the IRA. In addition to sales and surrender charges, variable annuities may impose other charges such as mortality and expense risk charges, administrative fees, etc. In less than 4 years, your client will have to begin making withdrawals regardless of any surrender charges the annuity may impose.

A customer has an investment objective of keeping pace with inflation while assuming moderate risk. Which of the following recommendations would best meet the customer profile? A) Variable annuity. B) Universal variable life policy. C) IPO. D) Money market fund.

A) Variable annuity. Insurance companies introduced the variable annuity as an opportunity to keep pace with inflation. For this potential advantage, the investor, rather than the insurance company, assumes the investment risk. A universal variable life policy should be purchased primarily for its insurance features, not its investment features.

When does pension payment liability affect the credit rating of a municipality? A) When funds needed to make payments exceed funds available. B) Pension liability cannot affect the rating of municipal debt. C) When the return on funds invested to meet future needs exceeds anticipated payments. D) When funds are invested presently to meet future pension needs.

A) When funds needed to make payments exceed funds available. The credit rating for a municipality's debt would be adversely affected if funds needed to make payments exceeded funds available. This is an unfunded pension liability and can result if monies set aside to make future payments are not enough or if poor investment decisions deplete the funds.

Mutual fund Class B shares assess: A) a deferred sales load. B) a level load. C) no load. D) a front-end load.

A) a deferred sales load. Class B shares carry a deferred sales load. This is sometimes referred to as a back-end load. Class A shares carry a front-end load. Class C shares carry a level load.

If a customer wishes to change a day order to a GTC order in the middle of the day, the registered representative should: A) allow the day order to expire at the end of the day and put in the GTC order before the next day's opening. B) enter the new order as GTC and immediately cancel the day order. C) enter a change notice immediately. D) enter the new GTC order immediately and do nothing about the day order.

A) allow the day order to expire at the end of the day and put in the GTC order before the next day's opening. The GTC order is treated as a new order. The registered representative should wait until the close of trading so as not to lose the time priority of the original order that day.

A broker/dealer informs registered personnel that, to satisfy the annual compliance meeting requirements, they have recorded a Webcast that individuals can view at their own convenience. A Webcast of this type would be: A) allowed for all registered personnel if informed of the opportunity to submit questions and receive answers in a timely fashion. B) prohibited because this is recorded and not a live Webcast. C) allowed for principals only. D) allowed for registered representatives but not for principals.

A) allowed for all registered personnel if informed of the opportunity to submit questions and receive answers in a timely fashion. Using a recorded Webcast for the annual compliance meeting is allowed for all registered personnel so long as certain safeguards are in place. One such safeguard is the opportunity to ask questions and receive answers in a timely fashion; the firm's registered individuals should be made aware of this opportunity.

Under FINRA rules, customers who are approved to trade options must receive a copy of the OCC Disclosure Booklet: A) at or before account approval. B) within 15 days of account approval. C) at or before the mailing of the next monthly statement. D) at or before the mailing of the confirmation representing the first options trade.

A) at or before account approval. All customers who are approved by the ROP to trade options must receive a copy of the OCC Disclosure Booklet at or before the time the account is approved to trade options.

When a member firm opens an account for a registered representative of another member, the employer-member must be sent written notification: A) before executing an order. B) either by the representative or by the firm opening the account. C) only if requested by the registered representative. D) within 5 business days.

A) before executing an order. When a registered representative of one member firm opens a brokerage account with another member firm, the firm opening the account must send written notification to the registered representative's employer before the execution of any transaction.

If a customer gives specific instructions to his registered representative to purchase a security that is clearly unsuitable in light of the customer's investment objectives, under FINRA rules, the registered representative: A) can enter the order. B) cannot enter the order. C) can only enter the order if the customer puts his verbal instructions into written form. D) can only enter the order with the prior approval of a principal.

A) can enter the order. Under FINRA rules, the representative may execute the trade at the customer's request; the trade ticket should indicate that the order was unsolicited.

If a married couple with a long-term growth objective is considering a mutual fund and they are concerned about the fund's annual expenses, they should select a: A) common stock fund with a low portfolio turnover. B) preferred stock fund. C) long-term corporate bond fund. D) common stock fund with a high portfolio turnover.

A) common stock fund with a low portfolio turnover. Of the choices given, common stock is the only vehicle capable of providing long-term growth. Preferred stock will provide dividends, but it will not provide much growth as it trades like a bond in line with interest rate changes. Of the two common stock funds, the one with the lower portfolio turnover will have lower annual expenses.

After-hours trading in large blocks of stock by institutional investors can be accomplished through: A) electronic communications networks (ECNs). B) the intermarket. C) any regional exchange. D) Nasdaq.

A) electronic communications networks (ECNs). Institutional investors can trade stock after hours through ECNs, which are open 24 hours per day

All of the following are nonexempt securities EXCEPT a: A) fixed annuity. B) variable annuity unit. C) municipal unit investment trust share. D) U.S. government bond mutual fund share.

A) fixed annuity. A fixed annuity is an insurance product exempt from registration with the SEC. Variable annuities, which carry investment risk, are nonexempt securities under the Securities Act of 1933 and must be registered before public sale. Similarly, unit trusts and mutual funds are nonexempt even though the underlying securities may be exempt, such as municipals and U.S. government securities.

Upon being informed that one party to a tenants in common account has died, a registered representative should: A) freeze the account. B) transfer half of the assets to the survivor. C) transfer all of the assets to the surviving tenants. D) allow the surviving tenants to continue trading.

A) freeze the account. The assets of a deceased tenant in a TIC account eventually go to his estate. A registered representative first freezes the account and then awaits the proper court documents. If the account was JTWROS, trading by the surviving tenants could continue.

A sharing arrangement in which only deductible costs are apportioned to the investor with the sponsor bearing all capitalized costs is called a(n): A) functional allocation. B) overriding royalty arrangement. C) carried interest. D) reversionary sharing arrangement.

A) functional allocation. Functional allocation is a sharing arrangement in which the general partner pays for all tangible drilling costs (capitalized costs) and the limited partners pay for all intangible drilling costs (deductible costs).

If another member broker/dealer has already received clearance from FINRA for a retail communication, filing the piece with FINRA so that your broker/dealer can now use it A) is not necessary if unaltered and used as originally intended B) must be done within 3 days after use by your broker dealer, even if unaltered C) must be done 10 days before your broker dealer can use it, even if unaltered D) must be done before publication by your broker dealer whether it is altered or unaltered

A) is not necessary if unaltered and used as originally intended If unaltered and used as it was originally intended, re-filing with FINRA is not required. If the piece had been altered or was intended to be used in a manner inconsistent with how it had been originally intended to be used, filing with FINRA would be required.

Sell order tickets must be: A) marked as either long or short. B) marked only if they are long sales. C) executed in accordance with the appropriate rules, but not necessarily marked. D) marked only if they are short sales.

A) marked as either long or short. Every sell order must be marked as either a long sale or a short sale.

A member firm receives a signed proxy from a customer who failed to indicate how his shares held in street name are to be voted at the annual shareholder's meeting. Under NYSE rules, the member firm: A) must vote the shares as recommended by management of the issuer. B) may vote the shares as it sees fit. C) may vote the shares as it sees fit only if a principal attends the meeting. D) cannot vote the shares.

A) must vote the shares as recommended by management of the issuer. If the beneficial owner of street name stock returns a signed proxy statement but fails to indicate how the shares are to be voted, the member must vote the shares as recommended by management of the issuer.

A purchase or redemption order for investment company shares must be executed at a price based on the: A) net asset value next computed after the fund receives the order. B) best net asset value computed the same day the fund receives the order. C) net asset value computed at the close of trading on the NYSE the day before the fund receives the order. D) net asset value last computed before the fund receives the order.

A) net asset value next computed after the fund receives the order. Purchase or redemption of mutual fund shares occurs at the net asset value next calculated after the fund receives the order; this is known as forward pricing.

Regulation T allows a customer to pay for securities with all of the following EXCEPT: A) newly purchased mutual funds pledged as collateral. B) cash. C) check. D) marginable securities.

A) newly purchased mutual funds pledged as collateral. Regulation T does not allow mutual funds to be used as collateral until they have been owned fully paid for 30 days.

If a ROP is asked to approve a discretionary order to buy 1 XYZ Oct 60 put and sell 1 XYZ Oct 55 put for a net debit of $5, he should: A) not approve the order. B) approve the order if the customer has sufficient funds in his accounts. C) obtain the best execution for the order. D) approve the order in writing.

A) not approve the order. Because this is a debit spread, the maximum gain occurs if both sides are exercised. If this occurs, the investor earns $5 (buy stock at 55 when the short put is exercised and sell stock at 60 by exercising the long put). Because the net premium paid for the spread is $5, there can never be any gain. This spread is not economical.

All of the following would flow through as a loss to limited partners EXCEPT: A) principal repayment on recourse debt. B) interest payments on recourse debt. C) accelerated depreciation. D) depletion.

A) principal repayment on recourse debt. Principal repayments are not deductible for tax purposes. The interest is deductible.

If a customer fails to return a proxy statement to a member firm by the 10th day before the annual meeting, the member may vote the shares: A) provided the matters to be voted on are of minor importance. B) without restriction. C) under no circumstances. D) with the permission of the issuer.

A) provided the matters to be voted on are of minor importance. If a customer signs and returns a proxy statement and fails to indicate how the shares are to be voted, the member must vote the shares as recommended by management of the issuer. If, however, the customer does not return the proxy by the 10th day before the annual meeting, the member may vote the shares as it sees fit as long as the matters to be voted on are of minor importance. If the matters are of major importance (e.g., a merger or the issuance of additional shares), the member may never vote the shares.

A corporate offering of 200,000 additional shares to existing stockholders may be made through a: A) rights offering. B) warrant. C) secondary offering. D) tender offer.

A) rights offering A rights offering is an offering of additional shares of stock to existing shareholders.

All of the following statements regarding OTC markets are true EXCEPT A) the OTC market is an auction market B) securities traded OTC include ADRs and municipal bonds C) a bid is the highest price a dealer will pay when buying D) an offer is the lowest price a dealer will accept when selling

A) the OTC market is an auction market The OTC market is a negotiated market in which market makers post their quotes in order to facilitate negotiating price. A bid is the highest price a buyer is willing to pay, and an offer is the lowest price a seller is willing to accept. Among the securities traded OTC, both American Depositary Receipts (ADRs) and municipal securities would be included.

The term "disintermediation" refers to: A) the flow of money from traditional, low-yielding savings accounts to higher- yielding money market instruments. B) the flow of money from money market instruments into traditional savings vehicles. C) using margin to leverage one's returns. D) protecting a diversified portfolio through the purchase of index put options.

A) the flow of money from traditional, low-yielding savings accounts to higher- yielding money market instruments. This is the industry-accepted definition of disintermediation, and it typically occurs when the Federal Reserve Board tightens the money supply and interest rates rise faster in the marketplace than at bank accounts.

Investment company shareholders must receive financial reports at least semiannually. All of the following are true regarding these reports EXCEPT A) the report is regulated under FINRA's rules regarding communications with the public B) one of the semiannual reports must be audited C) a statement of all compensation paid to the board of directors (BOD) must be included D) a valuation of all securities in the IC portfolio as of the date of the balance sheet provided in the report must be included

A) the report is regulated under FINRA's rules regarding communications with the public The reports are required to be supplied to IC shareholders under the Investment Company Act of 1940. They are prepared and distributed by the investment companies. Unless forwarded by broker/dealers to their customers, the semiannual reports would not be regulated under FINRA's communication with the public rules for broker/dealers.

Your next-door neighbor approaches you with a proposed security offering, knowing that you are a registered representative with a large, affluent client base. If he asks you to present this investment opportunity to your clients, you must tell him that: A) you must first show the offering to your broker/dealer and receive permission to proceed with it. B) you will present the offering only to those clients whose investment objectives make it a suitable purchase for them. C) you may make recommendations on the offering only after you have done a due diligence determination. D) you will present the offering to your clients only after you have established that it is either registered or exempt from registration requirements.

A) you must first show the offering to your broker/dealer and receive permission to proceed with it. You must receive permission from your broker/dealer before you sell or offer for sale any security to your clients. To avoid a charge of selling away, you must also see to it that any sales that result are carried on your broker/dealer's books.

According to FINRA, the maximum sales charge on a variable annuity contract is:

An amount that is fair and reasonable Explanation: There is no statutory maximum sales charge on variable annuities or variable life insurance policies. The sales charges must be fair and reasonable.

A customer buys 1 ABC Jul 55 Call @6 and 1 ABC Jul 65 Put @6 on the same day. Just prior to expiration, the stock is trading at $63 and the customer closes the positions at intrinsic value. The customer has a net loss of: A. $200 B. $400 C. $800 D. $1,200

A. $200 The customer purchased a long combination, for a combined premium of $1,200. When the stock is at $63, the long 65 put is 2 points "in the money", resulting in a 2 point gain to the shareholder while the long 55 call is 8 points "in the money", resulting in an 8 point gain to the holder. $1,200 paid in premiums minus a net of $1,000 received = $200 loss.

A customer buys 100 shares of ABC stock at $64 and buys 1 ABC Oct 65 put @ 3. ABC stock falls to $58 and just prior to expiration, the customer exercises the put, delivering the stock position. The customer has a: A. $200 loss B. $300 loss C. $400 loss D. $700 loss

A. $200 loss The customer bought the stock at $64 and sold at $65 by exercising the put. There is a 1 point gain on the stock position. However, the customer lost the $300 premium paid, so the customer loses the net amount of $200.

On the same day in a margin account, a customer sells short 100 shares of ABC at $44 and buys 1 ABC Jan 45 call @ 2.50. The customer will break even at: A. $41.50 B. $42.50 C. $46.50 D. $47.50

A. $41.50 Short Stock/long call B/E = short sale price - premium

On the same day in a cash account, a customer buys 100 shares of PDQ stock at $49 and sells 1 PDQ Jan 50 Call @2. The breakeven point is: A. $47 B. $48 C. $51 D. $52

A. $47 Long stock/short call breakeven: Stock cost - premium

A customer, age 30, has an investment objective of capital appreciation; but does not need current income. The BEST asset allocation mix to recommend to this customer is: A. 100% common stocks B. 50% common stocks; 50% bonds C. 10% common stocks; 90% bonds D. 100% bonds

A. 100% common stocks A customer who has an objective of growth; and is not concerned with current income; would invest the majority of funds in common stocks; principally growth stocks. The risk associated with such a strategy is that either the stock market takes a dive; taking these stocks with it; or that some of these companies "lose their way" and perform poorly.

Which of the following investment portfolios is LEAST liquid? A. An aggressive growth fund B. A U.S. Government securities fund C. A money market fund D. An income fund

A. An aggressive growth fund Aggressive growth stocks are usually too small to be NYSE listed; they might be found on NASDAQ, which has lower listing standards than the NYSE; or on the OTCBB - the Over-the-Counter Bulletin Board - which has no listing standards. The OTCBB is a much less liquid market than NASDAQ; and NASDAQ's liquidity is not as good as the NYSE's. Thus, aggressive growth stocks would be the least liquid securities of the choices offered. Government securities and money market securities are actively traded and are extremely liquid. Income funds are composed of high yielding preferred stocks and bonds. These are more liquid than aggressive growth stocks, though not as liquid as NYSE listed issues.

"CAPM" stands for: A. Capital Asset Pricing Model B. Computer Assisted Portfolio Management C. Corporate Allocation Portfolio Mechanism D. Computer Algorithm Pricing Module

A. Capital Asset Pricing Model "CAPM" stands for Capital Asset Pricing Model. This is a methodology for finding the most efficient investments - those that give the greatest return for the amount of risk assumed.

Which of the following statements are TRUE regarding MSRB rules on gifts and gratuities? I The maximum business related gift that may be given by a municipal securities employee is $100 per person per year II The maximum business related gift that may be given by a municipal securities employee is $200 per person per year III Business related entertainment is permitted under the rules, as long as it is not too excessive or frequent IV Business related entertainment is prohibited under the rules A. I and III B. I and IV C. II and III D. II and IV

A. I and III

A Series 11 assistant representative license permits an individual to: I accept unsolicited customer orders II solicit customer orders III make recommendations to customers IV approve new accounts A. I only B. I and II only C. II and IV only D. I, II, III, and IV

A. I only

Common shares of which of the following issuers are likely to have a Beta coefficient much higher than +1? A. Semi-conductor manufacturer B. Pharmaceutical manufacturer C. Public utility D. Food processor

A. Semi-conductor manufacturer

Establishing the structure of a portfolio to meet specific financial goals is called: A. Strategic allocation B. Tactical allocation C. Rebalancing D. Risk adjustment

A. Strategic allocation

A customer wishes to buy stock in a closely held corporation with a small amount of outstanding shares. The registered representative should advise him:

About the risks inherent in buying thin issues because of the probability of wide price fluctuations and possible difficulties in selling the stock because of the small amount of outstanding shares Explanation: The registered representative should inform the customer of the risks involved in buying thin issues because of the probability of wide price fluctuations (volatility) and because of the small amount of the shares outstanding.

In a direct participation program, the point at which revenues begin to exceed deductions is known as:

The crossover point Explanation: The crossover point is reached when the project's revenues exceed expenses and net income is produced

A registered representative conducts a seminar about investing in the meeting room of a local apartment complex. At the end of the talk, he hands out his business card and tells the attendees that if they want additional information, please write their contact information on the reverse side of the business card and return it to him. When he gets back to the office and starts to re-contact some of the attendees who returned the business card, he finds that one of them is blocked because the client name is on the National Do Not Call Registry. Which statement is TRUE? A. This prospect can be called by the registered representative B. This prospect cannot be called by the registered representative C. This prospect can only be called by the registered representative between the hours of 8:00 AM and 9:00 PM D. This prospect can only be called by the registered representative with written approval of the Branch Office Manager

A. This prospect can be called by the registered representative There are 3 exceptions provided for cold calls to individuals that are on the National Do-Not-Call list. These are the: Established Business Relationship ("EBR") Exception; Prior Express Written Consent Exception; and Personal Relationship With The Associated Person Exception.

If the real Gross Domestic Product of the G-20 countries is growing at a slower rate than real Gross Domestic Product growth in the United States, then the value of the U.S. dollar can be expected to: A. appreciate B. depreciate C. fluctuate D. stagnate

A. appreciate

Industrial development bonds are issued by municipalities to build facilities that are leased to: A. corporations B. government agencies C. municipalities D. sovereign governments

A. corporations

In order to open a new account for a customer that wishes to buy IPOs, the: A. customer must sign a representation letter B. registered representative must sign a representation letter C. Branch manager must sign a representation letter D. all of the above

A. customer must sign a representation letter

Fidelity bonds are maintained by brokerage firms to protect loss due to: A. embezzlement by employees B. catastrophic events C. severe market volatility D. regulatory changes

A. embezzlement by employees FINRA requires brokerage firms to maintain fidelity bond coverage to protect against loss due to securities loss, employee theft, or embezzlement of funds.

Speculators in foreign currencies would NOT be subject to which of the following risks? A. interest rate risk B. exchange rate risk C. market risk D. political risk

A. interest rate risk

Which statement is TRUE about selling short against the box? A. it "locks-in" a gain on the stock B. it defers taxation of a gain C. it stretches short term capital gain to long term D. it coverts ordinary income into a long term capital gain

A. it "locks-in" a gain on the stock

A customer, age 60, is looking for an investment that will provide life-long income at retirement. The BEST recommendation would be for the customer to: A. purchase a variable annuity and annuitize the separate account at retirement. B. purchase a variable annuity and take installments of a designated amount at retirement. C. invest in an income mutual fund and elect not to automatically reinvest distributions D. invest in a GNMA fund since GNMAs make monthly payments.

A. purchase a variable annuity and annuitize the separate account at retirement. The benefit of an annuity contract to an older person is the assurance of receiving income for life - however, this only happens if the customer annuitizes the contract. If the customer chooses installments, there is no guarantee of payments for life - when the money in the account is depleted, payments stop.

An investor has a broadly diversified portfolio of blue chip stocks. The use of index options to hedge the portfolio reduces: A. systematic risk B. non-systematic risk C. interest rate risk D. timing risk

A. systematic risk Index options can be used to hedge a portfolio. If index puts are bought, then a drop in the market lowering the portfolio's value will be offset by a gain in the value of the index puts. This strategy hedges against market risk, also known as systematic risk. Non systematic risk is the risk that any one security will perform poorly. The larger the portfolio, the lower the effect of non-systematic risk. Timing risk is the risk that trades will not be performed at the best market prices; interest rate risk is the risk that interest rates rise, forcing bond prices and stock prices down.

A woman will be retiring in 2020. She is interested in income and having her principal available at retirement. Of the following municipal bonds, you would recommend a(n): a. Aaa-rated 8.5% G.O. maturing in 2017 b. Aa-rated 10% G.O. maturing in 2039 which is callable in 2018 at 105 c. Aa-rated 8.5% G.O. maturing in 2020 d. Ba-rated revenue bond maturing in 2023

Aa-rated 8.5% G.O. maturing in 2020 Explanation: Since the woman wants her principal available at retirement, a bond maturing in 2020 (the year of her retirement), regardless of the yield, would be the best choice

Under the Uniform Transfer to Minors Act (UTMA), how can stock subscription rights be handled in a custodial account?

The custodian can exercise or sell the rights as he deems prudent

Many life insurance companies offer variable products. Determining benefits usually depends on the actual performance of the selected separate account subaccount(s) compare to an assumed interest rate (AIR). Which of the following statements reflects that determination?

Actual performance compared to the AIR affects the death benefit of a variable life insurance policy Actual performance compared to the AIR affects the value of an annuity unit of a variable annuity

When an investor sells an interest in a limited partnership, her cost basis for tax purposes is the:

Adjusted basis Explanation: An investor's basis will be reduced by any claimed losses and any cash distributions. This reduced (adjusted) basis is the cost basis at the time of sale.

All of the following statements are TRUE concerning a municipal bond issue having a serial maturity, EXCEPT: a. All of the bonds mature on one date in the future b. The bonds are priced on a yield-to-maturity basis c. The issue has a decreasing outstanding principal d. The issue has decreasing total interest payments

All of the bonds mature on one date in the future Explanation: Serial bonds mature in successive years and are priced on a yield-to-maturity basis. As a serial issue nears its final maturity, the outstanding principal and total interest payments decrease. Term bonds mature at one date in the future and are priced at a dollar price (percentage of par).

A customer has a Roth IRA with total assets of $350,000. When the customer is 65, how much may be withdrawn from this account tax-free?

All of the funds Explanation: Unlike a traditional IRA, contributions to a Roth IRA are not tax-deductible, so the investor is contributing after-tax dollars. The advantage of the Roth IRA is that all the money accumulated in the account may be withdrawn tax-free after age 59 1/2 if the first contribution was made at least five years prior.

A company based in Europe with offices located in New Jersey would like to have its stock traded on the NYSE. This most likely will be accomplished through the issuance of:

American Depositary Receipts Explanation: American Depositary Receipts (ADRs) facilitate U.S. investment in the stock of foreign corporations. When the foreign securities are deposited in a U.S. bank based in that country, a receipt for those securities is issued and traded in the U.S. as if it were the foreign security itself.

Which of the following is contained in an official notice of sale?

Amount of good-faith deposit required with the bid

Under Rule 144A, a registered representative is NOT permitted to sell unregistered securities to which of the following?

An accredited investor Explanation: Under Rule 144A of the Securities Act of 1933, unregistered securities may be resold only to qualified institutional buyers (QIBs). Qualified institutional buyers are entities that have at least $100 million of investable assets. The term institution includes insurance companies, trust funds, investment advisers, investment companies, employee benefit plans, or other types of institutional investors. Individual investors, even if they are deemed to be accredited investors, are not considered to be QIBs.

Which of the following securities is NOT suitable for an investor with $80,000 who will need the funds in three months to purchase a house? a. A 13-week Treasury bill b. An auction rate preferred stock that resets its rate every three months c. A three-month CD that is yielding 20 basis points above the prime rate d. A money-market fund

An auction rate preferred stock that resets its rate every three months Explanation: An auction rate security is a long-term security that resets its interest rate periodically through an auction process. Since the client will need the funds in three months, any investment should be free from potential loss of principal. There is no guarantee he will be able to sell this security at the price at which it was purchased. The other three investments will offer this client very little or no principal risk.

Which of the following statements is TRUE regarding stock index options? a. The index is affected if a stock in the index should split b. All index options use the European style of exercise c. The shortest initial expiration is three months d. An exercise is settled by cash instead of the delivery of securities

An exercise is settled by cash instead of the delivery of securities Explanation: The exercise of a stock index option is settled by cash instead of the delivery of securities. An index will not be affected if one of its components should split. Some index options are American style (may be exercised any day up to expiration), while others use the European style (may only be exercised on the last trading day prior to expiration). Stock index options have a monthly expiration cycle.

To which of the following firms could a member grant concessions or other allowances?

Another member firm A foreign nonmember broker-dealer ineligible for FINRA membership (concessions)

All of the following investment company terms are synonymous EXCEPT: A. Bid B. Redemption Price C. Net Asset Value D. Offering Price

D. Offering price Bid, redemption price, and net asset value are all the same terms for mutual fund shares.

Moody's Investment Grade (MIG) Ratings are applied to: A) municipal bonds. B) corporate bonds. C) municipal notes. D) money market instruments.

C) municipal notes. Moody's Investment Grade Ratings are applied to municipal notes which are short-term municipal debts, such as bond anticipation notes (BANs).

Foreign currency spot transactions normally settle in:

Two business days Explanation: Settlement for foreign currency spot transactions is usually two business days.

An investor in an oil and gas limited partnership program is subject to the economic consequences of all of the following EXCEPT: A) recourse loans. B) depreciation on tangible assets. C) nonrecourse loans. D) operating losses.

C) nonrecourse loans. Nonrecourse loans only have economic consequences for investors in real estate programs.

Customer account statements must include the per-share estimated value of a direct participation program (DPP) or unlisted real estate investment trust (REIT) security held in the account. Which of the following does FINRA accept as an estimated value methodology?

Appraised value

An investor established the following positions. Long 100 shares of XYZ at $37 per share Long 1 XYZ 35 put at 1.75 This investor prefers XYZ to:

Appreciate significantly Explanation: The investor purchased the stock at $37 per share and protected it by purchasing the right to sell XYZ at $35, paying a 1.75 premium (long XYZ 35 put). The investor loses money if the stock falls below the breakeven price of $38.75 ($37 purchase price of the stock + the 1.75 premium on the purchase of the put). The investor prefers XYZ to appreciate significantly since there is the potential for unlimited gains.

A collateralized debt obligation (CDO) is BEST defined as a type of:

Asset-backed security Explanation: A collateralized debt obligation (CDO) is a type of asset-backed security. A CDO is issued as a bond, which is backed (collateralized) by a pool of bonds, loans, and various other assets. Ownership of this type of security is typically in the form of a tranche (slice), with any given tranche from the CDO carrying a different maturity and risk level. The return an investor can expect from this type of investment is based on the credit quality of the underlying assets contained in the pool. CDOs are similar in structure to collateralized mortgage obligations (CMOs). These investment vehicles are broadly categorized as asset-backed securities.

Your client is president of XYZ Corporation and is selling XYZ shares pursuant to Rule 144. A filing must be made with the SEC:

At the time of the sale Explanation: The filing must be made at the time of the sale and is effective for 90 days.

Your customer, a small-business owner, likes investments that are short term, relatively safe from credit risk, and liquid. He's heard that higher rates of return can be realized from auction rate securities than the rates he is currently getting on the Treasury bills in his portfolio. He asks you to explain them to him. Which of the following would you note as being reasons why they are not suitable for him?

Auction rate securities are intended as long-term investments If the auction fails, holders of ARSs may not have immediate access to their funds.

Authorized Stock

Authorized stock is the total number of shares that the company is "authorized" to sell.

CMOs are often quoted on a yield spread basis to similar maturity:

U.S. Government bonds

A customer purchased on margin 100 shares of ABC stock at 120 and sold short 100 shares of XYZ stock at 100. The customer also wrote an ABC 120 call @ 3 and an XYZ 100 put @ 2. What is the margin requirement for the combined transactions? A) $10,500 B) $11,000 C) $11,500 D) $22,000

B) $11,000 The FRB margin requirement for the purchase or short sale of stock is 50%. Therefore, the margin requirement for the stock purchase is $6,000 (50% of $12,000) and for the short sale is $5,000 (50% of $10,000). The call is covered since the customer owns the underlying stock and the put is covered since the customer is short the underlying stock. Since there is no margin requirement for a covered call or put, the total margin requirement is $11,000. If the question had asked for the cash deposit, subtract the total premiums received ($500) from the margin requirement of $11,000.

In a new margin account, a customer buys 300 shares of ABC at $40 per share, 100 shares of the Ajax Mutual Fund at $24, and 10 PDQ Aug 30 calls at 4. The customer will receive a margin call for: A) 9200. B) 12400. C) 10800. D) 4400

B) $12400 The customer must pay 50% of the value of the stock, and 100% of the value of the mutual fund shares and the options because these securities are nonmarginable (NMS) and must be paid for in full (50% of $12,000 = $6,000). To calculate the total payment required, add $6,000 (stock) plus $2,400 (Ajax) plus $4,000 (PDQ calls) which equals $12,400.

A client has a margin account with the following positions: short 2,000 shares of EXA at $22 and long 40 EXA convertible bonds at $1,150 that are convertible at $20. If the client is using the convertible bonds as a hedge, the maintenance requirement is: A) $4,400 B) $4,600 C) $11,500 D) $13,200

B) $4,600 If a client is long a security that is convertible into an equal number of shares of a short position carried by the same client, the maintenance requirement is 10% of the current market value of the long position. This is an industry rule, not a Regulation T requirement. Each bond is convertible into 50 shares (the par value of $1,000 divided by the conversion price of $20). The client may convert the 40 bonds into a total of 2,000 shares (50 shares x 40 bonds), which is equal to the number of shares the client is short. The maintenance requirement is 10% of the long position, which is equal to $4,600 ($1,150 x 40 bonds x 10%).

A customer has an existing margin account with equity of $36,000. If the FRB requirement is 50% and the customer sells short 100 shares at $15.00 a share, the required deposit is: A) $450 B) $750 C) $1,500 D) $2,000

B) $750 The FRB requirement is 50%, which is the same for purchases as it is for short sales. Therefore, the required deposit is $750 (50% x $1,500). Since this trade is executed in an existing account, the only requirement is the FRB's. If this had been the initial trade in the account, the required deposit under industry rules would be $2,000.

How many business days after an index option is exercised should a cash settlement occur? A) 2. B) 1. C) 3. D) 5

B) 1. Exercised stock index options settle on the next business day.

Gargantuan Computers, Inc. (GCI) conducts a rights offering to its current shareholders at $50 per share, plus 1 right. If the current market price of GCI is $70, what is the value of one right before the stock trades ex-rights? A) 15 B) 10 C) 5 D) 3

B) 10 The stock is trading cum rights (before the ex-date). The formula to calculate the value of one right before the ex-date is follows: CMV − subscription price / Number of rights to purchase 1 share + 1. Therefore one right is valued at $10, computed as ($70 − $50) / 2 = $10

The longest initial maturity for U.S. T-bills is: A) 2 years. B) 26 weeks. C) 13 weeks. D) 39 weeks.

B) 26 weeks. Maximum initial maturity for T-bills is subject to change. Though T-bills have been issued in 1 yr (52 weeks) maturities, historically the longest initial maturity of T-bills has been 6 months (26 weeks); the shortest initial maturity is 4 weeks.

Under FINRA rules, delivery on a seller's option can be made no sooner than the: A) 5th business day following the transaction. B) 4th business day following the transaction. C) 3rd business day following the transaction. D) day following the trade date.

B) 4th business day following the transaction. Seller's option trades, under FINRA rules, allow a seller to settle later than a regular-way settlement. With T + 3 regular-way settlements, a seller's option would settle no earlier than 4 business days after the trade date. If the seller could deliver any earlier in compliance with regular-way, there would be no need for a seller's option contract.

In a new margin account, if a customer buys 300 shares of XYZ for 48 and simultaneously writes 3 XYZ Jan 50 calls at 1, the margin call will be for: A) 7500. B) 6900. C) 7200. D) 7350.

B) 6900. The Regulation T requirement is $7,200 (50% × $14,400). There is no Regulation T requirement for writing covered calls. The requirement to establish both positions is $7,200. However, the question asked for the margin call (a margin deposit). The requirement of $7,200 is reduced by the premium income received ($300). By depositing $6,900, the customer will have $7,200 in the account, the difference being the premium income credited to the account on settlement date.

Which statement is FALSE about OATS? A. Orders for NASDAQ-listed issues are entered into OATS B. Orders for OTCBB issues are entered into OATS C. Order for pink sheet issues are entered into OATS D. Orders for CBOE-listed options are entered into OATS

D. Orders for CBOE-listed options are entered into OATS. This is FALSE.

Which of the following permits the highest annual contributions? A) A traditional spousal IRA for which the contribution has been deducted. B) A SEP IRA. C) A Coverdell Education Savings Account. D) A traditional nondeductible IRA.

B) A SEP IRA. Under most circumstances, the annual contribution to a SEP IRA will be higher than those allowed for ESAs or traditional or Roth IRAs.

Which of the following persons may legally open an account to trade on margin? A) A minor child with approval of a court-appointed guardian. B) A corporation. C) A custodian of an UTMA account. D) An open-end investment company.

B) A corporation. A corporation may open an account to trade on margin if provided for in the charter and authorized in the bylaws. Both UTMA and UGMA specifically prohibit custodians from either engaging in speculative trading or borrowing money or securities in the name of the minor through trading on margin. Mutual funds are also prohibited from trading on margin.

Which of the following would be most likely to require a mandatory sinking fund? A) A GO. B) A water and sewer revenue bond. C) A TAN. D) A PHA.

B) A water and sewer revenue bond. Sinking funds force revenue bond issuers to set aside a portion of their revenue for debt retirement.

Which of the following securities has the LEAST amount of capital risk? A) Options B) Bonds C) Warrants D) Stocks

B) Bonds Capital risk is the risk of an investor losing her principal, the amount of funds invested in a security. When compared to the other securities, bonds have the least amount of capital risk. At maturity, the investor would receive the principal amount of the bond, thus minimizing the capital risk.

Your customer is interested in long-term corporate bonds. Which of the following interest-rate environments makes a call protection feature most valuable to your customer? A) Stable interest rates B) Declining interest rates C) Rising interest rates D) Volatile interest rates

B) Declining interest rates A call protection feature is an advantage to bondholders in periods of declining interest rates. When interest rates are falling, issuers are more likely to call in bonds previously issued at higher interest rates. For bondholders this creates reinvestment risk for them when the bonds are called as they are unlikely to be able to reinvest at the rate they had been earning. Call protection gives the bond holder a specified length of time during which the bond cannot be called.

Which of the following is an effect of advance refunding on a municipal bond issue? A) Cancellation. B) Defeasement. C) Lowered rating. D) Redemption.

B) Defeasement. Advance refunding (or pre-refunding) is in part the establishment of an escrow account in connection with an outstanding bond issue. Funds from a subsequent issue, deposited in the escrow account and generally invested in Treasury securities, will be used to call the bonds on the first available call date. When the bonds are escrowed to maturity, the outstanding debt is considered defeased.

Which of the following industries is most likely to be considered cyclical? A) Food. B) Durable goods. C) Utilities. D) Pharmaceutical.

B) Durable goods. The production of durable goods depends on whether the economy is in an expansion or a contraction phase. Pharmaceuticals, utilities, and food are always necessary.

If a corporation begins a nonqualified retirement plan, which of the following statements is TRUE? A) The employer must abide by all ERISA requirements. B) Employee contributions grow tax deferred if they are invested in an annuity. C) Employee contributions are tax deductible. D) Employer contributions are tax deductible.

B) Employee contributions grow tax deferred if they are invested in an annuity. Earnings accumulate tax deferred if the plan is funded by an investment vehicle that offers tax deferral, such as an annuity contract. Tax has been paid on all amounts the employees and the employer contribute to the plan. Nonqualified plans need not comply with all ERISA requirements.

Interest paid on I bonds is: A) Taxable at all levels. B) Exempt from state and local taxation. C) Exempt at all levels. D) Exempt at the federal level only.

B) Exempt from state and local taxation. Interest paid on I bonds is exempt from state and local taxes but is taxable at the federal level. Although, federal income taxes can be deferred for up to 30 years or until the bonds are redeemed, whichever comes first.

Two customers in their twenties, married only a few years, should select which investment for their IRAs? A) Oil and gas exploration limited partnerships. B) Growth-oriented mutual funds. C) High-tech funds. D) High yield bond funds.

B) Growth-oriented mutual funds. A growth mutual fund may be appropriate for a young couple's IRA account; all other selections incur high risk that is not appropriate for a retirement account.

The Nasdaq market includes securities in the: I. Global select market. II. Global market. III. Bulletin Board listings. IV. Electronic OTC Pink listings. A) II and III. B) I and II. C) III and IV. D) I and IV.

B) I and II. The Nasdaq Market includes securities in 3 market tiers; global select, global, and capital markets. Bulletin Board securities (OTCBB) are issues that are not listed on Nasdaq or any U.S. exchange. OTC Pink issues are unlisted and, like Bulletin Board stocks, are referred to as non-Nasdaq.

A customer purchases a 4% corporate bond yielding 5%. A year before the bond matures, new corporate bonds are issued at 3%, and the customer sells the 4% bond. Which of the following statements regarding the bond are TRUE? I. The customer bought it at a discount. II. The customer bought it at a premium. III. The customer sold it at a premium. IV. The customer sold it at a discount. A) II and III. B) I and III. C) II and IV. D) I and IV

B) I and III. A 4% bond yielding 5% has a YTM of 5%. If a bond's yield to maturity is higher than its coupon (or nominal yield), the bond is selling at a discount to par. If interest rates decline, new bonds will be issued with lower coupons, and outstanding bonds with higher coupons will trade at a premium.

Which of the following statements regarding nonsystematic risk are TRUE? I. It is the risk that an individual stock will not perform well. II. It is the same as market risk. III. Diversification reduces it. IV. Diversification does not reduce it. A) I and IV. B) I and III. C) II and IV. D) II and III.

B) I and III. Nonsystematic risk is company risk, the risk that an individual investment will perform poorly. Diversification can reduce most nonsystematic risks.

With ABC trading at 39, a customer buys 1 ABC March 40 call and sells 1 ABC March 35 call. A profit occurs if: I. the spread widens. II. the spread narrows. III. ABC declines sharply. IV. both contracts are exercised. A) II and IV. B) II and III. C) III and IV. D) I and III.

B) II and III. This investor established a credit spread because the premium he received for the 35 call is more than he will pay for the 40 call; a call with a lower strike always carries a higher premium. As a general statement, credit spreads are bearish, and are profitable if the spread narrows between the premiums or the contracts expire unexercised (this will happen if the stock falls).

When evaluating numerous mutual funds, what is meant by net investment income? A) Interest only B) Dividends only C) Interest + dividends - expenses D) Dividends + capital gains - expenses

C) Interest + dividends - expenses Net investment income of a mutual fund is derived from the total interest plus dividends earned by the fund's portfolio minus the expenses of the fund.

An investor purchased an interest in a limited partnership, paying $10,000 in cash and signing a recourse note to the partnership under a letter of credit for $40,000. Which of the following statements are TRUE? I. The investor's tax basis will be $10,000. II. The investor's tax basis will be $50,000. III. The investor's maximum loss will be $10,000. IV. The investor's maximum loss will be $50,000. A) I and IV. B) II and IV. C) I and III. D) II and III.

B) II and IV. A recourse note means that the limited partner agrees to pay the note no matter what happens. He is legally liable for the $40,000, which makes both his tax basis and maximum loss potential $50,000.

A 45-year-old investor takes a lump-sum distribution from a nonqualified variable annuity. How is the distribution taxed? I. The entire amount is taxed as ordinary income. II. The growth portion is taxed as ordinary income. III. The growth portion is taxed as a capital gain. IV. The growth portion is subject to a 10% penalty. A) I and IV. B) II and IV. C) II and III. D) III and IV

B) II and IV. On withdrawals from a nonqualified annuity, taxes are paid only on the amount that exceeds cost basis (the amount paid into the annuity). In this case, the investor is taking a lump-sum distribution before reaching age 59-½ and must pay an additional 10% penalty on the taxable amount.

Your client is interested in a direct participation program (DPP) limited partnership. Which of the following two are most likely to factor into a discussion on suitability of such an investment? I. Beta. II. Liquidity. III. Duration. IV. Age. A) II and III. B) II and IV. C) I and IV. D) I and III.

B) II and IV. The key here is to recognize that with DPPs, the customer's age is a relevant consideration in determining suitability. DPPs are long-term and illiquid. For example, it is unlikely that DPPs would be suitable for a customer near retirement age, regardless of the customer's financial situation. Beta, having to do with measuring an investment's volatility as related to the overall market, and duration having to with bonds are not factors that would be associated with DPPs.

Which of the following retirement plans is NOT legally required to establish vesting, funding, and eligibility requirements? A) Keogh plan. B) Payroll deduction plan. C) Profit-sharing plan. D) Defined benefit pension plan.

B) Payroll deduction plan. A payroll deduction plan is a retirement plan not subject to eligibility, vesting, or funding standards as required by ERISA plans. A payroll deduction plan is a nonqualified retirement plan. Profit-sharing, pension, and Keogh plans must have established standards.

A client owns shares of stock purchased at $46 a share. If the current market price is now $70 and the client wants to protect her profit if the price should fall 10%, the RR should recommend which of the following orders? A) A market order B) Sell stop $63 C) Sell limit $63 D) Sell stop-limit $63

B) Sell stop $63 Explanation:This client only wants to sell her position if the stock declines by 10% or $7.00. The RR should recommend a sell stop at $63. A market order is not suitable since the client does not want to sell unless the price declines. A market order will not allow the client to receive further profits if the stock increases above $70. A sell limit is an order to sell at a specified price or higher and is usually placed above the current market price. Therefore, a sell limit at $63 is not suitable. Since the client never mentioned a specific limit selling price she is willing to accept, a stop limit order should not be recommended. In addition, a stop limit order may be activated but never executed, and the client would not be able to protect her profit

Which of the following underwriting arrangements allows an issuer whose stock is already publicly traded to structure the timing of sales for an additional issue? A) Negotiated. B) Shelf. C) Standby. D) Competitive.

B) Shelf. A shelf registration with the SEC allows an issuer to sell the registered securities for up to 3 years from the effective date. This allows an issuer to time its sales with market conditions.

Sally is the named beneficiary of her grandmother's IRA. After the death of her grandmother who was 80 years old, what would Sally's options be regarding the IRA? A) Wait until age 59½ to begin taking distributions, to avoid the 10% tax penalty. B) Take minimum required distributions based on Sally's life expectancy. C) Roll over her grandmother's money into Sally's own IRA. D) Wait until age 70½, to maximize tax deferral, and then begin required minimum distributions.

B) Take minimum required distributions based on Sally's life expectancy Before age 80, Sally's grandmother would have already begun mandatory distributions. When someone inherits an IRA for which the initial owner has begun mandatory distributions, payout must continue but is now based on the life expectancy of the new owner.

Tenants in common ownership provides that a deceased tenant's fractional interest in an account is retained by which of the following? A) The surviving tenant. B) The deceased tenant's estate. C) Will be decided during probate. D) The registered representative for the account.

B) The deceased tenant's estate TIC (tenants in common) ownership of an account provides that a deceased tenant's interest in an account is retained by that tenant's estate and not passed on to the surviving tenant.

Nonmembers of a syndicate who are assisting in its sale of bonds buy the bonds at a discount called: A) the basis price. B) a concession. C) a net designated price. D) a takedown.

B) a concession. Members of the syndicate buy the bonds at the offering price minus the takedown, nonmembers buy at offering price minus a concession. The basis price is the yield to maturity.

The prospectus of the ABC Fund contains the phrase "will have at least one-quarter of common stock investments in the field of business machines." The ABC Fund is: A) a balanced fund. B) a specialized fund. C) a diversified fund. D) a growth and income fund.

B) a specialized fund. A fund that, as part of its investment policy, makes a commitment to invest 25% or more of its assets into a particular economic or geographical sector is a specialized fund. A balanced fund invests in a balance of bonds and common and preferred stocks. A diversified fund does not invest more than 5% of the fund's assets in any one issuer. A growth and income fund may invest in many industries, seeking both dividends and capital gains.

A customer buys a new issue municipal bond at a discount. If held to maturity, the amount of the discount is: A) accreted and taxed as ordinary income. B) accreted and is not taxed. C) taxed as a long-term capital gain. D) taxed as a short-term capital gain.

B) accreted and is not taxed. Original issue discounts are accreted, which allows for a step-up in cost basis. Accretion on original issue discount municipal bonds is not taxed.

A gain on the sale of a long equity put option is: A) a short- or long-term capital gain. B) always a short-term capital gain. C) ordinary income. D) always a long-term capital gain.

B) always a short-term capital gain. Any trading in options produces only short-term gains or losses; therefore any gain on the sale of a long put option must always be a short-term capital gain. (If a question wishes you to consider LEAPS, the question will refer to them.)

Continuing commissions in connection with the sale of investment company securities: A) are a form of deferred compensation; therefore, when a registered representative resigns, the registered representative must be paid all commissions due. B) may be paid to a retired employee provided a bona fide contract calling for such payment was entered into by the registered representative while employed by a member. C) is illegal. D) must be paid by a member whether or not the person receiving the commissions is a registered representative of a member

B) may be paid to a retired employee provided a bona fide contract calling for such payment was entered into by the registered representative while employed by a member. FINRA rules permit the payment of continuing commissions to retired registered representatives in connection with the sale of investment company securities if a bona fide contract to do so exists between the firm and the registered representative.

A limited partnership brought to market through a private placement may be sold to all of the following EXCEPT: A) 35 unaccredited investors. B) an unlimited number of unaccredited investors. C) an unlimited number of accredited investors. D) an investor with over $1 million net worth

B) an unlimited number of unaccredited investors. The primary sale of a limited partnership through a private placement is covered by Regulation D. Sales of the issue may be made to 35 individuals, which need not meet any financial standards. If, however, sales are made to more than 35 individuals, any other purchasers must meet certain standards of financial accreditation, known as accredited investors. An accredited investor would include an investor with $1 million or more in net worth not including net equity in a primary residence, or an individual who has earnings of $200,000 in the current year and $200,000 in the previous 2 years, or is an officer or insider of the offering. Also, any large financial institution, such as a bank, an insurance company, a savings and loan, etc., would be considered an accredited investor.

A buy stop order is elected (triggered) when the underlying stock trades A) at or below the stop price B) at or above the stop price C) through the stop price only D) anywhere below the stop price

B) at or above the stop price A buy stop order is placed above the prevailing market price and is elected (triggered), becoming a market order to buy when the stock trades at or through (above) the stop price.

The demand is far less than anticipated for a new issue of common stock. In this situation, the underwriter may stabilize the issue by placing bids in the open market: A) at or anywhere above the public offering price. B) at or slightly below the public offering price. C) slightly above the public offering price. D) at the public offering price only

B) at or slightly below the public offering price. When demand for a new issue is less than anticipated, thus causing the price to fall, an underwriter may place a stabilizing bid at or just below the public offering price.

A member firm's customer is requesting that IRA contributions converted from a traditional IRA to a Roth IRA now be moved back to a traditional IRA. This is A) never allowed under any circumstances B) called a re-characterization and is allowed by the IRS so long as certain requirements are met C) called a re-characterization and is permitted under all circumstances and within any time frame D) called a rollover and allowed by the IRS as long all requirements are met

B) called a re-characterization and is allowed by the IRS so long as certain requirements are met The IRS allows an individual to re-characterize contributions made to one type of IRA as if they had been made to another type of IRA as long as the requirements as to when the re-characterization can occur have been met.

The antifraud provisions of the Securities Exchange Act of 1934 apply to all of the following EXCEPT A) municipal bonds B) commodities C) options D) Nasdaq and exchange listed securities

B) commodities All securities are subject to the antifraud provisions of federal securities law. It should be recognized that commodities such as wheat or oil are not securities.

Each of the following is an example of a qualified retirement plan EXCEPT a: A) profit-sharing plan. B) deferred compensation plan. C) Keogh plan. D) defined benefit plan.

B) deferred compensation plan. A deferred compensation plan is considered a nonqualified plan because IRS approval is not required to initiate such a plan for employees.

Short against the box is a strategy associated with: A) avoiding the locate requirement when selling stock short. B) deferring capital gains. C) designating one tax ID number on a joint account for tax purposes. D) hedging short stock positions with options.

B) deferring capital gains. Selling short against the box is a strategy that is used to lock in a capital gain that will be deferred into a later tax period. For the deferral to be allowed, the IRS requires certain criteria to be met.

The manager will credit each syndicate member based on sales of that particular maturity allotted to the member, and such credits shall extinguish liability based only on such securities that are sold by the member. This statement describes an agreement among underwriters that is a(n): A) Eastern account. B) divided account. C) proportionate underwriting. D) undivided account.

B) divided account. This is part of an agreement for a Western (divided) syndicate

Regulation T permits borrowing money for the purchase of each of the following EXCEPT: A) listed warrants. B) listed options with expirations of less than 9 months. C) unlisted stocks and bonds. D) listed stocks and bonds.

B) listed options with expirations of less than 9 months. Options with expirations of less than 9 months must be fully paid without exception. With some exceptions, warrants, stocks, and bonds may be purchased on margin.

Establishing short positions is typical for all of the following EXCEPT: A) listed stock. B) municipal bonds. C) OTC common stock. D) preferred stock.

B) municipal bonds. Even though there is no regulation that prohibits short sales of municipal bonds, this is rarely done. To short a security, it must be borrowed and later covered. The general illiquidity of the municipal market makes this difficult.

Which of the following option positions obligates the investor to sell shares if exercised? a. Long a call b. Long a put c. Short a call d. Short a put

c. Short a call Explanation: A short call position obligates the investor to sell shares if the option is exercised.

Long an ABC Apr 60 call and short an ABC Apr 70 call is a: A) calendar spread. B) net debit spread. C) net credit spread. D) straddle

B) net debit spread. This is a vertical spread, not a calendar spread. To determine whether it is a net credit or debit, look at the strike prices. For call options with the same expiry month, the lower strike price will always have a higher value. In this case, the investor is long the higher valued option, which gives a net outflow of cash to enter the entire position (more money was spent on the lower strike price call than received for the higher strike price call). Therefore, the investor has a net debit for his account.

An order memorandum or ticket must be completed: A) by the close of business on T + 1. B) prior to order execution. C) by the close of business or trade date. D) by settlement date.

B) prior to order execution. Order tickets must be prepared prior to order execution.

Distributions from nonqualified variable annuities are: A) taxed as ordinary income. B) taxed as ordinary income only to the extent of earnings. C) tax free. D) taxed at a reduced rate.

B) taxed as ordinary income only to the extent of earnings. As contributions are made with after-tax dollars, only the earnings generated are taxed on withdrawal.

All of the following statements regarding a municipality's debt limit are true EXCEPT that: A) the purpose of debt limits is to protect taxpayers from excessive taxes. B) the debt limit is the maximum amount a municipality can borrow in any one year. C) revenue bonds are not affected by statutory limitations. D) unlimited GO bonds may be issued when a community's taxing power is not restricted by statutory provisions.

B) the debt limit is the maximum amount a municipality can borrow in any one year. The debt limit is the maximum amount of debt a municipality can have outstanding.

Variable annuity salespeople must register with all of the following EXCEPT: A) the SEC. B) the state banking commission. C) FINRA. D) the state insurance department.

B) the state banking commission. Variable annuity salespeople must be registered with FINRA and the state insurance department. Registration with FINRA is de facto registration with the SEC; no registration is required by the state banking commission.

An investor has purchased a municipal certificate of participation (COP). COPs can be characterized by all of the following EXCEPT A) they are a form of municipal revenue bond B) they would require voter approval before a municipality could issue them C) the holder of a COP could foreclose on the asset generating the revenue in the case of default D) the holder of the COP participates in lease or loan payments from a specific piece of equipment or facility purchased or built by the municipality

B) they would require voter approval before a municipality could issue them Certificates of participation (COPs) are considered revenue issues and, therefore, do not require voter approval. They are a form of lease revenue bond that allow the holders of the certificates to participate in some revenue stream (lease or loan payments) associated with land, equipment, or facilities purchased or built by the municipality. They are unique in that in the case of default, the holders of the COPs could foreclose on the asset associated with the certificate.

When a customer enters a sell order and the customer is in possession of the certificates, a broker/dealer must determine all of the following EXCEPT: A) the location of the securities. B) whether the transfer agent has accepted the securities. C) whether the securities are in deliverable form. D) whether the client can make delivery promptly .

B) whether the transfer agent has accepted the securities. A firm must make an affirmative determination and be reasonably sure the client can make prompt delivery. Conversely, approval of the transfer agent is not a factor when accepting a sell order.

Treasury STRIPS and Treasury receipts are quoted based on A) 0.03125 (1/32 of a point in dollars) B) yield to maturity C) amortization of premiums D) 0.125 (1/8 of a point in dollars)

B) yield to maturity Noninterest-bearing securities, like zeroes, are quoted based on their yield to maturity. They are sold at a discount and mature at par.

The designated Registered Options Principal is responsible for all of the following functions EXCEPT: A. approval of options advertising B. approval of options accounts C. writing of procedures for supervision of options accounts D. review of procedures for supervision of options accounts

B. approval of options accounts

If a technical analyst sees stock prices drop repeatedly to a certain level, from which they start rising again, then a market: A. "bottom" has been formed and stock prices can be expected to fall over the long-term B. "bottom" has been formed and stock prices can be expected to rise over the long-term C. "top" has been formed and stock prices can be expected to fall over the long-term D. "top" has been formed and stock prices can be expected to rise over the long-term

B. "bottom" has been formed and stock prices can be expected to rise over the long-term

A customer purchases $100,000 of corporate bonds at 80% in a margin account. The customer must deposit: A. $7,000 B. $16,000 C. $20,000 D. $40,000

B. $16,000 FINRA requirement is the greater 7% of face amount OR 20% of market value. The bonds are purchased at 80% of $100,000 par = $80,000. 20% of $80,000 = $16,000. 7% of $100,000 is $7,000. The greater amount is $16,000.

A waste disposal revenue bond issue is being underwritten on a negotiated basis. The offering consists of $10,000,000 par value of term bonds. The underwriter has agreed to a spread of $40.00 for each $5,000 bond. The manager has set the additional takedown at $15.00 per bond and the selling concession at $20.00 per bond. If a selling group member sells a $5,000 par value bond directly to the public, the selling group member earns: A. $5.00 B. $20.00 C. $35.00 D. $40.00

B. $20.00 When selling a bond directly to the public, a selling group member earns the selling concession of $20.00

A customer margin account shows: 100 shares of ABC @ 50 200 shares of DEF @ 60 200 shares of PDQ @ 75 Debit = $17,000 SMA = $5,000 Reg. T= 50% If the customer takes the SMA as cash, the account will be at which margin percentage? A. 25% B. 31% C. 40% D. 45%

B. 31% Long Market Value - Debit Balance = Equity 32,000 - 22,000 = 10,000

Trading in the Interbank market will affect all of the following directly EXCEPT: A. foreign currency prices in terms of U.S. dollars B. American Depositary Receipt prices in terms of U.S. dollars C. future economic growth D. future trade deficit or surplus figures

B. American Depositary Receipt prices in terms of U.S. dollars

A customer wishes to open an account to fund payment of private middle school tuition. If the customer does not wish to deposit more than $2,000 per year and wishes to get a tax benefit, the best advice is for the customer to open a: A. 529 Plan B. Coverdell ESA C. Trust account D. UTMA account

B. Coverdell ESA

All of the following are required to sell "144" stock EXCEPT: A. seller's representation letter B. buyer's representation letter C. issuer's representation letter D. broker's representation letter

B. buyer's representation letter

Under SEC rules, a customer short sale on an exchange floor can be executed on which of the following? I. Plus tick. II. Zero-plus tick. III. Minus tick. IV. Zero-minus tick. A) I and III. B) I and II. C) II and IV. D) I, II, III and IV.

D) I, II, III and IV. On an exchange floor, a customer short sale can be executed at any time in the trade sequence.

A corporation buys furniture and fixtures, paying cash. Which of the following choices are affected? I Current Assets II Current Liabilities III Net Worth IV Net Working Capital A. I and III B. I and IV C. II and III D. I, II, III, IV

B. I and IV If furniture is bought with cash, then cash goes down (a current asset) and property, plant and equipment increases (a long term asset). If current assets drop, then working capital drops. There is no effect on current liabilities because the furniture is paid for; net worth is only affected by a profit, loss, dividend payout, or capital structure change.

Prior to the opening of the options exchange, an investor wishes to place an order to sell an option contract at a premium that is higher than the previous day's close. The order type to be placed is a(n): A. at the open order B. Limit order C. stop order D. Not held order

B. Limit order The orders that are placed higher than the current market are "OSLOBS" - open sell limits open buy stops. Thus, to sell at a higher price than the current market, an open sell limit would be placed.

A 79-year old customer in the highest tax bracket with $1,000,000 to invest is risk averse. Which investment recommendation would be appropriate? A. Money market funds B. Municipal bonds C. A Dow Jones Industrial Average index fund D. Certificates of deposit

B. Municipal bonds Since this customer is in the highest tax bracket, and appears to be wealthy (with $1,000,000 to invest), tax-free municipal bonds are the best recommendation.

There is no real benefit for the manager of a pension plan to invest in which of the following securities? A. Corporate Bonds B. Municipal Bonds C. Government Bonds D. Government Agency Bonds

B. Municipal bonds Pension plans are "tax qualified" retirement plans. Earnings on securities held are tax deferred; so there is no benefit to investing in municipals, which have lower rates because their interest income is exempt from Federal corporate tax.

Which of the following is NOT defined as an "Investment Company" under the Investment Company Act of 1940? A. Face Amount Certificate Company B. Real Estate Investment Trust C. Management Company D. Unit Investment Trust

B. Real Estate Investment Trust

A customer sells stock out of his account receiving net proceeds of $28,000. The customer wishes to use the proceeds to buy ACME mutual fund shares. The fund has breakpoints of $5,000 intervals. The customer has no additional funds available for investment. You should recommend that the customer: A. reinvest $25,000 to ACME fund, and use the remaining $3,000 to buy common stocks. B. Sign a letter of intent to buy $30,000 of ACME fund C. invest an additional $2,000 in ACME fund in addition to the proceeds from the stock sale. D. refrain from investing in ACME fund until $30,000 has been accumulated

B. Sign a letter of intent to buy $30,000 of ACME fund

To handle increased trading volume in its stocks, NYSE introduced the: A. NASDAQ system B. Super Display Book system C. Order Support System D. Specialist System

B. Super Display book system

Which statement is TRUE concerning "wrap accounts"? A. Wrap accounts must be registered with the SEC and sold with a prospectus B. To sell a wrap account, the registered representative must also be licensed as an investment adviser representative by the State C. To sell a wrap account, the registered representative must also be licensed to sell insurance products by the State D. Wrap accounts cannot be sold by registered representatives

B. To sell a wrap account, the registered representative must also be licensed as an investment adviser representative by the State

A 70-year old client wants to invest in U.S. Treasury securities. When performing the suitability determination, the client informs the registered representative that he is looking for after-tax income, liquidity, and to avoid market risk. The registered representative should be LEAST concerned with the: A. client's tax bracket B. client's age C. coupon of recommended Treasury securities D. maturity of recommended Treasury securities

B. client's age Since Treasury securities are the safest security, they are an appropriate recommendation for a 70-year old client. So age really is not a concern with this recommendation.

The "effective" Federal Funds Rate is the: A. daily average rate of all commercial banks B. daily average rate of member banks of the Reserve System C. weekly average rate of all commercial banks D. weekly average rate of member banks of the reserve system

B. daily average rate of member banks of the Reserve System

The Bank of England is concerned that the British Pound is weakening against the U.S. Dollar. A method for the bank of England to strengthen its currency would be to: A. raise British interest rate levels B. lower British interest rate levels C. raise U.S. interest rate levels D. lower U.S. interest rate levels

B. lower British interest rate levels It can do nothing about U.S. interest rate levels.

A registered representative at a municipal securities firm receives a written complaint from a customer regarding a purchase of municipal bonds. The registered representative should: A. attempt to resolve the complaint prior to taking any other action. B. notify the municipal securities principal prior to taking any action. C.notify the municipal securities rulemaking board prior to taking any action. D. notify the SEC prior to taking any action

B. notify the municipal securities principal prior to taking any action. Written customer complaints should be resolved. The MSRB requires that every customer complaint be given to the principal for review and resolution.

In a firm commitment underwriting, the underwriter is acting as a(n): A. agent B. principal C. intermediary D. specialist (DMM)

B. principal

The "right of reclamation" in a municipal bond sale refers to the: A. refusal by municipal dealer to accept the delivery of bonds tendered to that firm by another municipal securities dealer. B. return of municipal securities that have been previously accepted on a delivery C. procedure where a municipal dealer that bought securities, but has not yet received them, can close-out the trade D. if the dealer does not have the monies, the delivery may be rejected

B. return of municipal securities that have been previously accepted on a delivery

A password protected website maintained by a member firm is considered to be: A. advertising B. sales literature C. public forum D. correspondence

B. sales literature FINRA defines communications with the public as either: Correspondence: A communication made available to 25 or fewer existing or prospective retail clients Retail Communication: A communication made available to more than 25 existing or prospective retail clients

At which Standard and Poor's rating is a bond considered to be speculative ("junk bond")?

BB The top 4 ratings are "investment grade" - AAA, AA, A, and BBB. Bonds below these ratings are speculative. The best speculative rating is, therefore, BB.

The lowest investment grade rating is:

Baa A bond's rating becomes speculative when it falls BELOW BBB or Baa rating, so these are the lowest investment grades.

Which of the following is part of the Federal Farm Credit System (FFCS)?

Bank for Cooperatives Federal Land Bank Federal Intermediate Credit Bank

Roundville Bank is considering an investment in Roundville County bonds. The bonds contain a provision that permits banks to deduct 80% of the interest cost being paid to depositors on the funds used to purchase the bonds. These securities are known as:

Bank-qualified bonds Explanation: Bank-qualified municipal bonds allow banks to deduct 80% of the interest cost paid to depositors on the funds used to purchase the bonds. This is done to encourage banks to invest in municipal securities. To qualify, a municipality may only issue up to $10,000,000 annually.

When the underlying common stock sells ex-dividend, a GTC buy limit order will:

Be reduced Explanation: All GTC (good-until-cancelled) orders entered below the current market are automatically reduced by the amount of the dividend on the ex-dividend date (unless they are market DNR -- Do Not Reduce). A buy limit order is entered below the current market and is reduced.

Super Entertainment Inc., a publicly traded firm on the NYSE, spins off its domestic syndication division, creating 1,000,000 new shares. To receive the new shares, investors must exchange 25% of their old shares. Investors who receive shares of the new company will:

Be required to receive a prospectus under the Securities Act of 1933 Explanation: This scenario is an example of an offering regulated by Rule 145. Rule 145 defines certain types of reclassifications of securities as sales subject to the registration and prospectus requirements of the Securities Act of 1933. Shares acquired through mergers, consolidations, and spinoffs involving exchanges of stock are all covered under the rule. The amount of shares is irrelevant.

TBill

Bid is greater than ask

A real estate limited partnership that does not specify the actual properties to be purchased is known as a:

Blind pool Explanation: If a real estate program's prospectus does not specify the actual properties to be purchased, it is known as a blind pool (or nonspecified property) program. An oil and gas program may also be considered a blind pool if the properties to be drilled are not specified in the prospectus.

A customer wishes to establish a tax loss and sells 100 shares of XYZ Corporation. The loss would not be allowed if the customer, within 30 days:

Bought an XYZ Corporation call Explanation: The IRS will not allow the loss if the same security or any security convertible into the same security is repurchased within 30 days of the sale. The customer must wait until the 31st day to buy back the security or its equivalent. In this example, the only choice given that could be converted into 100 shares of XYZ Corporation would be a call on XYZ Corporation. The loss would not be allowed if the customer, within 30 days, bought an XYZ Corporation call. This is known as a wash sale.

A Swiss company that is expecting payment from a customer in U.S. dollars is concerned that the dollar will decline in value. To hedge against a decline in the U.S. dollar, the Swiss company should:

Buy Swiss franc calls Explanation: If the value of the U.S. dollar declines, the value of the Swiss franc will increase. The company should buy Swiss franc calls since it will profit on the calls if the U.S. dollar declines, leading to a Swiss franc increase. The profit on the call could help to offset the loss on the U.S. dollars it is expecting to receive as payment.

An investor has sold stock short at $60. The current market price of the stock is $40 and the investor believes the stock will recover somewhat before going lower. The investor should:

Buy a call Explanation: The stock is currently trading at $40 and the investor is concerned that the price will rise. An investor who wants to limit the losses on an underlying short stock will buy a call.

An individual who is short stock and wants protection against an upside move in the market will probably:

Buy a call option Explanation: An individual who is short stock will buy a call option. If the market advances, the individual will exercise the call option to limit the loss if the market value of the short stock increases.

Which of the following orders would not be reduced on the order book on the ex-dividend date for a cash dividend?

Buy stop order Sell limit order

An article in The Wall Street Journal states that yields on Treasury bills have declined in the past month to 4.58% from 4.61%. This indicates that:

Buyers of new bills paid more than buyers paid the previous month Explanation: Treasury bills are purchased at a discount from the dollar amount on its face. The larger the discount, the higher the discounted yield to maturity. In this example, the discounted yield to maturity has gone down to 4.58% from 4.61% from the previous month. This indicates that buyers of new bills paid more for the Treasury bills (meaning the discount was less) than buyers paid the previous month.

A customer is considering writing an XYZ April 90 put for an $8 premium but is concerned about the risk of a large loss. Which of the following positions, when added, provides the BEST protection?

Buying an XYZ April 80 put for a premium of 2 Explanation: The short put position has a maximum risk of $8,200 if the stock declines to zero. If, in addition, the customer buys an XYZ April 80 put for 2, it becomes a credit spread. The maximum risk is reduced to $400, which is the difference between the strike prices (10) and the net premium received (6).

The FRB initial margin requirement is 50%. A customer's initial transaction in a margin account is a purchase of 100 shares of XYZ at $15 per share. The customer would need to deposit what amount in this new account? A) $375 B) $750 C) $1,500 D) $2,000

C) $1,500 Securities purchased in a new margin account require a minimum equity of $2,000. If the securities are worth less than $2,000, then the securities must be paid for in full. In this example, the purchase is for $1,500, requiring the customer to deposit the full amount of the purchase.

A corporation has pretax income of $2,000,000. In addition, it received dividends of $100,000 from the common stock of a corporation in which it had a 10% interest. If the corporation pays a 34% tax rate, what is its total tax liability? A) $680,000 B) $686,800 C) $690,200 D) $714,000

C) $690,200 If a corporation owns less than 20% of the distributing company, the corporation is required to pay tax on 30% of the dividends it receives on stock that it owns (70% is excluded). The company will have to add $30,000 (30% of $100,000) to its taxable income. The total taxable income will, therefore, be $2,030,000. The tax liability will be $690,200 ($2,030,000 times 34% tax rate). If the corporation owned at least 20% of the distributing company, only 20% of the dividends would be taxable.

Your customer has a Coverdell Education Savings Account for each of four preteen daughters. What is the maximum amount of pretax contributions that he can make to each ESA? A) 8000. B) 2000. C) 0. D) 500.

C) 0. Pretax contributions cannot be made to Coverdell ESAs. The customer is allowed to make a $2,000 after-tax contribution annually for each student until their 18th birthday.

LMN Securities is the managing underwriter for a new issue of one million MIC common shares. LMN has agreed to sell as much stock as possible in the market, and MIC has agreed to take back any unsold shares. If MIC has not specified a minimum amount of capital for LMN to raise, this is what type of offering? A) All-or-none. B) Contingency. C) Best efforts. D) Standby.

C) Best efforts. In a best efforts underwriting, any stock that remains unsold is returned to the issuing corporation.

To make a public offering, a registered investment company must have a minimum net worth of: A) $10 million. B) $100 million. C) 100000. D) $1 million.

C) 100000. Investment companies are not required to register an offering with the SEC unless they have a net worth of $100,000.

A registered representative who leaves the industry must requalify by examination to return to the industry if he is unaffiliated with a broker/dealer for more than: A) 5 years. B) 3 years. C) 2 years. D) 10 years.

C) 2 years. All securities licenses become null and void once an individual is unaffiliated for more than 2 years.

As an initial transaction in a margin account, a customer sells short 1,000 shares of a capital market stock at $2 per share. If Regulation T is 50%, how much money will the customer be required to deposit? A) 2000. B) 3000. C) 2500. D) 1000.

C) 2500. The industry requirement to short stocks below $5 per share is 100% of market value or $2.50 per share (whichever is greater).

On February 13, your customer buys an 8% Treasury bond maturing in 2009 for settlement on February 14. If the bonds pay interest on January 1 and July 1, how many days of accrued interest are added to the buyer's price? A) 45. B) 43. C) 44. D) 14.

C) 44. Accrued interest for government bonds is figured on an actual-days-elapsed basis. The number of days begins with the previous coupon date and continues up to, but not including, the settlement date. The bonds pay interest on January 1. There are 31 days of accrued interest for January. The bonds settle February 14. There are 13 days of accrued interest for February. Do not count the settlement date (31 + 13 = 44 days).

A customer invests $20,000 in a DPP and signs a recourse note for $50,000. During the first year of operation, the customer receives a cash distribution from the partnership of $15,000. At year end, the customer receives a K-1 statement reporting his share of partnership losses of $75,000. How much of the loss may the customer deduct from passive income? A) 0. B) 35,000. C) 55,000. D) 75,000.

C) 55,000. A limited partner can only deduct partnership losses to the extent of his basis. To determine basis, add the original investment ($20,000). to any recourse debt assumed by the investor ($50,000). Recourse debt adds to basis as the partner is liable for this amount. Cash distributions received reduce basis ($15,000). At year end, the investor's basis and the amount he can deduct from passive income is $55,000.

A customer purchased a municipal bond with a 6.50% coupon rate that was priced at a 6.95 basis. If the bond is currently trading at $945, the current yield is: A) 6.95% B) 6.73% C) 6.88% D) 6.50%

C) 6.88% The current yield is found by dividing the yearly interest payment of $65 by the market price of $945. This equals 6.88%. The fact that the bond was purchased at a 6.95 basis is not relevant.

A customer buys 10 DEC 91.50 calls on the Canadian dollar for 6.70. ($10,000 CD per contract). At the time of purchase, the spot rate for the Canadian dollar was 92.25. What is the margin requirement for the purchase? A) 9225. B) 9150. C) 6700. D) 3350.

C) 6700. The client purchased 10 calls at 6.70 for a total of $6,700. The margin requirement is 100% of the premium.

Cash dividends received from which of the following securities will be taxed as ordinary income? A) Preferred stock issued by a bank B) Common stock issued by an oil company C) A real estate investment trust D) Convertible preferred stock issued by a software company

C) A real estate investment trust Currently, dividends paid on both common and preferred stock are taxed at a maximum rate of 20% if the stock is held for more than 60 days. Dividends from a REIT are still taxed at the same rate as ordinary income since a REIT does not pay corporate income tax if it distributes a minimum percentage of its income. The type of company that issued the shares is not relevant to the tax status of the cash dividend.

Which of the following best describes a nominal municipal bond quotation? A) Likely bid or offer. B) Firm bid or offer. C) Approximate price reflecting current market value with no bid or offer. D) Subject bid or offer that must be reconfirmed before a trade.

C) Approximate price reflecting current market value with no bid or offer. A nominal quote is an indication of the approximate market value of a municipal bond, provided for informational purposes only. A nominal quote does not represent an actual bid or offer.

Which of the following is a true statement regarding payment to informants (whistleblowers)? A) Awards can only be paid for information regarding violations of the Insider Trading Securities Fraud Enforcement Act. B) If an award is paid to a "whistleblower", the amount of the award can not be based on any amount recovered. C) Awards may be paid in connection with original information concerning any violation of securities law that leads to successful enforcement. D) Awards are not paid for information regarding securities law violations regardless of the success of any enforcement action taken.

C) Awards may be paid in connection with original information concerning any violation of securities law that leads to successful enforcement. Awards to "whistleblowers" may be paid in connection with original information concerning any violation of securities law including but not limited to violations of the Insider Trading Securities Fraud Enforcement Act. Those awarded may be paid a percentage of the amount collected.

Which of the following would NOT be a valid use of the partnership democracy? A) Removing the general partner. B) Consenting to a legal judgment against the partnership. C) Deciding which partnership assets should be liquidated to pay creditors. D) Consenting to an action of a general partner that is contrary to the agreement of limited partnership.

C) Deciding which partnership assets should be liquidated to pay creditors. Deciding which partnership assets should be liquidated to pay creditors involves limited partners in the active management of partnership affairs. This would result in their being treated as general partners with respect to liability, and possible loss of limited partner status.

Which of the following types of retirement plans would be most beneficial to a young employee of a corporation? A) Keogh plan. B) Profit-sharing plan. C) Defined contribution pension plan. D) Defined benefit pension plan.

C) Defined contribution pension plan. The most beneficial corporate pension plan for a younger employee would be the defined contribution plan. The employee has many years to go in the workforce, so the investments made with the defined contributions will have a maximum time period to grow.

When determining whether a CMO is suitable, an RR must offer to a client all of the following information, EXCEPT a: A) Glossary of terms B) Discussion on how changing interest rates may affect the prepayment rates C) Discussion on how changing currency rates may affect the value of the securities D) Discussion on the relationship between mortgage loans and mortgage securities

C) Discussion on how changing currency rates may affect the value of the securities Broker-dealers must offer customers educational material about the features of CMOs. This material must include: A discussion of the characteristics and risks of CMOs. This includes: how changing interest rates may affect prepayment rates and the average life of the security, tax considerations, credit risk, minimum investments, liquidity, and transactions costs. A discussion of the structure of a CMO. This includes the different types of structures, tranches, and risks associated with each type of security. It is also important to explain to a client that two CMOs with the same underlying collateral may have different prepayment risk and different interest-rate risk. A discussion that explains the relationship between mortgage loans and mortgage securities A glossary of terms applicable to mortgage-backed securities Changing currency rates are not applicable to the risks associated with CMOs.

Which of the following statements is TRUE concerning a customer who purchases an out-of-state original issue discount (OID) general obligation bond? A) Each year the customer will pay both federal and state income tax B) Each year the customer will pay only federal income tax C) Each year the customer will pay only state and local income tax D) The customer will not pay any tax

C) Each year the customer will pay only state and local income tax The upward adjustment in the purchase price of an original issue discount bond is called accretion. The amount accreted each year is considered interest income, which may or may not be taxable depending on the type of security. The interest on an out-of-state municipal security is exempt from federal tax, but subject to state and local income tax. The tax rate is based on the state in which the customer maintains his primary residence.

MSRB rules for NYSE member firms are enforced by: A) the NYSE. B) the MSRB. C) FINRA. D) the SEC

C) FINRA. The board's rules are enforced by FINRA for securities firms. The MSRB has rule-making authority but no enforcement or examination authority.

Which of the following two are TRUE of a leveraged exchange traded fund (ETF)? I. The leverage ETF may be purchased on margin. II. Securities within the leveraged fund portfolio may be purchased on margin. III. The leveraged ETF may never be purchased on margin. IV. Securities within the leveraged fund portfolio may never be purchased on margin. A) I and IV. B) III and IV. C) I and II. D) II and III

C) I and II. Because an exchange traded fund is purchased and sold on an exchange, the rules generally applying to all exchange products such as purchasing them on margin, would apply. Leveraged funds can use a number of different securities types including derivative products, and trading techniques such as trading on margin as a means of attaining the leveraged returns they promise.

A customer buys 2 ABC Jan 35 calls @ 4 and buys 2 ABC Jan 35 puts @ 1 on the same day when the market price of ABC stock is $36. The customer has created a:

long straddle Buying a call and a put on the same stock with the same strike price and expiration is a long straddle.

In which of the following types of offerings does a brokerage firm have no financial obligation for unsold securities? I. All-or-none. II. Best efforts. III. Standby. A) II and III. B) I and III. C) I and II. D) I, II and III.

C) I and II. In a best efforts underwriting, the underwriter serves as an agent with no financial obligation for unsold securities. In an all-or-none (AON) offering, the underwriter agrees to devote its best efforts to sell the issue, but the entire offering is canceled if all shares cannot be sold. In a standby underwriting, the underwriter agrees to purchase any unsold shares remaining after the expiration of a rights offering (firm commitment).

An underwriting bid for a municipal GO issue would include which of the following? I. The dollar amount. II. The coupon rate. III. The yield-to-maturity. IV. The underwriting spread. A) II and IV. B) I and III. C) I and II. D) II and III.

C) I and II. The only information the underwriter must furnish to the issuer is the dollar amount of the bid (the amount the issuer will receive) and the coupon rate (the amount of interest the issuer will pay). From this the issuer can determine the lowest net interest cost and award the bonds on that basis.

An increase in the Federal Reserve Board's (FRB) reserve requirement has which of the following effects on total bank deposits? I. Decrease. II. Increase. III. Multiplier effect. IV. Logarithmic effect. A) II and IV. B) I and IV. C) I and III. D) II and III.

C) I and III. If the FRB raises the reserve requirement, total bank deposits decrease, with the overall impact being increased because of the multiplier effect. If banks must meet a higher reserve requirement, they will have less money available to lend.

An investor purchases 100 shares of XYZ common stock for $70 and sells it one year later for $50. Which of the following activities would violate the wash sale rule? I. Purchasing an XYZ call option 20 days after the sale. II. Purchasing an XYZ put option 20 days after the sale. III. Purchasing 100 shares of XYZ common stock 20 days after the sale. IV. Selling short 100 shares of XYZ common stock 20 days after the sale. A) III and IV. B) I and II. C) I and III. D) II and IV.

C) I and III. The wash sale rule is violated when an investor sells a security at a loss and purchases the same or a substantially identical security within 30 days of the sale date. The IRS considers a call option substantially identical to the underlying stock because it represents the right to buy the shares

Which statements are TRUE regarding contribution limits? I. The contribution limit to a Coverdell ESA can be reduced or eliminated for high-income individuals. II. The contribution limit to a Coverdell ESA cannot be reduced or eliminated for high-income individuals. III. The contribution limit to a Section 529 plan can be reduced or eliminated for high-income individuals. IV. The contribution limit to a Section 529 plan cannot be reduced or eliminated for high-income individuals. A) II and IV. B) II and III. C) I and IV. D) I and III.

C) I and IV. The after-tax contribution limit of $2,000 can be reduced or eliminated for high-income taxpayers. However, there are no income limitations placed on individuals opening Section 529 plans.

A member of the investment banking department of ABC securities is explaining some of the advantages and disadvantages of rights and warrants to the board of directors of XYZ Corporation. Which of the following statements could he make? I. The exercise prices of stock rights are usually below CMV of the underlying security at time of issue. II. The exercise prices of warrants are usually above CMV of the underlying security at time of issue. III. Both rights and warrants may trade in the secondary market and may have prices that include a speculative (time) value. IV. Warrants are often issued attached to a bond issue to reduce the interest costs to the issuer. A) I and II. B) I only. C) I, II, III and IV. D) I, II and III.

C) I, II, III and IV. All are true statements. The exercise prices of stock rights are usually below CMV of the underlying security at time of issue. The exercise prices of warrants are usually above CMV of the underlying security at time of issue. Both rights and warrants may trade in the secondary market and may have prices that include a speculative (time) value. Warrants are often issued attached to a bond issue to reduce the interest costs to the issuer.

The Securities Exchange Act of 1934: I. Created the SEC II. Provided for the regulation of credit III. Provided for the regulation of exchanges IV. Provided for the regulation of new issues A) I and III only B) I and IV only C) I, II, and III only D) II, III, and IV only

C) I, II, and III only The Securities Exchange Act of 1934 created the SEC and provided for the regulation of credit and exchanges. The Securities Act of 1933 provided for the regulation of new issues.

The Sarbanes-Oxley Act legislation was established to enhance standards for Market makers in U.S. exchange listed securities Boards of directors of U.S. publicly traded companies Public accounting firms Block traders in stocks of U.S. exchange listed securities A) I and IV B) I and III C) II and III D) II and IV

C) II and III The Sarbanes-Oxley Act of 2002 (SOX) was enacted in response to a number of major corporate and accounting scandals. The legislation established enhanced standards for all U.S. public company boards of directors, management, and public accounting firms.

Investment clubs: I. can take advantage of breakpoints on mutual fund purchases. II.cannot take advantage of breakpoints on mutual fund purchases. III. are permitted to purchase new equity issues at the POP. IV. are not permitted to purchase new equity issues at the POP. A) I and IV. B) I and III. C) II and III. D) II and IV.

C) II and III. Investment clubs are not considered restricted persons under the rules regarding sales of a new issue, and therefore are eligible to purchase new equity issues. Note that if a registered representative (a restricted person) were a member of an investment club, the club would be prohibited from buying a new equity issue. Investment clubs are never permitted to take advantage of breakpoints available on mutual fund purchases.

Which two statements are TRUE regarding margin calls? I. Customers are entitled to an extension of time. II. Customers are not entitled to an extension of time. III. Firms can sell securities without first contacting the customer. IV. Firms cannot sell securities without first contacting the customer. A) I and IV. B) II and IV. C) II and III. D) I and III.

C) II and III. Some customers mistakenly believe that a firm must contact them for a margin call to be valid, which is not the case. Most firms will attempt to notify their customers of margin calls, but are not required to do so. Also, there is no entitlement when it comes to an extension of time.

Which of the following securities is the least suitable recommendation for a qualified retirement account plan account? A) Treasury bill. B) Blue-chip common stock. C) Investment-grade municipal bond. D) A rated corporate bond.

C) Investment-grade municipal bond. Municipal bonds provide tax-exempt interest payments and, consequently, offer lower yields. Because earnings in a qualified retirement plan account grow tax deferred, the municipal bond is not a suitable investment. In addition, they will be fully taxed on withdrawal.

Which of the following may only be accomplished after applying the additional bonds test for a revenue bond? A) Increasing the project's user charges. B) Prerefunding an outstanding bond issue. C) Issuing new bonds with an equal lien on the project's revenues. D) Spending revenues already allocated for project expansion.

C) Issuing new bonds with an equal lien on the project's revenues. The additional bonds test must be met under the provisions of a revenue bond indenture before additional bonds with an equal lien on project revenues can be issued. The conditions under which additional bonds may be issued are specified in the bond indenture. This is an open-end covenant.

A resident of New York City purchases an Albany, New York general obligation bond and receives $600 of interest from that bond during the year. How is that $600 taxed? A) Taxation is deferred until the bond matures. B) It is subject to state income tax at ordinary rates. C) It is not subject to federal income tax. D) It is subject to federal income tax at ordinary rates.

C) It is not subject to federal income tax. Interest from public purpose municipal bonds is exempt from federal income tax and most states have chosen to make interest on their municipal bonds exempt from state income tax to residents of their states.

How often will the IRS allow a Health Savings Account (HSA) to be funded via an IRA distribution without paying federal taxes or penalties on the distribution? A) Never, taxes and penalties for early distributions are always due. B) Once each calendar year. C) One time. D) There are no funding limits when HSAs are funded from another qualified account.

C) One time. Health Savings Accounts (HSAs) are qualified employer sponsored plans. The IRS allows a one time funding distribution from an IRA to a qualified HSA without paying federal taxes or penalties on the IRA distribution.

Mr. Brown has several stock rights. Which of the following is NOT an alternative regarding these stock rights? A) Giving the rights to his son. B) Exercising. C) Redeeming them from the issuer for cash. D) Selling at the market.

C) Redeeming them from the issuer for cash. Rights are not redeemable by the issuer. They may be sold in the secondary market or be given to someone else to exercise. If exercised, rights are exchanged for an appropriate number of shares of the underlying common stock.

The legislation that required SRO's to establish research analyst conflict of interest rules for its members is: A) Securities Investors Protection Act. B) Securities Act of 1933. C) Sarbanes-Oxley. D) Regulation D

C) Sarbanes-Oxley. The research analyst conflict of interest rules were mandated by the Sarbanes-Oxley Act.

Sally is the named beneficiary of her grandmother's IRA. After the death of her grandmother who was 80 years old, what would Sally's options be regarding the IRA? A) Roll over her grandmother's money into Sally's own IRA. B) Wait until age 70½, to maximize tax deferral, and then begin required minimum distributions. C) Take minimum required distributions based on Sally's life expectancy. D) Wait until age 59½ to begin taking distributions, to avoid the 10% tax penalty.

C) Take minimum required distributions based on Sally's life expectancy. Before age 80, Sally's grandmother would have already begun mandatory distributions. When someone inherits an IRA for which the initial owner has begun mandatory distributions, payout must continue but is now based on the life expectancy of the new owner.

Investment companies with no management fee and low sales charges, which invest in a fixed portfolio of municipal or corporate bonds, are categorized as: A) Open-end investment companies B) Closed-end investment companies C) Unit investment trusts D) Face amount certificate companies

C) Unit investment trusts Investment companies with no management fee and low sales charges, which invest in a fixed portfolio of municipal or corporate bonds, are categorized as unit investment trusts (UITs). Investors can receive a reduced sales charge if they purchase a certain amount of a UIT

Of the following bonds, which has the greatest price volatility? A) Corporate bond fund. B) AA corporate bond with 7 years to maturity. C) Zero-coupon bond with 15 years to maturity. D) Zero-coupon bond with 5 years to maturity.

C) Zero-coupon bond with 15 years to maturity. The longer the duration of a bond, the greater the volatility will be of its market price when interest rates change. Because zero-coupon bonds do not make interest payments but are priced at a deep discount to par value, they are more volatile than coupon-bearing bonds.

An investor purchased a municipal bond at par to yield 5.5% to maturity. If, two years later, he sold the bond at a price equivalent to a 5% yield to maturity, the investor incurred: A) no taxable result at this time. B) a capital loss. C) a capital gain. D) taxable interest income.

C) a capital gain. Because the investor sold the bond at a price that will yield less than the yield when he purchased the bond, the bond must have been sold for more than the investor paid for it. Therefore, the investor incurred a capital gain-lower yield, higher price.

An instrument that illustrates the transfer of title to any dividend, interest, or right that pertains to securities contracted for is called: A) a warrant. B) a power of attorney. C) a due bill. D) a right.

C) a due bill. A due bill is an assignment of a forthcoming distribution from the seller to the new owner.

A hedge fund has contracted with your broker/dealer to handle all of its clearing functions and provide all back office support functions while it is executing transactions through numerous other broker/dealers whom your broker dealer will have agreements with. This type of account is known as A) a custodial account B) a numbered account C) a prime account D) a joint account

C) a prime account In a prime account, a customer contracts with one broker, the prime broker, to provide a list of support services, such as clearing and settlement of transactions, while contracting with numerous other brokers for executions services.

Which of the following accounts would a CMO Z-tranche be best suited for? A) an IRA account for a middle aged client B) a custodial account set up under the uniform transfer to minors act (UTMA) C) a professionally managed hedge fund specializing in real estate portfolio securities D) a joint account with a non working spouse

C) a professionally managed hedge fund specializing in real estate portfolio securities A zero tranche (Z-Tranche) CMO is considered to be among the most volatile CMO tranches because they receive no payments until all preceding tranches of the CMO are retired. Generally CMO tranches are not suitable for smaller or unsophisticated investors which is why customers are required to sign a suitability statement before purchasing any CMO tranche. Of the answer choices given the best suited account would be the one that is professionally managed and already specializing in real estate investments.

All of the following are restricted persons EXCEPT: A) finders and fiduciaries acting on behalf of the managing underwriter. B) portfolio managers. C) any persons owning 5% or more of a member firm. D) employees of members

C) any persons owning 5% or more of a member firm. Rules prohibit member firms from selling initial equity public offering stock to any account in which restricted persons are beneficial owners. Restricted persons include FINRA members, employees of member firms, finders and fiduciaries acting on behalf of the managing underwriter, portfolio managers, and any person owning 10% or more of a member firm. Also included are a restricted person's immediate family members.

One of the benefits of using arbitration to settle disputes between member firms is that it: A) gives more time to prepare arguments. B) is not binding on both parties. C) is relatively inexpensive. D) does not allow for arguments from parties outside the industry.

C) is relatively inexpensive. Arbitration is a method for settling disputes between member firms that is less costly than litigation. In addition, all decisions are final and binding on all parties.

All of the following are regulated by the MSRB EXCEPT: A) dealers. B) quotes. C) issuers. D) sales representatives.

C) issuers. Quotes, dealers, and sales representatives are regulated by the MSRB; issuers are not.

At 2PM ET a customer enters an order to buy GGZ at the close on the NYSE. GGZ traded between 70 and 71 all day. Then, after a last-minute rally, it closed up 4 points at 74. The customer should expect to pay the: A) opening price the next morning. B) average price calculated for the entire day. C) closing price. D) price as near to the close as possible, at the floor broker's discretion.

C) closing price. When an order is placed market at the close (MOC) on an exchange, a customer should expect execution at the closing price.

A nonqualified deferred compensation plan: A) guarantees payment to the employee even if the company becomes insolvent. B) must be offered to all employees. C) does not guarantee that the employer will fulfill the obligation. D) must be approved by the IRS.

C) does not guarantee that the employer will fulfill the obligation. Nonqualified deferred compensation plans are agreements between an employer and an employee in which the employee agrees to defer receipt of part of their salary. Nonqualified deferred compensation plans do not require IRS approval and may discriminate (need not be offered to all employees). In fact, they are generally offered only to officers and other high-ranking executives. In the event of a business failure, there is no guarantee that deferred amounts will be paid.

Before a firm distributes a prepared summary official statement for a new issue of municipal bonds to customers, it must have the written approval of the: A) bond attorney. B) issuer. C) firm's municipal securities principal. D) MSRB.

C) firm's municipal securities principal. A person qualified as a general securities or municipal securities principal must give prior written approval for municipal securities advertising or sales pieces that are intended to be used as communications with the public. Abstracts and summaries of official statements are included in the MSRB's definition of advertising.

All of the following events would cause FINRA to terminate quotations in a SmallCap stock EXCEPT A) the issuer's independent auditor rendering a disclaimer opinion B) FINRA deeming termination to be in the public's best interest C) having only 2 broker/dealers making a market in the stock D) the issuer declaring bankruptcy

C) having only 2 broker/dealers making a market in the stock Although 3 market makers are required initially for an issue to be included on Nasdaq, only 2 market makers are needed to continue being quoted on the system. The other answers, bankruptcy, a disclaimer opinion being rendered by auditors, or any reason FINRA deems termination to be in the general publics interest, are sufficient reason for termination of quotations.

If a corporation has a dividend payout ratio of 70%, the undistributed earnings will: A) decrease book value. B) increase capital surplus. C) increase retained earnings. D) increase earnings per share.

C) increase retained earnings. Retained earnings represent income that has not been paid out to shareholders.

A customer signs a trading authorization form granting written discretion over the account to the registered representative. All of the following practices are permitted EXCEPT: A) shifting funds among several mutual funds within a fund family, based upon the representative's expectations for market performance. B) notifying the customer periodically of the account's performance. C) indicating that trades in which the representative used discretion were unsolicited or solicited. D) buying or selling stocks without notifying the customer.

C) indicating that trades in which the representative used discretion were unsolicited or solicited. When discretion is used in a discretionary account, order tickets must be marked accordingly.

Variable rate municipal bonds are subject to all of the following risks EXCEPT: A) market. B) liquidity. C) interest rate. D) default.

C) interest rate. A variable rate bond is one whose coupon is adjusted periodically (semiannually or annually) to reflect current interest rates. Therefore, if rates rise, forcing prices down, the coupon on a variable rate bond will be adjusted upward, thereby tending to keep the bond's price at or near par. Therefore, no interest rate risk is associated with these bonds. However, if rates fall, the coupon will be adjusted downward, keeping the bond's price at or around par. Normally, a fall in rates will force prices up, but not with variable rate bonds.

A customer is interested in an IPO of a stock in registration. He requests that you highlight the important information on the preliminary prospectus and send an analysis of the company's past performance. You may: A) comply with the request because the customer solicited the information and analysis. B) comply with the request because it involves an IPO in which little information is known about the issuer. C) not comply with the request because the preliminary prospectus may not be altered. D) not comply with the request because the stock is not yet listed for trading on any exchange.

C) not comply with the request because the preliminary prospectus may not be altered. A prospectus, whether preliminary or final, is a legal document that cannot be altered by the registered representative. It is illegal for a registered representative to mark on or attach anything to a prospectus (even if requested by a client). Important information may be pointed out orally but not highlighted.

All of the following will affect the working capital of a corporation EXCEPT: A) declaration of a cash dividend. B) a decrease in liabilities. C) payment of a cash dividend. D) an increase in assets.

C) payment of a cash dividend Working capital is defined as current assets minus current liabilities. Payment of a cash dividend will reduce current assets (cash) and current liabilities (dividend payable) by the same amount, leaving working capital unchanged.

An annuity may be purchased under all of the following methods EXCEPT: A) single payment immediate annuity. B) periodic payment deferred annuity. C) periodic payment immediate annuity. D) single payment deferred annuity.

C) periodic payment immediate annuity. A periodic payment immediate annuity is a contradiction in terms. The annuitant may not contribute and withdraw simultaneously.

If a municipal firm purchases a block of municipal bonds in anticipation of a price increase, the firm is engaged in: A) hedging. B) short selling. C) position trading. D) arbitrage.

C) position trading. The dealer is buying for its inventory (position trading).

The 5% markup policy applies to: A) All of these. B) new issues. C) principal OTC trades. D) mutual funds

C) principal OTC trades The 5% markup policy applies to agency and principal nonexempt securities and transactions both exchange and OTC traded. It does not apply to prospectus offerings (mutual funds and new issues).

When determining whether a tax swap of municipal bonds will result in a wash sale, each of the following are considered EXCEPT: A) coupon. B) issuer. C) principal amount. D) maturity.

C) principal amount. In judging whether bonds purchased are substantially identical to bonds sold for a loss, the tax code considers maturity, issuer, and coupon rate. If at least two of the three are different, a wash sale will generally not result.

On the trading floor, the highest bid and offer receive first consideration. When several bids at the same price occur, the trade will be awarded based on: A) parity, precedence, then priority. B) priority then parity. C) priority, precedence, then parity. D) priority then precedence.

C) priority, precedence, then parity. When there are several bids or offers at the same price, the order in which they are filled is based upon time entered and size of order. This is known as priority, precedence and parity on the NYSE.

The manager of a portfolio that consists predominately of large- and mid-cap stocks could hedge against a market downturn and generate additional income by A) buying broad index puts. B) selling broad index puts. C) selling broad index calls. D) buying broad index calls.

C) selling broad index calls. The only way to generate income through the use of options is to sell them. If one is concerned that the market may fall, selling calls is the appropriate strategy.

The term that describes options of the same exercise price and expiration date for the same underlying security is: A) type. B) issue. C) series. D) class.

C) series. Options at the same exercise price and expiration date for the same underlying security are known as a series of options.

The last day that stocks can be bought for cash and still receive the dividend is: A) the business day prior to the regular way ex-date. B) on the ex-date. C) the record date. D) the day after the record date.

C) the record date. A cash trade settles the same day. Stocks bought for cash on the record date will be entitled to the dividend under an exception to the 2-business day rule for regular way transactions.

Mutual fund shareholders are NOT taxed on: A) reinvested dividends. B) interest distributions. C) unrealized capital gains. D) capital gains distributions.

C) unrealized capital gains. Interest, dividends, and realized capital gains are all taxed. However, unrealized capital gains are not taxed. Unrealized gains contribute to NAV appreciation and to a shareholder's capital gain upon redemption.

An outstanding municipal bond issue has the following characteristics: 7.50% coupon; maturity in 20 years; puttable in 5 years at 100; callable at 102 in 10 years; declining in a straight-line to maturity; yield-to-maturity is 6.50%. The issues should now be quoted: A) yield-to-put. B) yield-to-maturity. C) yield-to-call at 102. D) yield-to-call at par.

C) yield-to-call at 102. Since the bond issue is selling at a premium, the yield-to-call is less than the yield-to-maturity. The bonds must be quoted as yield-to-call at the earliest maturity, which would be the 10-year call at 102. If the bonds were selling at a discount, yield-to-maturity would be the proper quote. Yield-to-put is not required to be quoted.

A customer buys 1 Swiss Fran Jan 80 Call @ 7 when the Swiss Franc is trading at 78. The contract is: I. trading at parity II. "in the money" III. "out the money" IV. "at the money"

C. "out the money" Calls go "out the money" when the market price falls below the strike price.

A customer buys a $100,000 - 30 year U.S. Government bond with 10 years left to maturity in the secondary market at 80. The customer does not elect to accrete the discount annually. At maturity, the customer will have: A. no capital gain or loss B. a $20,000 taxable capital gain C. $20,000 of taxable interest income D. $a 20,000 capital loss

C. $20,000 of taxable interest income Treasury bonds bought in the secondary market at a discount are termed "market discount bonds". There is an OPTION to accrete the discount and paying tax annually on the accretion amount at full tax rates; OR of waiting until the bond is redeemed or sold to pay the tax on the earned income.

A customer buys 2 ABC Jan 60 puts @ 4 when the market price of ABC is $59. ABC stock falls to $40 and the customer buys the stock in the market and exercises the puts. The gain is: A. $800 B. $1,600 C. $3,200 D. $4,000

C. $3,200 The customer buys the stock at $40, and exercises the put to sell at $60 for a 20 point profit. Since 4 points were paid in premiums, the net profit per contract is 16 points, or $1,600 x 2 contracts = $3,200

An investor sells short 200 shares of ABC stock at $60 and sells 2 ABC Jan 60 Puts @ 4 on the same day in a margin account. The customer must deposit: A. $800 B. $2,400 C. $5,200 D. $6,000

C. $5,200 To sell stock short, initial margin is 50%. 50% of $12,000 is $6,000. No margin is required on the short puts since they are covered by the short stock position. The $800 premiums received from writing the puts is applied against the margin requirement of $6,000 for a deposit of $5,200.

A customer buys 100 shares of ABC stock at $58 and buys 1 ABC Jul 55 Put @ 2.50 on the same day. If the stock falls to $50 and the put is exercised, the customer will have a: A. $250 loss B. $300 loss C. $550 loss D. $750 loss

C. $550 loss If the stock price drops below $55, the customer will exercise the put and sell the stock (purchased at $58) at the $55 strike price. The customer will lose 3 points ($300) on the stock in addition to the $250 paid in premiums, for a total loss of $550.

On the same day a customer buys 1 ABC Feb 70 call @ 4 and sells 1 ABC Feb 80 call @ 1 when the market price of ABC is $70.50. The maximum potential gain is: A. $300 B. $400 C. $700 D. unlimited

C. $700 If the market rises, the long call is exercised and the stock is bought at $70. If it continues to rise, the short call is exercised and the stock is sold at $80 for a 10 point profit. Since 3 points were paid in net premiums, the max potential gain is $700.

A corporation has issues 50,000,000 shares of common stock at $2 par. The corporation has $10,000,000 shares of Treasury Stock on its books. The aggregate value of the outstanding shares is: A. $20,000,000 B. $40,000,000 C. $80,000,000 D. $100,000,000

C. $80,000,000 Outstanding stock is: Issued Stock (50,000,000) - Treasury stock (10,000,000) = 40,000,000 shares outstanding at $2 par = $80,000,000

A customer has $35,000 to invest in a mutual fund with a NAV per share of $9.42 and a Public Offering Price (POP) of $10.30. In the prospectus is the following breakpoint schedule: Purchase Amount Sales Charge $0-$10,000 8 1/2 % $10,001-$25,000 7 1/4 % $25,001-over 6 1/2 % How many shares of the fund can the customer purchase? A. 3,315 B. 3,398 C. 3,474 D. 3,746

C. 3,474 Ask Price = Bid (NAV)/100% - Sales Charge %

A customer buys 10 Allied Corporation 8% debentures, M '38, at 90 on Tuesday, April 18th in a regular way trade. The interest payment dates are March 1st and September 1st. How many days of accrued interest will the buyer pay to the seller? A. 29 B. 30 C. 50 D. 51

C. 50 Interest accrues on a 30 day month / 360 day year for corporate bonds. The bonds were purchased on Tuesday, April 18th. Settlement takes place 3 business days after trade date on Friday, April 21st. Interest accrues up to, but does not include, settlement date. Thus, 30 days are due for March; and 20 days are due for April.

In a deflationary period, which security would be most negatively affected? A. Long-term bond B. Intermediate-term bond C. Common stock D. Preferred stock

C. Common stock

All of the following securities delivers are "good' EXCEPT: A. trust securities with an assignment performed by the Trustee B. Guardian securities with an assignment performed by the legal guardian C. Custodian securities with an assignment performed by the recipient of the gift D. Partnership securities with an assignment performed by a partner designated in the Partnership Agreement

C. Custodian securities with an assignment performed by the recipient of the gift

All of the following statements are true about a tender offer for common shares EXCEPT: A. The offer must remain open for at least 20 business days B. Each "sweetening" of the offer must extend the offer for an additional 10 business days C. During the life of the offer, the issuer can buy the stock in the market in addition to buying shares via the offer D. During the life of the offer, any subscribing investors' shares that are tendered are held in escrow pending the outcome of the offer

C. During the life of the offer, the issuer can buy the stock in the market in a

When advertising the availability of a municipal security at a "yield," a municipal firm: I must own the security II does not have to own the security III must disclose whether the yield is the coupon rate or yield to maturity IV must disclose whether the yield is the coupon rate or current yield A. I and III B. I and IV C. II and III D. II and IV

C. II and III

Relative to stock index options, which of the following statements is NOT TRUE? a. There are options expiring each month b. If traded, settlement is the next business day c. If exercised, settlement is in cash in three business days d. The settlement amount is based on the closing index price on the day of exercise

C. If exercised, settlement is in cash in three business days Explanation: When exercised, a stock index option settles in cash on the next business day. (Equity options, if exercised, would settle in three business days.) The settlement amount is determined by the difference in strike price and the closing value of the index on the day of exercise. Index options expire each month. Option trades settle on the next business day.

Regarding options sales literature, which statement is TRUE? A. Options sales literature is prohibited from containing specific recommendations B. Options sales literature must be accompanied by complete documentation that supports any claims C. Options sales literature that shows performance must be accompanied or preceded by the latest Options Disclosure Document D. Options sales literature is prohibited from containing annualized yield calculations

C. Options sales literature that shows performance must be accompanied or preceded by the latest Options Disclosure Document

The interest rate that fluctuates the least is the: A. Federal Funds Rate B. Call loan rate C. Passbook savings rate D. London Interbank rate

C. Passbook savings rate The federal funds rate changes day to day, as does the London Interbank rate. Call loan rates also vary with the current level of interest rates.

All of the following must be registered under state blue sky laws EXCEPT: A. Sales Representatives B. Broker-Dealers C. U.S. Government Issues D. Real Estate Investment Trust Issues

C. U.S. Government Issues Issues that are exempt from registration under Federal laws are also exempt under state laws, so U.S. Governments do not have to be registered with the state. However, sales reps, broker-dealers, and non-exempt issues (REITS) must be registered.

A limited partnership account is being opened at your firm. The general partner has provided the partnership tax ID number and has signed the appropriate paperwork. What other documentation is needed to open the account? A. social security numbers of each limited partner B. signature of each limited partner C. certificate of limited partnership D. authorizing resolution

C. certification of limited partnership

Which of the following annuity payment options will continue payments to another person for their life after the annuitant dies? A. life annuity B. life annuity with period certain C. joint and last survivor annuity D. unit refund annuity

C. joint and last survivor annuity

Which statement is TRUE when a short sale is effected for a client? A. the broker-dealer can require the short seller to provide the stock to the lender within 3 business days. B. the broker-dealer can require the short seller to provide the stock to the lender within 5 business days. C. the broker-dealer can require the short seller to provide the stock to the lender at any time. D. the broker-dealer can require the short seller to provide the stock to the lender only when the client has a loss.

C. the broker-dealer can require the short seller to provide the stock to the lender at any time.

A new client with no other investment assets has just come into an inheritance of $500,000 of ABCD stock, a blue chip company listed on the NYSE. As the adviser to this customer, your IMMEDIATE concern should be: A. whether the company is a candidate for delisting B. the possibility that the value of the stock may decline C. the lack of diversification of the customer's investment D. whether the customer paid any estate tax liability due

C. the lack of diversification of the customer's investment

An officer of a company has acquired shares of that issuer in the open market. If the officer wishes to sell the shares, the officer must meet all of the following requirements EXCEPT: A. filing the form 144 with the SEC B. a maximum of 4 sales per year are permitted C. the stock must be held for 6 months, fully paid D. each sale is limited to the greater of 1% of the outstanding shares; or the weekly average of the prior 4 weeks' trading volume

C. the stock must be held for 6 months, fully paid

A registered representative receives an order from her customer to sell 600 shares of BWGF to be executed in her IRA account. The RR mistakenly executes the order in the wrong account. What action should be taken to correct the error?

Cancel the transaction and reenter it in the correct account after receiving approval by a registered principal Explanation: If a trade is executed in the wrong account due to an error on the part of the registered representative, the best course of action is to cancel the trade and reenter it to the correct account. This action must be approved by a registered principal or supervisor.

On a when-issued municipal bond transaction, the interim confirmation will show the:

Capacity in which the broker-dealer acted Explanation: A bond trades when-issued (WI) when a final settlement date has not been established. The term when-issued covers the period of a new issue of municipal securities from the original date of sale by the issuer to the delivery of securities to the underwriter. Without a settlement date, accrued interest cannot be calculated and, therefore, the final money amount cannot be calculated. The broker-dealer must disclose the capacity (principal or agent) in which it acted.

When a corporation seeks a bank loan, the bank will base its charge to the corporation on the:

Prime rate Explanation: A corporation pays the prime rate when borrowing from a bank if it is among the bank's best-credit-rated customers. Other corporations pay a higher rate, but it is based on the prime rate.

The recommendation to purchase a private activity bond would NOT be appropriate for a:

Client who is subject to the alternative minimum tax Explanation: A private activity bond is a type of municipal bond in which the funds being raised will be used to benefit a non-public (private) company (e.g., an airport terminal for an airline). If the person receiving the bond's interest payment is subject to the alternative minimum tax (AMT), the interest is taxable at the federal level. For this reason, these bonds are the least suitable for a client who is subject to the AMT.

Co. A Co. B Co. C Co. D Earnings per Share $2.00 $6.50 $5.20 $7.80 Dividends $0.10 $2.50 $2.60 $6.00 Percentage of Retained Earnings 95% 62% 50% 23% An investor has decided to diversify her portfolio into a more defensive position by including utility stocks. Which of the above companies is probably a utility?

Company D Explanation: Company D is probably a utility since utility companies usually have a high dividend payout ratio and a low percentage of retained earnings.

You discover that one of your clients is on the OFAC list. You must:

Contact federal law enforcement authorities immediately Explanation: You must contact the federal law enforcement authorities immediately if you discover that a client is on the list of suspicious persons and entities maintained by the Office of Foreign Assets Control (OFAC). You must also freeze the account and block further transactions.

Your client owns a convertible bond that has been called at 104. The bond is convertible at 40 and is selling in the market at 107. The common stock is selling in the market at 41. Which of the following choices is the LEAST attractive alternative to the client? a. Allow the bond to be called b. Sell the bond c. Convert the bond to common stock and sell the common stock d. All of the the alternatives are equally attractive

Convert the bond to common stock and sell the common stock Explanation: To find the conversion ratio, divide the par value of the bond by the conversion price ($1,000 divided by 40 = 25). The common stock is selling at $41. Converting the bond to common stock and selling the stock gives the client $1,025 (25 shares x $41 = $1,025). Since this is less than the client will receive by selling the bond ($1,070) or allowing the bond to be called ($1,040), it represents the least attractive alternative.

ABC Corporation has issued a call notice on its 5% convertible preferred stock. The preferred stock, which is convertible at $20, is being called at $110 and is currently trading at $111. If ABC's common stock is currently trading at $23, what should an RR recommend to an investor who holds the preferred stock?

Convert the preferred stock into common stock and sell the common stock Explanation: To determine the best choice for the investor, let's evaluate each possibility separately. The preferred stock may be converted into five shares of common stock ($100 par value ÷ $20 conversion price). The five common shares may then be sold at the market price of $23, which provides the investor with a total of $115. If the stock is sold, the investor will receive the current market price of $111. If the stock is called, the investor will receive the call price of $110. The investor is not able to continue holding the preferred stock since it is being called. By converting and selling the common stock, the investor will be provided with the greatest amount of proceeds.

A customer is short 1 ABC Jan 50 put @ 7. The put is exercised when the market price of ABC is $45. The customer liquidates the stock position 3 weeks later at $49 per share. Upon exercise, the tax consequence is a:

Cost basis of $43 per share if a short put is exercised, the writer is required to buy the stock at the strike price ($50). Since $7 was received in premiums, this reduces the cost of the stock in the cost basis. 50-7=43

Customers A, B, C and D have their portfolio assets allocated as follows: A B C D Money Markets 15% 5% 5% 0% Treasury Bonds 40% 10% 20% 20% Speculative Bonds 10% 30% 10% 30% Blue Chip Equities 15% 15% 20% 10% Small Cap. Equities 10% 10% 30% 5% Emerging Markets 10% 20% 10% 30% REITs 0% 10% 5% 5% Which customer's portfolio is MOST susceptible to political risk?

Customer D Political risk is only associated with making foreign investments in developing Third World countries where the political and legal systems are not strong. These weak governments can be overthrown, and the new governments can "nationalize" private industry, giving the shareholders essentially "nothing" for their investment. Since Customer D has the greatest percentage of assets invested in Emerging Markets stocks, this investor is most susceptible to political risk.

What is TRUE about the preliminary prospectus used during the period when a new issue is "in registration"?

Customers that would be prospective buyers of the offering may be sent the preliminary prospectus

If a customer wishes to open an account to trade options, the account must be approved:

Prior to the time an initial order is accepted Explanation: If a customer wishes to open an account to trade options, the account must be approved by an ROP prior to the time an initial order is accepted.

A customer interested in income and capital gains would most likely choose which of the following investments? (A) stripped treasuries (B) growth stocks (C) zero coupon corporate bonds (D) mutual funds

D - Mutual Funds

On the floor of the CBOE, duties similar to those performed by the NYSE specialist (DMM) are handled by the:

market maker

Your customer has a Coverdell Education Savings Account for each of four preteen daughters. What is the maximum amount of pretax contributions that he can make to each ESA? A) 2000. B) 500. C) 8000. D) 0.

D) 0. Pretax contributions cannot be made to Coverdell ESAs. The customer is allowed to make a $2,000 after-tax contribution annually for each student until their 18th birthday.

If an investor opens a new margin account and buys 100 shares of DEF at 39, with Regulation T at 50%, what is the investor's initial margin requirement? A) 3900. B) 1950. C) 975. D) 2000.

D) 2000. In a new margin account, for first trades between $2,000 and $4,000 the initial margin requirement is $2,000.

For reporting purposes, an order to sell 25 shares of an OTC equity security priced at $230 per share is: A) 1 round lot. B) 1 odd lot. C) 25 odd lots. D) 25 round lots.

D) 25 round lots. For OTC equity securities trading at or above $175 per share, 1 share is considered to be a round lot unit of trading. Therefore all last sale information will be disseminated for any transaction of one share or more.

If a member wishes to appeal an adverse decision in a Code of Procedure hearing, the member first must appeal to the National Adjudicatory Council within how many days of the decision date? A) 30. B) 40. C) 45. D) 25.

D) 25. If either side is displeased with a Code of Procedure decision, an appeal must be made within 25 days of the decision date.

Your 65-year-old client owns a nonqualified variable annuity. He originally invested $29,000 4 years ago; it now has a value of $39,000. If your client, who is in the 28% tax bracket, makes a lump-sum withdrawal of $15,000, what tax liability results from the withdrawal? A) 3800. B) 0. C) 4200. D) 2800

D) 2800 This annuity is nonqualified, which means the client has paid for it with after-tax dollars and has a basis equal to the original $29,000 investment. Consequently, the client pays taxes only on the growth portion of the withdrawal ($10,000). The tax on this is $2,800 ($10,000 x 28%). Because the client is older than age 59-½, he does not pay 10% premature distribution penalty tax.

An affiliate or insider holding unregistered shares can sell under Rule 144: A) 1 time a year. B) 2 times a year. C) 12 times a year. D) 4 times a year.

D) 4 times a year. Rule 144 allows an affiliate to sell the greater of 1% of the outstanding shares or the average of the last four weeks' trading volume with each Form 144 filing. The filing is good for 90 days, which would allow for as many as four filings per year.

A municipal bond with a 6% coupon is priced at a 7.20 basis. If the bond's yield to maturity increases by 40 basis points, the yield to maturity is: A) 5.60% B) 6.40% C) 6.80% D) 7.60%

D) 7.60% The term priced at a 7.20 basis refers to a serial bond that is priced to yield 7.20 or a YTM of 7.20%. If the bond's basis increased by 40 basis points, the new yield to maturity is 7.60%. The 6% coupon rate is relevant if the question asked about whether the bond was trading at a discount or a premium. Since the YTM is greater than 6%, the bond is trading at a discount.

A registered representative is provided with the following financial information concerning a company: Debt of $225 million, par value of the common stock $40 million, paid-in capital of $70 million, and retained earnings of $750 million. The common stock ratio is: A) 21% B) 26% C) 74% D) 79%

D) 79% The common stock ratio is found by dividing total shareholder equity by a company's total capital. Shareholder equity is equal to the par value of the common stock + paid-in capital + retained earnings, and the total capital is found by adding the debt to shareholder equity. The common stock ratio is 79% [par value of the common stock is $40 million + paid-in capital of $70 million + retained earnings of $750 million = $860 million / $1,085 million ($225 million + $860 million)]. The common stock ratio is used to analyze the capital structure of a company.

As a requirement of investing in a particular investment, your customer has just signed a statement attesting to his annual income, net worth and affirming that the risks associated with the investment are understood. Which of the following investments would have such a requirement? A) A variable annuity B) A collateralized mortgage obligation C) A hedge fund D) A direct participation program

D) A direct participation program Investors purchasing limited partnership participations or DPPs are required to sign a subscription agreement. In part the investor would be attesting to annual income, net worth and that they understand the risks associated with the type of program they are investing in. While suitability would be a factor for each of the investments listed, they do not require this type of statement be signed by the customer.

ABC Investors Group is looking to do a leveraged buyout of the XYZ Corporation. Which of the following would be the most likely source of capital to fund this takeover? A) XYZ's liquid assets. B) ABC's liquid assets. C) XYZ's creditors. D) A long-term bank loan.

D) A long-term bank loan. The word "leverage" generally implies the use of borrowed money. The acquiring company will either issue a bond or borrow from a bank and use that money to fund the acquisition. In most cases, the assets of the target company are used as collateral for the loan.

For which of the following would the net revenue to debt service ratio be applicable? A) GO bonds. B) School bonds. C) Tax anticipation notes. D) Hospital bonds.

D) Hospital bonds. This is the Coverage ratio. Because revenue bonds are only backed by funds generated by a specific source, it is important that net revenues exceed debt service requirements. Hospitals are often built with the proceeds of revenue bond issues.

Long term securities issued by municipalities that use a Dutch auction method to reset short term interest rates known as "clearing rates" are: A) American Depositary Receipts (ADRs). B) Real Estate Investment Trusts (REITs). C) Collateralized Mortgage Obligations (CMOs). D) Auction Rate Securities (ARSs).

D) Auction Rate Securities (ARSs). Auction Rate Securities (ARSs) are long term securities issued by municipalities that use a Dutch auction to reset interest rates at short term intervals. The reset rate is known as the "clearing rate" and establishes the rate paid during the period following the auction

A significant increase in which of the following types of orders may cause a bull market to accelerate? A) Short sales. B) Sell stops. C) Buy limits. D) Buy stops.

D) Buy stops. If the market is rising, only those orders on the order book above the current market will be executed. Buy stops and sell limits are both entered above the prevailing market price. Of these two, only buy orders (in this case buy stops) will accelerate a rise in the market.

An employee not covered under his company's pension plan has been contributing to a traditional IRA for 5 years. If he leaves his current job, starts a new job, and is covered under the new corporation's pension plan, which of the following statements is TRUE? A) The money in his IRA must be combined with any money he will receive from the pension plan. B) Contributions to his IRA must stop; the money in the account will be frozen, but interest and dividends can accrue tax-free until he retires. C) His traditional IRA must be closed. D) Contributions to his traditional IRA may continue.

D) Contributions to his traditional IRA may continue. An employee covered under a qualified retirement plan may continue to own and contribute to an IRA. The contributions to a traditional IRA may not be fully tax-deductible, depending on the amount of compensation earned, but the employee benefits from the tax deferral of IRA earnings.

Which of the following plans requires an actuary's services? A) Defined contribution. B) 401(k). C) Profit-sharing. D) Defined benefit.

D) Defined benefit In a defined benefit plan the payout is established, and employers must contribute annually to assure payment of the benefit amount. An actuary must calculate the annual contribution amount necessary to meet the benefit requirement.

Which of the following taxes does NOT impact the holder of an ADR? A) Foreign income tax. B) State income tax. C) Federal income tax. D) Excise tax.

D) Excise tax. Dividends on ADRs are subject to both federal and state income tax. In addition, the country of origin will frequently levy a tax which may be used as a credit on the investor's federal income tax return.

A municipal bond dealer is making a bona fide quote. Which of the following statements regarding such a quote is TRUE? The quote must have a reasonable relationship to fair market value. The quote may take into consideration any anticipated market movement. The quote cannot represent an offer to sell bonds the dealer does not currently own. The quote need not be one that the dealer is prepared to act upon (buy or sell). A) III and IV B) II and III C) I and IV D) I and II

D) I and II A bona fide quote is one the dealer is prepared to buy or sell on, as opposed to a workable, nominal or subject quote. A bona fide quote must have a reasonable relationship to fair market value and can be made in consideration of any anticipated market movement. On the offer side of a bona fide quote a dealer may make an offer to sell bonds that it does not hold in its own inventory but must know where to obtain the bonds if they are needed to complete the transaction.

An individual is employed as a research analyst for a member firm that specializes in investment banking and has just completed a research report comparing two companies in the semiconductor business. Which of the following would be considered prohibited activities by this analyst, under FINRA rules? I. Purchasing shares of a semiconductor company before that issuer's IPO. II. Trading in these two stocks or their derivatives in a manner inconsistent with that analyst's recommendation. III. Purchasing shares of either of these two stocks for a personal account after the research report has been issued. IV. Purchasing shares of the XYZ Semiconductor Fund, a fund qualifying as a diversified management investment company under the Investment Company Act of 1940 but not covered or analyzed in the research report. A) II and III. B) III and IV. C) I and IV. D) I and II

D) I and II FINRA rules restrict personal trading by research analysts. They are never permitted to acquire shares in advance of an IPO in a company in the same type of business that the analysts research. They are never permitted to engage in trading contrary to their opinions, as published in their firm's research reports. Once the report has been issued, they may trade in accordance with their recommendations. Purchasing shares of a mutual fund, even one that specializes in their field of research, is permitted.

Which of the following are TRUE of municipal securities quotations? I. A quotation can be an indication of interest. II. A quotation cannot be an indication of interest. III. A quotation can be a one-sided request for a bid or offer (bids wanted and offers wanted). IV. A quotation cannot be a one-sided request for a bid or offer (bids wanted and offers wanted). A) II and IV B) II and III C) I and IV D) I and III

D) I and III MSRB rules pertaining to quotations cover all bona fide bids and offers including one-sided requests for bids wanted and offers wanted which are considered indications of interest.

Which of the following statements describe the conduit theory of taxation? I. A fund is not taxed on earnings it distributes provided distributions equal 90% or more of net investment income. II. Earnings distributed by a regulated investment company are taxed three times. III. Dividends and interest are passed through to the investor without the fund being taxed. IV. Dividends and interest accumulate tax free to the shareholder. A) II and IV. B) II and III. C) I and IV. D) I and III

D) I and III Under the conduit, or pipeline, theory of taxation, a fund is liable for taxes only on the income retained, provided it distributes at least 90% of its net investment income. The investor benefits because the income is only taxed twice (at the corporate level and at the individual level), and avoids taxation at the fund level. There is no tax-free accumulation for the shareholder.

The initial confirmation of a when-issued municipal bond contains which of the following? I. Number of bonds involved in the transaction. II. Settlement date. III. Yield to maturity. IV. Total dollar amount due. A) I and II. B) III and IV. C) II and IV. D) I and III.

D) I and III. On a new municipal bond offering, where the customer receives a when-, as-, and if-issued confirmation, the final settlement date is not known; therefore, the amount of accrued interest is unknown (because it is payable up to but not including settlement). Thus, the total dollar amount is unknown because it includes accrued interest. The number of bonds purchased and the yield to maturity (price) are known and must be included on the confirmation.

A member firm has decided to allow a registered representative to operate from his residence. Which of the following statements are TRUE? I. Prior consent of the member's self regulatory organization (SRO) is required. II. Prior consent of the member's self regulatory organization (SRO) is not required. III. The residence may be advertised. IV. The residence may not be advertised. A) II and IV. B) II and III. C) I and IV. D) I and III.

D) I and III. Prior approval of the member firm's SRO is required when opening any office, including a private residence. The home address and telephone number may be advertised in any normal manner such as business cards, stationery, local newspapers, and so forth.

A customer calls his registered representative and asks that the firm hold his mail as he will be traveling for an extended period of time. Under the rules governing requests to hold mail, which of the following statements are TRUE? I. The request must be made in writing with a specific time period designated. II. For the sake of convenience, the customer can request any length of time for mail to be held. III. Under the rules, the broker dealer is obligated to grant any reasonable request to hold mail. IV. During the time that mail is being held, the broker dealer must still be able to communicate with the customer. A) II and III B) I and III C) II and IV D) I and IV

D) I and IV All requests to hold mail must be made in writing with a specific period of time designated up to three months. While requests for longer periods can be made and granted, the rules specifically prohibit it merely for the sake of convenience. Broker dealers must still be able to communicate with customers in a timely manner and are required to apprise the customer of ways in which that can or will be done. While these requests are commonly granted, doing so is not required under the rule. It is considered a courtesy that the a broker dealer may or may not accommodate.

Which of the following regarding a Roth IRA are TRUE? I. The contributions are nondeductible. II. Contributions must cease at age 70½. III. Withdrawals must begin at age 70½. IV. Withdrawals after age 59½ can be tax free. A) II and IV. B) I and III. C) II and III. D) I and IV.

D) I and IV. With a Roth IRA, the contributions are not deductible from current income. Withdrawals after age 59½ are tax free, provided the account has been open for at least 5 years. There is no age at which withdrawals must begin or contributions must cease.

The interest on which of the following municipal securities may be considered preference income for alternative minimum tax purposes?

Private purpose bonds

Which of the following are signs that a customer may be engaged in money laundering? I. Excessive journal entries between related accounts. II. Excessive journal entries between unrelated accounts. III. Lack of concern for risk, commissions, and other transaction costs. IV. Excessive concern for risk, commissions, and other transaction costs. A) II and IV. B) I and III. C) I and IV. D) II and III.

D) II and III. Examples of red flags include a lack of concern regarding risk, commissions, and other costs, and a large number of wire transfers to, or journal entries between, unrelated third parties.

Rank the following in the usual sequence of order allocation. I) Syndicate. II) Member at the take down. III) Presale. IV) Designated. A) II, IV, III and I. B) I, II, III and IV. C) III, II, IV and I. D) III, I, IV and II

D) III, I, IV and II The standard order priority for allocation of municipal bond issues is (as stated within the syndicate letter): presale, syndicate, designated, and member. Orders that benefit all syndicate members have the highest priority.

Which of the following bonds trade flat? A) GO bonds. B) Mortgage bonds. C) Revenue bonds. D) Income bonds.

D) Income bonds. Bonds that trade flat do not trade with accrued interest. These include income bonds (also known as adjustment bonds), zeroes, bonds in default, and bonds that settle on an interest payment date.

If an investor buys 300 shares of FLB, and one month later buys 1 FLB Jul 50 put, how does this affect the holding period on his or her stock? A) It stops the holding period on 300 shares. B) It ends the holding period on the put. C) It has no impact on the holding period for any of the shares owned by the investor. D) It stops the holding period on 100 shares.

D) It stops the holding period on 100 shares. The put purchase ends the holding period for any shares the put subsequently allows the holder to sell. Because the holder owns 1 put, this stops the holding period on 100 shares owned. The other 200 shares are unaffected.

At a social gathering, an officer of a publicly traded company confides to his neighbor, a registered representative, that his company will announce a major acquisition in the coming week. Which of the following statements regarding the SEC's insider trading rules is TRUE? A) The officer is in violation. B) Both the officer and the registered representative are in violation. C) The registered representative is in violation. D) Neither the officer nor the registered representative is in violation.

D) Neither the officer nor the registered representative is in violation. Simply giving someone material, nonpublic information (while imprudent) is not a violation. However, if the information is used to trade for profit or to avoid a loss, both the tipper and the tippee would have violated the law.

Which of the following is the computation for the coverage ratio for a municipal revenue bond issue? A) Revenues collected divided by annual interest expense. B) Annual interest and principal expense divided by revenues collected. C) Revenues collected divided by annual principal expense. D) Net revenue divided by annual interest and principal expense.

D) Net revenue divided by annual interest and principal expense. Debt service coverage measures the amount of money available for debt service compared to the annual debt service requirements. Annual debt service includes both interest and principal expense

Mr. Jones calls his registered representative and places an order to write an XYZ Oct 90 call and at the same time to write an XYZ Oct 80 put. The orders are executed at a premium of 5 for the call and 9 for the put. Which of the following best describes the customer's investment strategy? A) Bearish strategy. B) Mixed strategy. C) Bullish strategy. D) Neutral strategy

D) Neutral strategy A customer who writes both a call and a put on the same underlying security wishes for little or no market movement. This is referred to as a neutral strategy. Technically, the customer has created a short combination (an investment position very similar to a short straddle, with the same investment characteristics), and, in this case, a little less risk than a pure straddle because of the spread in the strike prices.

What is a bank-qualified municipal issue? A) One considered safe enough for a bank to invest in-same as investment grade. B) An escrow receipt. C) One in which the bank guarantees the payment of interest and principal. D) One that receives preferential treatment by allowing a bank to exclude from gross income 80% of the interest expense incurred to carry the bonds

D) One that receives preferential treatment by allowing a bank to exclude from gross income 80% of the interest expense incurred to carry the bonds A bank-qualified municipal issue is one that receives preferential treatment by allowing a bank to exclude from gross income 80% of the interest expense incurred to carry (issue) the bonds. An issue is qualified if it is for a public purpose and the issuer issues no more than $10 million in the calendar year of the issue. Bank qualified has no bearing on the quality of the issue.

Which of the following companies would probably be MOST leveraged? A) Software B) Biotech C) Consumer electronics D) Utilities

D) Utilities A leveraged company has a large amount of outstanding debt (bonds) and would be the most leveraged. Of the choices given, utilities are the heaviest users of debt and have the greatest amount of interest charges (fixed charges). The percentage of debt in a utility company's capitalization is usually greater than that of the other companies listed

Which of the following materials is subject to FINRA's filing requirements? A) Prospectus for a closed-end management investment company. B) Internal memo describing the benefits of an investment in a certain unit investment trust. C) Prospectus for a face amount certificate company. D) Retail communications for an open-end management investment company.

D) Retail communications for an open-end management investment company. Retail communications for investment companies are subject to filing requirements with FINRA. Those that include a performance ranking that is either created by the investment company or from a source that is not regularly published require pre-filing. Those that do not include such a performance ranking can be filed within 10 business days of first use. Prospectuses and internal memos need not be filed with FINRA.

For the purpose of reporting sales to the IRS, which method available to investors by the IRS offers the most flexibility in anticipation of the investor's year-end tax needs? A) None offer any flexibility in anticipation of year-end tax needs B) Average cost basis C) First in, first out (FIFO) D) Share identification

D) Share identification Share identification is the most flexible of the three methods. The investor keeps track of the cost of each share purchased and specifies which shares to sell based on his anticipated year-end tax needs. For investors, the idea is to minimize tax liability if able by limiting gains or maximizing loses in anticipation of what one's year-end tax liability might be.

Which of the following will halt trading in listed options when there is a trading halt in the underlying stock? A) The exchange on which the stock is listed. B) The SEC. C) The OCC. D) The options exchange on which the option is listed.

D) The options exchange on which the option is listed. If trading is halted in any stock on which options trade, trading in those options is also halted by the CBOE.

If a 40-year-old customer earns $65,000 a year and his 38-year-old spouse earns $40,000 a year, how much may they contribute to IRAs? A) Only the higher wage earner may contribute to an IRA. B) They may not contribute because their combined income is too high. C) They may contribute up to the maximum annual allowable dollar limit split evenly between both accounts. D) They may each contribute 100% of earned income or the maximum annual allowable dollar limit, whichever is less, to an IRA.

D) They may each contribute 100% of earned income or the maximum annual allowable dollar limit, whichever is less, to an IRA. No matter how much income individuals or couples receive, they may contribute to their IRAs if they have earned income. Each is entitled to contribute 100% of earned income up to the maximum allowed. However, if either or both of them are covered under a qualified plan, limits may exist on the deductibility of the contributions.

If a stock's ex-dividend date is Tuesday, January 13, when is the record date? A) Tuesday, January 20. B) Wednesday, January 7. C) Thursday, January 8. D) Thursday, January 15.

D) Thursday, January 15. The record date is two business days after the ex-dividend date (Thursday, January 15).

Each of the following is affected by the sale of securities in a restricted margin account

market value. SMA. debit balance

The Trust Indenture Act of 1939 covers all of the following securities transactions EXCEPT: A) a sale of an equipment trust bond issue worth $6 million. B) a public issue of debentures worth $5.5 million sold by a single member firm throughout the United States. C) a corporate bond issue worth $10 million sold interstate. D) a sale of an issue of $5 billion worth of Treasury bonds maturing in 2011.

D) a sale of an issue of $5 billion worth of Treasury bonds maturing in 2011. The Trust Indenture Act of 1939 requires all corporate debt issues of $5 million or more sold interstate to have a trust indenture; U.S. governments are exempt.

An accumulation unit in a variable annuity contract is: A) none of these. B) an accounting measure used to determine payments to the owner of the variable annuity. C) fixed in value until the holder retires. D) an accounting measure used to determine the contract owner's interest in the separate account.

D) an accounting measure used to determine the contract owner's interest in the separate account. When money is deposited into the annuity, it is purchasing accumulation units.

Customers could pay a commission, rather than a sales charge, for shares of a(n): A) open-end investment company. B) mutual fund. C) front-end load fund. D) closed-end investment company.

D) closed-end investment company Sales charges could be paid on all types of open-end funds. Commissions are paid on securities traded in the secondary market, such as closed-end investment company shares.

All of the following securities are exempt from the registration provisions of the Securities Act of 1933 EXCEPT: A) national and state bank securities. B) state and municipal bonds. C) commercial paper and bankers' acceptances that have maturities of no more than 270 days. D) commercial bank holding company securities

D) commercial bank holding company securities Commercial bank holding companies are corporations that have to register with the SEC. State and municipal bonds do not have to be registered under the Securities Act of 1933. Commercial paper and bankers' acceptances that have maturities of no more than 270 days are exempt from the registration provisions. National and state banks are regulated by various state and federal agencies.

If a customer buys 100 shares of stock and writes one out-of-the-money call against his long position, the breakeven point is the: A) cost of stock purchased plus premium. B) strike price less premium. C) strike price plus premium. D) cost of stock purchased less premium.

D) cost of stock purchased less premium. When the investor owns stock and sells a call, the call is covered. Breakeven is computed by subtracting the premium from the stock's purchase price.

A member firm broker/dealer wishing to go public may sell a new equity issue of its own securities to all of the following EXCEPT: A) public customers. B) owners, officers, and employees of the firm. C) family members of owners, officers, and employees of the firm. D) employees of other full-service member firms

D) employees of other full-service member firms Rules regarding restricted persons generally prohibit member firms from selling new issue securities to employees of member firms including their own. However, when member firms sell their own securities, rules regarding restricted persons do not apply to the issuer's own employees but still apply to the employees of other full-service member firms.

The issuer of an ADR is a: A) domestic branch of a domestic bank. B) domestic branch of a foreign bank. C) foreign branch of a foreign bank. D) foreign branch of a domestic bank.

D) foreign branch of a domestic bank. The American Depositary Receipt (ADR) is issued by a foreign branch of a domestic bank. Everything is in English and in U.S. dollars.

The Securities Exchange Act of 1934 regulates or mandates each of the following EXCEPT: A) extension of credit to customers. B) manipulation of the secondary market. C) creation of the SEC. D) full and fair disclosure on new offerings.

D) full and fair disclosure on new offerings. The Securities Exchange Act of 1934 created the SEC and regulates the secondary market. The Securities Exchange Act of 1934 does not address full and fair disclosure issues; the Securities Act of 1933 addresses such issues.

In a seller's option, securities may be delivered before the date specified if the seller A) cannot deliver on the specified date. B) wishes to be paid earlier. C) gives notice to the buyer on the day of delivery . D) gives 1 day's written notice to the buyer.

D) gives 1 day's written notice to the buyer. In a seller's option trade, the seller may (at his option) give the buyer written notice 1 day before making delivery.

Under FINRA rules, a registered representative is permitted to borrow money from a customer: A) under no circumstances. B) if written notification is given to the firm. C) without restriction. D) if written notification is given to the firm and the representative receives written approval.

D) if written notification is given to the firm and the representative receives written approval. Firms are not required to permit lending arrangements between registered representatives and their customers. If they do, they must have procedures in place to monitor such arrangements. If permitted by the firm, the arrangement must fall into one of five permissible categories: the customer is a member of the representative's immediate family; the customer is in the business of lending money; the customer and the representative are both registered with the same firm; the arrangement is based on a personal relationship outside of the customer/representative relationship; or the arrangement is based on a business relationship outside of the customer/representative relationship. If permitted by the firm, the representative must advise the firm in writing of the proposed borrowing, and receive written permission.

A workable indication for a block of municipal bonds is a: A) likely offer. B) firm bid. C) firm offer. D) likely bid.

D) likely bid. A bid is the price at which a dealer states its willingness to purchase securities from another broker/dealer. A dealer soliciting a workable indication is often working to satisfy a customer's order to sell securities. A dealer giving a workable indication is free to revise its bid if market conditions change.

All of the following orders could be placed on the specialist (designated market maker) order display book EXCEPT: A) stop limit orders. B) limit orders. C) stop orders. D) market orders.

D) market orders. Market orders are executed immediately. The order display book is for orders that are away from the current market, such as stop and limit orders.

An associated person of a broker dealer who wishes to accept an outside employment opportunity must notify the A) customers of the broker dealers in writing B) SEC in writing C) FINRA in writing D) member firm in writing

D) member firm in writing Notification must be made to the member firm in writing.

An investor seeking a high level of income combined with a moderate level of risk would purchase: A) income bonds. B) junk bonds. C) convertible bonds. D) mortgage bonds.

D) mortgage bonds. Bonds provide a semiannual stream of fixed income. Because convertible bonds normally have a lower coupon rate than nonconvertible bonds-and income bonds only pay interest if the company declares a payment-the best choice is the mortgage bond, which is secured by real estate.

All of the following kinds of orders may be turned over to the specialist (designated market maker) for execution EXCEPT: A) market. B) limit. C) stop. D) not-held.

D) not-held. A not-held order (NH) is a market order in which the investor has given the authority to choose the price and time to the floor broker to achieve the best possible execution.

Under FINRA rules, if a member firm receives an order to buy a new equity issue on behalf of an undisclosed principal from a bank, the member must: A) determine the identity of the purchaser. B) accept the order. C) reject the order. D) obtain a representation from the bank that the purchaser is not restricted.

D) obtain a representation from the bank that the purchaser is not restricted. If a member receives an order from a conduit such as a bank, the member must make an inquiry as to whether the ultimate purchaser is restricted. It is not necessary to determine the identity and business affiliations of the purchaser.

The Trust Indenture Act of 1939 applies to each of the following corporate debt offerings EXCEPT: A) interstate offerings. B) offerings over $5 million. C) nonexempt debt securities. D) offerings under $5 million.

D) offerings under $5 million. Corporate debt offerings under $5 million and exempt issues are not subject to the Trust Indenture Act of 1939.

An issuer may direct sales of a new issue to all of the following EXCEPT: A) officers of the issuer. B) officers of its largest customer. C) officers of its largest supplier. D) officers of the managing underwriter

D) officers of the managing underwriter Issuer-directed sales are permitted if the persons to whom the new issue is sold are not restricted. Officers of the managing underwriter are restricted.

The principal underwriter of an open-end investment company is also known as the: A) registrar. B) dealer. C) trustee. D) sponsor.

D) sponsor. A mutual fund's underwriter is also known as the sponsor or distributor of the fund

Margin requirements on exempt securities (U.S. government securities and municipal securities) are set by: A) the FRB. B) the DOE. C) the SEC. D) the DEA.

D) the DEA. The FRB sets the initial margin requirements for nonexempt securities. The margin requirements for exempt securities, such as U.S. governments, are set by a firm's SRO or DEA (designated examining authority).

The determination as to whether an OTC stock is eligible for purchase on margin is made by: A) FINRA. B) the SEC. C) the FDIC. D) the Federal Reserve Board.

D) the Federal Reserve Board. All decisions regarding initial margin eligibility are the role of the Federal Reserve Board.

A registered representative opens a new account for an investment club. His spouse is a member of the club and owns 15% of the club's assets. The registered representative wants to sell shares of a common stock IPO to the investment club. This is allowed: A) with written principal approval. B) with written notice to the SEC. C) only if the IPO is suitable for the investment club. D) under no circumstances.

D) under no circumstances. Rules prohibit member firms from selling common stock IPOs to restricted persons. Under the rules the account would not be restricted if the assets owned by the spouse composed less than 10% of the club's assets. Because the registered representative's spouse is a member of the investment club and owns more than 10% of the club's assets, the registered representative cannot sell shares of the IPO to the club.

To avoid tax and penalty, an IRA may be rolled over once each: A) quarter, by the end of the calendar quarter. B) 3 years, within 90 days. C) 5 years, by the end of the calendar year. D) year, within 60 days.

D) year, within 60 days IRA rollovers, which must be completed within 60 days, may be done no more often than once a year.

All of the following paperwork is required for trades to be effected in an account for a deceased person who held an individual account at a brokerage firm EXCEPT: A. Executor's authorization certificate B. Copy of the death certificate C. Affidavit of domicile D. Third party trading authorization

D.

All of the following would be purchasers of Eurodollar bonds EXCEPT: A. British investors B. French investors C. Japanese investors D. United States investors

D. Eurodollar bonds are not registered with the SEC, and they are not exempt from these registration requirements, they cannot be offered within the U.S.

Which of the following municipal bonds would MOST likely be refunded by the issuer? A. 5% G.O., M '37, callable in 2017 at par B. 6% G.O., M '37, callable in 2017 at 102 C. 7% G.O., M '37, callable in 2017 at 102 D. 8% G.O., M '37, callable in 2017 at par

D. In a refunding, an issuer refinances an outstanding debt by issuing new bonds. The bonds most likely to be refunded are those with the highest interest rates (to be replaced by lower interest rate bonds) and low call premiums (so it will not be too expensive to the issuer to call in the debt for refunding).

A customer sells 2 ABC Jan 15 puts @ 2 when the market price of ABC is $14. If the contracts are closed out at a premium of $4 when the ABC stock is at $13, the gain or loss is: A. $200 gain B. $200 loss C. $400 gain D. $400 loss

D. $400 loss Since the puts were originally sold for $2 and then bought back at $4, there is a 2 point loss per share. There are 200 shares covered by the 2 contracts, so the total loss is $400.

Which statement is FALSE when comparing Agency CMOs to Private Label CMOs? A. Agency CMOs carry the direct or implied guarantee of the U.S. Government while Private Label CMOs do not have such a guarantee B. Agency CMOs are backed by underlying mortgage backed pass-through certificates issued by that agency, while Private Label CMOs are backed only by mortgage backed securities issued by private lenders C. Agency CMOs take on credit rating of the underlying agency securities while Private Label CMOs are assigned credit ratings by independent credit rating agencies D. Agency CMOs are traded in the public markets while Private Label CMOs can only be sold in private placements and cannot be traded

D. Agency CMOs are traded in the public markets while Private Label CMOs can only be sold in private placements and cannot be traded

A customer holds a large portfolio of corporate bonds. The customer is worried about capital risk. Which diversification strategy would be least effective to minimize capital risk for this customer? A. Diversification among differing issuers in differing states B. Diversification among differing industries C. Diversification among differing maturities D. Diversification among differing coupon rates

D. Diversification among differing coupon rates

Which index is the narrowest measure of the market? A. Wilshire Index B. Value Line Index C. NYSE Composite Index D. Dow Jones Industrial Average

D. Dow Jones Industrial Average the DJIA consists of 30 stocks. This is the narrowest measure of the market. Value Line = 1700 issues NYSE = 2500 issues Wilshire = 3500 issues

A corporation issues $100 par convertible preferred stock, convertible at $10 per share, when the market price of the common is currently $5. The preferred is issued under an "anti-dilutive covenant." If the company declares a 25% stock dividend, which statements are TRUE? I. The conversion price remains at $10 II. The conversion price is adjusted to $8 III. The conversion ratio remains at 10:1 IV. The conversion ratio is adjusted to $12.5:1 A. I and III B. I and IV C. II and III D. II and IV

D. II and IV Under an "anti-dilutive" covenant, if there is a stock split or stock dividend resulting in the issuance of additional common shares, the conversion price and hence the conversion ratio are adjusted to reflect the fact that the market price of each common share will drop on the ex date. Prior to the stock dividend, the conversion price was $10 per share. If there is a 25% stock dividend, the new conversion price will be adjusted to $10/1.25 = $8 per share. Since each preferred share is $100 par, the new conversion ratio will be $100/$8 = 12.5

Which of the following is NOT required to be disclosed when a brokerage firm makes a securities recommendation? A. whether the firm is an investment banker for that issuer B. whether the firm is a market maker for that issuer C. whether the firm's officers own that issuer's securities D. Whether the issuer's employees own that issuer's securities

D. Whether the issuer's employees own that issuer's securities

Under NYSE rules, every "responsible broker or dealer" who communicates bids and offers on the exchange floor (also known as "addressing the crowd") must comply with all of the following rules EXCEPT: A. any bid or offer for less than the normal trading unit has no standing in the trading crowd. B. the highest bid and lowest offer have precedence in all cases. C. bids and offers must be publicly announced D. bids and offers are set by floor officials during unusual situations

D. bids and offers are set by floor officials during unusual situations.

An existing customer of a brokerage firm wishes to buy an initial public offering that he has heard good things about. The registered representative only has a limited number of shares to sell and explains to the customer that the offering is already "sold out". The customer tells the registered representative "If you can get me some of the stock, I will drop off cash payment in full today, with a little extra for you." The registered representative: A. can accept the customer's offer without restriction B. can accept the customer's offer if the cash payment does not exceed $500 C. can accept the customer's offer if the gift is reported to the IRS D. cannot accept the customer's offer

D. cannot accept the customer's offer

All of the following are terms that describe economic indicators EXCEPT: A. leading B. lagging C. coincident D. convergent

D. convergent Economic indicators either lag the economy, lead the economy; or are coincident with the economy. There is no such thing as a convergent indicator.

All of the following sources of REIT income are counted towards the 75% test required by Subchapter M EXCEPT: A. net rental income B. interest income from mortgages C. real estate tax refunds D. dividend income from investments

D. dividend income from investments

All of the following are true statements about variable annuities EXCEPT: A. the portfolio funding the separate account is professionally managed. B. the portfolio is invested in other management company shares. C. dividends and capital gains must be reinvested until annuitization occurs. D. investors get an interest rate guarantee

D. investors get an interest rate guarantee Variable annuity holders do not get an interest rate guarantee.

All of the following statements are true regarding the Order Book Official EXCEPT the OBO: A. is an exchange employee B. runs the book of public orders C. makes a market in options contracts D. does not maintain an options inventory

D. makes a market in options contracts The OBO is an employee of the CBOE who runs the book of public orders. The "OBO" does not make a market in the option contracts and does not maintain inventory - this function is performed by the MM (market maker).

What does a BDC invest in? A. hedge funds B. small cap stocks C. large cap stocks D. privately-held companies

D. privately-held companies A BDC is a business development company. It is a registered investment company under the 1940 Act that is listed and trades like any other stock.

A head and shoulders "bottom" formation is a(n): A. uptrend B. downtrend C. reverse upward trend D. reverse downward trend

D. reverse downward trend A head and shoulders bottom formation is bullish since the market has bottomed out and is now moving back upwards. It is a downtrend that has reversed itself.

Under FINRA rules, a member firm is allowed to vote the stock of securities held in street name: A. if the distributed proxy is not returned within 10 days of the annual meeting B. if the distributed proxy is not returned within 20 days of the annual meeting C. if the distributed proxy is not returned within 30 days of the annual meeting D. under no circumstances

D. under no circumstances

The dividend policy of most money-market funds is to declare dividends:

Daily and pay, credit, or reinvest the dividends on a monthly basis Explanation: Most money-market funds will declare dividends daily and pay, credit, or reinvest the dividends on a monthly basis. This information is usually found in the prospectus of the money-market fund.

Municipal securities Dealer A quotes a price for a block of bonds to Dealer B for one hour with a five-minute recall. This means:

Dealer A may recall Dealer B within the one-hour period and demand a decision of Dealer B in five minutes Explanation: When municipal bonds are offered on a firm basis to Dealer B for one hour with a five-minute recall, the offering dealer, Dealer A, may not offer the bonds to anyone but Dealer B without giving Dealer B the first opportunity to take the bonds. Since the recall period is five minutes, if Dealer A recalls Dealer B, Dealer B then has five minutes to take the bonds or else Dealer A is free to sell the bonds to someone else.

A secondary market exists for:

Dealer-placed commercial paper Explanation: A secondary market exists for owners of commercial paper to sell their investments to dealers or other investors. There is no secondary market for federal funds, repos, or U.S. savings bonds.

Which of the following would be considered in analyzing the credit worthiness of a revenue bond issuer?

Debt service coverage Management

An analysis of a revenue bond issue normally addresses the: a. Per capita debt of the municipality b. Per capita income of the municipality c. Assessed valuations of the municipality d. Debt service coverage ratio

Debt service coverage ratio Explanation: The debt service coverage ratio indicates the number of times pledged revenues cover interest and principal payments due on a bond. This is important for the analysis of a revenue bond, which is backed by a revenue producing facility. The other choices are important for the analysis of a general obligation bond, which is backed by the full faith, credit, and taxing power of the issuing municipality.

The official statement for a revenue bond issue states that the bonds are backed by a pledge of the project's net revenues. This means that the:

Debt service is the first item paid after operating and maintenance expenses Explanation: Most municipal revenue bonds are net revenue pledge bonds. This means that bond (debt) service is paid from net revenue (revenue after operating and maintenance expenses).

An announcement in the financial section of the newspaper states that the money supply (M1) has dropped for the week. This means that:

Demand deposits and currency in circulation in the banking system have declined Explanation: M1 is demand deposits, (checking accounts) checkable deposits, and currency in circulation in the economy, and constitutes what is called the money supply. If the announcement in the financial section of the newspaper states that M1 (the money supply) has dropped for the week, this means that demand deposits (checking accounts) and currency in circulation have declined.

An investor is expecting a sharp decline in interest rates in the near future. To capitalize on this situation, the investor should buy:

Discount bonds with long maturities Explanation: Long-term bond prices are more volatile than short-term bond prices. Discount bond prices are more volatile than premium bond prices. If the investor expects interest rates (yields) to decline, she is anticipating rising bond prices. The bonds that will rise (fluctuate) the most are long-term, discount bonds.

A member bank wishes to borrow money from the Fed. What rate will be charged?

Discount rate Explanation: The member bank is charged the discount rate when it borrows from the Fed.

Which of the following is NOT monitored by a technical analyst?

Dividend payout ratios Explanation: Technical analysts use price and trading volume information. Advance/decline data, chart patterns, and market momentum calculations are all methods used in analyzing this type of information. Dividend payout ratios would be important to a fundamental analyst.

Municipal term bond quotes are based on:

Dollar price Explanation: Municipal term bonds (bond issues that have one maturity date) are quoted based on a dollar price. Term bonds are also known as dollar bonds. Municipal serial bonds (that have several maturity dates) are quoted on a yield-to-maturity basis.

A married couple both have full-time jobs and are covered by their employers' pension plans. If they file a joint tax return and each wanted to open an IRA:

Each may do so in separate accounts and may contribute up to $5,500 in each account Explanation: Since they both have earned income, they may each establish an IRA and contribute $5,500 to each account. Joint accounts are not permitted for IRAs. A person who is covered by a corporate pension plan may continue to make contributions to an IRA.

If a syndicate is formed under a Western account arrangement, what is the treatment of any unsold securities?

Each member is responsible for its unsold securities. Western accounts are divided accounts. That means each member of the syndicate is responsible for any portion of its commitment that remains unsold.

Dynasty Corporation is planning to acquire Regal Corporation. If a trader purchased 12,000 shares of Regal Corporation and sold short 4,000 shares of Dynasty Corporation, the trader is:

Engaging in a risk arbitrage Explanation: When there is an acquisition or merger taking place, traders will try to take advantage of the activity between the common stocks of the two companies. The trader or risk arbitrageur will go long the company being acquired and sell short the shares of the acquiring company. This process is known as risk arbitrage. If the acquisition is successful, Regal Corporation's stock will increase and Dynasty Corporation's stock will decline.

The Order Audit Trail System tracks the:

Entire life of an order that is accepted by a member firm Explanation: The Order Audit Trail System (OATS) enables FINRA to effectively review market activity in regard to customer orders within a member firm, to conduct surveillance, and to enforce rules. OATS records the life of an order from receipt, to routing, to modification if applicable, and cancellation or execution.

When interest rates are trending upward, the economy will normally be in which phase of the business cycle?

Expansion Explanation: Increasing interest rates, along with increased costs and lower unemployment, are frequently associated with an expanding economy where there is an increasing demand for goods. As demand overtakes supply, prices begin to rise due to the scarcity of goods. This rise in prices is known as inflation. The Federal Reserve will look to raise interest rates in an attempt to curb demand and combat inflation.

The call feature on callable bonds is most relevant when the economy is:

Experiencing a slowdown and the FRB is trying to stimulate growth Explanation: The call feature on callable bonds is most relevant when the general level of interest rates is declining. Rates will tend to decline when the FRB is trying to stimulate the economy by increasing the money supply. The goal is to bring down interest rates to allow the economy to grow. Rising inflation usually causes the FRB to decrease the money supply in order to drive up interest rates. If the economy is growing and inflation is stable, this is a beneficial situation and the FRB may simply leave rates unchanged.

Which TWO of the following activities are normally functions of the investment banking department of a broker-dealer? I. Working with issuers to raise capital II. Selling securities to institutional investors III. Assisting companies with mergers and acquisitions IV. Making a secondary market for new issues

Explanation: A corporation that wishes to raise capital will typically employ the services of an investment banker and engage in an underwriting process. Investment bankers provide financing for corporations by bringing an issue, whether debt or equity, to market for the issuer. The investment banking department will also assist companies with mergers and acquisitions. Investment bankers do not make a secondary market for new issues or sell securities to institutional investors.

When opening an account for a customer, MSRB rules do not require the dealer to obtain the customer's: a. Financial condition b. Investment history c. Investment objectives d. Date of birth

Explanation: MSRB rules specifically state that a dealer should make every effort to obtain all of the information listed except the customer's date of birth. The dealer should determine that the customer is not a minor, but not specifically his date of birth.

Compared to selling short, buying a put option:

Explanation: Short selling requires the deposit of margin, whereas the premium on a put is usually substantially less than the Regulation T margin requirement. On a short sale, the seller's risk is unlimited, whereas on a put purchase, the risk is limited to the premium. Although a short sale may be effected only if the stock can be borrowed under Regulation SHO, a put may be purchased at any time. Puts can be purchased in a cash account, while selling short requires a margin account.

In an undivided (Eastern) municipal syndicate account, the remaining liability of an account member is computed:

From the number of bonds that are unsold in the account Explanation: An Eastern account is undivided as to liability. As long as any bonds in the account are still unsold, each member of the account is liable for his proportionate share of the unsold amount. If the member has a 10% participation in a $10,000,000 issue, originally his liability is for $1,000,000 of those bonds. If there is a balance of bonds unsold by other members, he will still have a liability of 10%, whether he sold all or none of his bonds.

FINRA's Trade Reporting Facility (TRF) electronically facilitates the reporting of trade data such as price and volume for A) brokers acting as agents in all order execution scenarios B) trades in Nasdaq-listed securities and exchange-listed securities when they occur off of the exchange trading floor C) trades in NYSE-listed securities occurring on the NYSE D) brokers executing orders as agents in an auction market on any exchange trading floors Your answer, brokers executing orders as agents in an auction market on any exchange trading floors, was incorrect. The correct answer was: trades in Nasdaq-listed securities and exchange-listed securities when they occur off of the exchange trading floor

FINRA's Trade Reporting Facility (TRF) is an automated electronic system that facilitates the reporting of data for transactions that occur in Nasdaq-listed stocks or in exchange-listed stocks when they occur off of the exchange trading floor. It is used for transactions that are negotiated between brokers, therefore acting as a dealer, rather than as an agent. Reference: 8.2.2.5 in the License Exam Manual.

Closing spot prices for foreign currencies are disseminated daily by the:

FRB Explanation: The Federal Reserve Board disseminates closing spot prices of foreign currencies daily.

One of your customers would like to purchase a government agency security for the UTMA account of her daughter. The daughter worked in construction over the summer and would like to use $1,275 of her savings for the purchase. Securities issued by which of these agencies could be purchased for this account?

Federal National Mortgage Association -securities have a min denomination of $1000

The interest rate that fluctuates the most is the:

Federal funds rate Explanation: Short-term rates fluctuate more than long-term rates. The federal funds rate, which is the rate of interest one bank charges another bank for the use of excess reserves for short-term periods (usually overnight), fluctuates the most since it has the shortest maturity. Although long-term bond prices fluctuate more than short-term bond prices, the yields of short-term securities fluctuate more than those for long-term securities.

Which of the following is NOT part of the Secondary Market?

Fifth market Secondary market is categorized into 4 sub-markets; The First, Second, Third, and Fourth Markets.

A broker-dealer is underwriting an initial public offering (IPO) for a company that will be listed on the NYSE. The broker-dealer is required to deliver prospectuses:

For 25 days after the effective date Explanation: When a company that is the subject of an IPO is listed, on the effective date of the offering, prospectuses must continue to be delivered on all purchases in the aftermarket for 25 days. The prospectus delivery requirement for an IPO that will not be listed on an exchange continues for 90 days after the deal closes.

A broker-dealer is underwriting an initial public offering (IPO) for a company that is not eligible to be listed on an exchange. The broker-dealer is required to deliver prospectuses:

For 90 days after the effective date Explanation: When a company that is the subject of an IPO is listed, on the effective date of the offering, prospectuses must continue to be delivered on all purchases in the aftermarket for 25 days. The prospectus delivery requirement for an IPO that will not be listed on an exchange continues for 90 days after the effective date.

A mutual fund buys stock from the portfolio of an insurance company. This is a trade executed in the:

Fourth market Explanation: When an institutional investor such as a mutual fund buys stock from the portfolio of an insurance company (another institution), it is considered a trade executed in the fourth market. This is the name given to the so-called market where institutions trade with other institutions.

In a strong bull market, which of the following positions utilizing leverage has the potential for the highest percentage gain?

Holding calls

A municipal bond dealer buys 100M of 30 year non-callable 9% General Obligation bonds at par less 1 point. After holding the bonds in inventory for a week, the dealer re-offers the bonds on an 8.90 basis. The dealer's approximate profit or loss per bond on this transaction is:

Gain of $21 The dealer purchases these bonds at par less 1 point, so the bonds were purchased at 99. Since these 9% coupon bonds were re-offered on an 8.90 basis, they must have been re-offered at a premium price. Since these are long term bonds (30 years), we can approximate the re-offering price by dividing 9% (nominal yield) by the 8.90 re-offering yield. 9/8.90 = 1.011. Thus, the bonds were re0offered at an approximate price of 1.011% of par. The bonds were re-offered at a price that is .021% higher than the cost to the dealer (.99 cost versus 1.011 re-offer price) .021% x $100,000 face amount = $21 gain on the transaction.

The member of the limited partnership who assumes liability for the debts of the entity and is usually concerned with its overall management is the: a. General partner b. Senior limited partner c. Management committee chosen by the limited partners d. None of the above

General partner Explanation: The general partner is the member of the limited partnership who assumes liability for the debts of the entity and is usually concerned with its overall management. Choices (b) and (c) have no bearing on this question.

A limited partner is considered accepted into a limited partnership when the:

General partner signs and approves the subscription agreement Explanation: The sale of a limited partnership interest is executed by means of a subscription agreement. It is signed by the limited partner, but is not final until the general partner signs the agreement which signifies the acceptance of the limited partner.

Government agency bonds issued by which of the following carry a minimum denomination of $1,000 with $1 increments?

Ginnie Mae

Which of the following securities are guaranteed by the federal government?

Ginnie Mae securities Explanation: Of the choices given, only Ginnie Mae securities or the Government National Mortgage Association securities (GNMAs) are fully guaranteed as to principal and interest by the federal government.

A "qualified" legal opinion is one in which:

Gives a conditional affirmation of the legality of the securities

Short-term municipal obligations payable from funds that usually are received from the federal government are:

Grant anticipation notes Explanation: Grant anticipation notes are short-term municipal notes issued on the expectation of receiving grant money, usually from the federal government.

A tombstone ad states that the McGee Oil Company is offering $200,000,000 of 8 1/2% bonds due July 1, 2038 at 99 1/2% of par value. The yield to maturity on the bonds is:

Greater than 8 1/2% Explanation: The 8 1/2% bonds are being offered at a discount at 99 1/2% of their $1,000 par value. An investor who purchased the bonds at the offering (at $995) and held the bonds to maturity will receive the par value of $1,000. The investor will, therefore, have a yield to maturity that is greater than the coupon rate (nominal yield) of 8 1/2%

A clause in an underwriting agreement that allows an underwriting syndicate to purchase additional shares from the issuer for sale to the public is a(n):

Green Shoe clause Explanation: A clause in an underwriting agreement that allows the syndicate to sell more of an issue than was originally available, and acquire those shares from the issuer, is known as a Green Shoe clause. This clause is found in the offering's overallotment provision and is limited to 15% of the offering.

A put option may be written in a cash account if the investor:

Has a cash balance in the account equal to the total exercise value of the contract Explanation: To write a covered put option in a cash account, the customer must have cash in the account equal to the total exercise value of the contract. If the writer is short the underlying stock, the put is considered covered for margin purposes, but this transaction may not be written in a cash account, only in a margin account.

If an investment representative hosts an investment seminar and intends to discuss general investment concepts and a specific mutual fund for which he has performance charts, which of the following are true?

He may discuss the investment returns of the mutual fund using a specific time frame. He must disclose all material facts regarding the mutual fund to the audience.

Which of the following statements regarding hedge funds is correct?

Hedge funds are usually structured as a partnership.

A bond with an 11% coupon is purchased at 103. The maturity of the bond is 20 years. The bond is callable in 10 years at par. The yield will be:

Higher if held to maturity Explanation: If the bond is held to maturity, the investor will be able to amortize the premium over a longer period, thereby realizing a higher yield. If the bond is called in ten years, the premium is amortized over half the time, resulting in a lower yield. A bond purchased at a premium and callable at par will always have a lower yield to call than to maturity. The opposite is true for a bond purchased as a discount callable at par. The yield to maturity will be lower than the yield to call.

A registered representative wants to take on a second job working part-time as a waiter in a restaurant. This is allowed as long as the individual notifies:

His employer Explanation: Prior written notification must be provided to the registered representative's employer.

The federal tax exemption for interest earned on an industrial revenue bond is NOT available if the:

Holder of the bond is a substantial user of the facility Explanation: If the holder of an industrial revenue bond is a substantial user of the facility, then the federal tax exemption on the interest earned does not apply.

When telemarketing, a registered representative is NOT required to provide a prospect with:

How he obtained the prospect's home number Explanation: Registered representatives, when telemarketing, are not required to tell prospects how their names were obtained. However, an RR must disclose his name, contact information, and the purpose of the call.

Which TWO of the following actions must be completed at or prior to an options trade? I. Send the customer a current copy of the risk disclosure document II. Have the ROP approve the account for options trading III. Deposit the customer's money in the account IV. Have the customer sign an options agreement

I and II Explanation: At or prior to the approval of an options account, a registered representative must send the customer a copy of a current risk disclosure document. Also, trades will not be executed until the account is approved for options trading by a registered options principal (ROP). Customers must sign and return the options agreement within 15 days of account approval.

To sell variable annuities, salespersons must be registered with (the): I. FINRA II. State Insurance Commission III. State Banking Commission

I and II only

Which of the following are defined as products offered by investment advisers? I. Non-managed fee based accounts II. Managed fee based accounts (wrap accounts) III. Per trade commission charge accounts

I and II only

Regulation T applies to: I. Cash accounts II. Margin accounts III. Commodity accounts IV. Municipal bond margin accounts

I and II only Explanation: Regulation T of the Federal Reserve Board applies to cash accounts and margin accounts. Regulation T does not apply to commodity accounts or municipal bond margin accounts. For municipal bond accounts, industry rules require a margin deposit of 7% of the market value of the bond. Margin requirements for commodity accounts are set by the individual commodity exchanges.

The Bond Buyer's 30-day visible supply includes: I. Competitive municipal bond issues II. Negotiated municipal bond issues III. Treasury bill issues IV. Corporate bond issues

I and II only Explanation: The Bond Buyer's 30-day visible supply is an indication used to reflect the amount of new offerings coming to the marketplace in the next 30 days. It carries figures for both competitive and negotiated municipal bond issues and notes maturing in 13 months or more.

According to the Telephone Consumer Protection Act, the recipient of a telephone solicitation must be: I. Told the identity of the broker-dealer on whose behalf the call is made II. Included on the caller's Do Not Call list upon request III. Sent the broker-dealer's most recent financial statements IV. An existing customer of the firm

I and II only Explanation: The Telephone Consumer Protection Act requires that the recipient of a telephone solicitation be told the name of the caller and the firm on whose behalf the call is being made. Those who do not wish to be called in the future must be included on a Do Not Call list. These rules apply to calls made to potential clients and not existing clients. However, an existing client may sever the relationship and request not to receive future calls. The Act does not require the mailing of financial information.

An investor who expects interest rates to rise will profit by: I. Buying yield-based calls II. Buying yield-based puts III. Selling yield-based calls IV. Selling yield-based puts

I and II only Explanation: Yield-based options are cash-settled options based on the movement in yield on a particular Treasury security. If an investor expects yields (interest rates) to rise, he will buy calls or sell puts.

The underwriting of most corporate issues is done on a negotiated basis. The investment banker who negotiates with the issuer on a firm commitment underwriting is known as

the syndicate manager (managing underwriter or the lead underwriter.)

A corporate issuer declares a reverse stock split. After the split is effected, which of the following statements are TRUE? I. the market price of the corporation's shares will increase II. the market price of the corporation's shares will decrease III. the reported earnings per share will increase IV. the reported earnings per share will decrease

I and III

When a stock and a put option on that stock are purchased on the same day, which of the following statements are TRUE? I. the put option is said to be "married" to the stock II. the put option is said to be "covered" by the stock III. the premium paid is treated as part of the cost basis IV. the premium paid is treated independently

I and III

Which statements are TRUE about banks that have customer accounts holding both exempt and non-exempt securities? I. The bank must be registered under Securities Exchange Act of 1934 II. The bank does not need to be registered under Securities Exchange Act of 1934 III. The bank must be a member of the Securities Protection Corporation IV. The bank does not need to be a member of the Securities Investor Protection Corporation

I and III

Which TWO of the following statements are TRUE concerning a Health Savings Account? I. The contribution is made in pretax dollars II. The contribution is made in after-tax dollars III. The funds grow tax-free if used to pay qualified medical expenses IV. The funds grow tax-deferred if used to pay qualified medical expenses

I and III Explanation: A Health Savings Account (HSA) is a tax-advantaged account that can be used by individuals to pay for qualified medical expenses. An HSA is not open to all individuals. It is generally open only to those persons who are not enrolled in any type of health plan other than a qualified, high-deductible health plan. Contributions are made in pretax dollars (which are limited under IRS guidelines), grow tax-free, and withdrawals are tax-free if used to pay qualified medical expenses. All funds withdrawn that are used for nonqualified medical expenses are taxable and subject to a 20% IRS tax penalty.

Mr. Jones is a small business owner who has purchased Treasury bills and other short-term securities during times when he has excess funds available in the business. He likes the aspects of liquidity and safety. A friend has told him he can get higher rates from auction rate securities. He wants to know why you have not recommended this investment to him. Which TWO of the following explanations would you cite as your reasons? I. Auction rate securities are long-term investments II. Interest or dividend rates are reset at established intervals based on a Dutch auction III. If the auction fails, the client may not have immediate access to his funds IV. The interest or dividend rate is set as the lowest rate to match supply and demand at the auction

I and III Explanation: Although auction rate securities are usually sold as an alternative to other short-term securities, they are long-term securities. An RR must disclose to a client that, if the auction fails, the client may not have immediate access to his funds. The RR also has a duty to disclose to clients any material fact relating to the specific features of the auction rate securities and the customer's need for a liquid investment when recommending this type of product. The fact that the interest or dividend rate is reset at specified intervals is a material fact, but is not a reason to avoid recommending the investment. The same reasoning applies to the fact that the rate is set at the lowest rate that matches supply and demand. These investments may not be suitable for investors who have a need for liquidity.

Which TWO of the following statements are TRUE regarding REITs? I. They may invest in both commercial and residential real estate II. They may retain a majority of their income III. Dividends paid are taxed as ordinary income IV. They may be sold only to retail investors

I and III Explanation: REITs invest in many different types of residential and commercial income-producing real estate such as apartment buildings, hotels, shopping centers, office complexes, storage facilities, hospitals, and nursing homes. Income is received from the rental income paid by the tenant leasing the real estate owned by the REIT. REITs must pay a minimum of 90% of their taxable income and the dividends received by investors are taxed at the same rate as ordinary income. The dividends paid to shareholders of REITs do not qualify for the lower 20% tax rate given other types of common and preferred stock. They can be suitable for both retail and institutional investors.

Which TWO of the following statements are TRUE concerning the auction process of Treasury bills? I. The 4-week bill is offered every week II. The 13-week and 26-week bills are auctioned every month III. The 4-week-bill is auctioned on a Tuesday IV. The 13-week and 26-week bills are auctioned on a Thursday

I and III Explanation: The 4-week Treasury bill is auctioned each week on Tuesday and is issued Thursday of that same week.13-week and 26-week T-bills are auctioned weekly on Monday and are issued on Thursday of that same week. All T-bills are issued (and traded) at a discount. Noncompetitive tenders are awarded first, but the price they will pay (the lowest accepted price of the competitive tenders) cannot be determined until after the competitive tenders are awarded.

Which TWO of the following statements concerning The Bond Buyer 20-Bond Index are TRUE? I. It is compiled weekly II. It consists of 20 GO and revenue bonds, with an average rating of AA III. It is used to show a trend in interest rates IV. It is used as an indication of the primary market in municipal securities

I and III Explanation: The Bond Buyer 20-Bond Index is compiled each week. It is calculated from the yields on 20 specific general obligation issues with an average rating of AA. There are no revenue bonds in the 20-Bond Index. The purpose of this index is to show trends in interest rates.

A dividend has been declared on ATT common stock. Which TWO of the following investors are entitled to the dividend? I. The owner of an ATT call option who exercised the option prior to the ex-dividend date II. The owner of an ATT put option who exercised the option prior to the ex-dividend date III. The writer of an ATT put option that exercised prior to the ex-dividend date IV. The writer of an ATT call option that was exercised prior to the ex-dividend date

I and III Explanation: The owner of a call and the writer of a put are entitled to receive the dividend if the option is exercised before the ex-dividend date. In both cases, the exercise results in the individual buying stock. To be entitled to a dividend, stock must be purchased prior to the ex-dividend date.

Which TWO of the following statements are TRUE concerning the Securities Act of 1933? I. Registration provisions apply if the securities beings sold are listed on the NYSE II. Antifraud provisions do not apply if the securities being sold are listed on the NYSE III. Registration provisions do not apply to securities issued by a municipality IV. Antifraud provisions do not apply to securities issued by a municipality

I and III Explanation: The registration provisions of the 1933 Act apply if securities sold are listed on the NYSE or Nasdaq, but do not apply to securities issued by a municipality. The antifraud provisions of the Securities Act of 1933 apply to all securities, even those exempt from registration.

Which statements are TRUE? I. Short information is found within EMMA II. EMMA information is found within SHORT III. RTRS information is found within EMMA IV. EMMA information is found within RTRS

I and III EMMA stands for Electronic Municipal Market Access. The EMMA web portal, makes available to the investing public: -Official statements -Real time reporting of muni bond trades -Real time reporting of muni note trades

A broker-dealer sending details of a trade to another broker-dealer would expect to receive either one of which TWO of the following in return? I. A confirmation II. A Regulation T margin call III. A DK notice IV. A loan consent agreement

I and III Explanation: A broker-dealer sending details of a trade (confirmation) to another broker-dealer would expect to receive either a confirmation or a signed don't know (DK) notice. In other words, either confirm the trade or send a notice regarding why the broker-dealer disagrees or otherwise has no knowledge of the trade.

When reviewing a company, a fundamental analyst will look at which TWO of the following choices? I. Financial reports II. Trading volume III. Management of the corporation IV. Short interest

I and III Explanation: A fundamental analyst is interested in the company, not technical factors relating to the stock. He would look at financial reports and the company's management.

Which TWO of the following types of securities may a municipal securities representative sell? I. General obligation bonds II. Treasury notes III. Variable-rate demand obligations (VRDOs) IV. Municipal unit investment trusts

I and III Explanation: A municipal securities representative may sell municipal bonds. General obligation bonds and VRDOs are types of municipal securities. The municipal securities representative, according to the MSRB, is not properly registered to sell government bonds, municipal unit investment trusts, or any type of corporate securities.

Penny stock rules apply under which TWO of the following conditions? I. The stock is priced below $5.00 per share II. The customer is an active trader of penny stocks III. The broker-dealer is a market maker in the security IV. The customer is an institutional investor

I and III Explanation: A penny stock, according to SEC rules, is a stock that sells for less than $5.00 that is not listed on Nasdaq or the NYSE. A stock quoted on the OTC Bulletin Board or OTC Pink Market (Pink Sheets) that has a bid price of less than $5.00 is defined as a penny stock. Penny stock rules would not apply under the following conditions. - The customer is defined as an existing customer, who is a person who has maintained an account with a broker-dealer for more than one year, or has previously engaged in 3 or more transactions involving penny stocks (i.e., an active trader of penny stocks) - In nonrecommended or unsolicited transactions - In transactions by a broker-dealer that is not a market maker in that security - In transactions by an institutional accredited investor

A corporation is planning to issue new stock to the public and has filed a registration statement with the SEC. As a registered representative of the firm that is expected to do the underwriting, you are permitted to: I. Obtain indications of interest from prospective purchasers II. Receive monies from customers who intend to purchase the issue III. Send a preliminary prospectus to retail investors IV. Guarantee a customer that he will be able to purchase 1,000 shares of the issue

I and III Explanation: A registered representative is permitted to send a preliminary prospectus to any type of investor and obtain nonbinding indications of interest. The RR cannot accept money nor guarantee a customer that he would receive a particular amount of the issue. If the corporation has not filed a registration statement with the SEC, none of the choices listed would be acceptable.

Which TWO of the following statements are TRUE when a broker-dealer creates a retail communication concerning the characteristics of exchange-traded funds (ETFs)? I. A principal is required to approve the communication prior to use II. The communication should be reviewed, but is not required to be approved by a principal III. The communication must be filed with FINRA IV. The communication does not need to be filed with FINRA

I and III Explanation: A retail communication is any written or electronic communication that is distributed or made available to more than 25 retail investors within any 30 calendar-day period. If the retail communication makes a recommendation, or promotes a product or service, prior principal approval is required. In addition, if the retail communication is created by the member firm and performance comparisons with other investment companies are used, it must be filed with FINRA at least 10 business days prior to first use. If the retail communication concerning investment companies does not included the member firm's performance comparisons, it still must be filed with FINRA, but it can be filed within 10 business days of first use. An ETF is a type of investment company and, since the communication does not contain any performance comparisons by the member firm, it can be filed within 10 business days of first use.

Which TWO of the following currency movements will cause U.S. exports to become more competitive than foreign exports? I. The U.S. dollar weakens when compared to foreign currencies II. The U.S. dollar strengthens when compared to foreign currencies III. Foreign currencies strengthen against the U.S. dollar IV. Foreign currencies weaken against the U.S. dollar

I and III Explanation: A weakening or devaluation of the dollar will make U.S. exports more competitive with foreign exports. The U.S. dollar will be worth less in relation to foreign currencies. Foreigners will then spend a relatively smaller amount of their currency to purchase U.S. products, making U.S. products more competitive. Likewise, a strengthening in foreign currencies to the U.S. dollar also causes U.S. products to become more competitive.

In order to have an issuer of securities exempt from the provisions of the Securities Act of 1933 under Regulation D, which TWO of the following statements are TRUE? I. The purchasers must sign an investment letter restricting the resale of the securities II. The size of the offering must be limited III. The number of accredited buyers is unlimited IV. The issuer must file an offering document with the SEC

I and III Explanation: According to Regulation D, certain conditions must be met for the securities to be exempt from the provisions of the Securities Act of 1933. The offering must be restricted to persons who are knowledgeable and experienced in business and financial matters and who are able to afford the economic risks involved. The issuer must provide the buyer with detailed financial information (this offering document does not need to be filed with the SEC). The number of nonaccredited purchasers must be limited to 35, and the offering must be made in direct negotiations between the issuer and the buyer or his purchaser representative. Also, the buyer must sign an investment letter stating that the purchase was made for investment and not for short-term trading purposes. The size of the offering is not limited and there is no limit as to the number of accredited investors.

PSE & G Utility is offering $975,000,000 worth of 4.30% subordinated debentures at a price of 99.25% of par value. An investor purchasing these bonds is: I. Subject to state income tax II. Exempt from state income tax III. Subject to federal income tax IV. Exempt from federal income tax

I and III Explanation: An issuer of debt offering subordinated debentures is a corporation, not a municipality. Therefore, interest paid on its debt obligations is subject to federal and state income tax.

Which TWO of the following choices can be calculated by examining the income statement of a company? I. The earnings before interest and tax (EBIT) II. The debt-to-equity ratio III. The operating profit margin IV. The amount of working capital

I and III Explanation: EBIT may be found by subtracting the operating expenses from the sales or revenue of a company, and the operating profit margin is found by dividing the sales by the operating expenses. All of this information can be found in the income statement. The debt-to-equity ratio and amount of working capital can be calculated by examining a company's balance sheet.

Lucretia has a significant gain in Jaymont shares that she has held for 10 months. If she buys a put on Jaymont, which TWO of the following statements are TRUE? I. She still has upside potential in the stock II. She can achieve a long-term gain by holding the put and stock for three months III. She terminates the holding period IV. She cannot suffer a loss on the investment

I and III Explanation: If a stock is held short-term (one year or less) and a put is purchased, the holding period is terminated and will not resume until the put is sold or expires. When the holding period resumes, it will do so as if the stock was purchased on that day. If a long-term holding period were established on the stock, then the acquisition of the put will not affect the investor's holding period. If the stock and the put are purchased on the same day, that is termed a married put. In that case, the price of the put will be added to the price of the stock to arrive at a cost basis for the entire position. When the put expires, there will not be a taxable event, since the stock must be sold to trigger a capital gain or loss. The purchase of the put will create a hedge by allowing the investor to sell at the strike price, but this does not necessarily mean there will be no loss. There is still potential for the stock to rise and for Lucretia to achieve large gains.

Interest rates had been very high. During the past three years rates have decreased dramatically, reaching historically normal level. The present yield curve would most likely be: I. Ascending II. Inverted III. Positive sloping IV. Negative sloping

I and III Explanation: If rates have declined for the past three years and reached a normal level, the present yield curve would most likely be ascending, which is also referred to as positive or upward sloping. This type of curve would have short-term rates lower than long-term rates, which is the way interest rates usually are. It is also referred to as a normal yield curve.

Which TWO of the following situations will cause U.S. exports to become more competitive than foreign exports? I. The U.S. dollar weakens when compared to foreign currencies II. The U.S. dollar stengthens when compared to foreign currencies III. Foreign currencies strengthen against the U.S. dollar IV. Foreign currencies weaken against the U.S. dollar

I and III Explanation: If the U.S. dollar weakens and foreign currencies strengthen, foreign consumers with strong foreign currencies can purchase more U.S. exports. This would have the effect of making U.S. exports relatively less expensive as compared to foreign goods and, therefore, more competitive.

In easy money periods, bonds of similar quality generally will have: I. Short-term yields lower than long-term yields II. Long-term yields lower than short-term yields III. Both short-term and long-term yields below normal IV. Both short-term and long-term yields higher than normal

I and III Explanation: In periods of easy money, there is availability of money. Therefore, interest rates will decline or be lower. In periods of easy money, bonds of similar quality generally will have short-term yields lower than long-term yields. Both short-term and long-term yields will be below normal. This situation creates a positively sloped yield curve where yields rise from short to long term.

A customer has purchased a new municipal issue during the underwriting period. According to MSRB rules, the customer must receive which TWO of the following documents? I. The final confirmation showing the aggregate price II. A copy of the notice of sale III. A copy of the official statement, if prepared IV. A list of syndicate members

I and III Explanation: MSRB rules require that a copy of the official statement be sent to each purchaser of a new issue. A confirmation must be sent on every transaction, whether a new issue or a secondary market trade. A copy of the indenture and a list of the syndicate members do not need to be sent.

A woman wishes to open an account at a municipal securities firm. She identifies herself as the spouse of a trader at another municipal securities firm. Which TWO of the following are TRUE? I. The representative must follow all instructions from the trader's employer II. The MSRB must be notified III. The carrying broker-dealer must send written notification of each transaction to the trader's employer IV. The trader's employer must approve all transactions of the spouse

I and III Explanation: MSRB rules require that when opening an account for an employee of another municipal firm, a municipal registered representative must: Notify the employer and follow all instructions (effectively receiving the employer's permission) Send duplicate confirmations to the employer Included in the definition of employee is spouse, minor children, and anyone else dependent on the employee. The MSRB need not be notified and the trader's employer is not required to approve each trade.

Which TWO of the following activities would require special disclosure documents? I. Penny stock trading II. Trading in high-yield bonds III. Day trading IV. Online trading

I and III Explanation: Regulators have singled out penny stock investing and day trading as presenting significant risks that warrant providing special risk disclosure documents.

A fundamental analyst is NOT interested in which TWO of the following metrics? I. Short interest II. The P/E ratio III. Trading volume IV. EPS

I and III Explanation: Short interest and trading volume are technical indicators. EPS and the P/E ratio are fundamental indicators.

The Daily Bond Buyer's 30-Day Visible Supply includes: I. Competitive municipal bond issues II. Auction rate securities III. Negotiated municipal bond issues IV. Tax anticipation notes

I and III Explanation: The 30-Day Visible Supply is all municipal bonds that are expected to be brought to market in the next 30 days. It is computed daily and includes all competitive and negotiated offerings that are anticipated to be brought to market. However, it does not include short-term notes or auction rate securities.

In May, a customer sells an STC July 40 listed call for a $6 premium and buys an STC July 30 listed call for $10. The customer has created a: I. Bullish spread II. Bearish spread III. Debit spread IV. Credit spread

I and III Explanation: The investor bought the more expensive call. Therefore, this is a debit spread. A call debit spread is a bullish strategy.

The term fast market is characterized by which TWO of the following descriptions? I. An imbalance of orders II. A very low number of trades III. Highly volatile prices IV. The quotes of market makers being updated very quickly

I and III Explanation: The term fast market is characterized by very heavy trading, fast moving prices, and high volatility. There also may be an imbalance in the number or shares clients are willing to buy or sell. For example, there are 500,000 shares to buy and only 100,000 shares to sell. Quotes may take a long time to update since prices and trades are moving so quickly. A client's order may take a longer time to execute, and if a market order is entered by a client, the price received may be significantly higher or lower then the quoted price.

Industrial development bonds: I. Are backed by rental revenues paid by corporate lessee II. Are backed by the municipality's ad valorem taxes III. Take on the credit rating of the corporate lessee IV. Take on the credit rating of the municipal lessor

I and III Industrial development bonds are backed by the rental revenues paid by the corporate lessee as well as the guarantee of the corporate lessee. These bonds, therefore, take on the credit rating of the corporation leasing the facility.

Which statements are TRUE about stabilizing bids? I. A stabilizing bid is placed by the syndicate manager II. A stabilizing bid is placed by each syndicate member III. Only 1 stabilizing bid is permitted at a time IV. Any number of stabilizing bids can be placed at a time

I and III Only 1 stabilizing bid is permitted at a time. The manager of the syndicate places the stabilizing bid on behalf of the syndicate.

Foreign currency options: I. Are quoted in U.S. dollars II. Are quoted in the underlying foreign currency III. Expire on the third Friday of the expiration month IV. Expire on the last Friday of the expiration month

I and III only Explanation: Foreign currency options are quoted in U.S. dollars (U.S. currency). Beginning February 15, 2015, the expiration date for foreign currency options is the third Friday of the expiration month, at 11:59 p.m. Eastern Time.

When examining an earnings report for National Corporation, a registered representative sees that earnings per share is reported on both a primary and fully diluted basis. This indicates that: I. The company has convertible bonds or convertible preferred stock outstanding II. The company has cumulative and participating preferred stock outstanding III. Earnings per share is calculated using current shares outstanding and also assuming that all convertible securities were converted IV. Earnings per share is calculated on a pretax and after-tax basis

I and III only Explanation: The calculation for earnings per share on a primary basis (before the possible dilution of convertible bonds, convertible preferred stock, stock options, or warrants) is computed based on the number of outstanding common shares only. The calculation for earnings per share on a fully diluted basis includes the outstanding shares if convertible bonds and preferred stock are converted into common stock.

Rising inflation tends to: I. Negatively impact the stock market II. Positively impact the stock market III. Negatively impact the bond market IV. Positively impact the bond market

I and III only Explanation: Rising inflation tends to have a negative impact on both the bond and stock markets. In the bond market, when there is rising inflation, bond investors look to trade out of fixed-income investments, which will have their returns eroded by rising consumer prices. This selling pressure negatively affects bond prices in the market. In the stock market, rising inflation often translates into an assumption that the Fed will raise interest rates to curb consumer borrowing and consumer demand. Rising rates should, in theory, lead to decreased economic activity and shrinking corporate earnings. This also tends to lead to some form of a sell-off and a falling stock market. From a technical perspective, the dividend discount model would project falling stock prices during inflationary times because the discount rate applied when calculating the present value of the projected dividend cash flows would be higher. This would lead to a lower present value.

On September 14, a customer purchases an ABC December 60 call and sells an ABC November 60 call. The customer: I. Has engaged in a debit spread II. Has engaged in a credit spread III. Wants the spread to widen IV. Wants the spread to narrow

I and III only Explanation: To determine whether the customer wants the spread to widen or narrow, it is necessary to determine whether the spread is a debit or credit spread. The premium for an option is determined by two factors—the in-the-money amount of the option (intrinsic value) and the time value. Since both options have the same strike price, the intrinsic values (in-the-money amount) are equal. Therefore, any difference in premium is the result of a difference in time value. Since the December contract has longer to go until expiration than the November contract, it has more time value. Therefore, the premium for the December contract will be larger than for the November contract. Since the customer purchased the December contract (higher premium), it is a debit spread and will profit if the spread widens.

A 65-year-old individual is in need of immediate cash to pay for repairs on his house and takes a lump-sum distribution from a nonqualified variable annuity. This withdrawal will be: I. Partially treated as ordinary income II. Partially treated as capital gain III. Taxed at the investor's tax bracket IV. Taxed at a reduced tax rate

I and III only Explanation: A nonqualified variable annuity is not used in conjunction with a qualified retirement plan (such as an IRA). Any contribution is made with after-tax dollars. Therefore, the appreciated value portion of withdrawals would be taxed as ordinary income and the remainder would be considered as return of capital (amount invested), which is not taxed. If a withdrawal is made prior to age 59 1/2, the ordinary income portion of the withdrawal is assessed a 10% penalty.

An investor purchases Swiss francs in the spot market at 61. As a hedge, the investor buys a Swiss franc June 60 put at 0.50. This strategy will be profitable if: I. The U.S. dollar weakens II. The U.S. dollar strengthens III. The spot price for the Swiss franc is 61.75 IV. The spot price for the Swiss franc is 59.25

I and III only Explanation: An investor who buys Swiss francs would purchase a put to provide protection if the U.S. dollar strengthens (which would cause the Swiss francs to decrease). If the Swiss francs decline, the investor could exercise the put or sell the put at a profit. The investor will break even when Swiss francs increase to a value equal to his cost (61) plus the premium paid (0.50). This makes the breakeven point 61.50 or $.6150 (since strike prices and premiums for Swiss franc options are in cents per unit, the decimal must be moved two places to the left). The investor will have a profit if the Swiss franc rises above the breakeven point (61.50) which would occur if the U.S. dollar weakens.

A transaction for a stock that is not DTCC eligible, settles on a regular-way basis. This means that settlement occurs: I. In three business days II. In five business days III. At the buyer's premises IV. At the seller's premises

I and III only Explanation: Regular-way settlement for stock transactions is in three business days. In most cases, settlement occurs electronically through DTCC. In this case, since the seller must make physical delivery of the securities, settlement takes place at the buyer's premises.

An investor enters an order to buy 400 shares of HRJ @ 56 on the NYSE. Which of the following statements are TRUE regarding this order? I. The designated market maker may hold this order in his book II. The order may only be executed at 56 III. A portion of the 400 shares may be purchased IV. The order must be executed immediately

I and III only Explanation: Since a price is specified, it is a limit order. A limit order may be executed at the limit price or better (lower for a buy order). It does not need to be executed at exactly the limit price. A designated market maker is permitted to hold a stop, limit, and stop-limit order. A portion of the order may be executed since the order was not marked AON (all or none). It does not need to be executed immediately since it was not marked IOC (immediate-or-cancel).

The Bond Buyer Revenue Bond Index is: I. A 30-year index II. A 20-year index III. A 25-bond index IV. A 30-bond index

I and III only Explanation: The Bond Buyer Revenue Bond Index (commonly referred to as the Revdex) is an index of the yields on 25 revenue bonds. It is compiled on a weekly basis by The Bond Buyer and contains 30-year maturity bonds with an average rating on S&P of A+ and on Moody's of A1.

A customer has 1,000 shares of stock and wishes to sell 100 of the shares. He places a limit order to sell 100 shares at $52. The order is placed, but is erroneously executed as "sell 1,000 shares at $52". The confirmation to the customer states that 1,000 shares were sold at $52. Which statements are TRUE? I. Any loss due to the firm's error must be absorbed by the firm. II. Any loss due to the firm's error must be absorbed by the customer. III. The firm must replace the shares that were erroneously sold. IV. The customer must accept the mistake

I and III.

Regarding Ginnie Mae Pass Through Certificates: I. The certificates pay holders on a monthly basis II. The certificates pay holders on a semi-annual basis III. Each payment consists of interest only IV. Each payment consists of a combination of interest and principal

I and IV

Rule 105 of Regulation M, covering transactions that occur in the secondary market during the 20-day cooling off period for "add on" securities offerings, requires that any short sales of the issue: I. that occur 5 business days prior to effective date II. that occur 20 business days prior to the effective date III. can only be effected on an upbid IV. cannot be covered by purchasing the issue from the syndicate

I and IV

A corporation would choose to refinance its debt for which TWO of the following reasons? I. To reduce its overall interest costs II. To be able to borrow funds at a higher rate III. To be able to reduce the number of persons on the board of directors IV. To remove restrictive provisions from the indenture

I and IV Explanation: A corporation would most likely refinance its debt to reduce its overall interest cost. This is most likely to happen when interest rates have declined and/or the credit strength of the issuer increases. It may also refinance to remove a restrictive provision from a bond's indenture. This restriction may have been included when the issuer sold bonds and it was not easy for the issuer to borrow funds. Refinancing does not alter the number of persons on the board of directors.

Which TWO of the following statements are TRUE regarding a variable annuity accumulation unit? I. It is an accounting measure used to determine an owner's interest during the pay-in phase II. It is an accounting measure used to determine an owner's interest during the payout phase III. The value of the units will remain fixed IV. The value of the units will fluctuate

I and IV Explanation: Accumulation units are an accounting measure used to determine an owner's interest in the separate account during the accumulation or pay-in phase. Their value will vary based on the performance of the separate account. (Annuity units are used during the annuity or payout phase.)

Blue-Sky laws apply to which TWO of the following choices? a. Registered representatives b. Securities issued by the City of Chicago c. Commercial paper d. Securities issued by a REIT

I and IV Explanation: Blue-Sky laws are state securities laws. These laws apply to the registration of sales personnel (registered representatives), the registration and sale of nonexempt securities. REITs (real estate investment trusts) are considered nonexempt securities and are, therefore, regulated by state laws. Municipal securities and commercial paper are considered exempt securities.

Which TWO of the following events may be reasons for a revenue bond issue to be called? I. There is a change in the tax status of the issuer II. Surplus funds are not available III. Interest rates rise dramatically IV. The facility is destroyed by fire

I and IV Explanation: Destruction by fire would be included in a catastrophe call provision and permit the issue to be called. If surplus funds are available (choice [II] states they are not available), the monies may be used to retire a portion of the outstanding bonds. If the tax status of an issuer is in doubt at the time of issuance, there is usually a provision requiring that the issue be called if the tax status of the issuer changes and the bonds become taxable. An issuer may refund an outstanding issue if interest rates are declining, not rising.

Which TWO of the following statements are TRUE regarding Eurodollar bonds? I. They are denominated in U.S. dollars only II. They are denominated in foreign currencies only III. They are traded only outside the U.S. IV. They are traded in the U.S. and in international markets

I and IV Explanation: Eurodollar bonds are dollar-denominated bonds issued and sold outside the U.S. They may trade in the U.S. after a seasoning period of 40 days after issuance.

A customer has a significant gain in shares of LRR that she has held for 10 months. If she buys a put option on LRR, which TWO of the following statements are TRUE? I. The customer still has upside potential in the stock II. The holding period on the stock is not affected III. The customer can achieve a long-term gain by holding the put and stock for three months IV. The customer has created a position that could reduce a loss

I and IV Explanation: If a stock is held short-term (one year or less) and a put is purchased, the holding period is terminated and would not resume until the put is sold or expires. When the holding period resumes, it will do so as if the stock was purchased on that day. If a long-term holding period were established on the stock, then the acquisition of the put would not affect the investor's holding period. If the stock and the put are purchased on the same day, that is termed a married put. In that case, the price of the put will be added to the price of the stock to arrive at a cost basis for the entire position. When the put expires, there will not be a taxable event since the stock must be sold to trigger a capital gain or loss. The purchase of the put will create a hedge by allowing the investor to sell the stock at the strike price, thereby reducing a loss. There is still potential for the stock to rise and for the customer to achieve large gains.

A registered representative has purchased two tickets to a football game for which he paid $85 per ticket. If the face value of the tickets is $45 per ticket, the RR may take which TWO of the following actions? I. Give one ticket to a customer II. Give two tickets to a customer III. Purchase two additional tickets and have two customers attend the game with their spouses IV. Give the two tickets to two different customers of the same firm

I and IV Explanation: Member firm personnel may not give, or permit to be given, a gift of material value exceeding $100 per recipient per year to personnel employed by another member firm. The gifts should be valued at the higher of the cost or market value. Since the tickets have a face value of $45 each, but the tickets were purchased at a value of $85 each, the higher value would be used. One ticket can be given to a customer, but two tickets exceed $100. If gifts are given to more than one customer, a prorated amount should be used, so the two tickets may be given to two different customers. If the RR attended the event with the customer, it would be a business expense and not a gift.

ABC Corporation bonds are convertible at $50. If the bonds are selling in the market for 90 ($900) and the common stock is selling for $43, which TWO of the following statements are TRUE? I. The stock is selling at a discount to parity with the bond II. The stock is selling at a premium to parity with the bond III. Liquidating the stock after converting the bond would be currently profitable IV. Liquidating the stock after converting the bond would not be currently profitable

I and IV Explanation: The conversion ratio, which is not given, is found by dividing the par value of the bond ($1,000) by the conversion price ($50). This equals 20 to 1 ($1,000 divided by $50 equals 20). The market price of the common stock is $43 per share. The stock is selling at a discount to parity with the bond ($43 stock x 20 shares = $860 which is below the $900 market price of the bond). If the bonds were converted and the stock was then sold at the market price, the investor would have a loss.

An investor takes the following position. Long 1 GHI Nov 65 put Short 1 GHI Nov 55 put Which TWO of the following statements are TRUE regarding this position? I. The investor paid money to create the position II. The investor received money to create the position III. The investor is bullish IV. The investor is bearish

I and IV Explanation: This position is referred to as a debit put spread. It cost the investor more than was received since the long put has a strike price greater than the short put. As a result, the long put is exercised first (since it has a higher strike price), allowing the investor to make money if the stock declines in value (a bearish move).

Upon written request, duplicate account statements would be required under which TWO of the following circumstances? I. The customer is an employee of a member firm and is opening a brokerage account at a bank II. The customer is an employee of a mutual fund and is on the board of directors III. The customer is an employee of a bank and is opening an account at a broker-dealer IV. The customer is an employee of a member firm and is opening a brokerage account at a financial institution

I and IV Explanation: Upon the written request by the employing member firm, duplicate account statements must be sent if an employee of a member firm opens a brokerage account at another member, investment adviser, bank, or other financial institution. There is no requirement to send duplicate statements if the customer is an employee at a financial institution.

When a stock sells ex-dividend, which TWO of the following orders on a designated market maker's (DMM's or specialist's) book will be reduced? I. Buy-limit order II. Sell-limit order III. Buy-stop order IV. Sell-stop order

I and IV Explanation: When a stock sells ex-dividend, the DMM (specialist) will reduce those orders on his book that were entered below the market. A buy-limit order and a sell-stop order will be reduced by the amount the stock sells ex-dividend since these orders are entered below the market.

A market maker that compensates a retail member firm for sending its customer orders to that market maker is: I. Paying for order flow II. Inter positioning III. Engaging in a prohibited practice under SEC rules IV. Permitted to do so, subject to best execution requirements

I and IV If a member firm chooses a market maker to execute its orders in return for compensation from that market maker, then the retail firm is earning so-called "payment for order flow". The SEC permits this practice, subject to the retail member firm always executing its trades at the best available price.

The "death benefit" associated with a variable annuity contract: I. applies prior to annuitization II. applies after annuitization III. means that, upon death, the insurance company will make a lump sum payment to complete the terms of the contract. IV. means that, upon death, the insurance company will pay a beneficiary at least the amount invested in the contract.

I and IV The "death benefit" of a variable annuity contract is not really much of one. If the contract holder died prior to annuitization, the insurance company pays the greater of current NAV or the amount invested to a beneficiary. If the contract holder dies after annuitization, there is no more "death benefit".

A convertible debenture is convertible at $25. It has a nondilutive feature in its indenture. If a stock dividend is distributed, which TWO of the following statements are TRUE? I. The conversion price will be reduced II. The conversion price will be increased III. The conversion ratio will be reduced IV. The conversion ratio will be increased

I and IV Explanation: A nondilutive feature means that if there is a stock split or stock dividend, the bond's conversion features must be adjusted. The bondholder would receive more shares upon conversion because the conversion ratio would be increased. The conversion price would be reduced to permit this increase in the conversion ratio.

Which TWO of the following statements are TRUE concerning bank-qualified municipal bonds? I. To qualify, the municipality may only issue up to $10,000,000 annually II. To qualify, the municipality must issue more than $10,000,000 annually III. Commercial banks are not permitted to purchase this type of security IV. Commercial banks are permitted to purchase this type of security

I and IV Explanation: Bank-qualified bonds are issued by small municipalities and, to qualify, a municipality may only issue up to $10,000,000 annually. This is done to encourage commercial banks to invest in locally issued municipal securities. Commercial banks that purchase this type of security are permitted to deduct 80% of the interest cost paid to depositors on the funds used to purchase the bonds.

Which TWO of the following activities are typically performed during the cooling-off period of an initial public offering (IPO)? I. A preliminary prospectus is prepared by the issuer II. The issuer will publish research on the securities to be offered III. The SEC reviews the issuer's registration statement and evaluates the investment merit of the issue IV. The issuer and underwriters hold a due diligence meeting

I and IV Explanation: During the cooling-off period, the SEC will review the issuer's registration statement for completeness. The SEC does not evaluate (pass on) the investment merits of the issue. Also, during the cooling-off period, the issuer will blue-sky the issue, send out a preliminary prospectus, and hold a due diligence meeting. Research is not permitted to be published by a broker-dealer until after the effective date of an IPO.

An officer of a company has been invited by a large mutual fund company to give a talk to the fund company's analysts about its business plans and prospects. At the talk, the officer inadvertently discloses material information that could affect the stock's price. Which statements are TRUE? I. A public announcement of the news must be made within 24 hours. II. A public announcement of the news must be made within 10 business days. III. The company must file an 8K with the SEC disclosing the information IV. The company must file an 10K with the SEC disclosing the information

I or III

Which TWO of the following statements regarding straddles are TRUE? I. An investor who does not anticipate that the price of the stock will change may sell a straddle II. An investor who anticipates a substantial advance in the price of a stock will buy a straddle III. An investor who anticipates a substantial decline in the price of a stock will buy a straddle IV. An investor who anticipates substantial fluctuations in the price of a stock will buy a straddle

I and IV Explanation: If investors wish to generate premium income, they will consider selling a straddle in order to generate income on both the put and the call. They will sell the straddle only if they do not anticipate significant price changes in the market price of the underlying security. The investor who anticipates significant changes in the price of a stock, but does not know if the price will advance or decline, will buy a straddle in order to be able to profit on both sides of the market. An investor who anticipates a substantial advance in the price of a stock will buy a call. An investor who anticipates a significant decline in the price of a stock will buy a put. Neither investor will buy a straddle if they anticipate that a price will move in only one direction, since the premium will be lost (assuming they are right) on the other side of the market. For example, an investor anticipates that XYZ stock will advance from $50 to $80. If a straddle is bought and the price did advance, there would be a profit on the call but there would be a loss of the entire put premium.

Which TWO of the following choices would be the most suitable purchasers of municipal zero-coupon bonds? I. An investor who does not seek present additional cash flow II. An investor who seeks the tax benefits of long-term capital gains III. An investor who needs cash for living expenses IV. A custodian account where the parent of the minor child is in the highest tax bracket

I and IV Explanation: In a custodian account, the minor is technically liable for taxes. Depending on the amount of income generated in the account and the age of the minor, taxes are calculated at the parents' rate. Therefore, parents may consider the purchase of municipal bonds in the custodian account for tax advantages. The zero-coupon bond will not produce cash flow during the holding period. This would be desirable for those who do not need cash income. (Funds are needed at a later date in the custodian account.) The zero-coupon municipal bond would be suitable for other accounts besides the custodian account, such as upper tax bracket earners during their peak earning years. Zero-coupon bonds are subject to annual accretion of the investor's cost basis. As such, at maturity, the investor's cost basis equals the par value of the bond. (There are no capital gains.) The accretion of the municipal bond is treated as interest income which, in the case of the municipal bond, is federally tax-free. This is a tax advantage, but it is not a long-term capital gain.

Which TWO of the following statements are TRUE about a divided account? I. It is called a Western account II. It is called an Eastern account III. Each member is responsible for the unsold bonds based on the member's original participation IV. Each member is liable only for its own participation in the syndicate

I and IV Explanation: In a divided account (Western account), the member is responsible for its own participation in the syndicate. If any bonds remain unsold, it is the responsibility of that member. In an undivided or Eastern account, any unsold bonds are the responsibility of the entire syndicate. Each member would then be liable for the same proportion as his original participation.

An employee of a corporation is enrolled in a noncontributory pension plan. Relative to the plan, which of the following statements are TRUE? I. Earnings in the plan accrue tax-deferred II. Earnings in the plan are taxed each year III. Benefits are taxed as a capital gain when received IV. Benefits are taxed as ordinary income when received

I and IV Explanation: In a noncontributory pension plan, the employee does not make contributions. Earnings in the plan accrue tax-deferred (are not taxed until received by the employee) and benefits received are taxed as ordinary income.

Which TWO of the following choices given to or shared with a customer are NOT a violation of MSRB rules? I. A gift of $100 II. A yearly financial magazine subscription worth $150 III. A gift of basketball tickets worth $200 IV. A business lunch that costs $125

I and IV Explanation: MSRB rules allow gifts of $100 or less. There is no limit on legitimate business expenses so the business lunch of $125 is not a violation even though it is more than $100. A subscription to any magazine in excess of $100 or a gift of tickets in excess of $100 is considered violations.

Which TWO of the following statements are TRUE concerning the characteristics of preferred stock? I. The securities do not have a fixed maturity date II. The price of these securities is more volatile than common stock III. The dividend will be paid annually IV. The price will fluctuate based primarily on changes in interest rates

I and IV Explanation: Most preferred stock does not have a maturity date and, therefore, one of the risks of purchasing this type of security is that there is no fixed date when you will receive your principal back. These securities are less volatile than common stock, and the prices of preferred stocks are inversely related to the movement of interest rates, as are bonds. The dividend usually is paid quarterly, not annually.

When comparing high-grade bonds to low-grade bonds, lower-grade bonds have which TWO of the following choices? I. Higher yields II. Lower yields III. Higher market prices IV. Lower market prices

I and IV Explanation: One of the basic principles of investing is the greater the risk, the greater the return, and the lower the risk, the lower the return. Since lower-grade bonds are riskier than higher-grade bonds, they will have higher yields and lower market prices.

Regulation NMS applies to which TWO of the following choices? I. Listed equity trades II. Listed debt trades III. Quotes available for manual execution IV. Quotes available for electronic execution

I and IV Explanation: One of the provisions of Regulation NMS (National Market System) requires a broker-dealer to provide its clients with the best price available for listed equity trades available for electronic execution. The best price is defined as the highest bid or lowest offer (inside market) from all available market centers. Reg NMS does not apply to securities subject to manual execution. Nor does it apply to debt securities, whether electronically or manually executed.

Information regarding new issue municipal offerings can be obtained from which of the following sources? I. EMMA II. Munifacts Wire III. Bond Buyer IV. Best's Ratings

I, II, III

Under SEC Rule 605 of Regulation NMS, market centers, in their monthly reports on order execution, must disclose which of the following information? I. Fill Rates II. Speed of executions III. Rates of price improvements IV. Trading volumes

I, II, III

Which TWO of the following investment companies are NOT open-end? NAV Offered I. $8.00 $ 7.00 II. $9.20 $10.00 III. $7.00 $ 7.00 IV. $8.00 $10.00

I and IV Explanation: Open-end companies are not offered below their current net asset value. According to the Conduct Rules, the maximum sales charge permitted for an open-end company is 8 1/2%. Choices (I) and (IV) must be closed-end companies. Choice (I) is a closed-end company because it is offered below its net asset value. Choice (IV) must be a closed-end company because when doing the sales charge calculation (sales charge divided by offering price), the result is a 20% sales charge, which is above the allowable maximum. Therefore, choice (IV) is a closed-end fund trading at a 25% premium to NAV.

A customer purchases 10 MMS May 20 puts at 2 in a cash account when the market price of MMS is 24. Which TWO of the following statements are TRUE regarding this transaction? I. The settlement date for the transaction is one business day II. The settlement date for the transaction is three business days III. The Regulation T payment date is three business days IV. The Regulation T payment date is five business days

I and IV Explanation: Option transactions settle on the next business day between brokerage firms and the Options Clearing Corporation. According to Regulation T, payments for transactions in cash and margin accounts must be made by the customer within two business days following the regular-way settlement date. Since regular-way transactions settle in three business days, customers have five business days in which to pay for purchases. Therefore, while option transactions settle next day, the Reg T payment requirements are based on a regular-way transaction. Hence, customers have five business days in which to pay for option purchases.

When evaluating two CMOs backed by GNMAs, one having a 6% yield and the other having a 10% yield, which TWO of the following statements are TRUE? I. Prepayment risk is greater for the CMO with the 10% yield II. Prepayment risk is greater for the CMO with the 6% yield III. Credit risk is greater for the 10% CMO IV. Credit risk is the same for both securities

I and IV Explanation: Prepayment risk measures the possibility that homeowners will refinance (prepay) their mortgages. Historically, the speed of prepayment increases when interest rates fall. If this happens, payments will flow into the CMOs at an accelerated rate, forcing investors to reinvest these monies at lower-than-anticipated rates. Therefore, the CMO with the higher interest rate will have higher prepayment risk. GNMA-backed CMOs are highly rated and, therefore, have little credit risk. Since both CMOs are backed by GNMAs, credit risk is minimal for both pools.

Which TWO of the following choices are characteristics of reverse convertible securities? I. They are short-term securities II. They are usually long-term securities III. The investor is guaranteed to receive his original principal back at maturity IV. The investor may receive less than the value of his original principal back at maturity

I and IV Explanation: Reverse convertible securities are short-term notes issued by banks and broker-dealers that usually pay a coupon rate above prevailing market rates. They are considered structured products because, in addition to the coupon rate, the investor may be required to purchase shares of an underlying asset at a fixed price. The underlying asset may be an equity security unrelated to the issuer, or a basket of stock, or an index. The issuer agrees to pay this higher coupon rate since it has an option to sell a security to the investor if the price of the security falls below a specified value known as the knock-in level. If the price of the underlying asset stays above the knock-in level, the investor will receive the high coupon and the full return of his principal. If the underlying asset falls below the knock-in level, the investor will be obligated to purchase shares of the underlying asset at a fixed price. The price of this asset may have depreciated below the knock-in level and the investor may receive substantially less than the original principal.

Which TWO of the following actions may an RR engage in when selling shares of a mutual fund? I. Tell a customer to invest in a family of funds to take advantage of a breakpoint II. Sell dividends III. Explain that the exact value can be determined at redemption IV. Allow a customer to sign a letter of intent two months after his initial investment

I and IV Explanation: Selling dividends means to suggest purchasing shares just prior to the ex-date and is a violation of securities rules since it does not benefit the investor. A letter of intent may be backdated up to 90 days. When redeeming shares, the price is based on the next calculated NAV so it is not known at the time of redemption. A family of funds allows an investor to take advantage of breakpoints although investing in more than one fund.

A bond is convertible into stock at $50 per share. The market price of the stock is 65. The market price of the bond is 120. To profit from this arbitrage opportunity, an investor should: I. Buy 5 bonds II. Buy 100 shares of stock III. Sell 5 bonds short IV. Sell 100 shares of stock short

I and IV Explanation: Since the bond is convertible into 20 shares of stock ($1,000 par divided by $50) and the bond is priced at 120, the parity for the stock is $60 per share ($1,200 bond price divided by 20 shares). An arbitrage situation exists because the stock is selling at a 5-point premium to parity (65 market price versus 60 parity price). An investor can profit from this situation by purchasing bonds at 120 and shorting the stock at 65. Each bond may be converted into 20 shares of stock at a cost of $60 per share. These shares may then be used to cover the short sale, establishing a 5-point profit (65 short sale price - 60 cost).

An individual is in the third year of accumulating an interest in a variable annuity with a deferred sales charge. A registered representative recommends a switch to a newly created variable annuity with a larger number of subaccount choices, also offered with a deferred sales charge. Which TWO of the following statements are TRUE of this switch? I. FINRA will probably consider the switch unsuitable II. FINRA will probably not consider the switch unsuitable, since both annuities are offered with a deferred sales charge III. The switch is taxable if it qualifies as a 1035 exchange IV. The switch is not taxable if it qualifies as a 1035 exchange

I and IV Explanation: Since the investor is in the third year of accumulation, there will be a deferred sales charge incurred. By switching to another variable annuity, the individual will now be subject to the highest deferred sales charge, thus making the switch unsuitable. If a registered representative recommends that a client switch from one annuity to another within a three-year period, the representative may be considered to be churning. An exchange of annuities that qualifies for IRS Section 1035 treatment is not a taxable event.

Federal Reserve actions can directly influence which of the following? I. Discount Rate II. Federal Funds Rate III. Money Multiplier IV. Money Velocity

I, II, III, IV

Stagflation is best defined as a period where the economy is experiencing which TWO of the following events? I. Inflation for a long period II. Deflation for a long period III. Low unemployment IV. High unemployment

I and IV Explanation: Stagflation is defined as a prolonged period of a high rate of inflation together with a high rate of unemployment. This does not happen too often since high unemployment usually leads to a period of low inflation or even deflation (falling prices) and the possibility of a recession. A period of low unemployment usually leads to rising prices and increased inflation.

Which TWO of the following statements are TRUE relating to the notes issued by the Federal Farm Credit Banks Consolidated System? I. They are issued at a discount II. They are issued at par III. They are interest-bearing IV. They are non-interest-bearing

I and IV Explanation: The Federal Farm Credit Banks issue consolidated systemwide notes that are issued at a discount (as with T-bills) and are non-interest-bearing. Bonds are also issued that are interest bearing (have a stated interest rate). Interest is subject to federal taxation but is exempt from state and local taxation.

A customer has purchased 10 ABC January 50 calls, paying a $2 premium, and 10 ABC January 50 puts, paying a $2 premium. The market price of ABC stock is $50 per share. The buyer's breakeven points are: I. $46 II. $48 III. $52 IV. $54

I and IV Explanation: The customer has the right to call the stock at $50. He has paid a $400 premium per straddle. The breakeven point on the call is determined by adding the 50 strike price to the premium of 4. This equals a breakeven of $54. The customer also has the right to put or sell the stock to the writer at $50, but has paid a $400 premium. The breakeven point on the put is four points below the strike price of $50, which equals $46. The buyer's breakeven points will, therefore, be $46 and $54.

A customer buys 10 ABC January 50 calls paying a $3 premium and 10 ABC January 50 puts also paying a $3 premium when the market price of the stock is $49 per share. The buyer's TWO breakeven points are: I. $44 II. $47 III. $53 IV. $56

I and IV Explanation: The customer has the right to call the stock at $50. The customer paid a $600 premium per straddle. The breakeven point on the call is determined by adding the $50 strike price to the premium of $6. This equals a breakeven of $56. The customer also has the right to sell the stock to the writer at $50, but has paid a $600 premium. The breakeven point on the put would be six points below the strike price of $50, which equals $44. The buyer's breakeven points, therefore, will be $44 and $56.

Which TWO of the following statements are TRUE concerning the death benefit on a variable annuity? I. The benefit skips the probate process II. The benefit must go through probate prior to distribution III. The beneficiary receives the proceeds tax-free IV. The beneficiary may have a tax liability when receiving the proceeds

I and IV Explanation: The death benefit on a variable annuity skips the probate process. Probate is a lengthy legal process in which the decedent's bills are paid and remaining assets distributed based on instructions generally left in a will. The recipient of a death benefit from a variable annuity may need to pay taxes on any amount above the contract's cost basis. For example, if a client invested $100,000 and died when the contract was worth $150,000, a nonspouse beneficiary may be required to pay taxes on the $50,000 above the decedent's contributions.

Which TWO of the following metrics can be calculated by examining the balance sheet of a company? I. The debt-to-equity ratio II. The operating profit margin III. The bond coverage ratio IV. The current ratio

I and IV Explanation: The debt-to-equity ratio is found by dividing the dollar amount of debt (bonds) by the dollar amount of shareholder equity (common stock + paid-in capital + retained earnings). The current ratio is found by dividing the current assets by the current liabilities. The operating profit margin and the bond coverage ratio can be calculated by examining the income statement.

If interest rates increase, which TWO of the following events will most likely occur? I. Yield-based call premiums will increase II. Yield-based put premiums will increase III. Bond prices will rise IV. Bond prices will fall

I and IV Explanation: The value of yield-based options is determined by the difference between the yield of a Treasury index and the strike price. Yield-based calls have intrinsic value when the Treasury index yield is above the strike price. Yield-based puts have intrinsic value when the Treasury index yield is below the strike price. When interest rates (yields) increase, yield-based call premiums will increase. Bond prices move in the opposite direction, falling when interest rates increase.

Which TWO of the following statements are TRUE regarding the buyer and writer of a straddle? I. The buyer of a straddle expects the market to fluctuate II. The writer of a straddle expects the market to fluctuate III. The buyer of a straddle expects the market to remain stable IV. The writer of a straddle expects the market to remain stable

I and IV Explanation: The writer (seller) of a straddle (call and put) believes the stock's price will remain stable. The buyer of a straddle expects that the market price of the underlying stock will be volatile.

A corporation purchases new machinery using cash. Which of the following choices are results of this transaction? I. Working capital is reduced II. Working capital remains the same III. Total assets are reduced IV. Total assets remain the same

I and IV Explanation: When purchasing machinery with cash, current assets (cash) are reduced and fixed assets (machinery) are increased by the same amount. Overall, total assets do not change. Since total assets (TA) and total liabilities (TL) remain the same, stockholders' equity (TA - TL) does not change. Working capital (current assets minus current liabilities) is reduced since current assets are reduced.

The bond counsel for a new municipal revenue bond issue may render a qualified legal opinion in which TWO of the following situations? I. The issuer does not have clear title to the property II. Construction of a competing facility may cut projected revenue flow III. The underwriting syndicate has not provided information that the MSRB requires IV. The project potentially could impact a national historic site

I and IV Explanation: When situations exist that could create potential problems for a proposed facility, the bond counsel will render a qualified legal opinion. These situations include (I) and (IV) in that both subject the issuer to potential legal action, whereas (II) and (III) do not deal with the legality, validity, enforceability, or tax-exempt status of the issue.

The primary difference between an underwriting syndicate member and a selling group member in a firm commitment underwriting is that

the syndicate member assumes liability for unsold shares and the selling group member does not.

A call is purchased in February and exercised in June. The following April, the stock is sold. Which TWO of the following statements are TRUE? I. The resulting gain or loss is short-term II. The resulting gain or loss is long-term III. The cost basis of the stock is the same as the strike price IV. The cost basis of the stock is the call premium plus the strike price

I and IV Explanation: When the buyer exercises the call option, the holding period for the stock begins. In this case, the period from February to June is ignored. The subsequent sale of the stock in April makes the holding period short-term (one year or less). The cost basis for the stock is the call premium plus the strike price.

A customer that wishes to effect a securities trade through an ECN in the "after hours" market should be advised that the market is: I. subject to high volatility II. subject to low volatility III. highly liquid IV. highly illiquid

I and IV (high volatility highly illiquid) ECNS are Electronic Communications Networks - which match trades for institutional customers at low cost. Trades that are effected through ECNs "after hours" are only filled if there is a "match". There is no specialist/DMM that must take the opposite side to the trade and that will "smooth" price movements. Thus, the market is not nearly as liquid as an exchange, and price movements will be volatile.

If a person under the age of 59 1/2 becomes disables and wishes to withdraw money from her IRA, which statements are TRUE? I. the withdrawal is subject to income tax II. the withdrawal is not subject to income tax III. the withdrawal is subject to a 10% penalty IV. the withdrawal is not subject to a 10% penalty

I and IV (subject to income tax and not subject to 10% penalty)

The Vice President of ACME Corporation, an NYSE listed firm, places an order to buy 10,000 shares of ACME common at the market. 3 months later, ACME stock's price has increased by 20% and the officer places an order to sell. Which statements are TRUE? I. the sale of the stock is subject to rule 144 II. the stock cannot be sold unless it's been held and fully paid for 6 months. III. the sale is prohibited IV. the officer must forfeit the profit on the sale

I and IV only

Variable annuities sold by insurance companies must be registered with: I. The SEC II. The FRB III. FINRA IV. The State Insurance Commission

I and IV only Explanation: Variable annuities are generally sold by agents of insurance companies. In recent years, more and more brokerage firms and banks have begun selling variable annuities. Variable annuities are considered securities by the SEC and, therefore, must be registered with the SEC. Variable annuities must also be registered with the State Insurance Commission. The agents that sell variable annuities must be registered representatives with a Series 6 or Series 7 registration and must be licensed insurance agents.

Briana Corporation, an existing public company, is offering 500,000 shares of common stock to the public through an underwriting syndicate. The prospectus states that 250,000 shares are being offered by selling stockholders and 250,000 shares are being offered by Briana Corporation. The effect of this offering will be: I. A dilution in the earnings per share II. An increase in the earnings per share II. The number of shares outstanding will increase by 500,000 IV. The number of shares outstanding will increase by 250,000

I and IV only Explanation: After the offering is completed, there will be 250,000 new shares outstanding (The shares sold by the selling stockholders were already outstanding.) This will result in the earnings per share being diluted because the earnings will now be divided by a greater amount (250,000 shares) of new outstanding stock.

A registered representative writes a letter to see if his clients have any interest in trading options. The letter is generic and describes the advantages and disadvantages of options trading. This letter: I. Must be approved prior to use by a ROP II. Need not be approved prior to use as long as it does not contain recommendations III. Must be accompanied by a risk disclosure document IV. Need not be accompanied by a risk disclosure document

I and IV only Explanation: All retail communications concerning options must be approved by a ROP prior to being sent to a customer. Since there are no specific recommendations, the OCC disclosure document does not need to precede or accompany the letter. However, the customer must receive the risk disclosure document at or before the account is approved for options trading.

Eurodollar deposits are: I. Denominated in U.S. currency II. Denominated in foreign currency III. Held in banks in the U.S. IV. Held in banks in foreign countries

I and IV. U.S. Currency and hold in banks in foreign countries.

A municipal securities representative does an analysis of an official statement and prepares a summary report. The report must be approved by: I. A municipal securities principal II. FINRA III. The Issuer IV. The MSRB

I only Explanation: The preliminary and final official statements are not considered advertising since they are prepared by or for the issuer. However, a summary of an official statement is considered advertising since it is prepared by the municipal representative and, therefore, must be approved by a municipal principal.

Which TWO of the following time limitations must be complied with regarding the delivery of a risk disclosure document? I. A brokerage firm must deliver a risk disclosure document to a customer at the time the account has been approved for options trading II. A brokerage firm must deliver a risk disclosure document to a customer prior to the time the account has been approved for options trading III. A brokerage firm must deliver a risk disclosure document to a customer after the account has been approved for options trading IV. A brokerage firm must deliver a risk disclosure document to a customer after the first order has been entered

I or II Explanation: According to the rules of the exchanges where options are traded, a brokerage firm must deliver a risk disclosure document to a customer at or prior to the account being approved for options trading.

Which of the following objectives is the least suitable reason for investing in a mutual fund? a. Diversification b. Professional management c. Short-term trading d. Liquidity

c. Short-term trading Explanation: Investors in mutual funds usually seek all of the objectives listed except short-term trading.

The FINRA suitability rule requires which of the following? I. Before a product or strategy can be recommended, a reasonable basis suitability determination must be completed, evaluating the investment's features, returns, costs, and risks. II. Before a product or strategy can be recommended, a customer, it must be determined that the investment is suitable, based on the clients new account profile. III. Before a product or strategy can be recommended to a customer, it must be determined that the customer has the ability to meet the financial commitment required by the recommendation. IV. Before a product or strategy can be recommended to a customer, the registered representative must understand, and be able to communicate the investment's features, returns, costs, and risks

I, II, III, IV

Which of the following are tax preference items included in the Alternative Minimum Tax? I. Excess depreciation II. Excess depletion III. Excess intangible drilling costs IV. Private purpose municipal interest income

I, II, III, IV

Which of the following recommendations are "red-flags" that are usually unsuitable for seniors? I. Variable annuities II. Structured products III. Mortgaging home equity for investment purposes IV. Using retirement savings to invest in high-risk stock

I, II, III, IV "No-no'" recommendations to seniors include: -purchase of variable annuities -purchase of equity indexed annuities -purchase variable life settlements -purchase complex structured products -mortgage their residence to obtain funds -use retirement savings, including early withdrawals to invest in high-risk stock

Limited partnership investors are subject to which of the following risks? I. Tax audit risk II. Marketability risk III. Legislative risk IV. management risk

I, II, III, IV Limited partnerships often suffer from marketability risk, since such securities are illiquid. They also suffer from the risk of further tax law changes, and increased chances of tax audits; and the risk of poor management by the general partner.

Which of the following securities can be traded in the over-the-counter market? I. Municipal bonds II. Treasury bonds III. Treasury bills IV Corporate bonds

I, II, III, and IV Explanation: All of the securities listed can be traded in the over-the-counter market.

A municipal securities principal must approve: I. Memos in response to customer complaints II. The opening of accounts III. Advertisements to be used for a seminar IV. Correspondence to customers

I, II, III, and IV Explanation: MSRB rules require a municipal securities principal to approve all the choices given. In addition, the principal must approve all transactions and must frequently review all discretionary accounts.

A municipal securities principal must review and approve municipal transactions made with: I. Individuals II. Trust departments III. Commercial bank portfolios IV. Casualty insurance companies

I, II, III, and IV Explanation: MSRB rules require a municipal securities principal to approve all transactions in municipal securities.

A municipal bond swap may be executed to: I. Establish a loss for tax purposes II. Increase cash flow III. Improve maturities IV. Improve yields

I, II, III, and IV Explanation: Municipal bond swaps may be executed to establish a loss, increase cash flow (increase income from larger coupon), improve maturities, improve yield, and improve quality.

A registered options principal (ROP) must review: I. Retail communication concerning options II. General options prospecting letters III. Option seminar transcripts IV. Allocation of exercise notices

I, II, III, and IV Explanation: The registered options principal (ROP) is specifically responsible for the firm's compliance program with respect to its options activities. The ROP performs an audit function to determine that these activities are conducted in compliance with current applicable regulations and rules. Some of the ROP's principal duties include establishing guidelines for options retail communication, and reviewing all such material before it is used. The ROP also reviews the method of allocation of exercise notices.

An uncle is a custodian for his nephew's account. The account receives rights to subscribe to additional stock held in the account. As custodian, the uncle may: I. Sell the rights II. Subscribe to the stock III. Buy additional rights to subscribe if additional rights are needed IV. Do whatever he considers to be in the best interest of the minor

I, II, III, and IV Explanation: The uncle may sell the rights, subscribe to the stock, or buy additional rights to subscribe if additional rights are needed. The uncle may do whatever he considers to be in the best interest of the minor.

Dealer A offers bonds on a firm basis to Dealer B with a recall. What does this mean? I. Dealer B has the right to buy the bonds before anyone else II. The price of the bonds has been set III. A time to sell the bonds has been set IV. A recall time has been established

I, II, III, and IV Explanation: When a dealer wants to buy bonds, such as Dealer B, he asks Dealer A to make a firm offer which will hold good for a specified time. The bonds are said to be out firm. Dealer B has the option or right to buy the bonds from Dealer A and sell the bonds before anyone else within the set time. A recall privilege can be set, which gives Dealer A the right to notify Dealer B that he has only a set time (for example, five minutes) to buy the bonds, after which time the firm pricing is cancelled.

Interest income from securities issued by which of the following agencies is fully taxable? I. Federal National Mortgage Association II. Government National Mortgage Association III. Federal Farm Credit Banks Funding Corporation IV. Federal Home Loan Mortgage Corporation

I, II, IV

Which of the following economic events would have a negative long term impact on common stock prices? I. Rising interest rates II. Rising capital gains tax rates III. Rising employment rates IV. Rising inflation rates

I, II, IV

The reoffering yield on a new municipal bond issue is

the yield at which the bonds are offered to the public.

Which of the following characteristics of Fannie Mae and Ginnie Mae pass-through certificates are the same? I. Certificates are issued in $25,000 denominations II. Certificates are backed by FHA and VA insured mortgages III. Certificates are backed by the direct guarantee of the U.S. Government IV. Certificate holders receive monthly payments for combined interest and principal

I, II, IV Both issues in $25,000 denominations Backed by FHA and VA insured mortgages Receive monthly payments for interest/principal **ONLY GINNIE-MAE IS BACKED BY U.S. GOV'T

An investor in the U.S. purchases the debt of a German company. The bonds are denominated in euros. Which of the following risks will the investor be exposed to? I. Interest-rate risk II. Credit risk III. Currency risk

I, II, and III

By buying a put option, an investor: I. Can avoid selling a security for a large capital gain and yet participate in additional gains if the security continues to increase in price II. Can avoid selling a security for a large capital gain and be assured of selling the security at the strike price during the term of the option III. Can protect a profit on his current stock position

I, II, and III Explanation: If investors are long stock, they can lock in an existing profit, or protect themselves against a market decline, by buying a put. If the market price continues to rise, investors will realize an additional profit, minus the premium paid for the put. For example, an investor owns stock that has a current market value of $60 and buys a July 60 put for 3. If the market price advances to $70, there will be an additional profit of $700 ($1,000 increase in profit minus $300 premium). The investor can provide protection against a loss by using a put. For example, if the stock in the preceding example dropped to $40, the investor would exercise the put and sell stock at $60. The entire cost would be the premium of $300.

Which of the following actions must a municipal dealer disclose on a confirmation? I. The municipal dealer acted as a agent II. The municipal dealer acted as a principal III. The municipal dealer acted as an agent for a third party IV. The municipal dealer acted as a bona fide market maker

I, II, and III only Explanation: The MSRB requires a municipal dealer to indicate to a customer through a written confirmation the capacity in which the dealer acted. The municipal dealer must disclose if it acted as an agent for the customer, as a principal for its own account, or as an agent for a third party. If the municipal dealer acted as an agent for a third party, the municipal dealer must disclose the name or promise to provide the name of the third party. Also, the amount of money received from the third party by the municipal dealer is required. A bona fide market maker is one who makes a market in over-the-counter stocks.

Prior to being listed on an exchange, which of the following factors must be evaluated? I. The number of shareholders II. The dividend payout III. The earnings record IV. The current market price

I, III, and IV only Explanation: Listing requirements include: a minimum number of round-lot shareholders, a minimum number of shares publicly held, minimum market values, a positive earnings history, and national interest in the stock. The amount of dividends paid or the dividend payout ratio is not a factor.

The ratio of net direct debt plus overlapping debt to assessed valuation is used to:

I. Analyze general obligation bonds II. Determine the municipality's ability to generate sufficient taxes to pay for debt service requirements

A pension fund manager has a large holding of 30-year Treasury Bonds, to fund a corporation's defined pension plan liability. The manager is concerned that market interest rates are going to rise, causing Treasury Bond prices to fall. To hedge the position, the manager should: I. Buy TYX Calls II. Buy TYX Puts III. Sell TYX Calls IV. Sell TYX Puts

I. Buy TYX Calls If the market interest rates rise, bond prices drop. To hedge, the contract must offer an offsetting profit during a period of rising interest rates -so buy TYX calls.

Step-down certificates of deposit:

I. Initial payments are made at an interest rate that is above the market rate II. At a predetermined time, the interest rate is decreased to a rate that is at, or below, the market

Which of the following statements are TRUE regarding debt obligations?

I. Municipalities issue revenue bonds II. Corporations issue income bonds

What would be considered an "overlapping" debt?

I. School district bond issue where the school district is coterminous with the town that 100% uses that school district II. School district bond issue where the school district encompasses three neighboring townships

Which statements are TRUE about CMBs? I. CMBs are sold at par II. CMBs are sold at a discount to par III. CMBs are sold at a regular weekly auction IV. CMBs are sold on an "as needed" basis

II and IV. Sold at a discount to par and sold on an "as needed" basis. CMBs are sold in $100 minimums at a discount to par value, just like Treasury Bills.

A self-employed individual has total income of $120,000. If the individual wants to open a Keogh plan: I. It must be opened by the time he files his tax return II. It must be opened by the end of the tax year III. A maximum deductible contribution of $24,000 is permitted IV. A maximum deductible contribution of $53,000 is permitted

II and III Explanation: A Keogh plan must be opened by the end of the tax year (December 31). However, contributions are permitted until the filing deadline for the tax return (April 15). A self-employed individual may deduct 20% of self-employed income or $53,000, whichever is less, to a Keogh plan.(For 2014, the limit was $52,000.) 20% of $120,000 is $24,000 and would be the maximum allowable deductible contribution.

Which TWO of the following statements are TRUE concerning Section 457 plans? I. These plans are state-sponsored and used to fund higher education expenses II. These plans are used to fund retirement III. These plans grow tax-deferred IV. These plans grow tax-free

II and III Explanation: A Section 457 plan is a type of qualified retirement plan used by many public sector workers. 457 plans grow on a tax-deferred basis and are generally subject to the same contribution limits as 401(k) and 403(b) plans. Each has similar tax features and contribution allowances. The difference between the plans is the type of employee who may use them. A 401(k) plan is used primarily by for-profit employees, a 403(b) plan by nonprofit employees, and a 457 plan by some local government workers. State-sponsored, higher education savings plans that may be opened by an investor are referred to as Section 529 plans, not 457 plans.

The tax treatment of a business development company is similar to which TWO of the following securities? I. A municipal bond II. A real estate investment trust III. A closed-end investment company IV. A variable annuity

II and III Explanation: A business development company (BDC) raises capital by selling securities to investors and is similar in structure to a closed-end investment company. A BDC will use the money it raises to invest in private companies, small and developing businesses, and financially troubled companies that have difficulty raising capital in public markets. The objective is to help these companies by providing funding when they may not be able to raise capital for themselves. Most BDCs trade on an exchange and, therefore, provide an investor with liquidity and, since they are structured as regulated investment companies, they are not taxed if they distribute at least 90% of their income to investors. For tax purposes, they are regulated similar to investment companies (mutual funds and closed-end funds) and to REITs that also must distribute a minimum of 90% of their income. Most have an investment objective of providing current income and capital appreciation, and will invest their funds in both debt (e.g., loans, subordinated and mezzanine financing) and equity of private small and middle-market companies. Since some of the funds are invested in the equity of nonpublic companies, a customer purchase of a BDC is similar to buying a publicly traded investment in a private equity firm.

A buyer of a call option is subject to which TWO of the following choices? I. Unlimited risk II. Protection for a short position III. A position that provides leverage IV. An obligation to buy stock

II and III Explanation: Buying a call option provides leverage because the buyer controls 100 shares of stock for a relatively small cost (the premium). The risk is limited to the premium since that is the maximum potential loss. If an investor is short stock, he risks a loss if the market price of the stock increases. Buying a call provides protection against this situation since he could buy stock at a set price by exercising the call. A buyer of a call option has a right to buy stock, not an obligation.

A Web site is being designed for a registered representative of a member firm. Which TWO of the following statements are TRUE regarding the design of this Web site? I. The FINRA logo must be displayed II. The registered representative's firm name must be displayed III. A reference to FINRA membership is permitted IV. Links to other Web sites are not permitted

II and III Explanation: Care should be taken in the design of Web sites. The name of the member firm with whom the registered representative is associated must be displayed. While the use of the FINRA logo is NOT permitted, the registered representative's association with a FINRA member firm is allowed. However, when a reference to FINRA membership is used, the Web site must provide a hyperlink to FINRA's home page. Links to other Web sites are allowed but care should be taken that these sites do not provide fraudulent or misleading information

Which TWO of the following securities will MOST likely be subject to a withholding tax? I. A bond issued by a U.S. company but sold to U.S. investors II. A bond issued by a foreign company but sold to U.S. investors III. Stock issued by a foreign company but sold to U.S. investors IV. Stock issued by a U.S. company but sold to U.S. investors

II and III Explanation: Choice (II) is an example of a Yankee bond and choice (III) is an example of an ADR. Dividends and interest paid to a U.S. investor on foreign securities may be subject to a withholding tax by the country from which they were paid. If the investor has securities that paid dividends and/or interest that were subject to a foreign tax, the broker-dealer will send the investor a form that will report the gross amount of the dividends or interest, and the amount of tax withheld by the foreign government.

ABC Corporation intends to make an initial public offering of 10,000,000 shares of common stock, 7,500,000 shares of which will be new stock being issued by ABC Corporation and 2,500,000 shares will be for selling stockholders. Which TWO of the following statements regarding this offering are TRUE? I. It is a primary distribution II. It is a primary and secondary distribution III. The proceeds of the sale will be shared by the corporation and the selling stockholders IV. The corporation will receive all of the proceeds of the sale

II and III Explanation: Since both the corporation and existing shareholders are selling stock, it is both a primary and secondary distribution. In a primary distribution, proceeds go to the corporation. In a secondary distribution, proceeds go to the selling shareholders.

Relative to a sales tax, which TWO of the following statements are TRUE? I. It is a progressive tax II. It is a regressive tax III. It affects low income individuals the most IV. It affects all individuals equally

II and III Explanation: A regressive tax applies the same tax rate regardless of a person's income. This has an adverse effect on a low-income person since the tax represents a higher percentage of income.

The market price of XYZ Company's stock is $60. The price-earnings ratio is 10 and earnings per share is $6.00. If the stock were to split 2-for-1, which of the following statements are TRUE? I. The price-earnings ratio will be reduced to 5 II. The price-earnings ratio will remain at 10 III. The earnings per share will be reduced to $3.00 IV. The earnings per share will remain at $6.00

II and III Explanation: A stock split will increase the number of shares outstanding while decreasing the market price of the stock. The split will also have the effect of reducing earnings per share since the number of shares outstanding will increase. The 2-for-1 split will reduce the market price to $30 ($60 x 1/2) and the earnings per share to $3.00 ($6.00 EPS x 1/2). However, the price-earnings ratio (market price/EPS), which was 10 before the split, will remain the same since both the market price and the earnings per share are reduced by the same percentage ($30/$3.00 EPS = 10).

A registered representative is the custodian of a UTMA account, which will be held at another member firm. Which TWO of the following statements are TRUE? I. There is no requirement for the RR to notify the employing firm before opening the account II. There is a requirement for the RR to notify the employing firm before opening the account III. The RR is required to notify the executing firm before opening the account IV. The RR is not required to notify the executing firm before opening the account

II and III Explanation: An employee of a FINRA member firm who wishes to open an account or place an order at another member firm must notify her firm and the executing firm, in writing, prior to opening a securities account. Since the RR is the custodian for the UTMA account, she will be placing orders for this account.

A client is seeking a mutual fund that will maximize its return by limiting expenses. As a result, he wants to invest in a portfolio that is passively managed. Which TWO of the following choices will help achieve this goal? I. A portfolio that invests only in fixed income securities II. An exchange-traded fund based on the Nasdaq 100 Index III. A mutual fund that tracks the S&P 500 Index IV. An account managed by an investment adviser

II and III Explanation: Passive investing or management is designed to minimize transaction costs and capital gains. This is accomplished by a portfolio manager trying to mirror an index, not outperfom an index. An exchange-traded fund or mutual fund that follows and is designed to replicate an index such as the S&P 500 or the Nasdaq 100 will accomplish this objective. Active management is a situation where the portfolio manager frequently trades the securities in a portfolio to achieve results that will outperform an index. The portfolio could be invested in equity or fixed-income securities.

XYZ Corporation has a 6 1/2% convertible bond outstanding that is convertible into 40 shares of common stock. The bond is currently selling in the market at 85 ($850) and the common stock is selling at 21. The XYZ Corporation is offering its existing bondholders a new straight (nonconvertible) bond paying 6 1/2% that matures at the same time as the convertible bond. The effect of the successful completion of the proposal would be to: I. Reduce interest costs II. Reduce potential dilution III. Have no effect on interest costs IV. Increase dilution

II and III Explanation: Prior to the refunding, if all of the bonds were converted into common stock, outstanding shares would increase causing earnings per share to decrease (dilute). The effect of the successful completion of the proposal (refunding) would be to reduce potential dilution because the conversion provision is being eliminated. There would be no reduction in interest costs since the new bonds are paying the same rate of interest as the old bonds (6 1/2%).

Which TWO of the following situations require written notification to an employer? I. A registered person is leaving the country on a business trip for more than three months II. A registered principal is serving on the board of directors of a private company III. A registered person wants to act as a consultant for a private placement of a security that is not being offered by her broker-dealer IV. A registered principal intends to purchase corporate securities in a personal account established at her employing broker-dealer

II and III Explanation: Registered personnel who pursue outside business interests and who will be compensated, or who participate in private securities transactions, must provide their employers with prior written notification. The employer may then approve or disapprove the participation. A registered person who has an existing account at her firm does not need to provide written notification to her employer for each transaction.

An insider of XYZ Corp. buys company stock in the open market at $63/share. Ten months later, the insider wishes to sell the stock at the current market price of $68/share. Which TWO of the following statements are TRUE regarding this transaction? I. The sale is subject to the six-month holding period under Rule 144 II. This sale is not subject to the six-month holding period under Rule 144 III. The sale is subject to the volume limitations under Rule 144 IV. The sale is not subject to the volume limitations under Rule 144

II and III Explanation: Rule 144 requires that restricted (unregistered) stock be held for six months before it may be resold. Control stock (registered stock purchased by insiders) is not subject to a holding period requirement under Rule 144. Both restricted and control stock are subject to the volume limitations under the rule.

Which TWO of the following statements are TRUE about SIPC? I. It was created by an Act of Congress and is considered a U.S. government agency II. It is a nonprofit organization that broker-dealers join III. It provides insurance for customer accounts in the event of bankruptcy by a broker-dealer IV. It provides insurance for customer accounts for fraud, embezzlement, and counterfeiting

II and III Explanation: SIPC can borrow from the U.S. government, but it is not an agency of the U.S. government. SIPC provides insurance coverage for customer accounts in the event of a brokerage firm's failure. Each brokerage firm must take out a separate insurance policy (known as a fidelity bond) to insure itself for fraud, embezzlement, and counterfeiting. This bonding is not provided by SIPC.

A customer wishes to make a purchase based on his belief that interest rates will decline over the next 15 years. The recommendation of which TWO of the following securities is NOT consistent with the customer's belief? I. A 5-year noncallable bond II. A tax anticipation note (TAN) III. Floating rate notes IV. A 15-year bond with a 5-year put feature

II and III Explanation: Since the customer believes interest rates will decline, he wants to lock in a high yield for the next 15 years. A TAN is a short-term security and a floating rate note's interest rate would be adjusted downward with prevailing interest rates. Neither would lock in the high return. The 5-year noncallable bond would lock in a high return without the possibility of being called prior to maturity. The 15-year bond locks in the high return and the 5-year put feature permits the investor to redeem the bond after 5 years or keep it to maturity. This decision would depend on the prevailing rates in 5 years.

An investor buys a DEF April 35 put at 3 and simultaneously writes a DEF April 30 put at 1. The investor has created a: I. Bullish put spread II. Bearish put spread III. Put debit spread IV. Put credit spread

II and III Explanation: Since the investor is paying more for the option purchased than was received for the option sold, it is a debit spread. When analyzing a debit spread, look at the buy side of the spread. Buying a put indicates that the investor is bearish (believes the stock price will decline).

Under the provisions of the Securities Act of 1933, which TWO of the following securities are exempt? I. Securities issued by broker-dealers II. A general obligation bond issued by the city of San Francisco III. Corporate debt maturing in six months IV. Securities issued by an investment company

II and III Explanation: The Securities Act of 1933 exempts government securities (both direct and agency), municipal securities (e.g., a G.O. bond), and corporate debt with a maturity of 270 days or less.

A registered representative wants to open a joint account for the dentists in his office building. Dr. White and Dr. Enamel will each contribute equally to the account but each dentist wants his portion of the account to pass to his own estate. Which TWO of the following statements are TRUE? I. The account should be established as Joint Tenants with Right of Survivorship II. The account should be established as Tenants in Common III. All dividends and capital gains in the account will be reported by the brokerage firm under one Social Security number IV. All dividends and capital gains in the account will be reported by the brokerage firm on a percentage-of-ownership basis

II and III Explanation: The dentists should open a Tenants in Common account. If a Joint Tenants with Right of Survivorship account is used, all assets pass to the surviving owner upon the death of one of the participants. All joint accounts use only one Social Security number for tax reporting purposes. The dentists must indicate the percentage of dividends, bond interest, and capital gains they are responsible for on their individual returns.

Which TWO of the following metrics may be calculated by examining the income statement of a company? I. The debt-to-equity ratio II. The operating profit margin III. The bond coverage ratio IV. The current ratio

II and III Explanation: The operating profit margin is found by dividing the sales by the operating income or profit. The bond coverage ratio is found by dividing the interest expense by EBIT. All of this information can be found in the income statement. The debt-to-equity ratio and current ratio can be calculated by examining a company's balance sheet.

A pension fund manager is holding a large portfolio of common stock. Many of these securities are represented in an index that has listed options trading. If a general market downturn is anticipated, the pension fund manager can: I. Buy stock index call options II. Write stock index call options III. Buy stock index put options IV. Write stock index put options

II and III Explanation: The pension fund manager can hedge against a decline in the market by writing stock index call options and buying stock index put options. If the market did decline, the stock index call option will expire unexercised and the fund will generate premium income. The put index option's value increases as the index declines, and can be sold by the pension fund manager at a profit. This profit, along with the premium income received from writing call index options, can help offset the loss incurred by the common stock in the pension fund's portfolio.

Which TWO of the following statements are TRUE regarding brokered CDs sold by registered representatives? I. A callable CD gives the investor the right to redeem the security prior to maturity II. If the CD is called by the issuer, the client may not be able to receive a comparable rate of interest III. A callable CD that is called prior to maturity may offer a client a return that is less than the yield to maturity IV. Since the security is issued by a bank, a callable CD will provide no limit to the amount of FDIC insurance

II and III Explanation: When a client purchases a brokered or long-term CD issued by a bank, the broker-dealer offering the product is required to provide a client with certain disclosures. These disclosures will be based on the potential risk and investment considerations relevant to the client. The following disclosures should be made concerning callable CDs. - The issuer at its sole discretion may decide to call in the CD prior to maturity. Therefore, the client does not have the right to redeem the CD prior to maturity. - If the CD is called prior to maturity, the client may be unable to reinvest the funds and receive a comparable rate of interest (if rates have declined). - A CD that is called prior to maturity may offer a client a return that is less than the yield to maturity. - The CD may not be called by the issuer, requiring the client to hold the security until maturity. All brokered CDs carry limited FDIC insurance if the amount of the CD at one bank exceeds the regulatory limits established by the FDIC.

tax treatments

corporate bonds: interest taxable on all levels municipal securities: federal exempt treasuries: state exempt mortgage backed: taxed on all levels US territories: tax exempt on all

Which statement is TRUE? I. Regular way trades of listed stock settle on the business day after the trade. II. Regular way trades of listed stocks settle 3 business days after the trade date. III. Regular way trades of listed stock options settle on the business day after trade date. IV. Regular way trades of listed stock options settle 3 business days after the trade date

II and III Regular way trades of listed stocks settle 3 business days later, and stock options settle 1 business day after.

If the market price is much higher than the strike price at the time of expiration, which of the following open options positions is likely to result in a gain? I. Long Put II. Short Put III. Long straddle IV. Short straddle

II and III (short put, long straddle) If the market price is higher than the strike price, the long put contract would expire "out the money", so the buyer will lose the premium paid for the position. Conversely, if a customer sells a put, and the market goes up, the contract will expire "out the money" and the customer earns the premium.

An income fund would likely invest in which of the following securities? I. Common stocks II. Preferred stocks III. Debentures IV. Income bonds

II and III only

The issuer of a variable annuity bears which of the following risks: I. Interest rate risk II. Expense risk III. mortality risk IV. Investment risk

II and III only

An investor owns stock that has increased in value. To protect his profit, he can: I. Enter a buy stop order II. Enter a sell stop order III. Buy put options on the stock IV. Buy call options on the stock

II and III only Explanation: A sell stop order can be used to protect a profit or limit a loss on an existing long position. It is not activated until the market declines to or below the stop price. By purchasing put options, the investor will have the right to sell his stock at a set price (strike price) and will establish a specific sales price

The French economy is on the verge of a recession. The Swiss government announces that there was another quarterly increase in its GDP figures. An investor wanting to act on this information will buy: I. Euro calls II. Euro puts III. Swiss franc calls IV. Swiss franc puts

II and III only Explanation: An increase in the Swiss GDP is a positive situation for the Swiss economy. This may cause the value of the Swiss franc to rise. If the Swiss franc rises, the holder of a Swiss franc call will profit. A looming recession in France may cause the euro to decrease in value. If the euro decreases, the holder of a euro put will profit.

Corporations repurchase their own stock in the open market to: I. Increase the number of voting shares that the corporation holds II. Increase earnings per share III. Have stock available for stock option plans for key employees IV. Make the stock more marketable

II and III only Explanation: Corporations repurchase their own stock in the open market to increase earnings per share and to have stock available for stock option plans for key employees. Stock repurchased becomes treasury stock, which does not have voting rights, and its marketability is very difficult to predict.

The tendering of equity securities is permitted if an individual is: I. Long investment-grade, nonconvertible securities II. Long call options or convertible securities that have been exercised III. Long a convertible security that has not been tendered for conversion IV. In possession of the securities by virtue of having borrowed them

II and III only Explanation: Short tendering of stock (i.e., tendering stock that a person does not own) is prohibited. Equity securities may be tendered only if the investor is long the stock, or long an equivalent security. Equivalent securities include rights, warrants, and other securities issued by the company that is the subject of the tender offer. All of these are immediately convertible into, or exchangeable or exercisable for, the subject security. Standardized call options are not equivalent securities, unless the option has been exercised.

An investor purchased 100 shares of ABC stock at $53 and on the same day purchased an ABC June 50 put at 2. After the put expired, the investor sold the ABC stock at $60. The investor's: I. Cost basis for tax purposes was 53 II. Cost basis for tax purposes was 55 III. Profit was $500 IV. Profit was $700

II and III only Explanation: When a married put (stock and put purchased on the same day) expires, the premium is added to the cost basis of the stock. The cost basis, therefore, will be $55 (53 + 2). The sale at $60 results in a profit of $5 per share (60 - 55), for a total of $500.

An elderly customer seeking extra income who has $100,000 to invest could be recommended which of the following? I. The $100,000 purchase of a variable annuity II. The $100,000 purchase of dividend paying blue chip stocks in a cash account against which calls are sold III. The $200,000 purchase of dividend paying blue chip stocks at 50% margin in a margin account IV. The $100,000 purchase of Treasury bonds

II and IV

During the accumulation phase of a variable annuity: I. funds can be distributed to unit holders II. funds cannot be distributed to unit holders III. as interest, dividends, and capital gains are received, the investor has the option of reinvesting in more shares IV. as interest, dividends, and capital gains are received, these must be reinvested in more accumulation units.

II and IV

Regarding an investor's basis in a limited partnership investment which of the following statements are TRUE? I. recourse financing is not included in the tax basis II. non-recourse financing is not included in the tax basis III. the investor is not "at risk" for any recourse financing IV. the investor is not "at risk" for any non-recourse financing

II and IV

When an investor purchases a municipal fund security, she will pay a sales load that is stated in the official statement. Which TWO of the following statements are TRUE regarding an advertisement for this municipal fund security? I. The minimum sales load should be stated in the ad II. The maximum sales load should be stated in the ad III. Sales charges may not be reflected in performance data IV. Sales charges may be reflected in performance data

II and IV Explanation: A 529 College Savings Plan is a type of municipal fund security. If sales loads are depicted in municipal fund securities advertising, the maximum amount of the sales charge should be stated. Additionally, sales charges may be reflected in the performance data of the investment and, if they are not, a statement to that effect should be made.

Under the Investment Company Act of 1940, which TWO of the following choices are considered investment companies? I. A bank holding company II. A face-amount certificate III. An insurance company IV. A management company

II and IV Explanation: A face-amount certificate and a management company are two types of investment companies. The third type is a unit investment trust. The Investment Company Act of 1940 does not consider holding companies and insurance companies to be investment companies.

Which TWO of the following taxes would best describe income taxes and estate taxes? I. Flat taxes II. Graduated taxes III. Regressive taxes IV. Progressive taxes

II and IV Explanation: A progressive tax is graduated (the tax rate increases as the taxable amount increases). Income taxes, estate taxes, and gift taxes are progressive. A flat tax is a situation where the tax rate remains constant regardless of the taxable amount. Flat taxes tend to be regressive in nature, which means that they have a greater effect on lower wage earners. Therefore, flat taxes are often categorized as regressive. Examples of regressive taxes are sales taxes and gasoline taxes.

Which TWO of the following statements are characteristics of REITs? I. They are formed as a limited partnership II. They provide limited liability for shareholders III. They invest only in mortgages IV. They are required to distribute a minimum percentage of their income

II and IV Explanation: A real estate investment trust (REIT) is organized as a corporation that offers stock to the public and, as such, provides investors with limited liability. REITs are not limited partnerships and, therefore, will not have a flow-through of losses. An investor's risk is limited to her investment. They can have an equity position in real estate (own the buildings) or be involved in mortgage activities (lend money). They must distribute 90% of their income in order to qualify for preferential tax treatment.

Which TWO of the following choices BEST describe a gasoline tax? I. Graduated taxes II. Flat taxes III. Progressive taxes IV. Regressive taxes

II and IV Explanation: A regressive tax is flat (i.e., the tax rate remains constant regardless of the taxable amount). Examples of regressive taxes are sales taxes and gasoline taxes. A progressive tax is graduated (i.e., the tax rate increases as the taxable amount increases). Income taxes, estate taxes and gift taxes are progressive.

A buy stop order is entered: I. Below a support level II. Above a resistance level III. To limit a loss on a long stock position IV. To limit a loss on a short stock position

II and IV Explanation: A stop order may be used to limit a loss or protect a profit on an existing position. If an investor is short stock, he can enter a buy stop order which, if activated, will cover his short position and protect the profit or limit the loss on the short position. A technical analyst may also place a buy stop above the resistance level to purchase the stock should there be a price breakout.

Which TWO of the following securities will enable an investor to both receive interest income and have a maturity date allowing their principal to be returned in one lump sum? I. A municipal bond fund containing mostly revenue bonds II. Municipal bonds that are subject to the alternative minimum tax III. A closed-end fund containing municipal bonds of one state IV. A portfolio of municipal bonds, some which have call provisions

II and IV Explanation: All of the securities listed will pay interest income to investors. The municipal bond fund and the closed-end fund invest in municipal bonds that pay interest. The funds will then pass through these payments to the holders of these securities, either monthly, quarterly, or semiannually. Only by investing in actual bonds will an investor be able to have her principal returned in one lump sum when the bonds mature. The type of municipal bonds, whether they are callable, or whether they are subject to the AMT, is irrelevant. One of the major differences between investing in actual bonds versus bond funds is that bonds will have a maturity date and bond funds will not mature. If a 20-year bond is purchased, 19 years later it will mature in one year. A 20-year bond fund will always have in its portfolio bonds that mature in approximately 20 years. In order for an investor to receive her principal with a bond fund, the investor is required to sell or redeem her shares of the fund.

Which TWO of the following statements are TRUE concerning an institutional communication? I. A principal is required to approve the communication prior to use II. The communication should be reviewed, but is not required to be approved by a principal III. The communication must be filed with FINRA IV. The communication does not need to be filed with FINRA

II and IV Explanation: An institutional communication is defined as any written or electronic communication that is distributed or made available only to institutional investors. Each broker-dealer is required to establish written policies and procedures to review institutional communications, but principal approval is not required. In addition, an institutional communication is not required to be filed with FINRA.

A registered representative has recommended the purchase of a variable annuity to a customer who subsequently completes the application. Which TWO of the following statements are TRUE concerning the application? I. The contract is forwarded directly to the insurance company II. The contract is forwarded to the representative's Office of Supervisory Jurisdiction (OSJ) III. A principal need not approve the transaction IV. A principal must approve the transaction within 7 business days

II and IV Explanation: Annuity suitability rules require that contracts sold through FINRA members be forwarded to the representative's OSJ and be approved by a principal within 7 business days of receipt before being sent to the insurance company.

A registered representative who fraudulently violates an MSRB rule may be disciplined by which TWO of the following organizations? I. The MSRB II. FINRA III. The CFTC IV. The SEC

II and IV Explanation: Both the SEC and FINRA may discipline a registered representative employed at a broker-dealer for fraudulent securities activities. The MSRB has no enforcement power and the Commodity Futures Trading Commission (CFTC) regulates the commodity and futures markets.

A registered representative who fraudulently violates an MSRB rule can be disciplined by which TWO of the following entities? I. The MSRB II. FINRA III. Nasdaq IV. The SEC

II and IV Explanation: Both the SEC and FINRA may discipline a registered representative employed by a broker-dealer for fraudulent securities activities. The MSRB has no enforcement power.

Which TWO of the following statements concerning convertible bonds are TRUE? I. Coupon rates are usually higher than nonconvertible bonds of the same issuer II. Convertible bondholders are considered creditors of the corporation III. Convertible bonds are usually issued by companies with strong credit ratings IV. It is possible that a convertible bond will sell at a price based solely on its inherent value as a bond

II and IV Explanation: Convertible bondholders are considered creditors of a corporation and provide investors with the ability to convert their bonds into shares of common stock of the same issuer at a set price (conversion price). This feature links these types of bonds to the equity markets and the price of a convertible bond is affected by the price of the underlying stock. However, if the price of the underlying stock declines to the point where there is no advantage to the conversion feature, the bond may sell at a price based on its inherent value as a bond, disregarding the convertible feature. Moreover, convertible bonds are issued by companies with weaker credit ratings and allow the issuer to sell debt at a lower cost. Since the conversion feature is a benefit to the bondholder, convertible bonds will have a lower coupon than similar nonconvertible bonds.

Which TWO of the following statements apply for a new issue to be sold in its home state and neighboring states? I. The issuer must file a Rule 147 Form II. Registered representatives selling the stock must be registered in whatever states in which they plan to make sales III. FINRA must approve the sale in all the states IV. The Blue-Sky laws (state securities laws) of all the states in which the securities will be sold must be observed

II and IV Explanation: For the issue to be sold in its home state and neighboring states, individuals selling the new issue must be registered to sell the securities in those states. The Blue-Sky laws (state securities laws) of those states in which the securities will be sold must also be observed.

Which TWO off the following sources of income would MOST likely be used by a school district to meet its debt service for general obligation bonds that it issued? I. Income tax II. Real estate tax III. Sales tax IV. Traffic fines

II and IV Explanation: General obligation bonds are backed by the full faith, credit, and taxing power of the municipality that issues the bonds. The income to pay debt service on these bonds is derived from taxes and other general revenues. For smaller local governments, such as school districts, it would include primarily real estate taxes (also called property or ad valorem tax). In addition, traffic and other types of local fines may also be used. Income taxes and sales taxes would most likely be used to meet the debt service of larger issuers, such as states and large cities.

Which TWO of the following statements are TRUE regarding Roth IRAs? I. Contributions are in pretax dollars II. Contributions are in after-tax dollars III. Qualifying distributions are taxable IV. Qualifying distributions are tax-free

II and IV Explanation: If an individual earns less than $116,000 (or $183,000 jointly), the full benefits of Roth IRAs can be realized. (For 2014, the limits were $114,000 and $181,000). This would allow the individual to contribute up to $5,500 (plus $1,000 for individuals age 50 and over) in after-tax dollars and receive qualifying distributions that are tax-free.

A customer owns a mutual fund that invests primarily in foreign securities. Which TWO of the following amounts will be reported to the customer concerning the tax treatment of interest and dividends? I. The net amount of dividends and interest II. The gross amount of interest and dividends III. The amount of tax paid to the Internal Revenue Service (IRS) IV. The amount of tax withheld by the foreign government

II and IV Explanation: If an investor owns a mutual fund that invests in foreign securities, and dividends and interest are paid to a U.S. investor, these earnings may be subject to withholding tax by the country from which they were paid. The broker-dealer will send the investor a form that will report the gross amount of the dividends or interest, and the amount of tax withheld by the foreign government.

According to MSRB rules, which TWO items need NOT be disclosed on a confirmation sent to a customer? I. The total dollars due including any accrued interest II. A rating provided by a nationally recognized municipal securities rating service III. The capacity in which the broker-dealer acted IV. The denominations of the securities to be delivered

II and IV Explanation: MSRB rules require that a confirmation be sent to a customer at or before the completion of the transaction (settlement date). The confirmation must include whether the customer purchased or sold, the par amount, and a complete description of the securities, including the coupon and the maturity date. Any pertinent call feature must be shown as well as the principal amount, accrued interest, and the total amount of the transaction. The broker-dealer must disclose if it acted as principal or agent and, if acting as an agent, the amount of the commission must be disclosed. Ratings and denominations are not included on the confirmation.

A registered representative is reviewing a corporation's financial statements. Which TWO of the following statements are TRUE concerning an issuer's bond interest expense? I. The annual interest payments are found on the balance sheet II. The annual interest payments are found on the income statement III. The interest payment is deducted from net income IV. The interest payment is deducted from EBIT

II and IV Explanation: The annual interest payment or bond interest expense may be found on a company's income statement. The amount of debt or bonds outstanding may be found on the balance sheet. The annual interest payment is deducted from the earnings before interest and tax (EBIT). Bond interest is paid in pretax dollars, whereas cash dividends are paid from net income or in after-tax dollars.

New Issue $50,000,000 City of Denver, Colorado Pollution Control Bonds Par Value $1,000 Amount Rate Maturity Date Price $50,000 5 1/2% July 1, 2027 101 $60,000 6 1/2% July 1, 2028 101 Based on the above information, which of the following statements may be made about the bonds maturing in 2027? I. The yield to maturity will be greater than 5.50% II. The yearly interest payment is $55 per bond III. The amount of principal the investor will receive at maturity will be greater than $1,000 IV. An investor holding the bond until maturity will have a yield of less than 5.50%

II and IV Explanation: The bonds maturing on July 1, 2027 have a nominal yield of 5 1/2% and have been issued at 101, which is 101% of their par value of $1,000, or $1,010. The yearly interest payment is 5 1/2% of par, or $55. The bonds are offered at a premium and an investor paying $1,010,will receive $1,000 at maturity. A bond offered at a premium and held to maturity will have a yield of less than the coupon (nominal yield) of 5.50%.

A corporation intends to sell 1,000,000 shares of stock through an underwriter. The prospectus states that 500,000 shares are being sold by the corporation from authorized but unissued stock. Also, 500,000 shares are being sold by officers and directors of the corporation. Which of the following statements are TRUE? I. This is a primary distribution for 1,000,000 shares II. This is a combined primary and secondary distribution of 1,000,000 shares III. All of the proceeds of the offering will go to the corporation IV. The proceeds of the offering will be divided between the corporation and selling officers and directors

II and IV Explanation: The corporation is selling 500,000 shares from authorized but unissued stock which is a primary distribution. The officers and directors are selling 500,000 shares that were previously issued to them and are already outstanding. This is a secondary distribution. The proceeds of the offering will, therefore, be divided among the corporation and the selling officers and directors of the corporation.

When analyzing the credit strength of a municipal issuer, consideration should include which TWO of the following factors? I. The condition of the U.S. economy II. The current financial status of the municipality III. Money supply figures IV. The general capability of the fiscal officers of the municipality

II and IV Explanation: The state of the local economy (not the U.S. economy) is an important factor in determining a municipality's creditworthiness. For example, communities at different stages of growth may require more or less debt and this must be understood in the analysis. The current financial status is also important to determine the credit strength of a municipality. The management capability of the fiscal officers is also important to insure they are able to implement the plans of the municipality. Money supply figures, which are published by the Federal Reserve Board are not relevant with regard to the credit strength of a municipality.

Which TWO of the following choices would be included in a subscription agreement for a direct participation program (DPP)? I. The priority provisions if the partnership is liquidated II. A statement indicating the purchaser understands the risks of this investment III. A statement listing the amount of tax credits or deductions the investor will receive IV. A statement that attests to the investor's ability to meet the financial requirements of this investment

II and IV Explanation: The subscription agreement will normally state the suitability standards for the program, specify who must sign the agreement, specify to whom the check must be made payable, and make inquiries of the purchaser to make sure that she understands the ramifications of the investment and can meet the financial requirements of this investment. Priority provisions for liquidating a limited partnership, and the tax implications, would be found in the offering documents.

Bergen County has issued Build America Bonds to improve its transportation system. Which TWO of the following statements are TRUE concerning these bonds? I. The bonds are federally tax-free II. The bonds are federally taxable III. The issuer will receive a federal tax credit IV. The issuer will receive a federal reimbursement

II and IV Explanation: These are an example of Direct Pay Build America Bonds (BABs). BABs are a type of municipal bond that pays taxable interest but the Treasury will reimburse 35% of the interest paid on the bonds to the issuer, which reduces the cost of borrowing. This would allow municipal issuers to compete with corporate issuers when raising capital.

A client owns 3,000 shares of stock in a company headquartered outside the U.S. The client receives a cash dividend and tax is withheld by the country where the company is located. Which TWO of the following statements are TRUE concerning the U.S. tax implications for the client? I. The dividend received is treated as a return of capital II. The taxes paid may be used as a credit III. The dividend paid is exempt from taxes IV. The taxes paid may be used as a deduction

II and IV Explanation: U.S. citizens and corporations owning foreign stock may receive dividends from which foreign taxes have been withheld. The investor still owes U.S. income tax on the net dividend. The amount of the foreign tax, however, may be claimed by the investor as a deduction against income or may be applied as a credit against U.S. income tax.

Broker-dealers are permitted to take part in which TWO of the following actions regarding the sale of mutual fund shares? I. Tell investors to buy mutual funds shortly before a dividend or capital gains distribution is to be paid II. Assign loan value to fully paid shares that have been owned for more than 30 days III. Provide a discount to nonmember broker-dealers when selling them investment company shares IV. Continue to compensate retired registered representatives for prior sales if a contract was signed with the registered representative who has retired

II and IV Explanation: Under industry rules, broker-dealers are permitted to continue to compensate retired registered representatives for sales made prior to retirement (i.e., to render continuing commissions or trails) if a contract is signed with the registered representative who has retired. To induce an investor to buy mutual fund shares shortly before a dividend or capital gain distribution is to be paid is a violation of the Conduct Rules and is called selling dividends. There is no benefit to the customer because the value of the mutual fund will decline when the fund sells ex- (without the) dividend or when there is a capital gain distribution. The customer could just as well have waited and received the same value in shares. The broker-dealer used the imminent payment of the dividend or capital gain distribution as a sales tool to sell the customer, whereas the customer's needs and objectives should have been the salesman's major consideration. Although broker-dealers may not obtain credit for a customer to buy open-end shares (because mutual funds may not be bought on margin), loan value may be assigned to fully paid shares that the customer has owned for more than 30 days. Also, broker-dealers may not give a discount to nonmember broker-dealers when selling investment company shares.

As the initial transaction in a new margin account, a customer buys 1,000 shares of XYZ stock at $30. The market value increases to $50. Which statements are TRUE? I. Equity increases by $10,000 II. Equity increases by $20,000 III. SMA increases by $5,000 IV. SMA increases by $10,000

II and IV. (Equity by $20,000, SMA by $10,000) In a long margin account, for every $1 rise in market value, equity will increase by $1. Since initial margin is set at 50%, for every $1 rise in market value, SMA increases by 0.50 (that is, 50% of the market value increase). The account's equity went up $20,000; so the SMA would go up by 1/2 the amount, or by $10,000. Initially: LMV - Debit = Equity 30,000 - 15,000 = 15,000 (50%) 0 SMA LMV - Debit = Equity 50,000 - 15,000 = 35,000 (70%) 10,000 SMA

When a major decline occurs within a few minutes of the close, trading is halted on all markets for the remainder of the trading day. Under the market-wide circuit breaker (MWCB) rules, market-on-close (MOC) orders pending at the time trading is halted

must be canceled.

When determining the position limit, the member firm will aggregate which TWO of the following positions? I. Long calls and long puts II. Long calls and short puts III. Short calls and short puts IV. Short calls and long puts

II and IV Explanation: When determining a customer's position in relation to the exchange's position limit, the member firm will consider all positions on one side of the market. The position may not exceed the limit on either the long side of the market or the short side of the market. When determining which is the long side of the market, it is NOT correct to aggregate long calls and long puts, or to aggregate short calls and short puts. Instead, the long side of the market is the side on which the investor acquires stock and the short side is the side on which stock is sold. We will examine first the long side of the market. If investors are long calls, they have the option to call away stock. They would, therefore, acquire stock. If investors are short puts, they have the obligation to accept stock if the put is exercised. In both instances, they will acquire stock. Therefore, long calls and short puts are on the long side of the market and are aggregated to determine the total position. In regard to the short side of the market, if investors are long puts, they have the option to sell stock by exercising the puts. If investors are short calls, they have the obligation to sell stock if the calls are exercised. In both instances, they will sell stock. Therefore, long puts and short calls are on the short side of the market and are aggregated to determine the total position.

Which of the following statements are TRUE regarding fixed unit investment trusts (UITs)? I. Fixed UITs are managed II. Fixed UITs are unmanaged III. The composition of the portfolio can be changed IV. The composition of the portfolio cannot be changed

II and IV Fixed UITs are unmanaged and CANNOT be changed.

A 200% Leveraged Dow Jones Industrial Average Index ETF would be expected to move: I. up 50% in price when the DJIA moves up 100% II. up 100% in price when the DJIA moves up 50% III. down 50% in price when the DJIA moves down 100% IV. down 100% in price when the DJIA moves down 50%

II and IV A leveraged ETF uses borrowing (margin) and options to magnify price movement as compared to the reference index. A 200% leveraged ETF can be expected to move 2 times as fast as the reference index, either up or down. A 300% leveraged ETF can be expected to move 3 times as fast as the reference index, either up or down.

New U.S. Government Agency Securities are: I. underwritten via competitive bid II. underwritten via negotiated offering III. offered through the Federal Reserve IV. offered through selling groups

II and IV (negotiated offering, selling groups)

When ABC is at $49 per share, a customer makes the following trades: Buy 1 ABC Jan 50 Put @ 4 Sell I ABC Jan 65 Put @ 11 The customer will profit if : I. The spread between the premiums widen II. The spread between the premiums narrow III. Both contracts are exercised IV. Both contracts expire

II and IV (spread narrows and contracts expire) Since this is a credit spread, the position would be profitable if both contracts expire allowing the customer to keep the net credit.

A municipal securities broker's broker will complete transactions for a: I. Retail customer II. Dealer bank III. Municipality IV. Municipal broker-dealer

II and IV only Explanation: A broker's broker works only for other professionals in the industry, executing trades for dealer banks or other broker-dealers, but not for retail customers or municipalities. The purpose of a broker's broker is to provide anonymity to market participants.

Retail communications regarding options: I. Must be submitted to the exchange 15 days prior to initial use II. Must be submitted to the exchange 10 days prior to initial use III. Must be kept on file by the member firm for six years IV. Must be kept on file by the member firm for three years

II and IV only Explanation: Retail communications regarding options must be submitted to the firms option regulator (an exchange or FINRA) for approval at least 10 days prior to initial use. All retail communications must be maintained on file by the member firm for three years.

If convertible bondholders convert their bonds into the common stock of a corporation, the effect on the balance sheet of the corporation will be: I. An increase in current assets II. A decrease in total liabilities III. A decrease in stockholders' equity IV. An increase in stockholders' equity

II and IV only Explanation: The conversion of bonds to common stock reduces the total debt of the corporation while increasing stockholders' equity (additional shares of common stock). The answer, therefore, will be a decrease in the total liabilities and an increase in stockholders' equity

Bud Jones purchased 100 shares of DEF at 20 on June 16 and passed away on July 27 when the market value of DEF was 25. If the 100 shares of DEF are inherited by Mr. Jones's daughter Mary, what are the tax implications? I. Mary assumes a cost basis of 20 II. Mary assumes a cost basis of 25 III. The holding period for the stock is short-term IV. The holding period for the stock is long-term

II and IV only Explanation: When securities are inherited, the recipient's cost basis is the market value of the securities at the time of the deceased's death. The recipient's holding period for the stock will be long-term, regardless of the deceased's actual holding period.

Under the short sale tender rule, a customer is prohibited from tendering common shares unless the customer is long a: I. call option for that security and intends to exercise the call. II. call option for that security and has exercised the call. III. warrant for that security and intends to exercise. IV. warrant for that security and has exercised the warrant

II and IV only Under the "short tender" rule, a customer is prohibited from tendering unless he has a net long position in the stock. The customer is not considered to be long the common stock until the call option, warrant, rights to buy that security are exercised.

Guarantees on insurance products

must be specific to the insurance contract and not associated investments.

When opening a new account, what is the order in which the following actions should take place? I. Obtain approval from the ROP II. Obtain essential facts from the customer III. Obtain a signed options agreement IV. Enter the initial order

II, I, IV, and III Explanation: When opening an account, the first step is to obtain the essential facts regarding the investor's investment objectives and financial means. The account is then approved by the manager and the initial order can be entered. The member firm has 15 days to obtain the signed options agreement.

Which of the following Moody's MIG ratings are considered non-investment grade? I. MIG 1 II. MIG 2 111. MIG 3 IV. SG

III AND IV (MIG 3 and SG) Moody's rates municipal anticipation notes under the "MIG" (Moody's Investment Grade) ratings scale, with MIG 1 and MIG 2 being the investment grades; and the non-investment grades being MIG 3 and SG (speculative grade).

Which TWO of the following securities pay a dividend that is NOT eligible for the corporate dividend exclusion? I. Common stock II. Preferred stock III. A real estate investment trust IV. A money-market fund

III and IV Explanation: Corporations are allowed an exclusion on dividends received from investments in common and preferred stock. Real estate investment trusts (REITs) make distributions in pretax dollars. The payout from a REIT normally results from collections of rent or mortgage interest. Money-market fund dividends are distributions of interest earned on short-term debt securities.

Which TWO of the following statements are TRUE regarding mutual funds? I. The maximum sales charge is 7% II. Investors can receive a reduced sales charge if they sign a 10-month letter of intent to purchase a certain dollar amount of mutual fund shares III. Under the right of accumulation, investors can receive a reduced sales charge on new purchases when a breakpoint is reached IV. No-load funds may charge a liquidation fee when an investor sells the fund

III and IV Explanation: The only true statements regarding mutual funds that are listed are that under the right of accumulation option, mutual fund investors can receive a reduced sales charge on new purchases when a breakpoint is reached, and no-load funds may charge a liquidation fee when an investor sells the fund. Under industry rules, the maximum sales charge for a mutual fund is 8 1/2%, not 7%. A letter of intent for a reduced sales charge is for 13 months, not 10 months.

Which of the following are non-exempt securities under the Securities Act of 1933? I. Government National Mortgage Association Mortgage Pass Through Certificates II. Small Business Investment Company Shares III. Commercial paper maturing over 270 days IV. Variable annuity contracts

III and IV Government National Mortgage Association is owned by U.S. Government. Its issues are exempt from the provisions of the Securities Act of 1933. Small business Investment companies are also exempt. For commercial paper to be exempt, its maturity must be over 270 days or less. Variable annuity contracts are also a non-exempt security that must be registered under the 1933 Act.

Which of the following are characteristics of Defined Benefit Plans? I. annual contribution amounts are fixed II. if the corporation has an unprofitable year, the contribution may be omitted III. the annual benefit amount is fixed at retirement IV. the adoption of this type of plan benefits key employees who are nearing retirement

III and IV only

For tax purposes, corporations may exclude a portion of the dividends received from: I. Municipal bonds II. Corporate bonds of other corporations III. Preferred stocks of other corporations IV. Common stocks of other corporations

III and IV only Explanation: Corporations may exclude a portion of the dividends received from equity investments in other corporations. This includes common stock and preferred stock.

When the economy is peaking, what will be the expected sequence of the next three stages of the business cycle? I. Trough II. Expansion III. Contraction

III, I, II Explanation: Historically, the business cycle has moved sequentially through four stages. An expanding economy will peak once the supply of goods and services surpasses demand. As the economy contracts, demand for products decreases causing a reduction in business activity. The economy will bottom out (forming a trough), leading the way to expansion and the beginning of a new cycle. Thus, if the economy is at its peak, the next three stages in succession will be contraction (recession), trough, and expansion.

A customer is most interested in safety of principal and wishes to avoid risk. List the securities you would recommend to the customer from those with the LEAST risk to those with the MOST risk. I. General obligation bonds II. Treasury notes III. Treasury bills IV. Revenue bonds

III, II, I, and IV Explanation: The safest security with the shortest maturity is Treasury bills, followed by the longer-term Treasury notes, followed by general obligation bonds, and then revenue bonds.

List the order of priorities that you would follow when opening an options account for a customer. I. Have the customer sign the options agreement II. Have the registered options principal approve the account III. Send the customer an Options Clearing Corporation risk disclosure document IV. Enter the order

III, II, IV, I Explanation: The customer needs to receive the Options Clearing Corporation risk disclosure document at or before the time the account is approved for options trading. The registered options principal then approves the account for options trading. Orders may then be entered. Finally, the customer has 15 days from the time the account is approved for options trading to sign the options agreement.

Regulation NMS required market centers to do all of the following EXCEPT: I. electronically link quotes accessible in real time II. provide for automated execution at best price within 1 second III. execute all short sales on stocks that are declining in price on an up bid IV. put procedures in place to prevent trade-throughs

III. They do not require market centers to execute all short sales on stocks that are declining in price on an up-bid.

Passive losses from an investment in a limited partnership can be offset by which of the following? I. Earned Income II. Interest Income III. Capital Gains Income IV. Passive Income

IV only. Passive losses can only be offset against other passive income - they cannot be offset against earned income; nor can they be offset against portfolio income such as interest.

An application to purchase a variable annuity need NOT be approved by a principal:

If the application is not submitted through a broker-dealer Explanation: An application for the purchase of a variable annuity that is received directly from the customer need not be approved by a principal. If the application is submitted through a broker-dealer, it must be approved by a principal or rejected.

According to MSRB rules, the delivery of a mutilated certificate is considered a good delivery:

If the certificate is authenticated by the issuer or transfer agent Explanation: A mutilated certificate may be authenticated by the issuer or an agent of the issuer (e.g., a transfer agent or paying agent). If authenticated, it is considered a good delivery. A mutilated coupon may be guaranteed by any commercial bank as well as the issuer or its agent.

A member firm is required to send duplicate account statements to FINRA:

If the customer is an employee of FINRA Explanation: A member firm is required to send duplicate account statements to FINRA when a customer of the firm is an employee of FINRA. The member firm would need written instructions from the employee of FINRA when opening an account in order to send the duplicate account statements.

A registered representative is permitted to contact a person whose name is on the national Do Not Call List:

If the person has provided prior written consent to be contacted Explanation: A person may register her number on the Federal Trade Commission (FTC) National Do Not Call Registry and have it remain on the list indefinitely. Broker-dealers are required to download the names on this list in order to prevent RRs from cold calling any of these persons. The following three exceptions exist which permit RRs to contact any person whose name is on this list. 1. The firm has obtained the person's expressed prior written consent as evidenced by a signed written agreement. The agreement must state that the person agrees to be contacted and the telephone number to which calls may be made. 2. There is an established business relationship between the RR and the person. 3. There is a personal relationship between the RR and the person.

Ms. Thomas calls her registered representative with an order to buy up to 2,000 shares of XYZ at $35 per share right now and do not leave the unexecuted portion as a day or open order. Ms. Thomas has entered a(n):

Immediate-or-cancel order Explanation: An order that dictates to fill as much of the order as you can right now and cancel the rest is called an immediate-or-cancel order. Limit orders are placed as either day or GTC orders and the unexecuted portions are placed on the designated market maker's book.

A customer who has purchased an exchange-traded fund (ETF) may be extended credit by a broker-dealer:

Immediately Explanation: Some investment companies, such as mutual funds, are marginable under Reg. T. However, they are considered new issues of securities. The Securities Exchange Act of 1934 prevents a dealer from extending credit on a new issue for at least 30 days. Once the mutual fund shares have been held for 30 days, these securities may be used as collateral for a loan in a margin account at the dealer. In this question, the client is purchasing an ETF, which is not considered a new issue. In this case, credit may be extended immediately.

A customer purchases 200 shares of XYZ at 17.50 and writes 2 XYZ Jan 20 calls at 1. At expiration, with the stock trading at 19, the options expire worthless. If the customer sells his long stock at the current market price, the gain is

The customer buys stock at $17.50 and sells his shares at $19 for a gain of $300. In addition, the customer keeps the $200 in premiums, for an overall gain of $500.

Level I of Nasdaq indicates the:

Inside market for the security listed Explanation: Nasdaq Level I provides subscribers with the highest bid and the lowest offer for a security (the inside market) in which there are at least two market makers. Actual market makers are not listed. Level I does not show cumulative trading volume nor the price of the transactions as they occur. Although nonmembers can also subscribe to Nasdaq Level I, it is typically used by branch offices of member firms.

A registered representative is sending an e-mail to banks and investment advisers in anticipation of a new product being offered by the firm. This is defined as a(n):

Institutional communication Explanation: FINRA's Communications with the Public Rule defines different types of communication. Correspondence, which is defined as any written or electronic communication that is distributed or made available to 25 or fewer retail investors within any 30 calendar-day period. Institutional communication, which is defined as any written or electronic communication that is distributed or made available only to institutional investors. This would not include any internal communication by the broker-dealer. Retail communication, which is defined as any written or electronic communication that is distributed or made available to more than 25 retail investors within a 30 calendar-day period. Public appearances are situations where employees associated with a broker-dealer or sponsor participate in a television or radio interview, seminar, or forum, or make a public appearance, or engage in speaking activities that are unscripted and are not otherwise considered retail communication. Social media sites, which permit real-time communication or interactive, electronic forums, fall under the guidelines of a public appearance (e.g., Facebook, Twitter, and LinkedIn). An institutional investor under this rule is any bank, S&L, insurance company, registered investment adviser or investment company (mutual fund), any person with total assets of at least $50 million, government entity, employee benefit plan, any member firm or registered person of the firm, and any person acting solely for any institutional investor. Since the e-mail is being sent only to institutional investors, it is defined as an institutional communication.

A fidelity bond is:

Insurance purchased by broker-dealers to protect them against fraud Explanation: Every broker-dealer is required to have a fidelity bond, which provides insurance in the event of a fraud judgement against the broker-dealer.

During the first year, an investment in an oil and gas drilling program will generate the largest deduction from:

Intangible drilling costs Explanation: Intangible drilling costs may be taken as an expense item. Therefore, these costs provide a large deduction in the first year. Depletion and depreciation provide deductions that are spread out over a period of years. Production is an income item, not a deduction.

Which of the following statements regarding Sallie Mae debentures are true?

Interest is generally paid semiannually. Interest is exempt from state and local taxation.

A newly issued bond has a provision that it cannot be called for five years after the issue date. This call protection would be MOST valuable to a recent purchaser of the bond if:

Interest rates are falling Explanation: The call protection provision of five years would be most valuable to a recent purchaser of the bond if interest rates are falling. If interest rates fall, outstanding bond prices will rise. Issuers of bonds will call or retire bonds when interest rates decline, and will issue new bonds with lower rates of interest. Bonds are usually callable at a small premium above par value. If the bonds are not callable, the investor can realize the full benefit of an increase in the market price of the bonds.

Charlene contacts her registered representative to buy an OTC stock. Rather than buying it directly from a market maker, Charlene's broker-dealer contacts another broker-dealer, who buys it from a market maker creating two levels of transaction fees. This is known as:

Interpositioning Explanation: Interpositioning occurs when a broker-dealer, executing an order for a customer, places another broker-dealer between itself and the market. This is generally prohibited.

Treasury arbitrage restrictions generally prohibit issuers of municipal securities from:

Investing bond proceeds in higher-yielding Treasury securities Explanation: Because of the tax exemption allowed on municipal bond interest, municipalities are normally able to issue bonds with coupon rates below those of Treasury securities. This presents an excellent arbitrage opportunity. A municipality can borrow at a low rate of interest and invest the money in higher-yielding risk-free Treasury securities. Congress has enacted laws, known as Treasury arbitrage restrictions, that prevent state and local governments from misusing the tax exemption.

The department of a brokerage firm that advises a corporation regarding the structure and timing of a future issue of stock, and assists in the underwriting of the securities is the:

Investment banking department Explanation: The investment banking department assists issuers who need to sell new securities to the public. The sales and trading department is involved in the secondary market trading of securities. The purchase and sales department and the reorganization department are both part of a firm's operations area involved in processing the trades and maintaining the books and records pertaining to customer accounts.

A registered representative receives an order from the president of XYZ Corporation to sell unregistered XYZ shares. The client purchased the shares in a private placement 90 days ago. This order:

Is a violation of Rule 144 if executed Explanation: According to Rule 144, an affiliated person (e.g., the president of a company) must hold unregistered (restricted) stock for at least six months before it may be sold. Since the president of XYZ Corporation owned the stock for only 90 days, the order to sell violates Rule 144, if executed.

Mr. Green is the manager for an asset allocation fund. In May, the fund's portfolio is allocated as follows. Cash (including T-bills) 5% Convertibles 12% Corporate Bonds 18% Common Stock 65% During the first week of June, Mr. Green shifted the assets in the portfolio to reflect: Cash (including T-bills) 65% Convertibles 5% Corporate Bonds 12% Common Stock 18% The reason for the change is most likely that Mr. Green:

Is bearish on bonds and stocks Explanation: The portfolio shift reflects significantly lower emphasis on stocks and a reduced position in bonds. If the manager anticipated a decrease in interest rates, he would be bullish on bonds. The bond allocation would then be expected to increase. Fundamental analysts are not market timers.

The following dividend information for New York Stock Exchange listed common stocks is reported in The Wall Street Journal. Quarterly Company Dividend Record Date Payable Date Cummings Corp. 50 cents 4/10 5/15 Federal Corp. 85 cents 4/13 5/25 General Electric 95 cents 4/8 5/21 A buyer of Cummings Corporation on May 10:

Is not entitled to receive the 50-cent quarterly dividend Explanation: A buyer of Cummings Corporation is not entitled to receive the 50-cent quarterly dividend because the purchase was made on May 10. This was after the stock had already sold ex-dividend (without the dividend). The ex-dividend date is not given but the record date of April 10 is. Stocks sell without the dividend (ex-dividend), on the second business day preceding the record date. This would be two business days prior to April 10, which is more than one month before the customer bought the stock. Even if the purchase was made for cash, which requires a same-day payment, it is still one month too late for the buyer to receive the dividend.

If the FOMC enters into a repurchase agreement, what is the immediate effect on the amount of money in the banking system?

It increases the amount Explanation: In a repurchase agreement (Repo), the Federal Open Market Committee first buys the government securities. This action adds money to the banking system. A short time later the dealer repurchases the securities from the Fed. Whether the transaction is a repo or reverse repo is determined by considering the dealers point of view.

The advance-decline theory states that:

It is bullish if more stocks go up than go down during the day Explanation: A technical indicator that measures the strength of the market by comparing the number of stocks that increase and decrease is called the advance-decline theory. It shows the general direction and breadth of a market movement on a given day.

Which of the following statements does NOT describe an equity-indexed annuity? a. It offer a guaranteed minimum rate of return b. It provides a return based on the performance of a stock market index c. It is considered a security d. It provides tax-deferred growth

It is considered a security Explanation: Equity-indexed annuities (EIAs) are annuities that provide a guaranteed minimum rate of return (unlike variable annuities), but can yield a greater rate of return based on the performance of a linked stock market index. They also provide tax-deferred growth. Currently, EIAs are not considered securities.

A company currently has $125,000,000 of 3 1/4% convertible bonds. The company is going to offer bondholders $125,000,000 of 3 1/4% nonconvertible bonds plus cash of $15,000,000 for the convertible bonds. How will this transaction, if successful, affect the company's financial status?

It will reduce the cash position and the potential dilutive effect on the common stock Explanation: The effect of the transaction will be to reduce the cash position and the potential dilutive effect on the common stock. The company is paying out cash and is also issuing nonconvertible bonds in place of convertible bonds (which could have been converted into common stock). This will reduce the cash position and the potential dilutive effect on the common stock.

According to MSRB rules, which of the following statements is TRUE regarding a secondary market joint account? a. It is a violation of the rules if it contains less than three members b. Its members are not permitted to disseminate more than one quote relating to the account's securities c. It needs to submit an underwriting fee to the MSRB d. It is considered to have a control relationship with the issuer

Its members are not permitted to disseminate more than one quote relating to the account's securities Explanation: Members of a secondary market joint account must publish the same offering price.

An investor buys T-bonds on Friday, January 16 for cash settlement. This transaction will settle on:

January 16 Explanation: Cash settlement for all securities takes place on the trade date.

Which of the following annuity settlement options would provide the longest stream of income over the lives of two individuals?

Joint and last survivor annuity Explanation: The joint and last survivor settlement option would provide the longest stream of income as it guarantees payments until the last annuitant dies. The life annuity with 20-year certain would result in payments ending after 20 years even if the survivor was still alive. The unit refund life annuity will only refund the balance of what is left over after the annuitant dies. Payments cease after the annuitant dies in a straight-life annuity.

JULY 20XX S M T W T F S 1 2 3 4 5 6 7 8 9 10 11 12 13 14 15 16 17 18 19 20 21 22 23 24 25 26 27 28 29 30 31 Use the calendar to answer this question. If a customer bought stock for cash (on a cash contract basis) on Friday, July 5, when will the trade settle?

July 5 Explanation: Securities bought or sold for cash or a cash trade have a same-day settlement for payment and delivery. The settlement date is July 5, which is the same day the trade was made.

If a customer buys 100 shares of ABC stock on Tuesday, July 2nd in a regular way trade, the trade will settle on:

July 8th Regular way trades settle 3 business days after trade date. Since July 4th is a legal holiday, 3 business days after July 2nd is July 8th for regular way settlement.

The major disadvantage to a limited partner in a DPP is:

Lack of liquidity Explanation: An investor has limited control (management) in equity investments and no control (management) in bond or DPP investments. The major disadvantage of a DPP is the lack of liquidity, meaning that the investor cannot easily sell his portion of ownership.

Which of the following assets is NOT permitted to be depreciated?

Land Explanation: All of the fixed assets listed lose value through usage or wear and are depreciated except land. The Internal Revenue Service does not allow for land to be depreciated.

An underwriting bid for a municipal general obligation issue would include which of the following?

The dollar amount The coupon rate

An investor purchases a bond on its initial public offering. Even though the bond has a maturity value of $1,000 in 10 years, the offering price is only $600. If the bond is held to maturity,

there are no tax consequences to report.

ABC Corporation has issued two $1,000 par value bonds with the same coupon rate, one paying interest annually and the other paying interest semiannually. If both bonds are held to maturity in 10 years, the bond paying interest annually will have a total return that is:

Less than the bond paying interest semiannually Explanation: The bond paying interest annually will have a yield to maturity that is less than the bond paying interest semiannually. Yields to maturity assume a reinvestment and compounding of interest. The compounding of interest will be greater for the bond paying semiannual interest.

Which of the following is a bull spread?

Long Jul 30 put, short Jul 35 put A debit call spread is bullish and a credit put spread is bullish. Long July 30 put, short July 35 put is the only bullish position in the answer choices.

Your client's position is long 100 MNO purchased at 90. Which of the following strategies will limit the customer's loss to $700?

Long one MNO 90 call at 4, long one MNO 90 put at 3

Under the 5% markup policy, which of the following determines the amount of markup in a principal transaction?

Lowest ask (inside offer) Markdowns are based on the highest bid (inside bid)

Rosewood Securities LLC has been accused of buying and selling securities for the purpose of creating artificial trading activity. Which of the following choices BEST describes this activity? a. Churning b. Matched orders c. Capping d. Stabilization

Matched orders Explanation: A matched order is also known as painting the Tape. It is an illegal activity based on a group of market manipulators buying and/or selling a security among themselves to create artificial trading activity. The intention of this activity is to lure unsuspecting investors into trading the stock because of the appearance of unusual trading volume. The manipulators have already taken a position in the stock, and hope to influence the market (illegally) to make their position profitable through this fake heavy trading volume. Churning is a violation in which a salesperson effects a series of transactions in a customer's account that are excessive in size and/or frequency in relation to the size and investment objectives of the account. A salesperson churning an account is normally seeking to maximize her income (in commissions, sales credits or markups) derived from the account. Capping is a situation where a manipulator is attempting to stop a securities price from rising. Stabilization is a practice used in connection with certain public offerings in which an underwriter posts an open bid for securities at a stated price. Stabilization is intended to maintain an orderly market for the securities during the underwriting and to prevent sharp fluctuations in the market for the securities due simply to supply factors. Properly disclosed, this is an acceptable practice.

A reverse repurchase agreement is sometimes called a(n): a. Repo b. Arbitrage c. Matched sale d. Treasury sale

Matched sale Explanation: A reverse repurchase agreement (matched sale) occurs when the Federal Open Market Committee (FOMC) sells securities to dealers with the intention of buying the securities back at a future date. This has the short-term effect of absorbing (removing) funds from the money supply.

An investor is a limited partner in a direct participation program that the IRS has determined to be abusive. This investor:

May be subject to pay back taxes as well as penalties and interest Explanation: If the IRS deems a direct participation program abusive, deductions previously claimed may be disallowed causing investors to pay back taxes as well as interest and penalties on the back taxes. A DPP may be considered abusive if it is based on a false assumption or if it overstated property values for the purpose of generating large deductions.

An aunt wishes to give her niece securities as a gift. The niece's parents have recently died and a court has appointed a guardian other than the aunt. The aunt:

May give the securities without the permission of the guardian Explanation: The aunt may give securities to the minor as a gift. There are no restrictions on a donor giving a gift.

A client's wife calls and wants to purchase 200 shares of XYZ in her husband's personal account. The registered representative handling the account knows that a favorable earnings report is about to be issued. The registered representative:

May not accept the order because the wife does not have trading authorization to enter orders for the husband's personal account Explanation: The registered representative may not accept the order because the wife does not have trading authorization to enter orders for her husband's personal account.

Which of the following investments is most suitable for an investor seeking monthly income?

Money market mutual fund

Collateralized mortgage obligations (CMOs) make interest payments to investors:

Monthly Explanation: CMOs are issued in minimum denominations of $1,000, are backed by pass-through securities (FNMA, GNMA, and FHLMC), and pay interest and principal monthly.

Which of the following municipal bonds may be paid by a state's legislative apportionment of funds to service the debt?

Moral obligation

A customer entered a market order to purchase 100 shares of XYZ Corporation. The brokerage firm confirms to the customer the purchase of 100 shares of XYZ Corporation at 28.25. The firm later finds that the purchase was actually executed at 28.75. The customer:

Must pay 28.75 Explanation: The customer must pay 28.75, which was the actual purchase price, even though the brokerage firm confirmed (erroneously) to the customer that the purchase was made at 28.25.

A variable annuity application sent by a FINRA member does not include a principal's approval. The insurance company:

Must reject the application Explanation: Annuity suitability rules require that contracts sold through FINRA members be forwarded to the representative's OSJ and be approved by a principal within 7 business days of receipt before being sent to the insurance company. If a principal does not approve the application, it must be rejected.

An uncle is the custodian for a nephew's account. When the nephew reaches majority, the uncle:

Must transfer all the securities in the account to the nephew Explanation: When the nephew reaches majority, the uncle must transfer all the securities in the account to the nephew.

Pickette Financial Services is participating in the IPO of Swank Tanks, Inc., as the managing underwriter. If a research analyst at Pickette wants to initiate coverage on Swank Tanks, she:

Must wait 40 calendar days after the offering date Explanation: A research analyst of Pickette Financial Services must wait 40 days after the date of the offering to publish a research report, or make a public appearance. This quiet period is applied to members that have agreed to participate as a manager or comanager of the IPO. Other participants, such as syndicate and selling group members, must wait 25 days to publish a research report or make a public appearance after the IPO date.

After a mutual fund's 10th year, performance statistics must show results for each of the following periods

Mutual fund performance statistics must show results for one, five, and 10 years, or for the life of the fund

A director of a publicly held company wants to sell 5,000 registered shares of that company's stock at $8 per share that she has held for 3 months. Does the Form 144 filing requirement apply to this sale?

No, because the shares are being sold under a "de minimis" exception. Rule 144 includes a "de minimis" exception, permitting the sale every 3 months of 5,000 shares or less, worth $50,000 or less, without filing a 144 form.

An investor purchased $200,000 of 6% general obligation bonds on margin. The customer has a debit balance of $50,000 and is paying interest of 10% yearly on the debit balance from the purchase of the municipal bonds. How much interest expense may the investor use as a deduction for federal income tax purposes?

None Explanation: The investor may not use any of the interest expense as a deduction against ordinary income. Interest charges on money borrowed to purchase federally tax-exempt municipal securities may not be used as an interest expense deduction for federal income tax purposes. The investor is already receiving the benefit of tax-free interest income from the municipal bond and the IRS will, therefore, not allow the interest expense to be deducted as well.

A customer has funded her IRA with pretax contributions. The account has total assets of $350,000. How much may be withdrawn from this account tax-free?

None of the funds Explanation: The earnings on an IRA account grow tax-deferred. If only pretax contributions are made, the entire withdrawal is taxable as ordinary income. None of the funds may be withdrawn tax-free. If an investor maintains an IRA account that has pretax and after-tax contributions and makes withdrawals, the IRS considers withdrawals to come from both sources. Therefore, a portion of the withdrawal is taxable and the other portion is tax-free. Withdrawals from a Roth IRA are tax-free if certain conditions are met.

The 5% markup policy applies to:

Nonexempt securities Explanation: The 5% markup policy does not apply to transactions requiring a prospectus (new issues, mutual funds, and registered secondaries) or transactions in certain exempt securities (such as municipal securities).

Sipcar is a stock listed on Nasdaq. The inside market is $3.40 - $3.45. A client contacts an RR and wants to place an order to buy the stock at $3.425. The BEST course of action for the RR is to:

Not accept the order Explanation: According to Regulation NMS, broker-dealers are prohibited from accepting bids, offers, or indications of interest for NMS stocks priced at $1.00 or more, in increments smaller than $0.01 and, for NMS stock less than $1.00, in increments smaller than $0.0001 (a hundredth of a penny). The RR is not permitted to accept an order to buy or sell in subpennies (more than two decimal places) if the stock is trading at or above $1.00. The RR is not permitted to accept this order.

John Jones, a shareholder of XYZ Corporation, reads in the newspaper that XYZ Corporation intends to issue new shares through a rights offering. The terms of the rights offering are as follows: 1. 10 rights plus $10.50 are required to subscribe to one new share of stock 2. Fractional shares become whole shares 3. The record date is Friday, October 17 4. JPMorgan Chase and Bank of America are the transfer agents 5. Goldman Sachs and Morgan Stanley are the standby underwriters John chose to subscribe to the rights offering and purchased additional XYZ Corporation common stock on October 16. Based on his latest stock purchase, he will:

Not be permitted to subscribe because the stock traded ex-rights on October 15 Explanation: John would not be permitted to subscribe to the rights offering for the new shares because he purchased the stock on October 16. The stock sold ex-rights on October 15. Therefore, he would not be a stockholder of record for these shares on the record date of October 17.

A customer enters a sell stop-limit order for 100 XYZ at 25.50. XYZ trades occur as follows: 25.50, 25.25, 25.13, SLD 25.50. The customer's order was:

Not executed Explanation: The first trade at 25.50 touched the stop price of 25.50 and the order became an active or live order to sell 100 shares of XYZ at a limit price of 25.50 or better. Thus, the stock must increase to at least 25.50 for an execution. The only other trade at 25.50 has the symbol SLD next to it, indicating that a trade occurred previously (assume prior to the other trades shown), was reported out of sequence, and is now being shown to indicate that fact. There is no trade at the customer's limit price of 25.50 after the customer's order became a live order. Therefore, the customer's order was not executed.

SIPC is a(n):

Not-for-profit corporation Explanation: The Securities Investor Protection Corporation (SIPC) is a nonprofit corporation that was created by an act of Congress in 1970. SIPC insures a customer's account for up to $500,000 in the event of a brokerage firm's failure. SIPC is not a government-sponsored agency or a regulatory body.

An individual sells investment company products after passing the General Securities Representative Examination (Series 7). According to MSRB rules, if this individual will be selling municipal securities:

Nothing more is required Explanation: Passing the General Securities Representative Examination (Series 7) satisfies the MSRB examination requirement for municipal securities sales limited representatives. This registration allows a person to engage in sales to, or purchases from, customers in municipal securities. In this case, the individual need not complete an apprenticeship period due to prior experience. Experience as a general securities representative, mutual fund salesperson, or government securities representative would fulfill the apprenticeship requirement.

Specialist/DMM

Obligated, under NYSE rules, to make a continuous market in the assigned stock. A customer order is always assured that the trade will be executed - however the price at which the trade is executed is always subject to market conditions.

The theory that states that the small investor is usually wrong, buying at market peaks and selling at market bottoms, is called the:

Odd-lot theory Explanation: The theory that states that the small investor is usually wrong because he is uninformed, buying at market peaks and selling at market bottoms, is called the odd-lot theory. According to this theory, the small investor can afford only to buy an odd-lot (less than 100 shares of stock). Odd-lot buying on balance (more buying than selling) is bearish and odd-lot selling on balance (more selling than buying) is bullish.

The most detailed financial information regarding a municipal securities issuer is found in the:

Official statement Explanation: Municipal securities are exempt from the registration provisions of the SEC. Therefore, a registration statement and prospectus are not required. Municipal issuers voluntarily provide the same financial information that would be found in a prospectus. This detailed financial information is found in the official statement. The notice of sale contains information pertaining to a competitive offering of bonds such as the time, place, date of sale, and type of offering.

A customer purchases 1,000 shares of an OTC equity in a cash account through an online brokerage firm on Wednesday, March 11th. The transaction will settle:

On March 16th Explanation: Regular way settlement of corporate securities is three business days. The transaction would settle on Monday, March 16th.

A when, as, and if distributed contract settles:

On the day determined by FINRA Explanation: When, as, and if issued contracts are contracts for securities that are trading but are not yet available for delivery. They are also referred to as a when-issued (WI) security. FINRA determines when the date of settlement will be, i.e., when a sufficient percentage of the issue is outstanding.

A customer owns a call on ABC Corporation that has a $60 strike price. ABC Corporation has announced a 5-for-4 split. After the split, the customer will own:

One call for 125 shares at a $48 strike price Explanation: The company has announced a 5-for-4 split. After the split, the customer will own one call contract representing 125 shares with a $48 strike price. In an odd split, the number of contracts remains the same. The number of shares per contract is increased by multiplying 100 times the split ratio (100 x 5/4 = 125). The strike price is reduced by multiplying it by the inverse of the split ratio (60 x 4/5 = 48).

The tool most commonly used by the FRB to regulate the amount of money and credit in the banking system is: a. Open market operations b. The discount rate c. Margin requirements d. Reserve requirements

Open market operations Explanation: Of all of the tools of the Federal Reserve Board listed, the one most commonly used is open market operations. This is the most flexible tool and can be changed or fine tuned very easily by buying or selling more or less U.S. government securities in the open market. The other choices are not as flexible, but are used to implement FRB policy. The margin requirement is another tool the FRB can use, but the margin requirement is the least likely tool the FRB would use since it only affects a small segment of the economy.

The proceeds of the sale of a municipal bond issue are invested in U.S. government securities that are sufficient to cover interest, principal, and call premiums on an outstanding bond issue. The outstanding bonds are called:

Prerefunded bonds Explanation: The outstanding bonds are called prerefunded or advance-refunded bonds. The new issue is called a refunding issue. This is usually done when the issuer can borrow funds at lower rates, thereby reducing its interest costs.

The tool most commonly used by the FRB to regulate the amount of money and credit in the banking system is: a. Open market operations b. The discount rate c. Moral suasion d. Reserve requirements

Open market operations Explanation: Of all the tools of the Federal Reserve Board listed, the one most commonly used is open market operations. This is the most flexible tool and can be changed or fine-tuned very easily by buying or selling more or less U.S. government securities in the open market. The other choices are not as flexible, but are used to implement FRB policy. The margin requirement is another tool the FRB can use, but the margin requirement is the least likely tool the FRB would use since it affects only a small segment of the economy.

A client buys 100 shares of XYZ Corporation at $27 per share and writes an XYZ October 30 call at a $3 premium.The option order ticket would be marked:

Opening sale, covered Explanation: The question states that the individual writes an option after buying the underlying stock. The initial sale of the option would be an opening sale. Since he is long the underlying stock, he is covered. If he did not own 100 shares of XYZ stock, his position would be uncovered and must be executed in the margin account.

Which of the following securities may not be used as collateral in a margin account?

Option contracts Explanation: Option contracts have no loan value and therefore may not be used as collateral in a margin account. The exception is LEAPS, which can be bought on margin and, therefore, have loan value. LEAPS are equity options that can have a maximum life of 39 months.

Expiration length of Options vs. LEAPs

Options: less than 1 year LEAPs: 1 year of longer

What term would apply to Authorized Stock?

Par value

An RR is hosting an event that is focused on the suitability of variable annuities. At the conclusion of the event, the RR provides a sheet for attendees to provide their contact information if they are interested in setting up a one-on-one meeting with the RR. In this situation, if some attendees provide their contact information, the RR is:

Permitted to contact the attendees even if they are on the National Do Not Call list since they have provided prior written consent Explanation: A person may register her number on the Federal Trade Commission (FTC) National Do Not Call Registry and have it remain on the list indefinitely. An RR is permitted to contact a person whose name is on this list if the person has provided her prior express written consent. In this question, since the attendees who are interested in being contacted are providing their contact information (including their telephone numbers), the RR is permitted to call them.

A registered representative has limited discretion over a customer's account. The registered representative may:

Place orders before the order has been approved by a principal Explanation: Limited discretion does not permit free withdrawal of funds. The account owner must receive confirmations. Buy stop orders are permitted. The RR may place orders which can be approved promptly afterward.

Which of the following CMOs has the LEAST prepayment risk? a. Sequential pay tranches b. Accrual or Z tranches c. Planned amortization class (PAC) tranches d. Support or companion tranches

Planned amortization class (PAC) tranches Explanation: The planned amortization class (PAC) is a type of CMO that is designed for more risk-averse investors and provides a predetermined schedule of principal repayment, as long as mortgage prepayment speeds are within a certain range. This greater predictability of maturity is accomplished by establishing a sinking-fund type of schedule. The PAC tranche has top priority and receives principal payments up to a specified amount. Any excess principal goes to a companion or support tranche that has lower priority. Holders of the companion tranche are generally compensated for this risk with higher yields.

A broker-dealer appears on the Nasdaq system as a market maker for DCIR common stock. An employee of the firm responsible for maintaining the firm's inventory in DCIR is known as a:

Position trader Explanation: A position trader is responsible for maintaining a broker-dealer's inventory as well as trading the firm's account.

A registered representative is taking over the business of another RR who is leaving the firm. Upon examining the accounts, the RR notices that the variable annuities owned by many of the clients have high expenses, mediocre performance, and few investment options. The RR decides that her first action will be to recommend that these customers redeem the old annuities and invest in the new Platinum One variable annuity that has substantially lower expenses, higher long-term performance, and many more subaccounts with varying investment strategies. This activity is:

Potentially acceptable if the benefits of the new annuity outweigh the possible taxes and additional sales charges the client might incur Explanation: The practice of recommending that a client redeem one annuity or mutual fund and invest the proceeds in another annuity (or fund) is called switching. When redeeming the first annuity, the investor might incur deferred sales charges and a tax liability. (The tax liability can be avoided if the switch is eligible to be treated as a Section 1035 exchange.) The reinvestment in another annuity might also involve sales charges or might subject the investor to an additional period when surrender charges could be imposed on redemptions. These disadvantages mean that switching is frowned on by regulators, who suspect the RR involved is often motivated by the prospect of additional commissions rather than the client's best interests. However, when the new annuity is clearly superior to the old product, the additional benefits might outweigh the disadvantages.

A proxy given to a caretaker to vote a stockholder's shares is a:

Power of attorney

Ms. Jones, a shareholder of XYZ Corporation, reads in the newspaper that XYZ Corporation intends to issue new shares through a rights offering. The terms of the rights offering are as follows: 1. 10 rights plus $10.50 are required to subscribe to one new share of stock 2. Fractional shares become whole shares 3. The record date is Friday, October 17 4. JPMorgan Chase and Bank of America are the transfer agents 5. Goldman Sachs and Morgan Stanley are the standby underwriters Ms. Jones also owns 87 shares of the preferred stock of the XYZ Corporation. How many additional shares can she subscribe to and at what cost?

Preferred stockholders are not permitted to participate in a rights offering Explanation: Preferred stockholders are not permitted to participate in a rights offering. Only the common stockholders are permitted.

An announcement in The Wall Street Journal states that New York State plans an advance refunding of its 7 1/2% Dormitory Bonds through the issuance of a special $50,000,000 bond issue. This means that:

Proceeds from the sale of a new bond issue will be put in an escrow account to retire the existing bond issue Explanation: Advance refunding means that proceeds from the sale of the new bond issue will be put in an escrow account to retire the existing bond issue. If a municipality wants to engage in advance refunding, as is the case in this example, the municipality will sell the new issue with the proceeds of the sale going into an escrow account containing U.S. government securities. The U.S. government securities would be purchased with a maturity date that coincides with the issue's call date. This allows the refunded issue to be retired using the proceeds from the matured government securities.

An ad valorem tax is based on:

Property values Explanation: An ad valorem tax is based on property values. The tax is based on the assessed value of the property and the millage rate (tax rate). It may also be referred to as a real estate or property tax.

A client would like to open a numbered account. An RR may open the account:

Provided the broker-dealer has a written statement on file signed by the client Explanation: Any client may open a numbered account for reasons of confidentiality. However, the registered representative should open the account only if the customer signs a written statement acknowledging ownership of the account. This document must be kept on file at the brokerage firm, but does not need to be filed with an SRO.

Many investors prefer to receive variable annuity payments under the straight-life payout option because this option:

Provides the maximum cash flow of all payout options Explanation: Annuitants will receive the greatest cash flow from the straight-life annuity payout option. This option allows an annuitant to receive payments for his lifetime. At death, the payments cease since no beneficiary is designated and, therefore, the insurance company is relieved of its obligation to make payments. The annuitant assumes the greatest degree of risk with this type of payout.

Which of the following risk factors is the MOST important for purchasers of long-term, high-grade bonds? a. The ability to pay interest when due b. The ability to pay principal upon maturity c. Limited marketability d. Purchasing power risk

Purchasing power risk Explanation: Long-term, high-grade bonds are relatively safe investments, but do have purchasing-power risk. Because the amount of interest income is fixed, the purchasing power of the interest income may decline over the long term because of inflation. A rise in inflation reduces the amount of goods and services that can be purchased with the fixed amount of dollars. (Also called Inflation Risk)

TCB Corporation wants to offer $75 million worth of common stock solely to residents of its home state. The issue will not be registered at the federal level. What type of registration will TCB use to register with the state?

Qualification If the registration is just with one state, the registration will be done through qualification

A term bond has a mandatory sinking fund call feature. What method will be used to determine which specific bonds will be called?

Random selection Explanation: Random selection is the method used to call term bonds

Income from which of the following investments is passive income?

Real estate direct participation programs (DPPs) Vacation cottage rentals

A charge to a customer for the collection of dividends, holding of securities, and other services must be:

Reasonably fair and not discriminate between customers Explanation: Under industry rules, a brokerage firm is allowed to charge a customer for collection of dividends, holding of securities, and other services. Most brokerage firms do not charge for these services, but if they do charge, the amount must be reasonably fair and not discriminate between customers.

A stop order will NOT be used to: a. Protect a gain when a long stock position appreciates b. Limit a loss if the market price of a short position increases c. Receive a specific price when buying or selling d. Limit a loss if the market price of a long stock position decreases

Receive a specific price when buying or selling Explanation: A knowledgeable investor will use a sell stop order to protect a profit (or limit a loss) in a long position and a buy stop order to limit a loss in a short position. A sell stop order is entered below the current market and becomes a market order when the stop price is reached or penetrated on the downside. A buy stop order is entered above the current market and becomes a market order when the stop price is reached or penetrated on the upside. Since it becomes a market order when the stop price is hit or penetrated, there is no guarantee as to execution price.

A brokerage firm accepted delivery of securities on settlement. Upon further inspection of the securities, the brokerage firm discovers a problem and wishes to return the securities to the selling dealer. The process of returning securities that have previously been accepted is known as:

Reclamation Explanation: Reclamation is the process of returning securities that were previously accepted on the settlement date. Rejection is when the brokerage firm refuses delivery of the securities on the settlement date.

Warrants will most likely be issued to:

Reduce the interest rate on an issue of debentures Explanation: Debentures may be issued with warrants attached. This allows the corporation to pay a lower interest rate on the debentures.

If a corporation attaches warrants to a new issue of debt securities, which of the following would be a resulting benefit to the corporation?

Reduction of the debt securities' interest rate

Size of the current market:

Refers to the highest bid and lowest ask currently on the book.

There is a halt in the trading of a security that underlies option contracts. Who makes the decision to stop trading the option contracts?

The exchange on which the option trades Explanation: Officials of the exchange on which the options trade make the decision to stop trading the options.

A member of a municipal new issue syndicate is entering an order for an accumulation account being used for a unit investment trust that the firm underwrites. This order must be entered as a(n):

Related portfolio order Explanation: MSRB rules require a syndicate member to disclose to the syndicate an order for a unit investment trust or an accumulation account to be used for a unit investment trust. The disclosure is accomplished by entering the order as a related portfolio order.

Money put aside on a municipal revenue issue for the betterment and improvement of the facility is placed in the:

Renewal and replacement fund Explanation: The renewal and replacement fund holds monies put aside for the improvement of the facility.

A tender offer has been made for PDQ common shares. The brokerage firm department that would handle the tendering of shares is the:

Reorganization department

Which of these securities are BEST described as collateralized loans?

Repurchase agreements Explanation: In a repurchase agreement (repo), a dealer sells securities to another dealer or investor and agrees to buy them back at a specific time and price. In effect, the selling dealer borrows money from another party and collateralizes the transaction with the securities. The other side is lending money and is receiving interest from the dealer creating the repo.

A customer, who is going on vacation, enters a GTC order to buy a stock. The order is executed. The customer tells the registered representative that he wants the stock but will not return in time to pay for the security by the payment date. The customer states he will send in a check a few days late. The registered representative should:

Request an extension Explanation: The customer has indicated that he wants to purchase the stock but will not be able to pay for it in time because he will be on vacation. The order was a good-until-cancelled (GTC) order, so the customer did not know if and when the order would be executed. The reason for the late payment is due to the customer being on vacation. This is a valid reason, and the registered representative should request an extension.

All of the following descriptions are TRUE about stopping stock on the NYSE, EXCEPT that it: a. Is permitted only for public orders b. Requires permission of an exchange official c. Is done by the designated market maker d. Will guarantee a price for the order

Requires permission of an exchange official Explanation: Stopping stock is done by the designated market maker (specialist) to guarantee a price for a public order. The designated market maker does not need permission of an exchange official to do so.

Sales literature includes:

Research reports, market letters or form letters delivered to more than 25 existing or prospective retail clients, scripted speeches delivered to more than 25 existing or prospective retail clients, password-protected websites

It is most beneficial to the holder of a call if the price of the underlying security is:

Rising Explanation: The holder (purchaser) of a call expects the market price of the underlying security to rise and, therefore, will profit from a rise in the security.

Which of the following choices is Standard and Poor's (S&P's) lowest rating for a municipal note?

SP-3 Explanation: Standard and Poor's best rating for notes is SP-1 and its worst rating is SP-3. AAA is S&P's best rating for bonds and D is its lowest rating for bonds.

The Federal Reserve Board was given the authority to set margin requirements according to the provisions of the:

Securities Exchange Act of 1934 Explanation: The Securities Exchange Act of 1934 gave the Federal Reserve Board the power to set margin requirements. This is done through Regulation T (for broker-dealers) and Regulation U (for banks and lenders other than broker-dealers).

Mr. Jones requests that his securities be held in street name. To honor his request, the broker-dealer must:

Segregate the securities from the B/D's own securities Explanation: The only requirement for holding securities in street name is that they must be segregated. A customer signs a margin agreement and hypothecation agreement only when opening a margin account.

A customer buys $10,000 worth of stock in a cash account. Two business days after the transaction settles, the customer calls the broker and tells the broker he does not have sufficient funds to pay for the stock. The brokerage firm will:

Sell him out and freeze the account according to Regulation T of the FRB Explanation: According to Regulation T of the Federal Reserve Board, the brokerage firm must sell out the securities in the account and freeze the account for 90 days.

The term all-or-none, in trading municipal bonds, applies to:

Sellers' offering terms Explanation: Offerings are sometimes made on an all-or-none basis (an AON offering), which is a situation where the offerer agrees to sell the bonds only if all that he has available will be bought.

When opening an account for an employee of FINRA, the member firm is required to:

Send duplicate account statements only Explanation: A member firm is required to send duplicate account statements to FINRA when a customer of the firm is an employee of FINRA. The member firm would need written instructions from the employee of FINRA when opening an account in order to send the duplicate account statements. There is no requirement to send duplicate confirmations or have the account approved by a chief compliance officer.

Sarah would like to help fund her 10-year-old niece's future education. Which of the following choices is her best option?

Set up a Coverdell ESA for her niece Explanation: Anyone may set up or deposit funds in a Coverdell Education Savings Account as long as the total amount deposited from all parties does not exceed $2,000 in any one year. Individuals without earned income may not contribute to an IRA. A minor may not participate in a joint account.

Which of the following is an overlapping debt?

Sewer district debt An "overlapping" debt is the debt of another issuer, that is the ultimate responsibility of property taxpayer.

An investor purchases stock on Monday, September 15. The settlement date on the purchase is:

Thursday, September 18 Explanation: The settlement date on a transaction is three business days following the trade date. Regulation T requires payment by customers for purchases in two business days following the settlement date, while the rules of the SRO require settlement between the buying and selling brokers in three business days from the trade date.

If your customer invests in a variable annuity and chooses to annuitize at age 65, which of the following statements are true?

She may choose to receive monthly payments for the rest of her life. The accumulation unit's value is used to calculate the total value of the account.

Which of the following positions would be profitable if the market is at $50?

Short 1 ABC Jan 50 straddle @ 5 Long 100 shares of ABC @ 50 / Short 1 ABC Jan 50 call @ 5

Which of the following would establish a covered put?

Short stock at 40, short put at 35 A covered put is created when a short stock is combined with a short put.

In a volatile market, which of the following option strategies carries the most risk?

Short straddle

A covered call writer can be described as being:

Short the call, and long the stock Explanation: When writing (or selling) the call, the investor is said to be short the call. A covered call writer will currently own the underlying securities, and hence be long the stock.

A client wants to purchase 10 RSR July 45 calls and 10 RSR July 45 puts. This transaction:

Should be executed on one order ticket Explanation: This type of option transaction is a long straddle. Advanced option strategies such as spreads and straddles should be executed on one order ticket. They do not need to be approved in advance by a registered options principal (ROP).

A customer requests that a broker-dealer sell stock that she owns and use the proceeds of the sale to purchase a different stock. In determining the amount of markup that he will charge, the broker-dealer:

Should only consider the amount of money involved in the sale to the customer Explanation: When the proceeds of the sale of one stock are used to purchase another stock from the same broker-dealer, the transaction is called a proceeds transaction. In determining the markup the broker-dealer will charge, industry rules state that the firm should consider only the amount of money involved in the sale to the customer.

Which of the following competitive bids on a new municipal issue is most likely to be awarded the bid?

Six percent coupon with premiums over par In a competitive bid bond sale, the winning bid is the one that provides the issuer with the lowest net interest cost. If the syndicate pays the issuer more than par for the bonds, the issuer is taking in more money than it must pay out at maturity. Therefore, its net interest cost is lower than the six percent coupon on the bonds.

In periods of easy money, when interest rates are declining, yield curves tend to:

Slope upward from the shorter to the longer maturities Explanation: In periods of easy money when interest rates are declining, yields on shorter maturities would be less than those of longer maturities. Yield curves tend to slope upward from the shorter to the longer maturities.

An investor whose portfolio consists of high-yield municipal bonds, equity securities, and futures and options MOST likely has an investment objective of:

Speculation Explanation: An investor whose portfolio consists of high-yield municipal bonds, equity securities, and futures and options most likely has an investment objective of speculation.

A customer of your broker-dealer is bullish on U.S. equity securities across a broad spectrum of industries. He would like to participate in an anticipated upward movement of an equity stock index. Which of the following investments would you recommend as being closely related to the movement of equities in general?

Standard & Poor's depository receipts (SPDRs)

The VIX option contract is based on the implied volatility of the:

Standard and Poor's 500 Index The VIX option is an up-to-the-minute market estimate of expected price volatility using real-time SPX (S&P 500 Index) option bid/ask quotes. The VIX calculates expected SPX price volatility over the next 30 days.

Which of the following issues will most likely have a mandatory sinking fund? a. Serial issues b. Balloon issues c. Term issues d. Convertible issues

Term issues Explanation: A term issue is one in which all the bonds mature in one specific year. To accumulate monies to help retire the bonds, the issuer will deposit monies (above the amount to pay interest) in a sinking fund. These monies will generally be used to retire some of the bonds prior to maturity. A serial issue is one in which a portion of the bond issue is paid off each year, versus all being paid in one specific year as with a term bond issue.

A client wants to make sure she does not pay more than $3,000 to execute a spread transaction. The RR should:

Submit one order for a net debit of $3,000 Explanation: Advanced option strategies such as spreads and straddles should be executed on one order ticket. In order to enable the investor to ensure the total cost, the order should be entered as a net debit, since the client does not want to pay more than $3,000. If both sides of the spread order are entered separately, market volatility may make it impossible to ensure the transaction will be executed at a net debit or credit. The client has the risk that the order will not be executed since it is possible that both sides of the transaction were not able to be filled at the net debit or credit.

To determine the winning bid on an net interest cost (NIC) basis, an issuer will do which of the following?

Subtract any premium from total interest cost Add any discount to total interest cost

A portfolio manager using index options is trying to reduce which of the following types of risks?

Systematic

If a new customer is preparing to buy his first home within the next year, and his investment objective is aggressive growth, which of the following investments would be most suitable for your customer's portfolio?

T-bills

Advertising includes:

TV, radio, newsprint, billboards, websites, internet bulletin boards

A bond swap is done for all of the following reasons, EXCEPT to: a. Increase the overall yield of the bond portfolio b. Increase the current income of a bond portfolio c. Establish a tax loss to offset income d. Take advantage of a large amount of accrued interest

Take advantage of a large amount of accrued interest Explanation: A bond swap occurs for all of the reasons given except to take advantage of accrued interest. The amount of accrued interest is not a factor in a municipal bond purchase or sale.

A legal opinion issued for a municipal bond covers which of the following?

Tax status of the municipal debt Constitutionality and legality of the municipal debt

A customer may contribute money to an IRA based on which of the following sources?

Taxable alimony Explanation: Contributions that are made to an IRA must be based on taxable compensation which includes salary or wages (part-time or full-time), bonuses, tips, commissions, net income from self-employment, and taxable alimony. IRA contributions may not be based on rental income from properties, funds received from annuity contracts, or funds received from dividends and interest from securities in a portfolio.

Which of the following securities is frequently offered with a 50-year maturity?

Tennessee Valley Authority bonds are the only government security available today with a maturity as long as 50 years

A bank or brokerage firm is applying to become a primary dealer in government securities. Which government body appoints the financial institution as a primary dealer? a. The Treasury Department b. The SEC c. FINRA d. The Federal Reserve Board

The Federal Reserve Board Explanation: The Federal Reserve Board appoints primary dealers in government securities.

Recently, the federal funds rate has been rising. This may indicate all of the following situations, EXCEPT: I. Rates for short-term loans have been increasing II. The Federal Reserve may be engaging in matched sales to absorb reserves from the banking system III. Banks will find it more expensive to obtain overnight loans to satisfy a minor deficit in their reserve accounts IV. The Federal Reserve is easing credit

The Federal Reserve is easing credit Explanation: The federal funds rate is the rate that one bank charges another bank for overnight borrowing. This borrowing is done when a bank is in need of reserves. If the fed funds rate is steadily rising, it indicates that the Federal Reserve is tightening credit. Therefore, banks may find difficulty in obtaining overnight loans to meet reserve requirements.

To whom may a registered representative appeal a finding by a Hearing Panel?

The National Adjudicatory Council Explanation: The first appeal of the finding of a Hearing Panel must be made within 25 days after service of a decision, to FINRA's National Adjudicatory Council (NAC). The NAC findings may then be appealed to the SEC, whose findings may be appealed to the federal court system.

A customer interested in a collateralized mortgage obligation (CMO) might look to which of the following for historical data or projections regarding mortgage prepayments?

The Public Securities Association (PSA) is the source of historical data for prepayment projections on CMOs.

A registered representative discovers that one of her customers is on the Office of Foreign Assets Control (OFAC) list. The RR or someone else at her firm must notify:

The Treasury Department Explanation: Firms are prohibited from transacting business with individuals and entities on the Office of Foreign Assets Control (OFAC) list. If a registered representative discovers that one of the owners or beneficiaries of an account is on the OFAC list (or if someone on the list tries to open an account with his firm), the RR or someone else from her firm should contact the U.S. Treasury Department immediately. The Financial Crimes Enforcement Network (FinCEN) and OFAC are part of the Treasury Department.

Trust Indenture Act of 1939

The Trust Indenture Act of 1939 is applicable to corporate debt issues that are nonexempt, are over $50 million to be issued within 12 months, are offered interstate, and have maturities of nine months or more.

A registered representative is preparing to leave her firm. Her clients will be assigned to another representative at the same firm. To accomplish this:

The account records must be amended to reflect the change Explanation: Account records must be amended whenever an internal transfer of an account is made. This change does not require approval of the customer, the completion of a new account form, or the notification of any regulatory authority.

A customer sells short 1,000 shares of DT at $60 a share on Monday, October 14 and deposits the Regulation T margin requirement. If on October 23 the stock is trading at $75 a share, which of the following statements is TRUE? I. The account will be closed by the broker-dealer II. The account will be adjusted on October 23 and no margin maintenance call will be issued III. The account will be adjusted on October 23 and a margin maintenance call will be issued IV. The account will be adjusted on October 24 and a margin maintenance call will be issued

The account will be adjusted on October 23 and a margin maintenance call will be issued Explanation: A short margin account is marked to the market once a day (daily) to make sure the account is above the maintenance requirement. The initial Regulation T margin requirement is 50% of $60,000, or $30,000. If the market value increases to $75 a share, the equity in the account will decline to $15,000. The current equity in the account is 20% of the short market value ($15,000 / $75,000), which is below the required 30% and, therefore, a margin maintenance call will be issued.

Several years ago, a client purchased 1,000 shares of RADAK common stock at $50 per share. Today, the stock is selling for $100 per share and the investor is nervous about the future for the market. An order is turned in to sell 10 RADAK 105 calls at a premium of 2 and buy 10 RADAK 95 puts at a premium of 2. This strategy is

The answer is a cashless collar. It is cashless is because the calls are sold for 2 and the puts bought for 2. That means no out-of-pocket cash.

An investor reading the newspaper sees that yesterday's effective federal funds rate was 3.47%. On the previous day, the rate was 3.41%. This information indicates:

The average rate charged on overnight loans throughout the country increased Explanation: The effective federal funds rate is the daily average rate that commercial banks charge throughout the country for overnight loans. It is influenced, but not set by, the Federal Reserve Board. An increase in the federal funds rate normally signifies that the Fed has taken money out of the banking system.

If your client's real estate limited partnership goes bankrupt, which of the following are paid before your client?

The bank that holds the mortgage on the property The bank that holds the unsecured loans on the property

A bond was issued 3 years ago with a coupon of 6%. The bond matures in 21 years and is callable at 103. Current market interest rates are 8%. Which of the following is most likely true?

The bond is selling at a discount (when rates rise, price declines)

A corporation has an outstanding issue of 8% convertible debentures with a conversion price of $25. The bond indenture contains an anti-dilutive clause guaranteeing the debt holders the right to maintain proportionate equity conversion in the corporation. If the company pays a 10% stock dividend to its common shareholders, how will that affect the debenture holders?

The bonds will now be convertible at approximately 22.73 (1000/ $25= 40) (40 x 10%= 4, 40+4= 44) (1000/ $44= 22.73)

If a customer enters an order that is good for one month only, who is responsible for cancelling the order at the end of the month if the order is not executed?

The brokerage firm that entered the order Explanation: A customer can enter an order good for a week, a month, or any specified time. If the order is not executed by the end of the specified time, the brokerage firm is responsible for cancelling the order.

Which statement is TRUE regarding a variable annuity offering a GMIB?

The contract guarantees a minimum growth rate for the separate account at the time of annuitization

Regulation NMS modernized the U.S. markets for trading equity securities and prohibited: a. Trading exchange-listed securities over-the-counter b. A broker-dealer from selling a security to a customer from its own inventory c. The execution of a buy order at one market center at a price above the lowest ask price in another market center d. Trading ADRs on U.S. exchanges

The execution of a buy order at one market center at a price above the lowest ask price in another market center Explanation: Regulation NMS modernized the U.S. markets for trading equity securities. Among other rules, Regulation NMS prohibits a trade-through of a protected quote. A protected quote is the highest bid and lowest offer (the inside market) in a market center that allows electronic executions. A trade-through occurs when there is an execution of a buy order at a price above the lowest ask price, or an execution of a sell order below the highest bid. For example, assume the same security is quoted on two market centers (NYSE and Nasdaq). If the NYSE has an ask price of $23.50 and Nasdaq has an ask price $23.60, it is a violation to execute an electronic buy order at $23.60 when there is a better or lower price of $23.50. The other choices are not prohibited. If a broker-dealer sells a security to a customer from its own inventory, it is acting as a principal. Trading exchange-listed securities over-the-counter is allowed and is referred to as a third-market trade. ADRs (American Depositary Receipts) may be listed and traded on U.S. exchanges, such as the New York Stock Exchange or Nasdaq. ADRs may also be quoted on the OTCBB or in the Pink Market and may trade over-the-counter.

An issuer making a tender offer for its non-convertible bonds and later increases the price being offered by 10%. What is TRUE?

The increase in the tender price increases the life of the offer by another 5 business days

A broker-dealer owns 100 shares of ABCO stock, which it purchased at 28. If the stock is sold to a customer, the broker-dealer will base a markup on:

The lowest offer on the Nasdaq system Explanation: When selling stock to a customer, a markup should be based on the lowest offer on the Nasdaq system, not the price the dealer paid to purchase the stock (dealer's inventory cost).

One of the computations in a margin account is that of the SMA. Which of the following actions would cause the SMA to increase?

The market value of short securities decreasing

Which of the following statements regarding limited partnerships are true?

The maximum commission in selling partnership offerings is 10%. Commissions taken are not deducted from the original investment to determine beginning cost basis.

In an UTMA account, the earnings are reported based on the Social Security number of:

The minor Explanation: In both UTMA and UGMA accounts, the earnings are reported based on the Social Security or tax ID number of the minor. This number must be disclosed at the time of the opening of the account.

An investor purchases a U.S. Treasury bond in the secondary market. When is settlement?

The next business day Explanation: Transactions for Treasury securities in the secondary market settle on the next business day.

A brokerage firm's research department has issued a buy recommendation for XYZ Corporation's common stock. The report need not contain which of the following information? a. The firm was the managing underwriter in a recent public offering of the stock b. The number of shares of the stock the firm owns c. The partners of the firm who hold options to purchase the stock d. The firm makes a market trading in the stock

The number of shares of the stock the firm owns Explanation: The report must contain all of the items listed except the number of shares of the stock the firm owns. The firm does need to disclose that it owns shares of the stock, but not the actual number.

Which statement is FALSE about a simple IRA?

The plan is available to any size employer **SIMPLE IRAs are only available to small businesses with 100 or fewer employees**

One of your customers has maintained a traditional IRA for the past 15 years. Some of her annual contributions were not tax deductible due to her income level and participation in another qualified plan. At age 60, the customer elects to make a lump-sum withdrawal. Which of the following statements is true?

The portion representing principal from the nondeductible contributions is tax free, while the balance is taxable as ordinary income.

A municipal bond dealer purchased $100,000 of municipal revenue bonds at a 3% yield less 3/8ths. The dealer marks the bond up by ½ point and reoffers them to his customers. The compensation to the firm on each bond is

The price paid for the bonds is irrelevant. The important fact is that the dealer put on a markup of ½ point. That is $5 per bond.

Which of the following descriptions best defines a tax swap? a. The purchase and sale of bonds to incur commissions b. The exchange of convertible bonds for stock to avoid the receipt of taxable interest income c. The purchase and sale of bonds to realize a capital loss to offset against a capital gain for tax purposes d. Exercising the exchange privilege of a mutual fund

The purchase and sale of bonds to realize a capital loss to offset against a capital gain for tax purposes Explanation: A tax swap is the sale and purchase of bonds (or other securities) to realize a capital loss that can be offset against a capital gain.

Because of its multiplier effect on the economy, the Federal Reserve Board is reluctant to change:

The reserve requirement Explanation: Changing bank reserve requirements has a multiplier effect. This means that a small change in the reserve requirement can have a large effect on the money supply and the economy. This makes the results of changing the reserve requirement difficult to control, and the FRB is hesitant to use this tool.

Which of the following statements regarding the suitability of municipal bonds are true?

The tax-free interest payments make them more suitable for those in higher tax brackets. The tax-free interest is why municipal bonds are not considered suitable investments to be included in one's retirement account, such as an IRA.

The penny stock rules would apply under which of the following circumstances? a. The stock is listed on the NYSE b. The broker-dealer is not a market maker in the stock c. The transaction is recommended by the broker-dealer d. The customer is an active trader in penny stocks

The transaction is recommended by the broker-dealer Explanation: A penny stock, according to SEC rules, is a stock that sells for less than $5.00 that is not listed on Nasdaq or the NYSE. A stock quoted on the OTC Bulletin Board or OTC Pink Market (Pink Sheets) that has a bid price of less than $5.00 is defined as a penny stock. Penny stock rules would not apply under the following conditions. - The customer is defined as an existing customer, which is a person who has maintained an account with a broker-dealer for more than one year, or has previously engaged in 3 or more transactions involving penny stocks (i.e., an active trader of penny stocks) - In nonrecommended or unsolicited transactions - In transactions by a broker-dealer that is not a market maker in that security - In transactions by an institutional accredited investor

On February 10, an investor sold 100 shares of ABC short at $50/share. The investor covers the position on November 1 by purchasing 100 shares of ABC at $58/share, establishing an 8-point loss. If, on November 15, the investor shorts 100 shares of ABC at $56/share:

The wash sale rule has been violated Explanation: Reinstating a position within 30 days of realizing a loss is a violation of the wash sale rule. The November 15 short sale creates a new short position in ABC only 15 days after establishing a loss on an original short position in ABC. Therefore, the loss may not be claimed at this time.

A fidelity bond insures against what?

Theft by employees

When comparing an Albany, New York hospital revenue bond to a Buffalo, New York hospital revenue bond, you notice that they have similar maturities but the Buffalo bond has a higher yield. A possible reason for this is:

There are more hospitals located in Buffalo than in Albany Explanation: Competing hospitals could affect the project's revenue and, therefore, could reduce the bond's security. Each of the other choices relates to taxes, which do not secure revenue bonds.

A bond counsel will issue an unqualified legal opinion for a municipal bond issue to state that:

There are no limitations or pending lawsuits that hinder the issuance of the bonds Explanation: A bond counsel renders an unqualified legal opinion if there are no situations in existence that could adversely affect the legality of the issue.

Which of the following descriptions characterizes leveraged exchange-traded funds (ETFs)?

They are designed to deliver a multiple of the performance of an index or other benchmark Explanation: A leveraged ETF is designed to deliver a multiple of the performance of an index or other benchmark. For example, a 3X leveraged ETF based on the DJIA seeks to deliver three times the performance of that index. So, if the DJIA rises or falls by 1%, a leveraged ETF would increase or decrease by 3% before fees and expenses. Choice (a) is a regular ETF, choice (c) is an inverse ETF which seeks to deliver the opposite of what it is tracking, and choice (d) is a leveraged inverse ETF.

Which of the following statements regarding Ginnie Maes are true?

They are quoted in 1/32nds. They are traded with an accrued interest computed on a 30/360 basis.

T-bills purchased at the weekly auction will have a settlement date on the:

Thursday following the auction Explanation: The auction for 13- and 26-week T-bills is held each Monday. Settlement is on Thursday of the same week.

A municipal bond backed by an insurance company has gone into default. The insurance carrier will provide:

Timely payment of principal and interest Explanation: Municipal bond insurance guarantees the timely payment of principal and interest. If a municipal bond has 10 years to maturity, the insurance company is obligated to make 20 interest payments as they come due and a lump sum at maturity.

If a member firm wishes to have a clearly erroneous trade reviewed, it must notify

To have a clearly erroneous trade reviewed, a member must notify Nasdaq Market Operations within 30 minutes of the trade. The NAC is where appeals of disciplinary hearings are made.

If an investor had cash and securities in his account, why would the investor write call options against the securities?

To increase the overall rate of return of the portfolio Explanation: The purpose of writing calls against securities owned is to increase the overall rate of return of the portfolio. The premium the purchaser of the call pays the writer will be added to whatever dividends the writer was receiving to increase the yield of the portfolio to the writer. If the stock declines in value, the writer will make the premium on the expiring call. However, the investor is still exposed to large downside risk in the stock. Therefore, generating income for the portfolio is a better choice than to hedge.

If a customer is in a low federal income tax bracket and his main investment objective is current income, which of the following securities should the agent recommend?

To maximize income, the best recommendation of the choices listed is the corporate bond, which offers a higher yield than a government bond with a similar maturity.

Regarding a company's financial statements, total assets are equal to:

Total Liabilities + Stockholders' Equity

The largest portion of the underwriting spread in a new municipal securities issue is the:

Total takedown Explanation: The largest portion of the underwriting spread in a municipal securities issue is the total takedown, which is made up of the additional takedown plus the concession.

Treasury receipts

Treasury receipts pay interest at maturity. Treasury securities held in trust collateralize the receipts. Treasury receipts are not backed by the faith and credit of the U.S. government.

A customer purchases a municipal security in the secondary market at a discount. At maturity the customer will:

Treat the discount as ordinary income Explanation: If a municipal bond is purchased at a discount in the secondary market and held to maturity, there will be reportable taxable income. The discount is taxed as ordinary income, not a capital gain. The investor may pay the tax each year or elect to report the entire amount at maturity. If a municipal bond is purchased at an original issue discount and held to maturity, there will be no federal tax liability.

When looking at a newspaper listing for foreign currency options, the spot prices for the underlying foreign currencies are quoted in:

U.S. terms Explanation: For foreign currency options, spot prices are quoted in U.S. terms (the cost in U.S. dollars to purchase one unit of the foreign currency). All of the spot prices are quoted in cents per unit except the Japanese yen (1/100th cent per unit).

What type of account allows for the irrevocable transfer of almost any kind of asset, including works of art and real estate, for the benefit of a minor?

UTMA

Under what circumstances may a husband and wife both be custodians in one minor's account?

Under no circumstances Explanation: There may be only one custodian for a minor's account. Either the husband or the wife may be the custodian. However, they may not both be custodians for the minor's account.

Which of the following would not be found in a municipal revenue bond resolution?

Underwriting agreement

The potential loss when writing uncovered straddles is:

Unlimited Explanation: A straddle involves the sale (writing) of a call and put with the same expiration and exercise price. Writing an uncovered call involves unlimited loss potential.

An investor purchasing a corporate bond regular-way will need to pay the market price of the bond plus accrued interest. Accrued interest will be calculated:

Up to but not including the settlement date Explanation: Accrued interest on corporate bonds is calculated from the last interest payment date up to but not including the settlement date (which is the day delivery is due). A regular-way delivery on a corporate bond is three business days.

A middle-aged widow customer has an investment objective of stable income and wants minimal market risk and liquidity risk. What type of preferred stock would be the best recommendation?

Variable rate preferred Variable rate preferred has a dividend rate that is tied to the market rate of interest, and the dividend rate varies as the rate varies. When market interest rates rise, the dividend rate rises, and vice versa. All preferred stock has minimal liquidity risk. Preferred shares are listed and trade, so the shares can be sold readily at a low cost.

Donald has just received a document describing his employer's 401(k) plan. If Donald wants to know when he will own his employer's contribution to his plan, he will look in the section of the document that discusses:

Vesting Explanation: A plan's vesting rules describe when employees will own their employer's contributions into the plan. Employees immediately own (are 100% vested in) their own contributions to the plan.

Which of the following statements about warrants is NOT true? A) Warrants may not be traded in the secondary market. B) Warrants have longer lifetimes than rights. C) Warrants have an exercise price above the current market price of the common stock when issued. D) Warrants may be attached to another of the issuer's securities. Your answer, Warrants have an exercise price above the current market price of the common stock when issued., was incorrect. The correct answer was: Warrants may not be traded in the secondary market.

Warrants usually have lifetimes of 2-10 years; rights expire in 30-45 days. A corporation may attach warrants to other securities, such as bonds, to make the bonds more marketable. Warrants have no intrinsic value when issued and may expire without ever having intrinsic value. Before expiration, they may be, and often are, traded in the secondary market. Reference: 1.7.2 in the License Exam Manual.

The purchase of a new issue prior to settlement with the issuer can BEST be described as a:

When-issued transaction Explanation: The term when-issued covers the period of a new issue of municipal securities from the original date of sale by the issuer to the delivery of securities to the underwriter. The purchase or sale of new issue securities prior to registration may be a violation of the 1933 Act.

A variable annuity has an AIR of 4%. This past year, the separate account grew at a rate of 12%. The appreciation in the separate account:

Will increase an annuitant's monthly payment from the annuity Explanation: If the separate account of a variable annuity grows at a greater rate than the AIR, monthly payments from the annuity will increase.

A customer opens a margin account and signs the basic customer agreement, which consists of a credit agreement, loan consent agreement, and hypothecation agreement. If the customer's first order is to buy 100 shares of XYZ stock at a price of 36, the customer:

Will pledge stock in order to receive a loan to buy the stock Explanation: Securities in a margin account are always held at the brokerage firm in street name to allow the firm to liquidate shares if necessary. Under the hypothecation agreement, the customer pledges securities as collateral for the loan. The loan consent agreement permits the firm to lend the securities to other customers or other broker-dealers. The credit agreement establishes the customer's responsibility to pay interest on the debit balance. Since the initial trade was for $3,600, industry rules require that the customer deposit at least $2,000 and may borrow up to $1,600.

The term opening sale applies to a:

Writer of an option Explanation: The term opening sale applies to the seller (writer) of a listed option. This designation must be written on the order ticket.

A company in Japan will be importing California wines. The company must pay in U.S. dollars and is, therefore, concerned that the U.S. dollar will appreciate in value. To provide protection in the event that the U.S. dollar does appreciate, the company can buy:

Yen puts Explanation: If the U.S. dollar appreciates, the value of the yen declines. Therefore, the company should buy puts on the yen. The company cannot buy U.S. dollar calls since there are no options on the U.S. dollar (trading on an options exchange in the United States). If the expectation is that the U.S. dollar will decline, the company could buy yen calls.

The City of Podunk has an outstanding 25-year maturity issue that is callable in seven years. It has prerefunded the issue and established an escrow account containing the proper government securities with face amounts and maturities approximating the call provisions of the original issue. In quoting the original issue, which of the following must be used?

Yield to call When a bond issue is prerefunded, the issuer is going to redeem the bond on the first call date. The yield must be quoted to call.

Someone who wants to hedge a portfolio of preferred stocks will buy:

Yield-based call options Explanation: The prices of preferred stocks are inversely related to the movement of interest rates, as are bonds. If the investor is concerned that rising interest rates will erode the value of the preferred stock portfolio, the purchase of an option that does well when interest rates rise will provide an effective hedge. Yield-based call options increase in value when interest rates rise, creating a viable hedge.

An investor buys 2 RST 40 calls and pays a premium of 4 each. He also buys 2 RST 40 puts and pays a premium of 2.50 each. When purchased, RST is trading at $40.75. On the expiration date, RST is trading at $32.50 and the investor closes his positions for intrinsic value. Excluding commission, the investor realizes a: A) $100 loss. B) $200 loss. C) $200 profit. D) $100 profit.

Your answer, $100 loss., was incorrect. The correct answer was: $200 profit. The cost of opening these two straddles is $1,300. On the expiration date, the puts are worth $750 each, for a total of $1,500, giving the investor a $200 profit. The calls will expire worthless. Alternatively, the breakeven points for this long straddle are 33.50 and 46.50 (add the combined premiums of 6.50 to the call strike and subtract combined premiums from the put strike). The investor profits in a long straddle when the stock moves outside the breakeven points. As the stock is at 32.50, the customer makes 1 point (33.50 − 32.50) on each straddle, resulting in a $200 profit. Reference: 4.4.2.1 in the License Exam Manual.

A customer owns 10M of 7% U.S. Treasury bonds. He is in the 28% federal tax bracket and the 10% state tax bracket. What is his annual tax liability on these bonds? A) $196. B) $266. C) $70. D) $98.

Your answer, $196., was correct!. His tax liability is as follows: $1,000 × 7% = $70 annual interest per bond; $70 × 10 = $700 annual interest, which is taxable only by the federal government; $700 × 28% = a $196 tax liability. Reference: 15.5.4.1.1 in the License Exam Manual.

Under SEC rules, a penny stock is defined as an unlisted, non-Nasdaq security trading at less than: A) $2 per share. B) $2.50 per share. C) $5 per share. D) $1 per share.

Your answer, $5 per share., was correct!. SEC rules define penny stocks as unlisted, non-Nasdaq stocks of less than $5 per share. Reference: 16.1.9 in the License Exam Manual.

ABC Corporation, whose common stock is trading at $32, has issued $40 million of 8-1/8% debentures due 10-1-14. Each bond issued has a warrant attached enabling the holder to buy 4 shares of ABC common at $40 per share. If all of the warrants are exercised, ABC Corporation will receive: A) $12.8 million. B) $6.4 million. C) $10 million. D) $20 million.

Your answer, $6.4 million., was correct!. There are a total of 40,000 warrants outstanding ($40 million of debentures / $1,000 par value per bond). Each warrant entitles the holder to buy 4 shares of common stock. Therefore, if all warrants are exercised, holders will be purchasing 160,000 shares (4 × 40,000) at $40 per share. 160,000 × $40 = $6.4 million. Reference: 1.7.2 in the License Exam Manual.

If the strike price of a yield-based option is 62.50, this represents a yield of: A) 0.0625. B) 0.00625. C) 0.625. D) 0.000625.

Your answer, 0.00625., was incorrect. The correct answer was: 0.0625. To calculate the percentage yield of the underlying Treasury security, divide the strike price by 10 (62.50 / 10 = 6.25%). Reference: 4.5.2 in the License Exam Manual.

Which of the following is a third-market trade? A) 12,000 shares of XYZ, a stock listed on the New York Stock Exchange, are sold over the counter. B) 10,000 shares of XYZ, a stock listed on the New York Stock Exchange, are sold on the Chicago Stock Exchange floor. C) 10,000 shares of LMNO, a stock listed on Nasdaq, are traded between two financial institutions via an electronic communications network (ECN). D) 12,000 shares of PQ, a stock listed on the Philadelphia Stock Exchange are sold on the Chicago Stock Exchange floor.

Your answer, 10,000 shares of LMNO, a stock listed on Nasdaq, are traded between two financial institutions via an electronic communications network (ECN)., was incorrect. The correct answer was: 12,000 shares of XYZ, a stock listed on the New York Stock Exchange, are sold over the counter. A third-market trade occurs when exchange-listed securities are traded over the counter. Reference: 8.2.1.3 in the License Exam Manual.

The S&P 100 index closed on August 10 at 536.04. "The Wall Street Journal" quotes the closing premium for the OEX September 510 call at 28.90. The time value of the contract is: A) 18.9. B) 26.04. C) 2.86. D) 7.14.

Your answer, 2.86., was correct!. To find time value, subtract the intrinsic value from the premium. The intrinsic value is the in-the-money amount. Calls are in the money if the market price exceeds the strike price. In this case, the intrinsic value of the 510 call is 26.04. If the premium is 28.90, the time value is 2.86. Reference: 4.1.6.2 in the License Exam Manual

Your client writes 2 ABC November 220 calls at 5, and buys 200 shares of ABC common stock at $220 in his margin account. What is the breakeven point for each covered call position? A) 210. B) 230. C) 225. D) 215.

Your answer, 230., was incorrect. The correct answer was: 215. The breakeven point for covered call writing is the cost of stock purchased less the premium (220 − 5). Reference: 4.3.2 in the License Exam Manual.

A margin account customer buys 100 shares of HEX at $70 and writes a HEX Oct 70 call for a premium of 8. What must he deposit? (Regulation T is 50%.) A) 3700. B) 4500. C) 2700. D) 2000.

Your answer, 2700., was correct!. The normal call would be 50% of $7,000 or $3,500. In this example, subtract the premium of $800 that the customer received. (Remember, in a covered call situation, no margin is required for the call.) Reference: 6.1.3.1 in the License Exam Manual.

The regular way ex-dividend date for cash dividends is the: A) 2nd business day following the record date. B) 3rd business day preceding the record date. C) 2nd business day preceding the settlement date . D) 2nd business day preceding the record date.

Your answer, 2nd business day following the record date., was incorrect. The correct answer was: 2nd business day preceding the record date. The regular way ex-dividend date is 2 business days before the record date. Reference: 1.6.3.2.2 in the License Exam Manual.

A customer writes 1 ABC July 45 put at 3 when ABC is trading at 46. Maximum potential loss is: A) unlimited. B) 4200. C) 300. D) 4300.

Your answer, 4200., was correct!. Writers of puts are bullish. If the stock falls below the strike price, the customer will be exercised and forced to buy stock at 45. If the stock becomes worthless, the customer will lose $4,500. However, the customer received $300 for writing the option. Maximum loss equals the breakeven point (SP minus premiums) multiplied by 100 shares. Reference: 4.2.4.5 in the License Exam Manual.

If a customer writes 10 DEF Aug 50 calls at 1 when DEF is trading at 44, what is the maximum gain? A) 500. B) Unlimited. C) 100. D) 1000.

Your answer, 500., was incorrect. The correct answer was: 1000. When writing options, the maximum gain is equal to the premium received. Because there are 10 calls with a premium of $100 each, the maximum gain is 10 multiplied by $100, or $1,000. Reference: 4.2.2.5 in the License Exam Manual.

If an investor buys 1 KLP Oct 95 put at 6.50, what is the investor's maximum potential gain? A) 8850. B) 9500. C) 9650. D) 10150.

Your answer, 8850., was correct!. The maximum gain on a long put is calculated by subtracting the premium from the strike price (95 − 6.50 = 88.50 per share). One contract represents 100 shares, so the buyer's maximum gain is $8,850 if the stock declines to 0. Because put buyers are bearish, they will make money if the stock falls below the breakeven point of 88.50. Reference: 4.2.3.4 in the License Exam Manual.

A corporate insider may profitably sell the stock of his company, without penalty, after the stock has been held for more than: A) 3 months. B) 9 months. C) 12 months. D) 6 months.

Your answer, 9 months., was incorrect. The correct answer was: 6 months. If corporate insiders sell their stock at a profit, they must do so after having held the stock for at least 6 months. This is termed the short swing profit rule. If they sell at a profit without having met the holding period requirement, any profit earned must be disgorged to the company. Reference: 7.6.2.4 in the License Exam Manual.

Letters of intent may be backdated up to how many days? A) 120. B) 60. C) 90. D) 30.

Your answer, 90., was correct!. The time limit for a letter of intent is 13 months, but the letter may be backdated up to 90 days from the date it was filed. In that case, the investor has 10 months to complete the letter. Reference: 10.7.5.1.1 in the License Exam Manual.

Relative to a corporate bond, what begins on the dated date? A) The computation of principal payment. B) The date of all mathematical computations. C) Accrual of interest. D) Settlement and delivery.

Your answer, Accrual of interest., was correct!. Interest begins accruing on the dated date on new bond issues. Reference: 2.7.1 in the License Exam Manual.

Which of the following would NOT be considered institutional communications with the public? A) A communication with an individual designated to act on behalf of your institutional customer B) A letter to a municipality offering your firm's services as an underwriter C) A letter to another broker/dealer D) An internal memo promoting a new product that will be offered to your firm's institutional customers only

Your answer, An internal memo promoting a new product that will be offered to your firm's institutional customers only, was correct!. Institution communications specifically exclude internal communications. Communications with another member firm, a government entity, such as a municipality or with someone designated to act on behalf of one of your firm's institutional customers, would all fall within the definition of institutional communications. Reference: 17.5 in the License Exam Manual.

Which of the following investors would be exempt from filing form 144 when selling securities they own? A) An investor selling shares acquired in a Regulation D private placement. B) An employee of the company selling registered shares. C) An affiliated person selling unregistered shares. D) An employee of the company selling unregistered shares.

Your answer, An investor selling shares acquired in a Regulation D private placement., was incorrect. The correct answer was: An employee of the company selling registered shares. Rule 144 regulates the sale of control or restricted securities. Securities bought in a registered public offering are not restricted and therefore an employee of the company selling registered shares need not file form 144. Unregistered shares or securities purchased in a private placement are restricted and Rule 144 would apply. Reference: 7.6.2.4 in the License Exam Manual.

A customer buys AC Growth Fund and enjoys a substantial paper capital gain. When he believes the market has reached its peak, he switches into AC Income Fund within the AC family of funds. He incurs a small service fee but is not charged an additional sales charge. What is the tax effect? A) Any gain or loss is deferred until he liquidates the AC Income Fund. B) Any gain in AC Growth Fund is taxable because the exchange is treated as a sale and a purchase. C) It is a tax-free exchange. D) The tax basis of AC Income Fund is adjusted to reflect the gain in AC Growth Fund.

Your answer, Any gain in AC Growth Fund is taxable because the exchange is treated as a sale and a purchase., was correct!. The exchange is treated as a sale of the growth fund shares followed by a purchase of the income fund shares. The gain or loss is determined by comparing the cost basis of the growth fund shares with the net asset value at the time of exchange. Any difference is a capital gain or loss, even though the proceeds were immediately used to purchase the income fund. Reference: 10.7.5.4.1 in the License Exam Manual.

What is the latest date that an IRA participant may make an IRA deposit for the current year? A) April 15 of the current year. B) April 15 of the following year. C) July 15 of the following year, if extensions have been filed. D) December 31 of the current year.

Your answer, April 15 of the following year., was correct!. Contributions to IRAs can be made up to April 15 of the year following the year for which the contribution is being made. Reference: 11.2 in the License Exam Manual.

An investor purchased 10 GO bonds at a discount of 2 points per bond. The bonds mature in 10 years. After holding the bonds for 5 years, they were sold at par. For tax purposes, the investor has

a $100 gain cost per bond is 980. accretion is 20. ($20 / 10yrs= $2 per year) ($2 per year x 5 years= $10 per bond accretion) this makes the adjusted cost 990. $10 per bond x 10 bonds = $100 gain

As the result of a conversation with an officer of a publicly traded company, a registered representative comes into possession of material, nonpublic information indicating a high probability that the company's stock will increase substantially in value. If the following morning the registered representative buys call options on the stock, which of the following statements is TRUE? A) Both the officer and the registered representative violated insider trading rules. B) Neither the officer nor the registered representative violated insider trading rules. C) The officer violated insider trading rules. D) The registered representative violated insider trading rules.

Your answer, Both the officer and the registered representative violated insider trading rules., was correct!. A violation occurs if insider information is used to trade for profit or to avoid a loss. In such cases, both the tipper and the tippee are liable. Reference: 16.1.8.2 in the License Exam Manual.

A technical analyst has been charting XYZ stock and notes that it fluctuates between $36 and $41. If the analyst expects a breakout through resistance, which of the following orders should be placed? A) Buy XYZ 35 GTC. B) Buy XYZ 42 Stop GTC. C) Buy XYZ 35 Stop GTC. D) Buy XYZ 42 GTC.

Your answer, Buy XYZ 42 Stop GTC., was correct!. A buy stop order is placed above a resistance level. It is triggered if and when the stock trades at or above the stop price. This allows an investor to participate in a bullish breakout through resistance. Reference: 8.4.2.4.1 in the License Exam Manual.

Which of the following would be the least appropriate investment in a traditional IRA for a 67-year-old client? A) Variable annuities. B) Treasury notes. C) Common stock. D) Corporate bonds.

Your answer, Common stock., was incorrect. The correct answer was: Variable annuities. Why buy a tax-deferred product in a tax-deferred account? A variable annuity will provide no additional tax savings and will likely increase the expense of the IRA. In addition to sales and surrender charges, variable annuities may impose other charges such as mortality and expense risk charges, administrative fees, etc. In less than 4 years, your client will have to begin making withdrawals regardless of any surrender charges the annuity may impose. Reference: 11.2.1.1 in the License Exam Manual.

The diversification and professional management offered by many investment companies tends to lower the investor's risk. That does not mean elimination of risk. Of the following, it is likely the greatest risk would be investing in

a 2x ETF.

Which of the following underwriting arrangements is associated with an invitation, typically found in The Bond Buyer, directed at investment bankers and broker dealers, intended to solicit interest in underwriting a new municipal issue? A) Negotiated. B) All or none. C) Competitive bid. D) Best efforts.

Your answer, Competitive bid., was correct!. With a competitive bid underwriting a municipality publishes invitations to bid in The Bond Buyer or other municipal bond publication. Investment bankers and broker/dealers interested in underwriting the new municipal issue would respond to the invitation to bid. Reference: 3.2.2 in the License Exam Manual.

Which of the following statements regarding Coverdell ESAs is TRUE? A) Contributions are tax deductible, and distributions for any reason are tax free. B) Contributions are tax deductible, and distributions are always taxable. C) Contributions are not tax deductible, and distributions for any reason are tax free. D) Contributions are not tax deductible, and distributions are tax free when used for qualified educational expenses.

Your answer, Contributions are not tax deductible, and distributions are tax free when used for qualified educational expenses., was correct!. Coverdell ESAs offer after-tax contributions of up to $2,000 per student per year for children under age 18. Distributions are tax free as long as the funds are used for education. Reference: 11.2.5.1 in the License Exam Manual.

The investment policy department of an investment firm forecasts that the current business cycle should reach its peak within the next 2 months. Under such circumstances, which of the following portfolio adjustments would be most suitable for the firm's customers who actively invest in common stocks? A) Corporate bonds. B) Defensive stocks. C) Cyclical stocks. D) Aggressive growth stocks.

Your answer, Defensive stocks., was correct!. Defensive stocks such as those in the food, pharmaceuticals, and energy industries would most likely be suitable for investors who actively manage their equity portfolios. Defensive stocks are least likely to be affected by a reversal in the business cycle. Reference: 14.5.1.1 in the License Exam Manual.

Your client, age 62, single and just retired with no mortgage, currently owns some growth stocks and AAA rated corporate bonds. He would like to diversify in a way that might add to his current income in order to supplement his pension plan distributions. He tells you he is financially comfortable and willing to accept moderate risk. Of the following choices, which would be the most suitable recommendation for this individual? A) Corporate bonds with non-investment grade ratings and higher yields B) Selling naked call options to generate income C) New property direct participation real estate program (DPP) D) Equity income fund

Your answer, Equity income fund, was correct!. Of these choices, an equity income fund would be the most appropriate. These funds seek current income through dividend-paying stocks and generally have a secondary objective of moderate growth. None of the remaining choices would align with the client's moderate risk criteria. Reference: 15.2.2.2 in the License Exam Manual.

Which of the following affects the holding period of XYZ stock, a position that has been held for 6 months? Buying an in-the-money put Buy an out-of-the-money put Writing an in-the-money call Writing an out-of-the-money call. A) III and IV. B) II and III. C) I and IV. D) I and II.

Your answer, I and II., was correct!. Buying a put (in or out-of-the-money) on a stock held short term (one year or less) stops the holding period until the put is disposed of. Reference: 4.7.1 in the License Exam Manual.

An investment company registered under the Investment Company Act of 1940, whose specific purpose is to aid in the promotion and development of small businesses, is

a business development company.

A customer opens a margin account by purchasing 300 shares of XYZ at $60 and deposits the required margin. The stock rises to $70 on the following day. On the third day, after the release of a disappointing earnings report, the stock falls to $50. Which of the following describes the account after the changes in market value? A) Equity of $9,000; SMA of $0. B) Equity of $6,000; SMA of $1,500. C) Equity of $12,000; SMA of $1,500. D) Equity of $6,000; SMA of $0.

Your answer, Equity of $6,000; SMA of $1,500., was correct!. The account starts out as follows: $18,000 − $9,000 = $9,000 (LMV − DB = EQ). After the stock rises to 70, the account looks like this: $21,000 − $9,000 = $12,000; SMA = $1,500. For every $1 increase in market value, 50 cents of SMA is created. After the stock falls to 50, the account looks like this: $15,000 − $9,000 = $6,000; SMA = $1,500. An increase in market value creates SMA but a subsequent decline has no effect. Reference: 6.3 in the License Exam Manual.

Your customer redeemed 200 of her 500 Kapco common shares without designating which shares were redeemed. Which of the following methods does the IRS use to determine which shares she redeemed? A) Wash sale rules. B) FIFO. C) LIFO. D) Identified shares.

Your answer, FIFO., was correct!. When a customer does not choose a method, the IRS uses FIFO (first in, first out). This will likely result in shares with the lowest cost basis being redeemed first, which creates a greater taxable gain. Reference: 10.8.1.5.1 in the License Exam Manual.

If the holder of a call tenders an exercise notice after the ex-dividend date for a cash dividend, which of the following statements is TRUE? A) He is not entitled to the dividend. B) He is entitled to the dividend. C) He must pay the dividend to the writer. D) He is entitled to the dividend only if he sells the underlying stock.

Your answer, He is entitled to the dividend., was incorrect. The correct answer was: He is not entitled to the dividend. If the holder of a call exercises before the ex-date, the trade settles on or before the record date and he is on record for the dividend. If the holder exercises on or after the ex-date, the trade settles after the record date and he is neither on record for the dividend nor entitled to it. Reference: 4.6.1.3 in the License Exam Manual.

Which of the following would make an employee ineligible to participate in a company's qualified retirement plan? A) He is not a member of the company's management team. B) He has been with the company for only 2 years. C) He is only 20 years old. D) He works only 1,200 hours a year for the company.

Your answer, He is only 20 years old., was correct!. Under the Employee Retirement Income Security Act, anyone over the age of 21, management or not, who has been with the company for at least 1 year, and who works 1,000 or more hours per year for the company, must be allowed to participate in the company's qualified plan. Reference: 11.6 in the License Exam Manual.

Which of the following govern(s) the sale of a publicly offered direct participation program? FINRA Securities Act of 1933. Blue-sky laws. A) I and II. B) I, II and III. C) I only. D) II only.

Your answer, I, II and III., was correct!. The sale of a publicly registered DPP, like any other newly issued nonexempt security, is governed by the Securities Act of 1933, FINRA, and any applicable blue-sky laws. Reference: 13.1.1.2 in the License Exam Manual.

An individual is employed as a research analyst for a member firm that specializes in investment banking and has just completed a research report comparing two companies in the semiconductor business. Which of the following would be considered prohibited activities by this analyst, under FINRA rules? Purchasing shares of a semiconductor company before that issuer's IPO. Trading in these two stocks or their derivatives in a manner inconsistent with that analyst's recommendation. Purchasing shares of either of these two stocks for a personal account after the research report has been issued. Purchasing shares of the XYZ Semiconductor Fund, a fund qualifying as a diversified management investment company under the Investment Company Act of 1940 but not covered or analyzed in the research report. A) I and IV. B) II and III. C) III and IV. D) I and II.

Your answer, I and II., was correct!. FINRA rules restrict personal trading by research analysts. They are never permitted to acquire shares in advance of an IPO in a company in the same type of business that the analysts research. They are never permitted to engage in trading contrary to their opinions, as published in their firm's research reports. Once the report has been issued, they may trade in accordance with their recommendations. Purchasing shares of a mutual fund, even one that specializes in their field of research, is permitted. Reference: 17.5.5.1.4 in the License Exam Manual.

Which of the following are required to be given to retail customers at settlement in municipal new issue transactions? Confirmation showing the purchase price. Official statement. Names of syndicate members with their participation amounts. Copy of the agreement among underwriters. A) I and III. B) II and IV. C) III and IV. D) I and II.

Your answer, I and II., was correct!. MSRB rules state that a confirmation and an official statement must be sent to the investor no later than at settlement. Reference: 3.2.8.1 in the License Exam Manual.

The letter of intent in a corporate underwriting is typically signed by which of the following parties? Issuer. Managing underwriter. Syndicate members. Selling group members. A) I and II. B) I and III. C) III and IV. D) II and IV.

Your answer, I and II., was correct!. The letter of intent initiates the underwriting process and is signed by the issuer and managing underwriter. Reference: 7.5.1 in the License Exam Manual.

Which of the following are TRUE of the GO "Bond" Index? It includes only GO bonds. It includes both GO bonds and revenue bonds. It is computed weekly. It is computed monthly. A) II and III. B) II and IV. C) I and III. D) I and IV.

Your answer, I and III., was correct!. The Bond Buyer Index measures secondary market yields of GO bonds. It consists of 20 GO bonds, A-rated or better, each with approximately 20 years to maturity. The index is updated each week. Reference: 3.2.3.1 in the License Exam Manual.

Which items would change if a company buys equipment for cash? The working capital. The total assets. The total liabilities. The shareholders' equity. A) II and IV. B) I only. C) IV only. D) I and II.

Your answer, I and II., was incorrect. The correct answer was: I only. The general balance sheet formula is assets = liabilities + shareholders' equity. A purchase of equipment for cash would affect working capital by reducing current assets. However, it would not affect total assets since it is an exchange of one asset (cash) for another asset of equal value (equipment). Since no loan was needed, it does not affect total liabilities, nor does it affect equity. Reference: 14.6.1.3.5 in the License Exam Manual.

An investor's cost basis in a real estate program could be increased by: cash contributions. contributions of property. nonrecourse financing. distributions of cash to the partner. A) I and II. B) I, II and III. C) I and III. D) I, II, III and IV.

Your answer, I and II., was incorrect. The correct answer was: I, II and III. An investor's original cost basis will be increased by cash contributions made by the limited partner, property, fully paid securities and in the case of real estate programs, nonrecourse financing. A distribution of cash to a limited partner will decrease the investor's cost basis.

An investor's cost basis in a real estate program could be increased by: cash contributions. contributions of property. nonrecourse financing. distributions of cash to the partner. A) I and II. B) I, II and III. C) I and III. D) I, II, III and IV.

Your answer, I and II., was incorrect. The correct answer was: I, II and III. An investor's original cost basis will be increased by cash contributions made by the limited partner, property, fully paid securities and in the case of real estate programs, nonrecourse financing. A distribution of cash to a limited partner will decrease the investor's cost basis. Note: This may be an error. Recourse financing increases an investor's cost basis, not non-recourse loans.

An investor who has purchased a nonqualified variable annuity has the right to: vote on proposed changes in investment policy. approve changes in the plan portfolio. vote for the investment adviser. withdraw funds without any tax consequences. A) II and IV. B) I and IV. C) I and III. D) II and III.

Your answer, I and III., was correct!. Owners of variable annuities, like owners of mutual fund shares, may vote on changes in investment policy and for an investment adviser. Withdrawals from a nonqualified variable annuity are made on a LIFO basis, so the taxable earnings are considered taken out before principal. Reference: 12.1.4 in the License Exam Manual.

If a customer with an unrealized gain on a short stock position wishes to protect her profit, she should enter

a buy stop order. A buy stop order can be placed above the current market to protect the short stock position. If the stock trades at or above the stop price, the order is elected and becomes a market order to buy the stock, which will be used to cover the short position.

Which of the following statements regarding qualified retirement plans are TRUE? Contributions are made with pretax dollars. Contributions are made with after-tax dollars. Distributions are 100% taxable. Distributions are taxable only to the extent of earnings. A) I and IV. B) II and IV. C) II and III. D) I and III.

Your answer, I and III., was correct!. With qualified plans, participants receive a tax deduction for contributions to their plan. As earnings accumulate tax-deferred, distributions, which consist of tax-deferred earnings and contributions for which the participant received a tax deduction, are 100% taxable. Reference: 11.1.1 in the License Exam Manual.

U.S. government securities that are deposited with a trustee against which certificates are sold representing principal payments only on the securities are: clipped bonds. stripped bonds. subject to annual taxation on the per year accreted amount. subject to taxation at maturity. A) II and IV. B) I and IV. C) I and III. D) II and III.

Your answer, I and III., was incorrect. The correct answer was: II and III. U.S. government securities that are deposited with a trustee and against which certificates are sold representing principal payments only on the securities are referred to as Treasury STRIPS. These are zero-coupon bonds issued by the U.S. government and are subject to annual taxation on the per-year accreted amount. Reference: 2.6.1.4.1 in the License Exam Manual.

Which of the following statements regarding red herrings are TRUE? They may be used to obtain indications of interest. They may be sent out with sales literature. They contain the final offering price. Their use ends when the offering becomes effective. A) II and III. B) I and IV. C) I and III. D) II and IV.

Your answer, I and IV., was correct!. A preliminary prospectus, or red herring, is used only during the cooling-off period. The red herring does not contain the final price; offerings are priced immediately before the effective date. Reference: 7.2.2.1 in the License Exam Manual.

A variable annuity's separate account is: used for the investment of funds paid by contract holders. used to escrow late or otherwise delinquent premium payments. required to be located off of the company's premises. regulated under both securities and insurance laws. A) I and III. B) II and III. C) I and IV. D) II and IV.

Your answer, I and IV., was correct!. The separate account is used for both variable life insurance and variable annuity investments. The nature of the securities invested in-bonds and growth stocks-makes it necessary that sales representatives and their principals be licensed in securities as well as insurance. Reference: 12.1.2.1.1 in the License Exam Manual.

Which of the following orders are NOT placed on the order display book? Buy stop limit. Buy stop. Market. Not held. A) II and III. B) I and IV. C) III and IV. D) I and II.

Your answer, I and IV., was incorrect. The correct answer was: III and IV. Market orders are executed immediately and are not placed on the order book. Not held orders are not presented to the order book. Reference: 8.4.2.5 in the License Exam Manual.

Which of the following are acceptable forms of collateral in a margin account? Listed bonds. Listed stocks. Listed put or call options. A) I and II. B) I and III. C) II and III. D) I, II and III.

Your answer, I, II and III., was incorrect. The correct answer was: I and II. Put and call options have no loan value, and therefore are not acceptable as collateral in a margin account. Reference: 6.1.3.1 in the License Exam Manual.

The Securities Exchange Act of 1934: created the SEC. regulates trading in the secondary market. prohibits fraud in the distribution of new issues. A) I only. B) I and II. C) II and III. D) I, II and III.

Your answer, I, II and III., was incorrect. The correct answer was: I and II. The Securities Exchange Act of 1934 regulates secondary market activity. It created the SEC, which oversees all trading activity. The Securities Act of 1933 prohibits fraud in the distribution of new issues, whereas the Act of 1934 prohibits fraud in the trading of securities. Reference: 7.1.1.2. in the License Exam Manual.

Under SEC rules, which of the following events require a broker/dealer to furnish a copy of the account record to a customer? The opening of a new account. Change of customer's name or address. Change of customer's investment objectives. Change in customer's employment or financial status. A) II and III B) I, II, III and IV C) I and II D) I only

Your answer, I, II, III and IV, was correct!. Any change in a customer's status that may impact the suitability of recommendations requires a broker/dealer to update customer account records. Reference: 5.1.1.3 in the License Exam Manual.

Policies and procedures regarding instructions received from customers by email should identify as red flags which of the following scenarios? An email received that can not be verified by the registered representative as having actually come from the customer An emailed request to liquidate certain account holdings and transfer the sales proceeds to a third party A request by email regarding a joint account with instructions to sell all securities and to forward a check made payable to only one party to the account An urgent request that funds be sent to an overseas bank account not previously known to be associated with the client A) I, II, III and IV B) II and IV C) I and IV D) I and III

Your answer, I, II, III and IV, was correct!. Each of the scenarios should raise a red flag regarding unusual instructions received by email or otherwise. All firms must have in place written supervisory policies and procedures for reviewing and monitoring the transmittal of funds and a method of verifying that instructions received by email came from the customer and not an unknown third party. Urgency that might be intended to circumvent or deter broker/dealer verification procedures should always be viewed with extreme caution. Reference: 5.1.2.4 in the License Exam Manual.

A zero-coupon callable revenue issue has been pre-refunded. Under the rules of the MSRB, which of the following are required to be on a customer's confirmation? Zero coupon. Callable provisions. Pre-refunded. Yield. A) II, III and IV. B) I and IV. C) I, II, III and IV. D) II and III.

Your answer, I, II, III and IV., was correct!. On a customer's confirmation, it is necessary to include the zero-coupon rate, the yield, callable provisions, and whether or not it is pre-refunded, because all of these affect the price of a bond. Reference: 3.4.5.1.2 in the License Exam Manual.

Under SEC rules, a customer short sale on an exchange floor can be executed on which of the following? Plus tick. Zero-plus tick. Minus tick. Zero-minus tick. A) I and III. B) I, II, III and IV. C) I and II. D) II and IV.

Your answer, I, II, III and IV., was correct!. On an exchange floor, a customer short sale can be executed at any time in the trade sequence. Reference: 8.4.4 in the License Exam Manual.

A broker/dealer may charge customers for which of the following services? Safekeeping of customer's securities, exchange, or transfer. Collection of customer's interest or dividends. Forwarding of proxy material to the customer. Loans made to customers. A) I, II, III and IV. B) I, II and IV. C) I and II. D) II and IV.

Your answer, I, II, III and IV., was incorrect. The correct answer was: I, II and IV. The Conduct Rules permit FINRA members to charge reasonable fees to their customers for a variety of services. Included in chargeable services are the collection of interest and dividends, finding a buyer for a client who wishes to sell a relatively illiquid investment, and holding customers' securities in safekeeping. Members may also charge interest on loans made to customers. However, the expenses associated with sending proxy statements to customers whose securities are being held in trust by the member cannot be charged to the customer. Instead, the member may seek (and receive) reimbursement from the issuer for these expenses, since the issuer would have incurred these expenses had the customer/shareholder been holding the securities directly. Reference: 9.1.3.3 in the License Exam Manual.

You have 4 clients each expressing interest in a variable annuity contract. Which 2 of the 4 client profiles would a VA be least suitable for? A 45-year-old employed individual with no other retirement accounts in place A 58-year-old individual near retirement who is in good health and anticipates a lengthy retirement A 32-year-old with a company-sponsored 401k plan and will need a lump sum soon to finance graduate school tuition A 60-year-old individual, nearing retirement who has both IRAs and a 401k in place, is comfortable with market risk associated with the stock market, and has a lump sum in cash available to fund the annuity A) II and IV B) I and III C) II and III D) I and II

Your answer, II and III, was incorrect. The correct answer was: I and III VAs are less suitable for individuals who have not yet made maximum contributions to other retirement accounts such as IRAs and 401ks. They are also not considered suitable for anyone who anticipates needing a lump sum within a short time frame to fund other endeavors. They are more suitable for individuals who can fund the annuity with cash, want to supplement existing retirement benefits they have already funded, are comfortable with the market risk associated with a VA separate account portfolio and anticipate a long retirement. Reference: 12.3.4 in the License Exam Manual.

A company's changing from straight line to accelerated depreciation will: increase income in the early years. decrease income in the early years. increase income in the later years. decrease income in the later years. A) II and IV. B) I and III. C) I and IV. D) II and III.

Your answer, II and III., was correct!. Accelerated depreciation increases charged expenses during the early years of equipment life but decreases charged expenses during the later years. Reference: 14.6.1.4.2 in the License Exam Manual.

Twenty-five basis points on a par bond with 1 year to maturity are equal to: $.25 per $1,000. $2.50 per $1,000. 0.25%. 2.5%. A) I and IV. B) II and IV. C) I and III. D) II and III.

Your answer, II and III., was correct!. If 1 basis point equals .01%, 25 basis points equal .25%. .25% of $10 (which is the value of one full point for a bond) = $2.50. Reference: 2.1.5.1 in the License Exam Manual.

A retail customer purchases a municipal bond from your firm. According to MSRB rules, the confirmation must disclose which of the following? Where your firm acquired the bonds. Whether your firm acted as agent or principal. Your firm's address. The price your firm paid for the bonds. A) II and III. B) II and IV. C) I and IV. D) I and III.

Your answer, II and III., was correct!. The broker/dealer must always disclose the capacity in which it acted (principal or agent). The confirmation must show the name of the person for whom the trade was executed (the customer). The name, address, and telephone number of the broker/dealer must be shown so the customer may easily contact the firm. The settlement date is also required. The broker/dealer is not required to disclose where it acquired the bonds or the price it paid. Reference: 3.4.5.1.2 in the License Exam Manual.

Which of the following statements regarding a Rule 144 sale of restricted stock are TRUE? Stock sold through a 144 sale is considered registered stock after the sale. After holding the stock for 6 months, nonaffiliates may sell unrestricted. After holding the stock for 6 months, there are no volume restrictions for affiliates. Form 144 must be filed with the SEC at least 10 business days before a 144 sale made by an affiliate. A) II and III. B) I and III. C) III and IV. D) I and II.

Your answer, II and III., was incorrect. The correct answer was: I and II. Stock sold through a 144 sale is considered registered stock after the sale. When required to be filed by affiliates or insiders, Form 144 must be filed with the SEC on or before the date of sale. After holding the stock fully paid for 6 months, nonaffiliates may sell unrestricted but affiliates are subject to the volume restrictions of Rule 144. Reference: 7.6.2.4 in the License Exam Manual.

An investor purchases a municipal bond at par to yield 5.5% to maturity. Two years later, if he sells the bonds at a price equivalent to a 5% yield to maturity, the investor incurs

a capital loss, YTM dropped so the bond is trading at a higher price

Which of the following statements regarding UGMA accounts are NOT true? Only one custodian may be appointed for an UGMA account in the name of a minor. A gift under UGMA can be revoked. A custodian is liable for imprudence. Short sales are permitted in an UGMA account. A) II and IV. B) I and II. C) I and IV. D) II and III.

Your answer, II and IV., was correct!. An UGMA account is allowed only one custodian and one minor. Short selling is prohibited in UGMA accounts. Once a gift is given, it is irrevocable. The custodian has fiduciary responsibility toward the minor to invest conservatively, and the custodian is liable for imprudence in handling the entrusted funds. Reference: 5.3.1.5 in the License Exam Manual.

In a competitive bid, which of the following would the issuer need to determine net interest cost? Coupon rates. Basis. Dollar price. Spread. A) I and III. B) I and II. C) II and IV. D) III and IV.

Your answer, II and IV., was incorrect. The correct answer was: I and III. The coupon rate and dollar price are important to the issuer because they determine the actual cost of borrowing. The spread and basis at which the bonds will be resold are important to the underwriters, but not to the issuer.

Which of the following fundamental analysis theories might rely on lowering and raising taxes to stimulate or cool down an economy? A) Supply side. B) Breakout. C) Keynesian. D) Short interest.

Your answer, Keynesian., was correct!. Keynesian theory is interventionist. Supply side theory calls for low taxes and low government spending. Both short interest and breakout theories are technical rather than fundamental analysis theories. Reference: 14.1.2.1 in the License Exam Manual.

A couple in their early 30s has been married for 4 years, their disposable income is relatively high, and they are planning to buy a condominium. If they need a safe place to invest their down payment for about 6 months, which of the following mutual funds is the most suitable for these customers? A) XYZ Investment-Grade Bond Fund. B) LMN Cash Reserves Money Market Fund. C) ABC Growth & Income Fund. D) ATF Capital Appreciation Fund.

Your answer, LMN Cash Reserves Money Market Fund., was correct!. These customers are preparing to make a major purchase within the next few months, so they require a highly liquid investment to keep their money safe for a short amount of time. The money market fund best matches this objective. Reference: 15.2.1.2 in the License Exam Manual.

An option writer liquidates a position by purchasing an option. This order must be marked as

a closing purchase.

For the underwriting of a municipal bond issue, competitive bids are submitted by underwriters as

a firm commitment.

Which of the following order types is permitted in Nasdaq markets but NOT in NYSE equity markets? A) Fill or kill (FOK). B) Limit. C) Market. D) Immediate or cancel (IOC).

Your answer, Market., was incorrect. The correct answer was: Fill or kill (FOK). Fill-or-kill (FOK) and all-or-none (AON) orders may no longer be entered in the NYSE equity market but are still accepted in both the bond market and Nasdaq. Reference: 8.4.3.5 in the License Exam Manual.

In its notice of sale in the "Bond Buyer", an issuer states that it will take into consideration the timing of interest payments when evaluating bids. The issuer will be using which of the following methods in its bid selection? A) Net interest cost. B) Real interest cost. C) Low interest cost. D) True interest cost.

Your answer, Net interest cost., was incorrect. The correct answer was: True interest cost. The true interest cost method (TIC) takes into consideration the time value of money. The issuer discounts future interest payments to arrive at a present value. Reference: 3.2.5.1 in the License Exam Manual.

Performance of the terms of a standardized listed option contract are guaranteed by the: A) OCC. B) CBOT-CME. C) SEC. D) NYBOT.

Your answer, OCC., was correct!. The Options Clearing Corporation issues, guarantees, and handles the exercise and assignment of listed options. Reference: 4.6.3 in the License Exam Manual.

At which stage of the money-laundering process are illicit funds most susceptible to detection? A) Relocation. B) Placement. C) Layering. D) Integration.

Your answer, Placement., was correct!. Placement, the first step in laundering money, when no attempt is made to hide its origins, is recognized as the step during which the illegal funds are most vulnerable to detection. There is no step called relocation. Reference: 16.1.10.1.1 in the License Exam Manual.

Growth companies tend to have all of the following characteristics

a high earnings retention ratio. potential investment return from capital gains rather than income. low dividend payout ratios.

Which of the following statements regarding investment companies is TRUE? A) Pre-filing for investment company retail communications is never required. B) Pre-filing for investment company retail communications is always required. C) Retail communications for investment companies must be filed within 10 business days of first use if the communication does not include a performance ranking. D) Retail communications for investment companies must be pre-filed 10 business days before first use when the communication does not include a performance ranking.

Your answer, Pre-filing for investment company retail communications is always required., was incorrect. The correct answer was: Retail communications for investment companies must be filed within 10 business days of first use if the communication does not include a performance ranking. Retail communications for investment companies must be filed within 10 business days of first use if the communication does not include a performance ranking. For retail communications that do include a performance ranking, a 10 day pre-filing is required. Reference: 17.5.2.1 in the License Exam Manual.

John is the custodian of his niece's UGMA account. John places an order for 100 shares of a very speculative stock for the account. What should the representative do? A) Refuse to enter the order, because it is unsuitable for a UGMA account. B) Enter the order as requested, without comment. C) Advise the custodian as to the suitability of the stock. D) Consult with his principal before entering the order.

Your answer, Refuse to enter the order, because it is unsuitable for a UGMA account., was incorrect. The correct answer was: Advise the custodian as to the suitability of the stock. The custodian should not place unsuitable orders in a UGMA account. The representative should inform the custodian of this. Reference: 5.3.1.5 in the License Exam Manual.

Which of the following entities was created to protect investors who have money and/or securities on deposit at member firms? A) ERISA. B) CU.S.IP. C) AGA. D) SIPC.

Your answer, SIPC., was correct!. The Securities Investor Protection Act of 1970 created the Securities Investor Protection Corporation (SIPC) to protect securities investors against losses resulting from the financial failure of broker/dealers. SIPC does not provide protection against securities losses arising from decreases in market value. Reference: 16.1.6 in the License Exam Manual.

Which of the following statements regarding SMA balances is TRUE? A) SMA balances may be withdrawn provided the withdrawal does not bring the account below minimum maintenance. B) SMA balances may be withdrawn without restriction. C) SMA balances are free credit balances available upon demand. D) SMA balances may only be used to purchase additional securities.

Your answer, SMA balances may only be used to purchase additional securities., was incorrect. The correct answer was: SMA balances may be withdrawn provided the withdrawal does not bring the account below minimum maintenance. SMA is a line of credit that may always be withdrawn (even in a restricted account), provided the withdrawal does not bring the account below minimum maintenance.

A customer calls you and expresses interest in purchasing nonmarketable U.S. government debt. His primary concern is purchasing power risk. You should advise the customer to consider purchasing: A) Series HH bonds. B) any of these. C) Series I bonds. D) Series EE bonds.

Your answer, Series I bonds., was correct!. Series I bonds have been designed for investors seeking to protect the purchasing power of their investment. The interest rate on these bonds has two components: a fixed rate and an inflation rate tied to the Consumer Price Index. The inflation component is recalculated every six months. Series I bonds are an accrual-type security, meaning the interest is added to the bond's face value each month and is paid when the bond is sold or redeemed. If the customer were interested in buying marketable U.S. government debt, the customer would purchase Treasury Inflation Protection Securities (TIPS). Reference: 2.9.3 in the License Exam Manual.

Which of the following statements regarding revenue bonds issued by a state or municipality is TRUE? A) Interest and principal payment is guaranteed. . B) The bonds carry an unqualified promise to pay interest and principal backed by the power of the issuer to levy taxes. C) Interest and principal payment is backed by the full faith and credit of the issuer. D) Interest will be paid only if the enterprise owned and operated by the state or municipality has sufficient earnings to cover the interest payments or the debt service reserve.

Your answer, The bonds carry an unqualified promise to pay interest and principal backed by the power of the issuer to levy taxes., was incorrect. The correct answer was: Interest will be paid only if the enterprise owned and operated by the state or municipality has sufficient earnings to cover the interest payments or the debt service reserve. Because revenue bonds are not backed by the full faith and credit of the municipality that issues them, the earnings of the revenue-producing project must be large enough to cover the interest and principal payments. Reference: 3.1.2.2.2 in the License Exam Manual.

Your customer owns 100 shares of ABC Corporation being held in street name. What procedure will apply regarding your customer's proxy? A) The customer is required to sign one proxy card for his 100 shares. B) The brokerage firm must forward the proxy to your customer. C) The brokerage firm holding the shares must vote the proxy. D) ABC Corporation must send a proxy to your customer.

Your answer, The brokerage firm must forward the proxy to your customer., was correct!. The broker/dealer must forward the proxy to the beneficial owner. Reference: 9.1.3.3 in the License Exam Manual.

Which of the following is TRUE of principal protected notes? A) They are considered to be equity securities. B) They are unsecured debt obligations backed by the full faith and credit of their issuer. C) The investor is always guaranteed that the note is 100% principal protected and, therefore, they cannot, in any circumstance, lose their initial investment. D) They are comprised of 2 bonds linked together and sold as one.

Your answer, The investor is always guaranteed that the note is 100% principal protected and, therefore, they cannot, in any circumstance, lose their initial investment., was incorrect. The correct answer was: They are unsecured debt obligations backed by the full faith and credit of their issuer. A principal protected note (PPN) is a fixed-income security that promises a minimum return equal to the investor's initial investment if held to maturity. It is considered to be a structured product and is comprised of a bond and an option component. PPNs are unsecured debt obligations backed only by the full faith and credit of their issuer Reference: 2.12.1.1 in the License Exam Manual.

All of the following must be considered by an investment adviser representative before recommending a municipal security to a customer EXCEPT: A) Customer's state of residence. B) Customer's tax status. C) Municipal security's rating. D) The municipality's coverage ratio.

Your answer, The municipality's coverage ratio., was correct!. The coverage ratio is specific to revenue bonds only and tells how many times annual revenue from that issue will cover the debt service of the issue. It is not a factor of suitability to be considered when recommending a municipal bond but more of a factor to consider when comparing two municipal revenue bonds. The customer's state of residence and tax status are essential when determining suitability for a municipal security. The security's rating is also important because it measures the overall safety and quality of the bond. Reference: 3.4.3.2 in the License Exam Manual.

If a representative possesses material inside information about a publicly traded company, under which of the following circumstances may he communicate this information to a customer? A) If the customer enters an unsolicited order. B) If the information will be made public the next day. C) If the customer knows that it is inside information. D) Under no circumstances.

Your answer, Under no circumstances., was correct!. Inside information may never be divulged to a client. Violations are punishable with both civil and criminal penalties. Reference: 16.1.8.2 in the License Exam Manual.

In a variable life annuity with 10-year period certain, a contract holder receives

a minimum of 10 years of variable payments, followed by additional variable payments for life.

Crossover refunding, which is a type of advance refunding, is best described by which of the following statements? A) The revenue stream is halted completely from the project until the new bonds are issued. B) Revenues can never cross over to fund a new issue. C) The new issue will not be funded by the revenue stream from the project that funded the initial bond offering. D) The revenue stream originally pledged to secure the refunded issue continues to pay debt service on those bonds until they mature or are called.

Your answer, The revenue stream originally pledged to secure the refunded issue continues to pay debt service on those bonds until they mature or are called., was correct!. Crossover refunding is a method of advance refunding in which the revenue stream originally pledged to secure the refunded bonds continues to be used to pay debt service on those bonds until they mature or are called in by the issuer. Reference: 2.1.7.3.1 in the License Exam Manual.

Which of the following documents sets forth the priority of sale of securities? A) A tombstone. B) An offering circular. C) The official notice of sale. D) The syndicate letter.

Your answer, The syndicate letter., was correct!. The syndicate letter lists the terms under which members will conduct the sale of the bonds. It also describes each member's sharing of profits and expenses, the type of business entity (i.e., joint venture or partnership), and the good faith deposit required. Reference: 3.2.4 in the License Exam Manual.

A tombstone for a new bond issue announces that 5-year warrants to purchase shares of the company's common stock at $75 are attached to the bonds. The current market value of the company's stock is $45. For what reason were the warrants attached to the bonds by the issuer? A) To increase the dilution of the current shareholders. B) To improve the marketability of the bond issue. C) To make the bonds convertible into the issuer's common stock. D) To decrease the dilution of the current shareholders.

Your answer, To improve the marketability of the bond issue., was correct!. Warrants are often issued as a bonus (or sweetener) to entice investors to purchase new bond issues. Dilution may occur at the time the warrants are exercised (if ever), but this would not be a reason for their issuance. A warrant has nothing to do with the bond's convertibility into the underlying common stock. Reference: 1.7.2.1 in the License Exam Manual.

Which of the following is limited in the case of a limited tax municipal bond? A) Type of tax that can be used to service the debt. B) Number of bonds issued. C) Number of taxpayers. D) Number of buyers.

Your answer, Type of tax that can be used to service the debt., was correct!. A general obligation (GO) bond may be backed by a specific tax. For example, a limited tax GO may be serviced only from sales tax revenue, not income tax revenue. As the source of debt service is limited (it is not backed by the full taxing authority of the issuer), these bonds are sold with higher yields than conventional GOs. Reference: 3.1.2.1.2 in the License Exam Manual.

A customer purchases ten 8% Treasury notes at 101-16. What is the dollar amount of this purchase?

a quote of 101-16 means 101 plus 16/32. 101 plus 1/2 = $1,015; $1,015 × 10 bonds = $10,150.

A customer wishes to close a short option position. The order ticket must be marked as: A) a closing purchase. B) an opening sale. C) an opening purchase. D) a closing sale.

Your answer, a closing purchase., was correct!. The investor opened with a sale, so the position must close with a purchase. Reference: 4.6.3.4 in the License Exam Manual.

Nonmembers of a syndicate who are assisting in its sale of bonds buy the bonds at a discount called: A) the basis price. B) a concession. C) a takedown. D) a net designated price.

Your answer, a concession., was correct!. Members of the syndicate buy the bonds at the offering price minus the takedown, nonmembers buy at offering price minus a concession. The basis price is the yield to maturity. Reference: 3.2.6.3 in the License Exam Manual.

When a customer wishes to open an account as guardian, you would ask for: A) a copy of the court order and other guardianship papers. B) a full power of attorney. C) a signed customer account card, credit agreement, and loan consent. D) a limited power of attorney.

Your answer, a copy of the court order and other guardianship papers., was correct!. Guardianship is one of several account titles that require legal papers to open the account. Because a guardian is appointed by court order, the broker/dealer would need a copy of that court order. Reference: 5.2.1.4 in the License Exam Manual.

An investor has a large portfolio of Blue Chip common stocks and expects the market to remain stable or decline slightly. He'd like to increase the rate of return on his portfolio. Which of the following would be the best choice for this investor? [A] Buy Calls on the portfolio [B] Write Covered Calls on the portfolio [C] Establish a Call Spread [D] Write Covered Puts on the portfolio

[B] Write Covered Calls on the portfolio

A municipal bond underwriter looking in the bond buyer would recognize the percentage of new issues sold versus new issues offered for sale the prior week as the: A) visible supply. B) acceptance or placement ratio. C) GO index. D) revenue index.

Your answer, acceptance or placement ratio., was correct!. The placement ratio also known as the acceptance ratio is compiled weekly and reflects the municipal bonds sold divided by the municipal bonds offered in the previous week. Reference: 3.2.3.1 in the License Exam Manual.

A fund must inform its shareholders of their right to reinvest dividends at NAV: A) quarterly. B) only at the time of original purchase. C) annually. D) at the time of each distribution.

Your answer, annually., was correct!. A fund must notify shareholders of their right to reinvest at least annually (usually communicated through the annual report). Reference: 10.7.5.3 in the License Exam Manual.

A registered representative would recommend a customer establish a short straddle on T-bonds when interest rates are expected to: A) rise. B) be volatile. C) decline. D) remain unchanged.

Your answer, be volatile., was incorrect. The correct answer was: remain unchanged. Any straddle writer is always looking for a stable market. Volatility is the biggest enemy of the writer. Since this question is referring to debt options, their price movements are based upon changes in interest rates. No fluctuations in interest rates means no price changes. Reference: 4.4.2.2 in the License Exam Manual.

If a customer wishes to purchase a nonexempt security in a cash account, Regulation T requires a broker/dealer to receive payment in full: A) within 3 business days. B) within 10 business days. C) within 5 business days. D) before the purchase.

Your answer, before the purchase., was incorrect. The correct answer was: within 5 business days. Regulation T requires that a broker/dealer receive payment in full, from a customer making a purchase of this type in a cash account, no later than 5 business days after the trade date. Reference: 6.1.5 in the License Exam Manual.

The placement ratio, as shown in the "Bond Buyer", is: A) bonds issued/bonds unsold. B) bonds issued/bonds sold. C) bonds sold/bonds issued. D) bonds sold/bonds unsold.

Your answer, bonds issued/bonds sold., was incorrect. The correct answer was: bonds sold/bonds issued. The placement ratio is a measure of investor demand for new issue municipal bonds. It is computed by dividing the amount of bonds sold each week by the amount issued that week. Reference: 3.2.3.1 in the License Exam Manual.

If an American exporter will be paid 25 million Japanese yen when her goods arrive in 45 days, her best hedge is to: A) sell yen puts. B) sell yen calls. C) buy yen calls. D) buy yen puts.

Your answer, buy yen calls., was incorrect. The correct answer was: buy yen puts. The exporter does not want to see the value of the yen fall. If she owns yen puts and the yen does fall, her profit on the puts would help compensate for the decrease in the value of the yen. Selling yen calls would also provide protection if the yen fell in value, but only to the extent of the premium received. Exporters buy puts in order to hedge; importers buy calls on the foreign currency to hedge. Reference: 4.5.3.1.8 in the License Exam Manual.

In a repurchase agreement between a broker/dealer and a large institutional customer, the broker/dealer: A) sells securities to the customer with an agreement to buy them back. B) buys securities from the customer with an agreement to sell them back. C) sells securities to the customer who, with prior agreement, resells the securities to an unrelated third party. D) buys securities from the customer who, with prior agreement, repurchases the securities from an unrelated third party.

Your answer, buys securities from the customer with an agreement to sell them back., was incorrect. The correct answer was: sells securities to the customer with an agreement to buy them back. The dealer agrees to buy back the securities at a specified date at a higher price than they were sold for. The difference represents interest to the party who bought the securities (the lender). Reference: 2.10.2.1 in the License Exam Manual.

In a seller's option, securities may be delivered before the date specified if the seller A) gives notice to the buyer on the day of delivery . B) gives 1 day's written notice to the buyer. C) wishes to be paid earlier. D) cannot deliver on the specified date.

Your answer, cannot deliver on the specified date., was incorrect. The correct answer was: gives 1 day's written notice to the buyer. In a seller's option trade, the seller may (at his option) give the buyer written notice 1 day before making delivery. Reference: 9.1.2.3 in the License Exam Manual.

If a married couple with a long-term growth objective is considering a mutual fund and they are concerned about the fund's annual expenses, they should select a: A) preferred stock fund. B) common stock fund with a low portfolio turnover. C) common stock fund with a high portfolio turnover. D) long-term corporate bond fund.

Your answer, common stock fund with a low portfolio turnover., was correct!. Of the choices given, common stock is the only vehicle capable of providing long-term growth. Preferred stock will provide dividends, but it will not provide much growth as it trades like a bond in line with interest rate changes. Of the two common stock funds, the one with the lower portfolio turnover will have lower annual expenses. Reference: 10.5.1.1 in the License Exam Manual.

All of the following records must be kept for 6 years EXCEPT A) customer account records, like monthly statements B) the general ledger C) blotters D) communications with the public

Your answer, communications with the public, was correct!. Of the answer choices listed, it is important to recognize that communications with the public must be maintained on file for 3 years by a member firm, and therefore, would not fall under the 6-year record retention requirement. Reference: 17.5.3.1 in the License Exam Manual.

All of the following statements are true of the risks of investing in an oil and gas limited partnership EXCEPT: A) development programs have higher risk than exploratory programs. B) income programs have fewer tax benefits than exploratory programs. C) wells may not have sufficient reserves to return drilling costs. D) development programs may involve acquisition of expensive leases

Your answer, development programs have higher risk than exploratory programs., was correct!. Exploratory programs have the highest risks, rewards, and tax benefits. Development wells are drilled to develop a reserve that is already known to be present. Reference: 13.2.2.5 in the License Exam Manual.

Mutual fund shares represent an undivided interest in the fund, which means that: A) the fund can only hold securities of certain companies . B) the number of shares outstanding is limited to a predetermined maximum. C) investors can only purchase full shares. D) each investor owns a proportional part of every security in the portfolio.

Your answer, each investor owns a proportional part of every security in the portfolio., was correct!. Each mutual fund shareholder owns an undivided interest in the investment company's portfolio. Because each share represents one class of voting stock, the investor's interest in the fund is reflected by the number of shares owned. Reference: 10.5 in the License Exam Manual.

In a single day, a customer purchases 15 ACM Sep 50 puts at 6 and 15 ACM Sep 50 calls at 1. The customer would profit from the positions if ACM traded: A) between $43 and $57. B) either below $38 or above $52. C) either below $43 or above $57. D) between $38 and $52.

Your answer, either below $43 or above $57., was correct!. The customer paid $6 for the Sep 50 puts and $1 for the Sep 50 calls. A long straddle is profitable on the call side if the price rises above the strike price plus the combined premiums paid ($57). On the put side, it is profitable if the price falls below the strike price minus the combined premiums paid ($43). Reference: 4.4.2.1 in the License Exam Manual.

The interest that municipal securities pay is: A) not taxed at the state, local, or federal levels. B) exempt from both state and local taxation. C) fully taxed. D) federally tax exempt.

Your answer, federally tax exempt., was correct!. Interest paid on securities issued by municipalities is generally exempt from taxation at the federal level. It may also be exempt from state and local taxation if the purchaser resides in the issuing state. Reference: 3.1.1.1 in the License Exam Manual.

An ADR represents a: A) foreign security trading in both the U.S. and a foreign market. B) foreign security trading in the U.S. market. C) U.S. security trading in a foreign market. D) U.S. security trading in both the U.S. and a foreign market.

Your answer, foreign security trading in the U.S. market., was correct!. ADRs are receipts issued by U.S. banks that represent ownership of a foreign security and are traded in U.S. securities markets. Reference: 1.8.1 in the License Exam Manual.

Upon being informed that one party to a tenants in common account has died, a registered representative should: A) allow the surviving tenants to continue trading. B) transfer all of the assets to the surviving tenants. C) transfer half of the assets to the survivor. D) freeze the account.

Your answer, freeze the account., was correct!. The assets of a deceased tenant in a TIC account eventually go to his estate. A registered representative first freezes the account and then awaits the proper court documents. If the account was JTWROS, trading by the surviving tenants could continue. Reference: 5.2.1.2.1 in the License Exam Manual.

A customer buys 1 XYZ Aug 60 call at 4 and 1 XYZ Aug 60 put at 2 when XYZ is at 61.25. If the stock rises to 68 and the customer lets the put expire and closes out the call at intrinsic value, the result is a: A) gain of $200. B) loss of $200. C) gain of $600. D) loss of $600.

Your answer, gain of $200., was correct!. The customer has established a long straddle. To determine profit or loss, compute the breakeven points by both adding and subtracting the combined premiums (6 points) from strike (the breakeven points are 54 and 66). Because the customer profits if the stock moves outside these points, at 68, the customer has a 2-point ($200) gain. Reference: 4.4.2.1 in the License Exam Manual.

In a 3-for-2 stock split, an investor will: A) have 50% more shares at half the price. B) have two-thirds fewer shares at a 50% higher price. C) have 50% fewer shares at twice the price. D) have 50% more shares at two-thirds the price.

Your answer, have two-thirds fewer shares at a 50% higher price., was incorrect. The correct answer was: have 50% more shares at two-thirds the price. If a stock splits 3 for 2, an investor will receive an additional 50 shares for every 100 shares owned. The price will decline by one-third, but the total value of the position will stay the same. For example, if a shareholder owns 100 shares before the 3 for 2 split, the shareholder will have 150 shares after the split (3 / 2 × 100 = 150). Reference: 1.2.4.1 in the License Exam Manual.

A client of your broker/dealer is interested in collateralized mortgage obligations (CMOs). While determining suitability for the client all of the following should be discussed EXCEPT A) how currency exchange rates may affect the value of the securities B) the tax consequences of CMOs C) the relationship between mortgage loans and mortgage securities D) how changing interest rates may affect the prepayment rates

Your answer, how currency exchange rates may affect the value of the securities, was correct!. Currency exchange rates are not applicable to the risks associated with CMOs. However, when determining suitability, a discussion of all of the characteristics and risks of CMOs, should occur. This would include how changing interest rates may affect prepayment rates and therefore the average life of the security, tax considerations (CMOs are taxable at all levels), and the relationship between actual mortgage loans and mortgage-backed securities. Reference: 2.8.2 in the License Exam Manual.

All of the following statements regarding bonds with both a convertible and callable feature are correct EXCEPT: A) after the call redemption date, interest payments will cease. B) dilution of company stock will occur on conversion of the bonds. C) if called, the owners have the option of retaining the bonds and will continue to receive interest. D) the coupon rate on a convertible bond would be less than the rate for comparable nonconvertible debt.

Your answer, if called, the owners have the option of retaining the bonds and will continue to receive interest., was correct!. After bonds are called, the issuer no longer pays interest. Conversion of convertible bonds causes more shares outstanding, resulting in a reduced proportionate ownership interest (dilution) for current shareholders. The coupon rate paid on convertible bonds is lower than the coupon for nonconvertible bonds. There is a trade-off in the amount of interest for the ability to convert the bonds into common stock. Reference: 2.1.7.1.6 in the License Exam Manual.

As the underlying stock price increases, the premium of a call option generally: A) fluctuates. B) increases. C) remains the same. D) decreases.

Your answer, increases., was correct!. The premium of an option changes as the market price of the underlying security moves; therefore, if the stock price increases, the premium of a call also increases. Reference: 4.1.6.2 in the License Exam Manual.

All of the following appear on a corporation's balance sheet as fixed assets EXCEPT: A) real estate. B) computer equipment. C) inventory. D) furniture.

Your answer, inventory., was correct!. Inventory is considered a current asset, not a fixed asset, because the company expects to convert its inventory into cash within a short period of time. The other choices are fixed assets and cannot be liquidated easily. Reference: 14.6.1.2.1 in the License Exam Manual.

The Nasdaq stock market permits listing for all of the following EXCEPT: A) warrants. B) common stock. C) convertible bonds. D) nonconvertible debt securities.

Your answer, warrants., was incorrect. The correct answer was: nonconvertible debt securities. The Nasdaq stock market is an equity and equity equivalent market. Listed are common stock, preferred stock, warrants, limited partnerships, ADRs, and convertible bonds. Straight debt securities are not part of Nasdaq. Reference: 8.8.1 in the License Exam Manual.

All of the following statements regarding the over-the-counter market are true EXCEPT: A) it is an auction market. B) it trades listed securities. C) more issues trade OTC than on the exchanges. D) it trades unlisted securities.

Your answer, it trades unlisted securities., was incorrect. The correct answer was: it is an auction market. The OTC market is a negotiated market. The exchanges are auction markets. Reference: 8.8.1.1 in the License Exam Manual.

A member firm receives a signed proxy from a customer who failed to indicate how his shares held in street name are to be voted at the annual shareholder's meeting. Under NYSE rules, the member firm: A) cannot vote the shares. B) may vote the shares as it sees fit only if a principal attends the meeting. C) must vote the shares as recommended by management of the issuer. D) may vote the shares as it sees fit.

Your answer, may vote the shares as it sees fit., was incorrect. The correct answer was: must vote the shares as recommended by management of the issuer. If the beneficial owner of street name stock returns a signed proxy statement but fails to indicate how the shares are to be voted, the member must vote the shares as recommended by management of the issuer. Reference: 9.1.3.3 in the License Exam Manual.

With XYZ trading at $47.50, your customer writes 1 XYZ January 50 put and simultaneously writes 1 XYZ January 45 call receiving $600 in combined premiums. Your customer's market attitude is: A) bearish. B) bullish. C) speculative. D) neutral.

Your answer, neutral., was correct!. This position is a short combination where both contracts are in the money. Breakeven points are 51 and 44. Above or below these points, the customer will lose money. Reference: 4.4.2.3 in the License Exam Manual.

Stop orders may be used for each of the following EXCEPT: A) lock in a specific price to close out a position. B) protect profits on long positions. C) protect profits on short positions. D) establish positions.

Your answer, protect profits on long positions., was incorrect. The correct answer was: lock in a specific price to close out a position. Stop orders are contingent orders that are triggered when the stock trades at or through a stated price. When triggered, they become market orders to buy or sell. They are used by technical traders to establish positions above or below resistance and support levels, respectively. Stop orders never guarantee a specific execution price. Reference: 8.4.2.4 in the License Exam Manual.

Customer statements must be sent at least: A) semiannually. B) monthly. C) annually. D) quarterly.

Your answer, quarterly., was correct!. The SEC and FINRA require member firms to send customer account statements at least once per calendar quarter. Reference: 9.1.1.5 in the License Exam Manual.

A corporate offering of 200,000 additional shares to existing stockholders may be made through a: A) rights offering. B) secondary offering. C) warrant. D) tender offer.

Your answer, rights offering., was correct!. A rights offering is an offering of additional shares of stock to existing shareholders. Reference: 7.3.3 in the License Exam Manual.

Your customer owns shares of LMN stock that have gone up in value. He does not wish to sell the shares now because he does not want to realize the capital gain. To lock in the gain without selling those shares, he sells shares of LMN stock short, holding both the long and short positions simultaneously. You recognize this tax strategy as A) a wash sale B) commingling C) selling or shorting against the box D) advance or pre-refunding

Your answer, selling or shorting against the box, was correct!. Selling or shorting against the box is a tax strategy used to defer capital gains into the next tax year. Selling shares short of a company when you are already long effectively locks in any gain you have on the long position. For every dollar gained in the long position, you lose one in the short position, and vice versa. Ultimately, in the next tax year, the long shares are used to replace the borrowed shares for the short position, which effectively closes both positions, and any gain would then be taxable. The IRS mandates that certain other criteria be met to utilize this tax strategy. Reference: 15.5.9.4 in the License Exam Manual.

All of the following positions have limited loss potential EXCEPT: A) long stock/long put. B) long stock/short call. C) short stock/long call. D) short stock/short put.

Your answer, short stock/short put., was correct!. If the stock rises the put will expire leaving the customer short stock with an unlimited loss potential. Reference: 4.3.4 in the License Exam Manual.

Each of the following statements regarding OTC markets is true EXCEPT: A) the OTC market has no exchange trading floor. B) the bid is the highest price at which a dealer will buy. C) the OTC market is an auction market. D) securities traded OTC include ADRs and municipal bonds.

Your answer, the OTC market is an auction market., was correct!. The OTC market is a negotiated market in which market makers may bargain during a trade. Stock exchanges like the NYSE are an auction market. Reference: 8.8.1.1 in the License Exam Manual.

If an investor is long 5 Dec puts on the Canadian dollar, these options will expire in December on: A) the Friday preceding the third Wednesday. B) the Saturday after the third Friday. C) the Wednesday after the third Saturday. D) the third Friday of the month.

Your answer, the Saturday after the third Friday., was correct!. Currency options, like equity options, expire on the Saturday following the third Friday of the expiry month. Reference: 4.5.3.1.5 in the License Exam Manual.

All of the following are required by the MSRB on customer confirmations EXCEPT: A) the amount of markdown or markup on a principal transaction. B) the source of any commission received on an agency transaction. C) whether the bond is registered or in book-entry form. D) the amount of any commission received on an agency transaction.

Your answer, the amount of markdown or markup on a principal transaction., was correct!. The amount of markup or markdown must be fair and reasonable, but need not be disclosed. Reference: 3.4.5.1.2 in the License Exam Manual.

All of the following would be found in a bond resolution for a new municipal issue EXCEPT: A) covenants to which the issuer must adhere. B) a description of the issue. C) the issuer's obligations to bondholders. D) the costs to be incurred by the issuer in connection with the offering.

Your answer, the costs to be incurred by the issuer in connection with the offering., was correct!. The bond resolution (or the bond contract) spells out the characteristics of the issue (maturities, call features, etc.), the issuer's responsibilities to bondholders, and any restrictive covenants to which the issuer must adhere. Costs to be incurred by the issuer have no impact on bondholders. Reference: 3.1.3.3 in the License Exam Manual.

Yield quotes on CMOs are based on the: A) tranche's expected life. B) underlying mortgages' interest rate. C) underlying mortgages' average life. D) underlying mortgages' maturity.

Your answer, underlying mortgages' interest rate., was incorrect. The correct answer was: tranche's expected life. Yield quotes on CMOs are based on the tranche's expected life, not the average life of the mortgages in the pool backing all of the tranches. Reference: 2.8 in the License Exam Manual.

The latest issue of a newsletter your firm subscribes to is especially relevant to one of your firm's investment products. If you decide to send it to clients and prospects, you must disclose that: A) the newsletter's purpose is to provide your clients with a choice of products that are suitable for all of their portfolios. B) the newsletter discusses only those products which you have available through your firm . C) the newsletter is written and produced by a third party. D) future articles sent will provide similar discussions and information.

Your answer, the newsletter is written and produced by a third party., was correct!. If a third party is the creator of the newsletter, that fact must be disclosed together with the name of the third party and the date of publication. Reference: 17.5.3.1 in the License Exam Manual.

Revenue bond rate covenants require the user fees to be high enough to cover all of the following obligations of the issuing authority EXCEPT: A) the optional call provisions. B) the operations and maintenance. C) the debt service. D) the debt service reserve fund.

Your answer, the optional call provisions., was correct!. Optional call provisions are at the option of the issuer. Rate covenants of an issue will not require enough to be collected to cover a call on the bonds. Reference: 3.1.2.2.4 in the License Exam Manual.

All of the following are true regarding nonqualified deferred compensation plans EXCEPT: A) the plans need not be offered to all employees. B) income taxes on compensation are not due until constructive receipt. C) IRS approval is not needed for deferred compensation plans. D) employees may use accumulated funds as collateral for a bank loan

Your answer, the plans need not be offered to all employees., was incorrect. The correct answer was: employees may use accumulated funds as collateral for a bank loan. Deferred compensation is a promise made by an employer to defer a certain amount of an employee's salary upon retirement. The employee has no rights to the money until retirement, death, or disability, and thus cannot use it as collateral. Reference: 11.1.2.1 in the License Exam Manual.

The risk that time value may erode the premium of an equity option even while the underlying issuer remains financially sound is an example of: A) timing risk. B) interest rate risk. C) capital risk. D) currency risk.

Your answer, timing risk., was incorrect. The correct answer was: capital risk. Capital risk is generally associated with equity instruments, such as common stock, and equity-related derivatives, such as options. It is the risk that invested dollars can be lost as the result of circumstances unrelated to an issuer's financial strength. Reference: 15.3.2.2 in the License Exam Manual.

An outstanding municipal bond issue has the following characteristics: 7.50% coupon; maturity in 20 years; puttable in 5 years at 100; callable at 102 in 10 years; declining in a straight-line to maturity; yield-to-maturity is 6.50%. The issues should now be quoted: A) yield-to-call at 102. B) yield-to-maturity. C) yield-to-call at par. D) yield-to-put.

Your answer, yield-to-maturity., was incorrect. The correct answer was: yield-to-call at 102. Since the bond issue is selling at a premium, the yield-to-call is less than the yield-to-maturity. The bonds must be quoted as yield-to-call at the earliest maturity, which would be the 10-year call at 102. If the bonds were selling at a discount, yield-to-maturity would be the proper quote. Yield-to-put is not required to be quoted. Reference: 3.4.5.1 in the License Exam Manual.

You are a registered representative servicing a joint tenants with right of survivorship account for a married couple, Byron and Shelly Nelson. You also maintain an account for their adult son Frank. This morning you receive a phone call from Frank informing you that Byron has died. As part of helping his mother sort out the many details associated with his father's death, Frank requests that the assets in the account be transferred to an account in Shelly's name only. The first course of action is to inform Frank that:

Your firm must receive a death certificate before it may transfer the assets Explanation: The broker-dealer must receive a death certificate as proof of death before it may mark an account deceased, or change the account title in a joint account.

A broker/dealer under Reg T is not required to liquidate a customer's positions if the margin call in the account is for: [A] $1,000 or less [B] $2,000 or more [C] $5,000 [D] Any unpaid amount

[A] $1,000 or less

A client purchased 200 shares of ABC for $22,500 two years ago. Over the next few years the client sold call options for a total of $1,500. All the calls expired worthless. For tax purposes, the cost basis for the 200 shares of ABC is [A] $22,500 [B] $21,000 [C] $23,250 [D] $24,000

[A] $22,500

An investor buys 100 shares of ABC at $50 per share. Later, the investor sells 1 ABC Jul 55 call at 3. What is the investor's breakeven point as the contract nears expiration? [A] $47 [B] $50 [C] $53 [D] $55

[A] $47

An investor asks an RR for his opinion on the purchase of 500 shares of a very closely held stock. The investor thinks it has good near term investment potential. He should be advised that [A] A thin market for such a stock is a major risk factor. [B] Closely held companies have low growth potential. [C] He would have no voice in the company's affairs. [D] He should not buy stock in a closely held company

[A] A thin market for such a stock is a major risk factor.

An investor asks an RR for his opinion on the purchase of 500 shares of a very closely held stock. The investor thinks it has good near term investment potential. He should be advised that [A] A thin market for such a stock is a major risk factor. [B] Closely held companies have low growth potential. [C] He would have no voice in the company's affairs. [D] He should not buy stock in a closely held company.

[A] A thin market for such a stock is a major risk factor.

Which of the following would NOT be considered a "bullish" options strategy? [A] An investor establishes a short straddle. [B] An investor buys a call. [C] An investor establishes a call spread at a net debit. [D] An investor sells a put.

[A] An investor establishes a short straddle

When a client opens an options account at a firm, regulations dictate that the firm must receive an account agreement signed by the customer in what time frame? [A] Following approval of the account, the firm must receive the signed account agreement within 15 days. [B] The firm must receive the signed account agreement prior to approval of the account. [C] Following approval of the account, the firm must receive the signed account agreement within one month. [D] The firm must receive the signed account agreement no fewer than 10 business days before approval of the account.

[A] Following approval of the account, the firm must receive the signed account agreement within 15 days.

One of your clients wants to free up some capital and decides to sell a municipal bond out of their portfolio. The bond last paid interest a couple of months ago and will not pay interest again for a few months. In this situation, who pays the accrued interest at the time of settlement? [A] The buyer of the bond pays the accrued interest. [B] The seller of the bond pays the accrued interest. [C] The agent will pay the accrued interest. [D] The issuer will pay the accrued interest.

[A] The buyer of the bond pays the accrued interest.

Which of the following BEST describes what earnings coverage means when dealing with a revenue bond for a municipality? [A] The term describes available revenue that the issuer can use to pay debt service. [B] The term describes the funds set aside to replace or repair the facility. [C] The term describes the funds that an issuer sets aside in a sinking fund. [D] The term describes the amount of insurance carried to cover catastrophic events that may damage or destroy the facility.

[A] The term describes available revenue that the issuer can use to pay debt service.

Which of the following is TRUE in relation to securities being pledged as collateral for loans when the securities are pledged by a customer to the broker/dealer firm? [A] This type of pledge would be called hypothecation. [B] This type of pledge would be called re-hypothecation. [C] These securities would be considered "tendered" by the issuer. [D] These securities would be considered "treasury stock" by the issuer.

[A] This type of pledge would be called hypothecation.

Which of the following U.S. Government securities are classified as money market instruments? [A] Treasury bills [B] Treasury notes [C] Treasury bonds [D] Treasury strips

[A] Treasury bills

The negotiated underwriting of municipal securities usually involves all of the following EXCEPT: [A] a bid form [B] a legal opinion [C] an underwriters agreement [D] a bond purchase contract

[A] a bid form

If a long call option is exercised by the holder, the holding period on the acquired securities begins: [A] on the date the call is exercised [B] the day after the option is exercised [C] the day the call option was acquired [D] the day after the call option was acquired

[A] on the date the call is exercised

A municipality has come out with a new issue which includes a Call feature. The Call feature means that: [A] the issuer can redeem the bonds held by the investor. [B] the broker dealer can redeem the bonds held by the investor. [C] the investor can redeem the bonds back to the issuer. [D] the investor must hold the bonds until the final maturity date.

[A] the issuer can redeem the bonds held by the investor.

FINRA regulations apply to communications with the public and advertising by broker/dealer firms. Of the following communications, which would be labeled as an advertisement and would have to abide by advertising regulations? [A] A web log (blog) which has been set up for established clients of the firm and requires a password to view pages. [B] A listing in a local phone book which contains a description of the firm and various services provided by the firm. [C] A letter following a specific form which is sent to a limited group of potential clients which discusses the services provided by the firm. [D] A report prepared by the firm's research department which discusses various banks in the area and is only sent to existing clients of the firm.

[B] A listing in a local phone book which contains a description of the firm and various services provided by the firm.

Which of the following funds would be the least suitable for a retiree in need of a steady stream of income? [A] A bond fund [B] A sector fund [C] A balanced fund [D] A growth and income fund

[B] A sector fund

529 college savings plan accounts have all of the following features EXCEPT: [A] Beneficiaries of the 529 accounts can be changed by the owner. [B] Annual contributions to the account can total no more than $2,000. [C] The owner of the account maintains control over the account. [D] As long as withdrawals are qualified, federal income tax does not apply.

[B] Annual contributions to the account can total no more than $2,000.

Which of the following bids may be considered stabilizing bids? I.Bids at the public offering price II.Bids just above the public offering price III.Bids just below the public offering price [A] I only [B] I and III [C] III only [D] II and III

[B] I and III

Jimmy is a client of yours and he has some holdings in U.S. T-Bonds. Over the past year, he received $500 in interest from these bonds. His accountant has informed him that he is in the 28% tax bracket with all sources of income considered. He lives in the State of California, so his State Income Tax Rate is 9%. What kind of tax liability is Jimmy looking at in relation to the interest received from his U.S. T-Bonds? [A] Jimmy has a tax liability of $185.00 total on the interest received. [B] Jimmy has a tax liability of $140.00 total on the interest received. [C] Jimmy has a tax liability of $45.00 total on the interest received. [D] Jimmy has no tax liability on the interest received.

[B] Jimmy has a tax liability of $140.00 total on the interest received.

A client recently purchased a sizeable number of mutual fund shares and knows that the Net Asset Value will change daily. The customer asks the RR how the NAV number is computed. Which of the following best describes the computation to arrive at NAV per share? [A] NAV per share is calculated by finding the average value on a per share basis over the previous quarter. [B] NAV per share is calculated by subtracting the liabilities of the fund from the total assets of the fund, then dividing this number by the total number of shares currently outstanding. [C] NAV per share is calculated by taking the total value of the portfolio of the fund, adding sales charges received, and then dividing this figure by the total number of shares currently outstanding. [D] NAV per share is calculated by finding the total appreciation of the portfolio and dividing this figure by the total number of shares outstanding.

[B] NAV per share is calculated by subtracting the liabilities of the fund from the total assets of the fund, then dividing this number by the total number of shares currently outstanding.

A registered representative recommends that a client purchase shares of a mutual fund today rather than tomorrow because the fund shares will go ex-dividend tomorrow. The registered representative explains that if the client waits until tomorrow to purchase the shares, the client will not receive the dividend. Such a recommendation by the registered representative is: [A] Allowed, because shares purchased on or after ex-date do not receive dividends and are not in the client's best interest. [B] Prohibited, because such recommendation is selling dividends and is not in the client's best interest. [C] Not regulated because it would have no effect on the client's potential position. [D] Acceptable, provided it is first approved in writing by the registered representative's branch manager.

[B] Prohibited, because such recommendation is selling dividends and is not in the client's best interest.

An investor purchases the following bonds, all at a premium above par value: •XYZ 5% non-callable bonds maturing in 15 years •XYZ 5.10% non-callable bonds maturing in 20 years •XYZ 5.25% non-callable bonds maturing in 25 years Several months after these bonds are purchased, the going rates on bonds have moved an average of 20 basis points. Of this investor's purchases, which will show the largest adjustment in terms of price because of the change in the going rate? [A] The bonds are all trading in the secondary bond market and will all be affected equally. [B] The 25-year bonds will be affected the most. [C] The 15-year bonds will be affected the most. [D] The bonds are all trading in the secondary bond market and have fixed coupons, so their prices will not be affected by fluctuations in new bond rates.

[B] The 25-year bonds will be affected the most.

Which of the following statements is FALSE when discussing the Federal Funds Rate? [A] Changes to the supply of money and demand for money cause fluctuations in the federal funds rate. [B] The Federal Reserve directly sets the Federal Funds Rate. [C] Historically, the discount rate has been lower than the Federal Funds Rate. [D] The calculation of the Federal Funds Rate takes place daily.

[B] The Federal Reserve directly sets the Federal Funds Rate.

During a discussion with a client to determine investment objectives, a client indicates a strong desire to preserve capital. The client also would like to see some current income from their investments. Which portfolio recommendation would BEST suit this customer's stated investment objectives? [A] The RR recommends a portfolio comprised of an equal mix of junk bonds, highly-rated municipal bonds, and 30-year corporate bonds. [B] The RR recommends a portfolio comprised of money market funds, short-term debt instruments issued by the Treasury and federal agencies, and T-Bills. [C] The RR recommends a portfolio comprised of equity securities such as common and preferred stock, as well as a mutual fund focused on large-cap growth stocks. [D] The RR recommends a portfolio comprised of an international mutual fund, investments in real estate investment trusts, and individual securities issued by foreign corporations

[B] The RR recommends a portfolio comprised of money market funds, short-term debt instruments issued by the Treasury and federal agencies, and T-Bills.

One of your clients placed a trade in their cash account without the funds to pay for the securities. No additional funds were added to the account and no extension was obtained for additional time to deposit funds. The firm liquidates the purchased position. In order to comply with Reg T guidelines, the customer's account must be frozen for what amount of time? [A] The account must be frozen for 180 days. [B] The account must be frozen for 90 days. [C] The account must be frozen for 30 days. [D] The account need not be frozen.

[B] The account must be frozen for 90 days.

You are talking to a client who is making a few changes to their account information. The client recently retired. The client has a need for taxable monthly income from investments and would like to keep their investments conservative in nature. Which of the following investments would NOT be appropriate for this client? [A] The client could invest in the bonds of several conservative Blue Chip companies. [B] The client could invest in several leveraged, closed-end funds made up entirely of bonds. [C] The client could invest in the preferred stocks of several conservative Blue Chip companies. [D] The client could invest in CDs (Certificates of Deposit).

[B] The client could invest in several leveraged, closed-end funds made up entirely of bonds.

The factor listed below which would have a direct impact on the alpha of a particular stock would be: [A] Investors on a general scale have changed their minds from being pessimistic about the market to being optimistic. [B] The company whose stock is in question has made significant changes to the management strategy for the upcoming year. [C] With recent over-buying in the technology sector, the market has now corrected itself and technology stocks have come back to reasonable levels. [D] The index where the company is listed has been on a downward trend.

[B] The company whose stock is in question has made significant changes to the management strategy for the upcoming year.

The price of two funds are as follows at the end of the day of Tuesday, January 15th: NAV Offering Price Net Asset Change Fund X 10.20 10.95 -0.04 Fund Z 11.80 12.57 -0.08 One of your clients called and entered a redemption order for 250 shares of Fund X on that Tuesday morning. What would this investor receive? [A] The investor will receive $2,550, less any change in market value. [B] The investor will receive $2,550. [C] The investor will receive $2,540 less any change in market value. [D] The investor will receive $2,540

[B] The investor will receive $2,550.

An investor who holds a variable annuity decides to annuitize and start receiving monthly annuity payments. The client is currently 55 years of age and the variable annuity is non-qualified. What can this investor expect to pay in terms of tax liability? [A] The 10% penalty on early withdrawals will apply. [B] The monthly payments will be partially taxable on earnings in the annuity and will be partially tax-free with regards to principal that was invested in the variable annuity. [C] The portion of payments that was principal that was invested in the variable annuity will be fully taxable. [D] The full amount of payout each month will be fully taxable.

[B] The monthly payments will be partially taxable on earnings in the annuity and will be partially tax-free with regards to principal that was invested in the variable annuity.

Which of the following is measured by a mutual fund's expense ratio? [A] The mutual fund's profitability [B] The mutual fund's operating costs [C] The mutual fund's dividend payout percentage [D] The mutual fund's stability

[B] The mutual fund's operating costs

What is the typical make-up of a balance sheet? [A] The overall values of outstanding bonds plus outstanding stocks is equal to the total assets of the company. (Bonds + Stocks = Total Assets) [B] The total liabilities plus the company's net worth equals the total assets of the company. (Liabilities + Net Worth = Total Assets) [C] The capitalization less its liabilities set equal to the assets of the company. (Capitalization - Liabilities = Assets) [D] The current liabilities added to the fixed liabilities of the company set equal to the assets of the company. (Fixed Liabilities + Current Liabilities = Assets)

[B] The total liabilities plus the company's net worth equals the total assets of the company. (Liabilities + Net Worth = Total Assets)

A member of the board of directors of the Able Baker Charlie Company (ABCC) took her director's fees and purchased 200 shares of ABCC on the Nasdaq Stock Market at $20 per share. If she wished to sell these shares, compliance with Rule 144 would entail

meeting neither a size limit nor a time limit.

One of your clients refers a friend. After talking with the referred friend, she opens an account with you, deposits $20,000, and states that she wants you choose some stocks that will have good growth potential whenever you feel it is the right time to buy. The next step you should take would be to: [A] choose appropriate stocks, execute the orders, and report the transactions back to the new customer. [B] get written discretionary authorization from her prior to executing any transactions. [C] advise her that she needs to pick the stocks and enter the orders herself using the firm's online trading platform. [D] check with your original client to obtain his assessment of her risk tolerance prior to executing orders in the new client's account.

[B] get written discretionary authorization from her prior to executing any transactions.

When a bond is trading at a discount, the current yield on the bond is: [A] unrelated to its yield to maturity [B] less than its yield to maturity [C] equal to its yield to maturity [D] more than its yield to maturity

[B] less than its yield to maturity

A customer's account appears as follows: $100,000 MV Long $ 40,000 DB $ 60,000 EQ The securities in this customer's account INCREASE in value by $25,000, and the debit balance is decreased by $5,000. After these changes, what is the buying power in the account? [A] $27,500 [B] $35,000 [C] $55,000 [D] $90,000

[C] $55,000

Which of the following would be the MOST typical portfolio make up for a mutual fund that is categorized as "balanced"? [A] A portfolio consisting of bonds that are all investment grade [B] A portfolio consisting of municipal bonds with diversified ratings and maturities [C] A portfolio consisting of equity securities such as common and preferred, along with bonds from various issuers including government, municipal, and corporate [D] A portfolio consisting of large-cap "blue chip" equity securities

[C] A portfolio consisting of equity securities such as common and preferred, along with bonds from various issuers including government, municipal, and corporate

A municipal bond has a provision for redeeming a certain number of bonds at par in ten semi-annual installments. This type of call feature is best described as: [A] An installment call [B] An optional call [C] A sinking call [D] A catastrophic call

[C] A sinking call

All of the following statements about municipal auction-rate securities are TRUE EXCEPT: [A] At auction, a rate is determined which will be the lowest interest rate which would provide for the sale of all of the securities in the market. [B] The rates on these securities typically are reset at regular intervals such as 7 days, 28 days, 35 days, or another pre-determined interval. [C] All auction-rate securities are putable, meaning investors can force the selling dealer to buy them back at any time. [D] When selling auction-rate securities, the broker/dealer seeks buyers who are interested in holding the securities over the next period after the rate has been reset.

[C] All auction-rate securities are putable, meaning investors can force the selling dealer to buy them back at any time.

In terms of features that would limit the credit risk that would apply to a municipal bond issue, which of the following would do so most effectively? [A] Being backed by general obligation [B] A legal opinion that is un-qualified or non-qualified [C] An issue which is escrowed to maturity [D] A putable bond feature at par value at one year

[C] An issue which is escrowed to maturity

Which of the following is the MOST accurate description of "beta"? [A] Beta measures the returns that a company has against the stocks of individual companies both inside and outside of the same sector. [B] Beta measures the price-to-earnings ratio of one company in relation to the average price-to-earnings ratio of companies on a particular index. [C] Beta measures the volatility of an individual company in comparison to the volatility of the overall market. [D] Beta measures the volatility of one class of securities such as common stock against other classes of securities such as bonds, preferred stock, and derivatives.

[C] Beta measures the volatility of an individual company in comparison to the volatility of the overall market.

When a registered representative is trying to determine if a stock is a growth stock, public utility, or blue chip, they would most likely look at the: [A] Earnings per share [B] Current Ratio [C] Dividend Payout ratio [D] Advance/Decline ratio

[C] Dividend Payout ratio

REITs (Real Estate Investment Trusts) have which of the following features? I.Much like limited partnerships, REITs pass profits and losses on to their shareholders. II.90% of all investment income must be passed on to shareholders in order to ensure special tax treatment. III.Any profits passed on to shareholders of a REIT are not taxable until the REIT shares are sold. IV.Investment in other REITs is acceptable as a real-estate-related activity. [A] I and II [B] I and III [C] II and IV [D] III and IV

[C] II and IV

Of the following types of orders, which would generally be used to protect an existing position or profit? I.buy limit II.sell stop III.buy stop IV.sell limit [A] I, III [B] I, IV [C] II, III [D] II, IV

[C] II, III

An investor, Mrs. Smith, is seeking a mutual fund offering a high yield. Which of the following is the best choice for Mrs. Smith? [A] Aggressive Growth Fund [B] Hedge Fund [C] Income Fund [D] Emerging Growth Fund

[C] Income Fund

A younger client with a moderate amount of funds is considering the purchase of a home in the near-term future. For this reason, the investor wishes to keep their assets as liquid as possible in the event that the right deal on a home comes along. As this client's registered representative, which of the following would carry the LEAST amount of liquidity and therefore be unsuitable for this client? [A] Investment in bonds and bond funds focused on municipal securities [B] Investment in equity securities such as preferred stock [C] Investment in a DPP (Direct Participation Program) [D] Investment in CDs (Certificates of Deposit) with varying short-term maturities

[C] Investment in a DPP (Direct Participation Program)

Which of the following risks is NOT normally a major factor when analyzing ADRs (American Depositary Receipts)? [A] Risks associated with changes in inflation [B] Risks associated with changes to legislators and political climate Which of the following risks is NOT normally a major factor when analyzing ADRs (American Depositary Receipts)? [C] Risks associated with the call of such securities by the issuers [D] Risks associated with fluctuations in currency exchange rates

[C] Risks associated with the call of such securities by the issuers

A registered representative suggests and then implements a strategy in a client's portfolio. This strategy involves the RR coming up with certain determinations in relation to an appropriate distribution of investments in the client's account and the maintenance of this mix of investments over time. Which of the following most accurately describes the strategy that has been implemented? [A] The RR is using only a technical analysis approach to allocation. [B] The RR is using capital asset pricing model to determine allocation. [C] The RR is using a form of asset allocation for the client. [D] The RR is using a strategy purely focused on diversification for the client.

[C] The RR is using a form of asset allocation for the client.

MSRB Rules dictate that certain aspects of a trade may be modified with the agreement of both parties, while other aspects of a trade may not be modified. Which of the following is NOT permitted to be modified, even if all persons involved in the trade agree? [A] Determination of who will pay expenses related to shipping certificates, etc. [B] The various factors which will determine whether or not the delivery of the securities constituted a "good delivery" [C] The information which is provided on confirmations sent to both sides of the trade [D] Determination as to where and when the securities will be delivered

[C] The information which is provided on confirmations sent to both sides of the trade

If an investor is seeking capital gains and income from their investments, what type of investment would BEST suit these objectives? [A] The purchase of a block of growth stock [B] The deposit of funds into a money market account [C] The purchase of a mutual fund heavily invested in equity securities. [D] The purchase of STRIPs on U.S. Treasury Securities

[C] The purchase of a mutual fund heavily invested in equity securities.

Excise taxes on cigarettes, alcohol, and gasoline secure certain securities issued by a municipality. Which of the following is the MOST appropriate statement regarding this type of municipal issue? [A] These would be BABs (Build America Bonds). [B] These would be Special or Additional Assessment Bonds. [C] These would be Special Tax Bonds. [D] These would be Tax or Bond Anticipation Notes.

[C] These would be Special Tax Bonds.

An investor's investment objectives are related to all of the following circumstances EXCEPT: [A] time horizon [B] marital status [C] level of education [D] number of dependents

[C] level of education

When an investor buys a call option, regardless of whether it is an Equity or Stock Index option, the maximum risk or loss potential to the investor is the [A] premium that was paid for the contract plus any out-of-the-money amount [B] premium that was paid for the contract less the in-the-money amount [C] premium that was paid for the contract [D] exercise price on the contract plus the premium paid for the contract

[C] premium that was paid for the contract

One of your clients regularly trades options. The customer feels that QRS is going to remain relatively stable or have a slight downturn and he decides to sell 1 QRS May 55 call for $2.50 when QRS is at $54.50. About a month later, the customer receives an exercise notice when QRS is trading at $60.75 per share. In terms of the exercise, what price will the client report for tax purposes in relation to the price at which the stock is sold to the buyer of the call? [A] $52.50 per share [B] $54.50 per share [C] $57.00 per share [D] $57.50 per share

[D] $57.50 per share

When a corporation is investing funds, which of the following types of securities provides the best tax incentives for the corporation? [A] Investments in convertible fixed income debt securities [B] Investments in Treasury Securities [C] Investments in corporate debt securities [D] Investments in corporate preferred stock

[D] Investments in corporate preferred stock

A client is evaluating purchasing corporate bonds versus purchasing municipal bonds. In their evaluation, the client wishes to evaluate the equivalent yield required by a corporate to compensate for the tax benefits of the municipal bonds. Which of the following is the formula used to determine tax-equivalent yield? [A] The client should multiply the tax-exempt yield of the muni by their tax rate. [B] The client should divide the tax-exempt yield of the muni by their tax rate. [C] The client should multiply the tax-exempt yield of the muni by (100% - their tax rate). [D] The client should divide the tax-exempt yield of the muni by (100% - their tax rate).

[D] The client should divide the tax-exempt yield of the muni by (100% - their tax rate).

Most CMO's, Collateralized Mortgage Obligations, as an investment have all of the following risks EXCEPT: [A] The risk that mortgages will be paid off early [B] The risk that interest rates will fluctuate and investors may be stuck with a lower return [C] The risk that inflation will make the investment less profitable [D] The risk that the CMOs will go into default

[D] The risk that the CMOs will go into default

An investor, whose investment objective is speculation, would MOSTLY be concerned with which of the following? [A] Their exposure to reinvestment risk [B] Their exposure to currency risk [C] Their exposure to interest rate risk [D] Their exposure to capital risk

[D] Their exposure to capital risk

Of the following, which offers the MOST accurate description of a municipal bond that is considered "double-barreled"? [A] These bonds are backed by the unlimited taxing power of a municipality along with the ability of State-level appropriations in the event of default. [B] These bonds are backed by project revenues and receive additional backing in the form of the moral obligation of the State in which the municipality is located. [C] These bonds are backed by income received from securities held in a municipality's portfolio, as well as a gross revenue pledge. [D] These bonds are backed by specific revenues from a project along with the municipality's general obligation pledge.

[D] These bonds are backed by specific revenues from a project along with the municipality's general obligation pledge.

All of the following are components of the calculation to find compound accreted value (CAV) on zero-coupon bonds EXCEPT? [A] the maturity date of the bond [B] the dated date of the bond [C] the bond's original yield at offering [D] the bond's current market yield

[D] the bond's current market yield

One of your customers has recently celebrated a 58th birthday. The investor began a regular investment program into shares of the KAPCO Growth Fund over twenty years ago. The account is showing a substantial gain. Because retirement is getting closer, you suggest using the exchange privilege offered by the KAPCO fund group. Your recommendation is to place half of the holdings into the KAPCO Balanced Fund. Following this recommendation would result in

a taxable transaction for those shares exchanged.

An investor receiving a quote of 102 for a municipal security is probably interested in

a term bond (1,020 is a dollar quote) dollar bonds have term maturity

Which of the following choices is a type of retirement plan associated with individuals who work for a private company? a. A 401(k) plan b. A 403(b) plan c. A 457 plan d. A 529 plan

a. A 401(k) plan Explanation: A 401(k) plan is a tax-advantaged retirement account established by corporations, both private and publicly traded. A 457 plan is a tax-advantaged retirement plan that may be established by a governmental body (a municipality) or nongovernmental (nonprofit) employers. 403(b) plans are established by certain nonprofit organizations, such as religious organizations or public schools. A 529 plan is a tax-advantaged savings plan used by individuals to fund higher education needs.

An investor is in the 28% tax bracket. Which of the following investments affords him the BEST tax advantage? a. A 5% municipal bond b. A 5 3/4% corporate bond c. A 6 1/2% preferred stock d. A 6 3/4% convertible bond

a. A 5% municipal bond Explanation: The 5% municipal bond offers the best tax advantage because the interest income is completely free from U.S. government taxes. The income received on the other investments is subject to U.S. taxes at the 28% tax rate. The taxable equivalent yield of the 5% municipal bond is 6.9% (5% municipal yield divided by 72%, the complement of tax bracket), which is greater than the other choices.

A customer has a federal tax rate of 35% and a state tax rate of 7%. Which of the following investments would afford him the BEST after-tax yield? a. A 6.25% in-state municipal bond b. A 6.65% out-of-state municipal bond c. A 9.95% investment-grade corporate bond d. A 10.35% mortgage bond

a. A 6.25% in-state municipal bond Explanation: The major advantage of municipal bonds for most investors is that the interest received from the bond is exempt from federal taxes. In addition, most states also exempt interest from bonds issued within their state from a resident's state and local income taxes. However, if a state resident earns interest from an out-of-state municipal security, that interest is usually subject to state and local taxation. If an investor in a particular tax bracket would like to compare the benefit of tax-free interest income to after-tax income of a taxable bond, it is necessary to find the equivalent taxable yield. The mortgage bond is a type of corporate bond and both are fully taxable. Since the investor can purchase an in-state municipal bond and out-of-state municipal bond, we use the combined rate of 42% for the in-state bond and the federal rate of 35% for the out-of-state bond. The formula is: Municipal Bond Yield / (100% - Investor's Tax Bracket) = Equivalent Taxable Yield The customer is in the 42% combined tax rate. The municipal bond has a yield of 6.25%. 6.25% (Municipal Bond Yield) / 58% (100% - 42%) = 10.78% Equivalent Taxable Yield The out-of-state municipal bond has a yield of 6.65% and the equivalent taxable yield is 10.23% (6.65% / 65%). The in-state municipal bond has the best or highest after-tax yield.

Which of the following bonds results in the highest real interest rate? a. A bond yields 8% when inflation is at 3% b. A bond yields 12% when inflation is at 8% c. A bond yields 10% when inflation is at 7% d. A bond yields 6% when inflation is at 4%

a. A bond yields 8% when inflation is at 3% Explanation: The real interest rate, also called the real rate of return, refers to yields adjusted for inflation (yield minus inflation rate). Choice (a) provides the highest real interest rate (8% bond yield minus 3% inflation rate equals 5% real interest rate).

A variable annuity would be MOST suitable for which of the following customers? a. A client in a high tax bracket who is purchasing the annuity for his spouse's retirement needs b. A client in a high tax bracket who is purchasing the annuity for short-term liquidity needs c. A client who is purchasing the annuity in a 401(k) for his retirement needs d. A client who is purchasing the annuity in order to have the funds available by the age of 50

a. A client in a high tax bracket who is purchasing the annuity for his spouse's retirement needs Explanation: A variable annuity is most suitable for an investor seeking long-term, tax-deferred income for retirement. A tax-deferred investment, as with a variable annuity, becomes more advantageous for an investor with a higher tax bracket. A variable annuity is unsuitable for customers that have short-term needs since the insurance company may impose surrender charges if the annuity proceeds are withdrawn early. It would also be unsuitable for a client purchasing the annuity in a tax-qualified account such as a 401(k) or IRA, since these accounts already have the benefits of tax-deferred growth. If a client withdraws the proceeds of a variable annuity prior to age 59 1/2, a 10% tax penalty applies.

Which of the following statements is TRUE concerning periodic payment variable annuities? a. A client's number of annuity units never changes b. A client's number of accumulation units never changes c. Annuity contracts never have a beneficiary d. The monthly payout is fixed by the inflation index

a. A client's number of annuity units never changes Explanation: During the pay-in period of a variable annuity, the client is continually purchasing accumulation units. These accumulation units are then exchanged for a fixed number of annuity units when the payout period begins. The monthly payout is determined actuarially and is based on the performance of the separate account.

A corporation wishes to open a cash account. Which of the following documents is required? a. A corporate resolution b. A copy of the corporate charter c. A hypothecation agreement d. A risk disclosure document

a. A corporate resolution Explanation: A corporate resolution authorizing a person to trade for the account is necessary to open a corporate cash account. A risk disclosure document may be required but only if options or penny stocks are going to be traded in the account. A hypothecation agreement and corporate charter are required to open a margin account.

Which of the following descriptions regarding the Capital Asset Pricing Model (CAPM) is NOT TRUE? a. It predicts future values for the stock b. It was developed to explain the behavior of security prices c. It provides a mechanism to assess risk and return d. It is based on the efficient market theory and assumes investors act rationally

a. It predicts future values for the stock Explanation: CAPM does not establish a price objective for the stock. All of the other descriptions listed are correct.

SEC regulations state that a brokerage firm must provide a current financial statement (balance sheet) to: a. A customer when requested by the customer b. A noncustomer who requests it c. Both of the above d. Neither of the above

a. A customer when requested by the customer Explanation: SEC regulations states that a brokerage firm must provide a current financial statement (balance sheet with net capital computation) to a customer upon request. The customer has the right to know the financial condition of the company with which she is doing business.

Which of the following corporate retirement plans is designed to provide employees with a fixed amount of funds at retirement? a. A defined benefit plan b. A defined contribution plan c. A 401(k) plan d. An IRA

a. A defined benefit plan Explanation: A corporate retirement plan may be established as either a defined benefit or a defined contribution plan. A defined benefit plan is designed to provide employees with a fixed monthly stipend at retirement. This is generally a percentage of the employee's salary, the exact amount of which is determined by the employee's age and years of service.

A customer contends that his registered representative made unauthorized trades in his account and will take this matter to an arbitration panel. Regarding the makeup of this panel, which of the following statements is TRUE? a. A majority of the arbitration panel must come from outside the securities industry b. A majority of the arbitration panel will come from within the securities industry c. All arbitrators must come from inside the securities industry or must be attorneys d. All arbitrators must come from outside the securities industry

a. A majority of the arbitration panel must come from outside the securities industry Explanation: Under the Code of Arbitration, if a public customer takes a member firm to arbitration to resolve a dispute, the majority of the panel must come from outside the securities industry, unless the customer requests a panel with a majority of industry arbitrators. Neither the broker-dealer nor the customer may actually pick the arbitrators and arbitrators do not need to be attorneys.

Which of the following choices is LEAST important to an investor considering a bond swap? a. Accrued interest b. Annual income c. Capital loss d. Maturity dates

a. Accrued interest Explanation: A bond swap involves selling one bond and using the proceeds to buy another bond with either a different yield, interest rate, or maturity date. This is usually done to establish a capital loss for tax purposes. Of the choices given, the least important factor to consider in the swap or exchange is accrued interest since accrued interest paid will be included in the next interest payment and all accrued interest received has been earned prior to the bond being sold.

A REIT is NOT used for a tax shelter because it does NOT: a. Allow flow-through of losses b. Allow flow-through of income c. Provide limited liability d. Offer income potential

a. Allow flow-through of losses Explanation: A REIT allows the flow-through of income, but not losses. Shareholders have limited liability. While a REIT is similar in structure to a mutual fund, it is not defined as an investment company. Also, while it may invest in real estate properties, it is not considered to be a limited partnership. Partnerships can pass through losses.

Which of the following lists assists a broker-dealer in making a reasonable determination that a security is available to be borrowed from another broker-dealer in order to effect a short sale transaction? a. An Easy-to-Borrow List b. A Hard-to-Borrow List c. A Threshold Security List d. A Restricted Stock List

a. An Easy-to-Borrow List Explanation: In order to aid in the process of locating securities, the SEC has accepted the use of Easy-to-Borrow lists. These lists, which must be less than 24 hours old, provide reasonable grounds for belief that a security on the list will be available to be borrowed. The securities on the list must be readily available to avoid fails to deliver. Use of an Easy-to-Borrow list expedites the fulfillment of the locate provision. A Hard-to-Borrow list refers to securities that a clearing broker-dealer may have difficulty in borrowing.

A customer has $350,000 to invest and would like to hold a diversified portfolio of stock, bonds, and money-market instruments. She wants the percentage invested in each of these asset categories to be adjusted as financial markets change. However, her business keeps her too busy to adequately monitor her holdings and make the appropriate changes. Which of the following investments are MOST suitable for this customer? a. An asset allocation fund b. An S&P Index fund c. A bond index fund d. A variable annuity

a. An asset allocation fund Explanation: Asset allocation funds hold diversified portfolios of stocks, bonds, and money-market instruments. The percentage of the portfolio invested in each of these categories is shifted by the fund manager from time to time, often according to computer models.

An investor with an investment objective of speculation wants to purchase a security that will increase by the same percentage as a decline in the S&P 500 Index. Which of the following securities would you recommend? a. An inverse exchange-traded fund (ETF) b. A leveraged exchange-traded fund (ETF) c. A leveraged inverse exchange-traded fund (ETF) d. An exchange-traded fund (ETF)

a. An inverse exchange-traded fund (ETF) Explanation: An inverse ETF is designed to deliver the opposite of the performance of an index or other benchmark. An inverse ETF based on the S&P 500 Index seeks to deliver the opposite performance of that index. So, if the S&P 500 rises by 1%, an inverse ETF would decrease by 1%, and if the S&P 500 falls by 1%, the inverse ETF would increase by 1% before fees and expenses. Choice (b) would be suitable if the customer anticipated an increase in the S&P 500 and wanted a multiple of that increase. Choice (c) would be suitable if the customer wanted a return that was a multiple or higher return and anticipated a decrease in the S&P 500, and choice (d) would be suitable if the customer only wanted to track the return of the S&P 500.

The credit rating agencies have downgraded an issuer of an exchange-traded note. Which of the following statements is TRUE? a. It will have a negative impact on the security b. It will have a positive impact on the security c. It will have no impact on the security d. The issuer will be obligated to repay the investor his principal immediately

a. It will have a negative impact on the security Explanation: Exchange-traded notes (ETNs) are a type of unsecured debt security. ETNs carry issuer risk that is tied to the creditworthiness of the financial institution backing the note. If the issuer's financial condition deteriorates and the credit rating agencies downgrade the issuer of the ETN, it would impact the value of the ETN negatively.

Which of the following statements is NOT TRUE concerning VIX options? a. An investor will buy VIX puts if he expects an increase in the volatility on the S&P 500 Index b. An investor will buy VIX calls if he expects the S&P 500 Index to fall c. An investor will buy VIX calls as a hedge if he expects the S&P 500 Index to fall d. VIX options can be used if an investor expects an increase or decrease in the volatility on the S&P 500 Index

a. An investor will buy VIX puts if he expects an increase in the volatility on the S&P 500 Index Explanation: The VIX (volatility index) is often referred to as a gauge of investors' fear. The index tends to move inversely with the S&P 500 Index. The VIX usually rises when the S&P 500 Index falls, and falls when the S&P 500 Index increases. An investor will buy calls when he expects the market to decline and volatility to increase. An investor will buy puts on the VIX if he expects the market to rise and volatility to decrease. Many investors will buy VIX call options as a hedge against a possible decline in the market. The VIX option can be used by investors who expect either an increase or a decrease in volatility.

Which of the following money-market securities assists in financing importing and exporting operations? a. Bankers' acceptances b. Treasury bills c. Eurodollar CDs d. Revenue anticipation notes

a. Bankers' acceptances Explanation: Of the choices given, a banker's acceptance (BA) is the only instrument that is used as a means of financing foreign trade. Do not confuse a BA with an ADR (American Depositary Receipt) which facilitates the trading of foreign securities in U.S. markets. A Eurodollar certificate of deposit pays interest and principal in Eurodollars (U.S. dollars deposited in nondomestic banks) and is not used to finance importing and exporting operations. A revenue anticipation note is a short-term municipal security issued in anticipation of receiving revenues from the federal or state government.

According to technical analysis, a head and shoulders top formation indicates a trend that is: a. Bearish b. Bullish c. Neutral d. Highly unpredictable

a. Bearish Explanation: A head and shoulders chart formation is one of the classical patterns agreed upon by technical analysts or chartists as being a reversal of a trend in the price of a stock. If the head and shoulders pattern appears at the top of an upward trend (head and shoulders top), as in this example, it indicates the reversal of an upward trend (bearish indicator). If the head and shoulders pattern appeared at the bottom of a downward trend (head and shoulders bottom), it indicates a reversal of a downward trend in the price movement of a particular stock (bullish indicator).

A high put/call ratio would MOST likely be associated with a(n): a. Bullish indicator b. Bearish indicator c. Indicator that the market will trade within a narrow range d. Indicator that the trading volume will be increasing

a. Bullish indicator Explanation: The put/call ratio is a technical market indicator and is found by dividing the volume of all put transactions by the volume of all call transactions on a daily basis. Technical analysts view the put/call ratio as a contrarian indicator. The higher the ratio, the more oversold the market, and the higher the probability that the market will reverse course and turn bullish. The opposite is true for a low put/call ratio, which is viewed as a bearish indicator.

An American company has provided its services to a British consulting firm and is expecting payment from a customer in British pounds. To hedge against an increase in the U.S. dollar, the American company should: a. Buy British pound puts b. Buy British pound calls c. Write British pound calls d. Write British pound straddles

a. Buy British pound puts Explanation: If the value of the U.S. dollar increases, the value of the British pound will decrease. The American company should buy British pound puts since it would profit on the puts if the U.S. dollar increased, leading to a decrease in the British pound. The profit on the put could help to offset the loss on the British pounds the American company is expecting to receive as payment.

An individual who is bearish or wants protection against a downside move in the market will probably: a. Buy a put option b. Sell a put option c. Buy a call option d. Write an uncovered call

a. Buy a put option Explanation: An individual who is bearish will buy a put option. If the market declines, the individual will purchase the security in the market at a lower price and put (sell) it to the writer at the higher agreed-upon strike price.

All of the following derivatives are created by an issuer of securities, EXCEPT: a. Call options b. Warrants c. Rights d. Convertible preferred stock

a. Call options Explanation: Call options are issued by the Options Clearing Corporation (OCC) and not by an issuer of securities. The other products are created by an issuer of securities.

A customer has a nondiscretionary account at a broker-dealer. The customer has received a research report and has indicated that she may want to purchase a stock on the recommended list. Which of the following actions is MOST appropriate for the registered representative to take? a. Contact the customer and ask her to place an order to buy the security b. Purchase a small amount of the stock and contact the customer no later than the same business day c. Purchase a small amount of the stock and have the customer sign a written authorization form no later than the same business day d. Purchase the stock on behalf of the customer and have the order approved promptly by a principal

a. Contact the customer and ask her to place an order to buy the security Explanation: This is a nondiscretionary account and, therefore, no shares may be purchased unless the customer gives the broker-dealer an order to purchase the security.

Which of the following choices is NOT present in an equipment leasing program? a. Depletion b. Depreciation c. Interest expense d. Possibility of recapture

a. Depletion Explanation: Depletion relates to natural resources only (oil and gas, for example).

The largest deduction generated by a DPP in real estate is: a. Depreciation b. Depletion c. Recapture d. Tax credit

a. Depreciation Explanation: The largest deduction in a real estate program is generally depreciation.

Which of the following statements is NOT a characteristic of an electronic communication network (ECN)? a. ECNs act as market makers b. ECNs permit trading electronically c. ECNs permit trading anonymously d. ECNs permit trading after-hours

a. ECNs act as market makers Explanation: Electronic communication networks allow market participants to display quotes and execute transactions. These participants are referred to as subscribers and pay a fee to the ECN to trade electronically through the system. ECNs allow subscribers to trade after-hours, and to quote and trade without disclosing their names (anonymously). ECNs act in an agency capacity and will not buy or sell for their own account as with a market maker.

Which of the following statements is NOT TRUE about exchange-traded notes (ETNs)? a. ETNs generally pay a fixed coupon rate b. ETNs may be sold at any time in the secondary markets or held until maturity c. ETNs carry issuer risk that is tied to the creditworthiness of the financial institution backing the note d. If the issuer's financial condition deteriorates, it could negatively impact the value of the ETN, regardless of how its underlying index performs

a. ETNs generally pay a fixed coupon rate Explanation: All of the statements about ETNs are true except ETNs pay a fixed coupon rate. ETNs do not usually pay an annual coupon or specified dividend. They are a type of unsecured debt security. This type of debt security differs from other types of bonds and notes since ETN returns are linked to the performance of a commodity, currency, or index minus applicable fees. Similar to ETFs, ETNs are traded on an exchange, such as the NYSE, and may be purchased on margin or sold short. Investors may also choose to hold the debt security until maturity. ETNs carry issuer risk that is tied to the creditworthiness of the financial institution backing the note. If the issuer's financial condition deteriorates, it could negatively impact the value of the ETN, regardless of how its underlying index performs.

Which of the following statements is TRUE concerning a customer who purchases an original issue discount (OID) corporate bond? a. Each year the customer will pay both federal and state income tax b. Each year the customer will pay only federal income tax c. Each year the customer will not pay any tax d. The customer will only pay tax at bond maturity

a. Each year the customer will pay both federal and state income tax Explanation: The upward adjustment in the purchase price of an original issue discount bond is called accretion. The amount accreted each year is considered interest income, which may or may not be taxable depending on the type of security. The interest on corporate bonds is subject to both federal and state income tax.

A customer in his late twenties wants capital appreciation and is willing to take a moderate degree of risk in his initial investment. The customer is also concerned about the inflationary risk to his portfolio. Which of the following investments is MOST suitable? a. Equities b. Corporate debt c. Municipal debt d. Variable annuities

a. Equities Explanation: Since the investor is concerned about inflationary risk, and is willing to accept a moderate degree of risk to his initial investment, equities would be the most appropriate investment. If the investor wanted a tax-deferred investment with the same investment objectives, variable annuities would be the most suitable choice.

SPDR is considered a type of: a. Exchange-traded fund b. Index option c. World currency option d. Mutual fund

a. Exchange-traded fund Explanation: Standard & Poor's Depositary Receipt (SPDR) is a type of exchange-traded fund (ETF). It can be used to refer to a specific exchange-traded fund that tracks the S&P 500 or a group of ETFs.

Which of the following statements is TRUE concerning gift and estate taxes paid by a husband and wife? a. Gifts between spouses are unlimited and no gift or estate taxes will be paid when one spouse passes away b. Gifts between spouses are unlimited and there is no gift tax, but estate taxes must be paid when one spouse passes away c. Tax-free gifts between spouses are limited to $14,000 per year, but taxes are not due on any excess until one spouse passes away d. Tax-free gifts between spouses are limited to $14,000 per year and taxes must be paid on any excess in the year the gift is given

a. Gifts between spouses are unlimited and no gift or estate taxes will be paid when one spouse passes away Explanation: There is no limit on the amount of a gift between spouses. A husband or wife may give any amount to a spouse without incurring a tax liability. An estate tax may be levied on the value of a decedent's assets, but there will not be any estate tax due when one spouse dies if all the assets are passed along to the surviving spouse. The entire estate may pass tax-free to the survivor. If assets are distributed to persons other than the decedent's spouse, the estate may be taxed if the amount exceeds the allowable limits established by current tax law. Gift taxes must be paid by a donor, not the recipient of the gift. An individual may give a gift of $14,000 per person, per year ($28,000 for joint returns) without incurring a gift tax. The donor must pay the gift tax on amounts given over this figure.

A firm that is planning an offering of common stock has not filed a registration statement. Which of the following actions on the part of a registered representative are NOT a violation of the Securities Act of 1933? a. Having the registered representative contact an investment banker at the firm b. Informing a customer that he may receive as many shares as he desires c. Accepting orders for the shares to be offered d. Attempting to obtain indications of interest for the shares to be offered

a. Having the registered representative contact an investment banker at the firm Explanation: There is no prohibition restricting an RR to contact an investment banker at the firm. The other actions listed are in violation of the Securities Act of 1933, if a registration statement has not been filed with the SEC. A registered representative may not inform a customer that the customer may receive as many shares as desired. Nor may the registered representative solicit buy orders or solicit indications of interest from the customer. A registration statement needs to be filed before indications of interest may be accepted, and only indications of interest will be acceptable at this time, not orders.

A corporation has raised money to use for expansion of its plant within the next six months. In which of the following securities should the corporation invest the funds until they are used? a. High-quality commercial paper b. Long-term municipal zero-coupon bonds c. U.S. Treasury bonds d. High-quality preferred stocks

a. High-quality commercial paper Explanation: The corporation intends to use the money in a short period and does not want to assume undue investment risks. Of the choices given, the most suitable investment is high-quality commercial paper since it is extremely safe and can be purchased with a short maturity to match the corporation's needs.

A customer has two children and wants to open one UTMA account which will be shared by the two of them. The RR should take which of the following actions? a. Instruct the customer to open two UTMA accounts, one for each child b. Instruct the customer that the account may be set up for both children provided it is preapproved by the firm's compliance department c. Instruct the customer that one UTMA account may be shared by both children as long as they are at least 14 years of age d. Instruct the customer that one UTMA account may be opened if the customer's spouse also signs the new account agreement

a. Instruct the customer to open two UTMA accounts, one for each child Explanation: A custodial account (UTMA or UGMA) may be established for the benefit of one minor only. In this question, the customer should open two UTMA accounts, one for each of his children.

James Hendricks wants to open a Coverdell Education IRA for his three-year-old son. Which of the following statements is TRUE? a. James may contribute up to $2,000 per year b. At least 50% of the investments in the account must be conservative c. The account becomes the property of the child upon reaching the age of majority d. Only parents or grandparents may contribute for the benefit of the minor child up to the age of majority

a. James may contribute up to $2,000 per year Explanation: Anyone may contribute to a Coverdell Education IRA for a child, but the total contributions to the account are limited to $2,000 per year. Choice (c) is true of a UGMA or UTMA account, not a Coverdell Education IRA. There is no percentage requirement for investments in the account.

Which of the following choices is Moody's BEST rating for a municipal note? a. MIG 1 b. MIG 3 c. Aaa d. AAA

a. MIG 1 Explanation: MIG stands for Moody's Investment Grade and is used to rate municipal notes. There are three MIG ratings, with the best rating being MIG 1 and the lowest rating being MIG 3. Aaa is Moody's best rating for bonds.

If the S&P 500 has been increasing on high volume for several days, what term would BEST define this situation? a. Market momentum b. An efficient market c. Market neutral d. A resistance level

a. Market momentum Explanation: The term market momentum is used to describe a situation where prices are moving in a certain direction and there is a high level of trading volume. There is also an expectation that this pattern will continue in the near future. For example, if the S&P 500 Index has been trading up or down significantly over a period of days along with heavy trading volume, some traders will anticipate this pattern may continue for a few more days. Market neutral is used to describe attempting to profit by buying some securities while at the same time selling short others. A resistance level is a point on a chart where the price of a security stops increasing. Efficient market is a term used to define that stock prices already represent all available information and there is no benefit that may be gained by using professional analysis.

In the Interbank market, foreign currency transactions: a. May settle on a spot or forward basis b. Occur on exchanges throughout the world c. Are regulated by the SEC d. Are reported on the Nasdaq system

a. May settle on a spot or forward basis Explanation: The Interbank market is an unregulated over-the-counter market in which currencies of different countries are bought and sold. Foreign currency transactions may settle on a spot or forward basis. Spot transactions settle in two business days from the trade date. In a forward transaction, the exchange rate is established on the trade date but settlement occurs in more than two business days. While foreign currency transactions are not reported on Nasdaq, spot quotes are available from information vendors such as Knight-Ridder Financial Information Systems, Reuters, and Telerate.

To sell a security in a given state, a registered representative: a. Must be registered in that state b. Must be registered in that state only if he is a resident of that state c. Need not be registered in that state if his brokerage firm is registered in that state d. Does not need to be registered in that state if he is registered with FINRA

a. Must be registered in that state Explanation: Any broker-dealer or registered representative selling securities in a particular state must be registered in that state. Being registered with FINRA does not necessarily register an individual in a given state. Many states have additional registration requirements besides joining FINRA.

Listed below are a group of mutual funds. Net Asset Value Offer Price Net Change Dreyfus 11.55 12.67 -.05 Wellington 12.70 13.85 +.07 Lenox 5.14 5.14 +.09 Sentry 13.42 14.63 -.08 Lenox fund is most likely a: a. No-load fund b. Closed-end fund c. Balanced fund d. Growth fund

a. No-load fund Explanation: The Lenox fund is a no-load fund. The net asset value (bid price) and offering price (asked price) of a no-load fund are the same. There is no sales charge.

A designated market maker (DMM) may not accept which of the following orders? a. Not-held order b. Market order c. Good-'til-cancelled (open) order d. Day order

a. Not-held order Explanation: A designated market maker (formerly known as a specialist) may accept all of the orders listed except a not-held order, which allows a floor broker to use discretion in executing an order. If the question asked which orders may be accepted and placed on his book, the answer would be open (GTC) and day orders only. A DMM may accept a market order but must execute it immediately and may not place it in his book.

Which of the following risks for an agency-backed CMO is LEAST important to an investor in a rising interest-rate environment? a. Prepayment risk b. Credit risk c. Interest-rate risk d. Extension risk

a. Prepayment risk Explanation: Prepayment risk is associated with a falling interest-rate environment in which mortgage holders refinance or repay their mortgages at a faster rate. The holder of a CMO, therefore, receives a larger portion of the principal earlier than anticipated and is forced to reinvest at lower rates. Many CMOs are created from government agency mortgage-backed securities (MBS), which have a minimal amount of credit risk. Some CMOs are constructed without this backing and, therefore, credit risk is a greater concern. CMOs, as with most fixed-income securities, carry interest-rate risk. Extension risk is the opposite of prepayment risk, where interest rates are rising and the CMO holder receives a smaller portion of her principal back.

When must a customer be sent a current Options Clearing Corporation risk disclosure document? a. Prior to or at the time the account is approved for options trading b. Not later than the time the customer receives a confirmation of the initial option transaction c. Not later than 15 days following the time the account is approved for options trading d. At least 15 business days prior to the time an initial option transaction is executed

a. Prior to or at the time the account is approved for options trading Explanation: A customer must be sent a current Options Clearing Corporation risk disclosure document prior to or at the time the account is approved for options trading.

Which of the following methods is used by the Options Clearing Corporation in assigning exercise notices? a. Random selection b. First-in, first-out c. To the member firm holding a long position that first requests an exercise d. On the basis of the largest position

a. Random selection Explanation: The OCC assigns exercise notices on a random selection basis only.

An investor would purchase an inverse exchange-traded note if: a. She anticipates a decrease in the benchmark that is being tracked b. She anticipates an increase in the benchmark that is being tracked c. She is interested in obtaining a long-term capital gain d. She is interested in obtaining income on a regular basis

a. She anticipates a decrease in the benchmark that is being tracked Explanation: Exchange-traded notes (ETNs) are a type of unsecured debt security. ETNs carry issuer risk that is tied to the creditworthiness of the financial institution backing the note. These securities are not like traditional fixed-income securities since they typically do not make interest payments to investors. The returns are linked to the performance of an index, currency, or commodity and would be suitable for investors who want to speculate on the value of an index. An inverse ETN would pay the opposite of the benchmark that is being tracked and would be suitable for a person interested in short-term trading. Most ETNs are traded on a national exchange (NYSE) and, therefore, an investor can quickly sell the security to earn a short-term gain.

Which of the following factors is LEAST important when recommending a long-term brokered CD to a client? a. The CD was issued by a bank located in a different state from where the client lives b. The CD has a feature in which the interest rate is based on a percentage increase in an equity index c. The client will be purchasing the CD in a retirement account d. The firm may make a market in this CD, but is not obligated to do so

a. The CD was issued by a bank located in a different state from where the client lives Explanation: The state in which the client or issuing bank is located is not an important factor when recommending a long-term brokered CD. The features that establish the interest rate of the security, such as an index of fixed-income or equity securities, is relevant to the client. The amount of FDIC insurance and tax considerations are different depending on whether the CD is purchased in a retirement account. In addition, a broker-dealer is not required to maintain a secondary market or act as a market maker in a CD that was sold to the client. This will limit the liquidity of the security if the client needs the funds prior to maturity.

The initial FRB margin requirement is 50%. A customer purchases 1,000 shares of XOP at $48 per share and makes the necessary deposit. If XOP increases in value to $57 per share and later declines to $45 a share, which of the following statements is TRUE? a. The SMA in the account is $4,500 and the equity is $21,000 b. The SMA in the account is $4,500 and the equity is $33,000 c. There is no SMA and the equity is $21,000 d. The SMA is $9,000 and the equity is $33,000

a. The SMA in the account is $4,500 and the equity is $21,000 Explanation: First, determine the amount of the debit balance. If the customer purchased $48,000 worth of stock at a 50% margin requirement and deposited $24,000, the debit balance is $24,000 ($48,000 market value - $24,000 margin requirement = $24,000 debit balance). XOP increased to $57 per share, making the market value $57,000. The equity increases to $33,000. The excess equity (SMA) is found by subtracting the FRB-required equity of $28,500 (50% of $57,000) from the actual equity in the account of $33,000. The SMA is, therefore, $4,500. The SMA remains in the account until it is used. The SMA balance will never decrease because of market movements. Securities held in a margin account that increase in value can create excess equity (SMA) but, if these securities later decline in value, this will not decrease SMA. The equity decreases since the market value declined to $45 per share and is now $21,000 ($45,000 - $24,000).

A customer fails to pay for securities by payment date. Which of the following statements is TRUE? a. The account is frozen for 90 days b. The customer is prohibited from opening another account for 90 days c. The customer may trade after 30 days d. The account is closed because it is in violation

a. The account is frozen for 90 days Explanation: When a customer fails to pay for securities, the account is restricted (frozen) for 90 days. Before the customer may buy additional securities, the customer must deposit the full purchase price of the securities in the account. This is a Regulation T requirement. The other choices are incorrect.

Which of the following factors is NOT used in determining the value of an annuity unit? a. The assumed interest rate b. The value of the separate account c. Income distributions from securities held in the separate account that are reinvested d. Capital gain distributions from securities held in the separate account that are reinvested

a. The assumed interest rate Explanation: The assumed interest rate (AIR) is used to determine the subsequent payments made to the annuitant. The value of the annuity unit is determined by the value of the separate account, including all reinvested distributions.

A 3-month Treasury bill is issued at a discount to yield 9.5%, and a corporate bond is issued to yield 9.5%. The bond is to mature in 10 years. If both are offered on the same day on a bond equivalent yield basis, which of the following statements is TRUE? a. The bill has a greater yield than the bond b. The bond has a greater yield than the bill c. The yield is the same for both d. The bond equivalent yield and tax equivalent yield are equal

a. The bill has a greater yield than the bond Explanation: T-bills are issued and quoted on a discount yield basis, whereas corporate bonds are quoted on a yield-to-maturity basis. These yields are calculated in different manners. The bond equivalent yield of a T-bill is always higher than its discount yield.

A customer buys a premium bond that is callable. Which of the following is LEAST beneficial for the customer? a. The bond is called at its par value in five years b. The bond is called at its par value in ten years c. The bond is called at its par value in fifteen years d. The bond is called at its par value in twenty years

a. The bond is called at its par value in five years Explanation: If the bonds are called in five years at par, the premium paid for the bond will be amortized over the shortest period. This results in the investor realizing a lower yield than if the bond were called after a longer period. It is important to note that rules require a firm to disclose to a customer the lowest possible yield that the customer can realize. On a premium bond (as in this example), the lowest yield will result from the bond being called at par in the shortest period.

An investor has sold a stock short. If the present market value is $2.00 per share, the minimum maintenance requirement will be: a. 50% b. $2.50 per share c. $2.00 per share d. 30%

b. $2.50 per share Explanation: When selling short securities that have a market value less than $5 per share, a minimum maintenance requirement of $2.50 per share or 100% of the market value, whichever is greater, applies. Since $2.50 a share is greater than $2.00 per share, this is the correct answer.

A bond that trades "flat" means that:

no accrued interest is added to the price of the trade

A client contacts an RR after reviewing the financial statements of the S-Works Carbon Company. The client is confused since the company paid a cash dividend but had a loss for the last fiscal year. Which of the following statements is TRUE? a. The company is permitted to pay a cash dividend even though it had a loss b. The company is not permitted to pay a cash dividend if it had a loss c. If the company has a loss in its last fiscal year, it may pay a cash dividend only with prior approval from shareholders d. If the company has a loss in its last fiscal year, it may pay a cash dividend only with prior approval from the SEC

a. The company is permitted to pay a cash dividend even though it had a loss Explanation: A company is permitted to pay cash dividends in excess of its net income even if it had a loss. In terms of financial accounting, cash dividends are paid out of retained earnings that are part of shareholders' equity. Therefore, cash dividends paid will reduce shareholders' equity. The company could have paid the cash dividend easily based on retained earnings from previous years.

Which item of the following information is NOT generally included on an options confirmation? a. The contrabroker b. The price of the option c. The month and the strike price of the contract d. The name of the underlying security

a. The contrabroker Explanation: The contrabroker is the broker on the other side of the trade. The contrabroker's name is not generally included on a confirmation. All of the other items are included on an options confirmation.

Manny Ortiz, a registered representative with Red Sock Financial, has recommended the purchase of a variable annuity to Derek Pettitte, one of his clients. After agreeing to purchase the contract, which of the following actions must be taken? a. The information concerning the recommendation must be documented and signed by Manny b. The application must be forwarded by Manny directly to the issuing insurance company c. The principal only signs the recommendation if the purchase is approved d. The principal must review the application within five business days of receipt from Manny

a. The information concerning the recommendation must be documented and signed by Manny Explanation: If the purchase of a variable annuity is made after a recommendation by a registered representative, several steps must be taken before the application is submitted to the issuing insurance company. In this example, Manny must document and sign the recommendation before forwarding the application to the Office of Supervisory Jurisdiction (OSJ) of the member firm. The principal at the OSJ reviews the application to determine suitability. The principal must approve or reject the application within seven business days of receipt and must document and sign the approval or rejection.

What information would NOT be found in a subscription agreement in a direct participation program (DPP)? a. The name of the limited partner's certified public accountant b. The suitability standards c. The signature of the limited partner d. To whom the check is made payable to

a. The name of the limited partner's certified public accountant Explanation: The subscription agreement will normally state the suitability standards for the program, specify who must sign the agreement, specify to whom the check must be made payable, and make inquiries of the purchaser to make sure that he understands the ramifications of the investment and can meet the financial requirements of this investment. The name of the limited partner's accountant would not need to be included in this agreement.

Which of the following statements is TRUE concerning periodic payment variable annuities? a. The number of a client's annuity units never changes b. The number of a client's accumulation units never changes c. They never have a beneficiary d. The monthly payout is fixed by the inflation index

a. The number of a client's annuity units never changes Explanation: During the pay-in period of a variable annuity, the client is continually purchasing accumulation units. These accumulation units are then exchanged for a fixed number of annuity units when the payout period begins. The first monthly payout is determined actuarially and thereafter is based on the performance of the separate account.

A customer without a discretionary account gives a registered representative the following verbal instructions: Buy 1,000 shares of General Electric whenever you think the price is right. Under current regulations: a. The order may be executed by the RR only on the same trading day b. The order must be marked discretionary and approved by a branch manager c. The order may be executed by the RR on any trading day d. The RR is not permitted to accept this type of order

a. The order may be executed by the RR only on the same trading day Explanation: The order may be executed by the RR, but only on the same trading day. It is not a discretionary order, which requires written power of attorney. The customer indicated to the registered representative the name of the specific stock (GE), the action (to buy), and the amount (1,000 shares), so this is a not-held order. Allowing the RR to make decisions limited to price and time does not constitute discretion. Absent written instructions from the client, this type of time and price discretion is only valid the same trading day.

Which of the following statements is TRUE regarding a registered representative who has not completed the Continuing Education Regulatory Element training within 120 days of his registration anniversary? a. The representative will be placed in inactive status b. The broker-dealer must request an extension from an SRO c. The representative will be suspended d. The representative has 30 days to complete the requirement

a. The representative will be placed in inactive status Explanation: FINRA will notify a representative within 30 days of the second anniversary date of initial registration, and every three years thereafter. If the representative then fails to complete the required training within 120 days of the anniversary date, that person's registration will become inactive and any activity that requires registration, including receipt of commissions, will be prohibited.

Kyle, a client at TLC brokerage firm, anticipates a decline in the earnings of LPOP. LPOP is a thinly traded issue. Which of the following statements BEST describes what the RR should disclose to Kyle? a. The stock may be difficult to sell short because the shares may not be available to borrow b. All securities may be sold short provided the client has a margin account c. As long as the order ticket is marked sell long, the stock could be sold short d. Exchange-traded put options are available on all securities and are a less risky method to profit

a. The stock may be difficult to sell short because the shares may not be available to borrow Explanation: A client may sell short or buy a put to profit from a decline if the value of a security is anticipated. In order to sell short, the broker-dealer is required to borrow the security. Although short sales may be executed only in a margin account, if an issue is thinly traded, it may be difficult or impossible to borrow the security. A put option may be an attractive alternative to selling short. However, put options are unlikely to be available on a thinly traded security.

In an underwriting of a new issue by a syndicate, which of the following statements is TRUE? a. The underwriting spread is larger than the selling concession b. The selling concession is larger than the underwriting spread c. The reallowance is larger than the underwriting spread d. The reallowance is larger than the selling concession

a. The underwriting spread is larger than the selling concession Explanation: In the underwriting of a new issue, the underwriting spread is larger than the selling concession. The underwriting spread is larger because members of the underwriting syndicate assume the risks of the underwriting. The selling concession given to the selling group is less because the selling group acts in a best-efforts capacity and does not assume the risks involved in a firm-commitment underwriting. A reallowance is compensation given to broker-dealers who are nonmembers of the syndicate or selling group who would like to participate in the underwriting. The reallowance given is less than the amount members of the syndicate or selling group receive.

An investor establishes a long margin account and buys 1,000 shares of TMP at $55. The value of the securities increases and SMA is created. All of the following actions affect SMA, EXCEPT: a. The value of the securities declines b. The value of the securities increases c. Cash is withdrawn from the account d. The buying power of the accounts used

a. The value of the securities declines Explanation: The SMA remains in the account until it is used. The SMA balance will never decrease because of market movements. A decrease in the market value of the securities does not affect the SMA in a long account since, once created, SMA is reduced only when used. An increase in the market value of the securities can increase SMA, since equity increases. Withdrawing cash and buying of additional securities for the account will reduce the SMA since the SMA is used.

Private label mortgage-backed securities have all of the following characteristics, EXCEPT: a. They are not rated by a credit rating agency b. They may be issued by investment banking firms c. They may contain securities that are issued by U.S. government agencies d. They are not considered obligations of the U.S. government

a. They are not rated by a credit rating agency Explanation: Mortgage-backed securities (MBS) may be issued by a U.S. government agency, such as the Government National Mortgage Association (GNMA or Ginne Mae), or a government-sponsored enterprise (GSE), such as the Federal National Mortgage Association (FNMA or Fannie Mae) or the Federal Home Loan Mortgage Association (FHLMC or Freddie Mac). The securities issued by these three entities are commonly referred to as agency securities and receive high ratings (e.g., AAA). A collateralized mortgage obligation (CMO) is an example of this form of MBS. Mortgage-backed securities are also issued by financial institutions such as commercial banks, investment banks, and home builders. These securities are referred to as private label MBS and may contain some agency securities, however, they typically contain other types of mortgage loans that are not agency securities. A private label MBS is not an obligation of the U.S. government or any GSE and its credit rating is assigned by an independent credit agency. A private label MBS has higher credit risk and is generally not given a AAA rating.

Investors may receive disclosure and secondary market information concerning municipal securities: a. Through the EMMA system b. Through the TRACE system c. Directly from the issuer d. From the OATS system

a. Through the EMMA system Explanation: The MSRB has established the Electronic Municipal Market Access (EMMA) system as the primary market disclosure service for official statements, other related primary market documents, and information. The EMMA system also contains information related to the continuing disclosure requirements submitted by municipal issuers and secondary market transactions submitted by municipal securities dealers. EMMA receives transactional information from the MSRB's Real-Time Transaction Reporting System (RTRS).

Which of the following choices best describes the primary cause of inflation? a. Too much money compared to the goods that are available b. Too many goods compared to the money that is available c. Foreign investments appreciating faster than U.S. investments d. U.S. investments appreciating faster than foreign investments

a. Too much money compared to the goods that are available Explanation: Inflation is a general rise in the prices of goods and services. If there is too much money in the economy, the demand for goods and services will increase, pushing up prices. This is sometimes described as too much money chasing too few goods.

The Federal Reserve Board's Open Market Committee (FOMC) buys and sells which of the following securities most often to accomplish its aims? a. Treasury bills b. Treasury notes c. Treasury bonds d. Agency bonds

a. Treasury bills Explanation: The Federal Reserve Board's Open Market Committee (FOMC) purchases and sells U.S. government securities in the open market to accomplish the Federal Reserve Board's aims of influencing the money supply. The securities most often used are Treasury bills.

An "accumulation unit" of a variable annuity contract is a(n):

accounting measure of the owner's interest in the separate account

After the bid is won in a municipal underwriting, a syndicate member that places an order with the manager for a selling group member would earn the:

additional takedown

In a best efforts underwriting, the underwriter is acting as an:

agent

The effect of using the first in, first out (FIFO) method for a sale of some of the securities that were purchased separately during a period of rising prices will be

an increase in the taxable profits of the investor.

A retired person seeking to maximize income with reasonable safety and liquidity should most likely consider investing in

an intermediate-term, high-grade corporate bond fund.

SEC Regulation FD is best described as a rule requiring disclosure by

an issuer of securities.

Although municipal securities are exempt from SEC registration, full disclosure is still required with the use of

an official statement.

An investor purchased a REIT from her broker-dealer in an SEC-registered public offering. The following year, she asked her registered representative for a quote. When told there is no current quote available, you would gather that this is

an untraded REIT.

All U.S. exchange-listed foreign currency options

are settled in cash (U.S. dollars). expire on the third Friday of the expiration month.

Which of the following securities have the most interest-rate risk? a. 2X short-term bond leveraged ETFs b. 2X long-term bond leveraged ETFs c. Long-term bond leveraged ETFs d. Short-term bond leveraged ETFs

b. 2X long-term bond leveraged ETFs Explanation: An essential bond concept is that long-term bonds have a higher degree of interest-rate risk than short-term bonds. A leveraged ETF seeks to deliver a multiple of the performance of an index or other benchmark. Therefore, a 2X long-term bond leveraged ETF will have the highest degree of interest-rate risk of the answer choices.

Which of the following bonds has the most interest-rate risk? a. A 3-month Treasury bill b. A 30-year Treasury STRIP c. A 6%-coupon, 30-year Treasury bond d. A 3%-coupon, 5-year Treasury note

b. A 30-year Treasury STRIP Explanation: The bond with the most interest-rate risk or price volatility is the one with the longest maturity and lowest coupon. Zero-coupon bonds would have the most interest-rate risk and a STRIP is a type of zero-coupon bond.

An investor with an investment objective of speculation wants to purchase a security that will increase three times as much the Russell 2000 Index. Which of the following securities would you recommend? a. An inverse exchange-traded fund (ETF) b. A leveraged exchange-traded fund (ETF) c. A leveraged inverse exchange-traded fund (ETF) d. An exchange-traded fund (ETF)

b. A leveraged exchange-traded fund (ETF) Explanation: A leveraged ETF is designed to deliver a multiple of the performance of an index or other benchmark. For example, a 3X leveraged ETF based on the Russell 2000 Index seeks to deliver three times the performance of that index. So, if the Russell 2000 Index rises by 1%, a leveraged ETF would increase by 3% before fees and expenses. Choice (a) would be suitable if the customer anticipated a decrease in the Russell 2000, choice (c) would be suitable if the customer wanted a return that was a multiple or higher return and anticipated a decrease in the Russell 2000, and choice (d) would be suitable if the customer only wanted to track the return of the Russell 2000.

An investment adviser opens an account for one of his clients. The registered representative opening the account would NOT need which of the following items? a. Written authorization from the client b. A listing of all the investment adviser's clients c. The client's name d. The client's Social Security number

b. A listing of all the investment adviser's clients Explanation: Since the account is being opened for the investment adviser's client, the registered representative would require the client's name and Social Security number. Since a person other than the account holder (the adviser) will be entering orders, the registered representative would require written authorization from the client granting the adviser the right to enter orders. The client list of the investment adviser is not required by the registered representative.

A person maintains an IRA account and has contributed both pretax and after-tax dollars. Withdrawals from this account will be treated as which of the following? a. A return of capital b. A portion will be taxable as ordinary income and a portion will be tax-free c. A portion will be taxable as a capital gain and a portion will be tax-free d. The entire withdrawal will be taxable as ordinary income

b. A portion will be taxable as ordinary income and a portion will be tax-free Explanation: The earnings on an IRA account grow tax-deferred. If an investor maintains an IRA account that has pretax and after-tax contributions and makes withdrawals, the IRS considers withdrawals to come from both sources. Therefore, a portion of the withdrawal is taxable and the other portion is tax-free. If only pretax contributions are made, the entire withdrawal will be taxable as ordinary income. Withdrawals from a tax-deferred account are never taxable as a capital gain.

According to MSRB rules, which of the following documents need not be approved by a principal prior to being sent to a customer? a. An abstract of an official statement b. A preliminary official statement c. An advertisement regarding the firm's products and services d. A research report

b. A preliminary official statement Explanation: A preliminary official statement is prepared by or for the issuer. Since the MSRB does not have the power to regulate issuers, a preliminary official statement cannot be considered advertising under MSRB rules. However, an abstract (summary) of the official statement is prepared by a dealer and is, therefore, considered advertising. A final official statement and a firm's offering list are also not considered advertising.

A registered representative opens an option account for a customer on October 1 and buys 5 ABC November 30 calls at 4. On October 16, the premium of the calls has decreased to 2 and the registered representative has not received a signed options agreement. The registered representative may: a. Not accept any orders from the customer until the signed options agreement is received b. Accept an order to sell the 5 ABC November 30 calls that were previously purchased c. Accept an order to buy 5 additional ABC November 30 calls d. Accept an order to buy 5 XYZ December 40 calls

b. Accept an order to sell the 5 ABC November 30 calls that were previously purchased Explanation: If the customer does not return the options agreement within 15 days of the approval of the account, the customer is permitted only to close out existing positions. Since the account was approved on October 1, the customer must sign the options agreement and return it to the firm by October 16. As of October 16, the customer may only open new options positions after the signed form is returned to the firm.

Government-sponsored enterprise securities are comparable to direct government obligations with regard to all of the following statements, EXCEPT: a. They trade in the over-the-counter market b. All are government guaranteed c. Short-term securities are quoted on a discount yield d. Long-term securities are quoted as a percentage of par

b. All are government guaranteed Explanation: Government-sponsored enterprise securities are not guaranteed by the government. The other statements are true.

A client wants to invest $250 a month and have broad exposure to the U.S. equity market. Which of the following recommendations is the most suitable for this client? a. A managed closed-end fund b. An S&P 500 Index mutual fund c. An S&P 500 Index exchange-traded fund d. An DJIA exchange-traded fund

b. An S&P 500 Index mutual fund Explanation: Although all of these investments are suitable for a client seeking broad exposure to the U.S. equity market, the mutual fund is the most cost-effective method for an investor to accomplish this goal with $250 per month. The closed-end fund and ETFs are purchased on an exchange and the client pays the current market price plus a commission. Most index mutual funds do not charge the client a sales charge (no-load). If the investor were to purchase a large dollar amount at one time, any of these funds may be appropriate.

An investor with an investment objective of tax-exempt income will need access to the funds in four months. An RR should NOT recommend which of the following municipal securities? a. A variable-rate demand obligation (VRDO) b. An auction-rate security (ARS) c. A tax-anticipation note (TAN) d. A bond anticipation note (BAN)

b. An auction-rate security (ARS) Explanation: A VRDO and an ARS are both long-term securities with short-term trading features. A VRDO has a put feature that permits the holder to sell the securities back to the issuer or third party. An auction rate security (ARS) does not have this feature and, if the auction fails, the investor may not have immediate access to his funds. TANs and BANs are short-term municipal notes and, if their maturities extend four months, these securities can easily be sold in the secondary market.

Suitability in variable annuity transactions does not apply in which of the following situations? a. A purchase of a new variable annuity b. An employee's contribution to his 403(b) plan c. A 1035 exchange from one variable annuity to another d. The allocation of funds to the subaccount products within a variable annuity

b. An employee's contribution to his 403(b) plan Explanation: FINRA focuses on the suitability of annuity transactions in the purchase of new contracts, exchanges, and the allocation of funds to the subaccount products within the annuity. Annuity transactions in tax-qualified, employer-sponsored annuity programs (e.g., 403(b) plans) is not subject to FINRA's rules. However, the allocation of funds to the various subaccount products within the qualified plans is covered.

A registered representative has a 33-year-old client with a stable income with no foreseeable need to access money. The client is looking for a long-term investment that will offer a guaranteed rate of return, that can also share in the performance of the stock market, and offers some form of death benefit. Which of the following investments is MOST suitable for this client? a. A fixed annuity b. An equity-indexed annuity c. A variable annuity d. A varialble life insurance policy

b. An equity-indexed annuity Explanation: An equity-indexed annuity will satisfy the objectives of this client. It is a hybrid investment which offers the benefits of a fixed annuity -- guaranteed rate of growth -- as well as those of a variable annuity -- growth potential in the market. These, like most annuities, are not designed as short-term investments. The variable life insurance policy is designed to provide death benefits that can increase because of growth in the market.

An investor is looking for a fund that, with little risk to her principal investment, will supplement her current wages. Which of the following funds best suits this investor? a. A growth fund b. An income fund c. A sector fund d. A no-load fund

b. An income fund Explanation: A mutual fund investor most interested in current yield (i.e., regular dividend checks) as an investment objective will most likely purchase an income fund. A growth fund invests in companies that are growing rapidly and pay out a small percentage of earnings in dividends. Investors seeking capital gains will most likely purchase a growth fund. A no-load fund is an open-end investment company that does not have a sales charge and whose investment objectives may be income or capital gains. A sector fund is a mutual fund that invests primarily in a particular industry or geographical area, such as the energy or high technology industries.

Which of the following persons is normally compensated by receiving a fee based on a percentage of the assets under management? a. A broker's broker b. An investment adviser c. A designated market maker d. An order book official

b. An investment adviser Explanation: Investment advisers are often compensated based on a percentage of assets under management. For example, mutual fund managers are usually compensated in this way.

A registered representative has established a discretionary municipal bond account for a customer. Which of the following bonds may the RR purchase for the account? a. General obligation bonds only b. Any suitable bonds c. Bonds rated Baa or higher d. Any bond that is not subject to the alternative minimum tax

b. Any suitable bonds Explanation: An RR must be sure to recommend only bonds that are suitable for a customer in any type of account.

A mutual fund shareholder is NOT required to report which of the following events for tax purposes? a. Receiving a dividend that is subsequently reinvested in the fund at the net asset value b. Appreciation in the value of the shares c. Exchanging shares of one fund for another fund within the same family of funds d. Receiving a capital gains distribution that was not reinvested in the fund

b. Appreciation in the value of the shares Explanation: Dividends and capital gains distributions are taxable to the investor regardless of whether they are reinvested in the fund. Exchanging shares for another fund within the same family of funds must also be reported on the investor's tax return since shares of one fund are being sold to buy shares in another fund. Appreciation in the value of fund shares is not taxable until the shares are sold to establish a capital gain.

An investor who sells a July 50 put and buys a July 60 put on the same stock is establishing a: a. Bull spread b. Bear spread c. Long straddle d. Short straddle

b. Bear spread Explanation: A bear spread always involves buying the higher exercise price and selling the lower exercise price. This applies to both call spreads and put spreads. A bull spread always involves buying the lower exercise price and selling the higher exercise price. This applies to both call spreads and put spreads.

Joseph Carlyle is a customer of a municipal securities firm. Based on his existing account documentation, he is clearly unsuitable for securities with a speculative credit rating. However, he has entered an order to purchase a bond that is rated BB by Standard and Poor's. His representative, Bob Thomas, has communicated to him that this transaction is not in his best interest based on the information that the firm has on file. Regarding this situation, which of the following statements is TRUE? a. Bob should process the order because a BB rating is not speculative b. Bob should process the order because MSRB rules allow the order to be filled if the representative explains to Joseph that the trade is unsuitable c. Bob should process the order because registered representative are not fiduciaries and, therefore, must always do what the customer says d. Bob should not process the order because MSRB rules prohibit the processing of a clearly unsuitable transaction

b. Bob should process the order because MSRB rules allow the order to be filled if the representative explains to Joseph that the trade is unsuitable Explanation: According to recent interpretations of the MSRB's suitability rule, Bob should process the customer's order as long as he takes the time to explain to Mr. Carlyle why he believes the investment is unsuitable for him.

Someone who wishes to hedge a portfolio of preferred stocks will: a. Buy yield-based puts b. Buy yield-based calls c. Write a yield-based straddle d. Write a yield-based combination

b. Buy yield-based calls Explanation: The prices of preferred stocks are inversely related to the movement of interest rates, as are bonds. Therefore, if the investor is concerned that rising interest rates will erode the value of the preferred stock portfolio, the purchase of an option that does well when interest rates rise will provide an effective hedge. Yield-based calls (which are yield-based options) increase in value when interest rates rise, also creating a viable hedge.

Class A shares of an open-end investment company are different from Class B shares in that: a. Class A shares are common shares, while Class B shares are preferred shares b. Class A shares have a front-end sales charge, while Class B shares have a contingent deferred sales charge c. Class A shares pay quarterly dividends, while Class B shares pay a monthly dividend d. Class A shares can be purchased directly from the fund, while Class B shares are offered through broker-dealers

b. Class A shares have a front-end sales charge, while Class B shares have a contingent deferred sales charge Explanation: The difference between Class A and Class B shares is normally the fact that A shares have a front-end sales charge while the B shares have a contingent deferred sales charge (CDSC). A CDSC is deducted only when the investor redeems shares. Generally, if the investor holds the B shares for a sufficient period, there is no sales charge deducted upon redemption.

An RR receives unsubstantiated information that a biotech company trading on Nasdaq has been approached by a large pharmaceutical company as a possible acquisition. The news seems to indicate that the price of the biotech company will increase greatly in value. Which of the following choices is the most inappropriate action for the RR to take? a. Take no action b. Contact clients and indicate that this biotech company is heading upward and that they should place a buy order c. Wait for the news to break and then see if the stock is worth buying d. Contact your firm's compliance department

b. Contact clients and indicate that this biotech company is heading upward and that they should place a buy order Explanation: If an RR receives unsubstantiated news (possibly a rumor), he should take no action until the information becomes public. At that time, the RR will be in a better position to be able to recommend whether clients should purchase the stock. If he received material, nonpublic information on a company, the best course of action is for the RR to contact a principal or compliance person associated with his firm. Please note that this question is asking for the most inappropriate action for the RR to take making the best answer, choice (b).

Which of the following option strategies is most suitable in a retirement account under ERISA guidelines? a. A long straddle b. Covered call writing c. A credit spread d. No strategies, since options are not permitted in retirement plans

b. Covered call writing Explanation: Options may be used in the investment strategies of retirement plans subject to ERISA provisions, but they tend to be limited to the most conservative approaches, such as the sale of covered call options.

A retail investor usually is NOT able to purchase: a. Money-market fund shares b. Dealer-placed commercial paper c. Municipal bonds d. Treasury STRIPS

b. Dealer-placed commercial paper Explanation: The minimum requirement for an investment in dealer-placed commercial paper is normally $100,000. Commercial paper is not typically purchased by retail investors. The minimum requirement for investment in Treasury STRIPS is usually $100 and the minimum denomination for municipal bonds is $1,000.

Which of the following information does NOT have an effect on the credit quality of an airport revenue bond? a. Tourism b. Debt per capita c. Airport traffic d. Energy costs

b. Debt per capita Explanation: Debt per capita is used when analyzing a general obligation bond and would not be considered for a revenue issue.

Which of the following deductions applies to oil and gas programs, but not to real estate programs. a. Depreciation b. Depletion c. Recapture d. Tax credits

b. Depletion Explanation: Depletion is a deduction available to programs that extract oil and gas and other types of wasting assets. It does not apply to real estate programs.

Which of the following statements is TRUE concerning a customer who purchases an original issue discount (OID) U.S. government security? a. Each year the customer will pay both federal and state income tax b. Each year the customer will pay only federal income tax c. Each year the customer will not pay any tax d. The customer will only pay tax at maturity

b. Each year the customer will pay only federal income tax Explanation: The upward adjustment in the purchase price of an original issue discount bond is called accretion. The amounted accreted each year is considered interest income, which may or may not be taxable depending on the type of security. The interest on U.S. government securities is subject to federal income tax, but exempt from state and local income taxes. Technically should be c. because OID's are for municipal bonds

A company has chosen accelerated depreciation instead of straight-line depreciation. Which of the following statements is TRUE? a. Earnings are overstated in the early years and understated in later years b. Earnings are understated in the early years and overstated in later years c. Earnings are understated in both early and later years d. Earnings are not impacted by the method of depreciation

b. Earnings are understated in the early years and overstated in later years Explanation: Accelerated depreciation allows a company to take a larger amount of the cost of an asset as a deduction in the early years and less in the later years. Since large deductions are taken, earnings will be understated (reduced) in the early years. Small deductions in later years will overstate earnings.

Which of the following features is NOT a benefit of a laddered portfolio of bonds? a. A higher average yield b. Elimination of interest-rate risk c. Improved liquidity d. Lower reinvestment risk

b. Elimination of interest-rate risk Explanation: A laddered portfolio is created by investing in the same type of bonds with different maturity dates. This approach reduces, but does not eliminate, interest-rate risk. Some of the benefits of laddering include a higher average yield since there is a mix of both short-term and long-term bonds, improved liquidity since bonds will be maturing on a regular basis, lower reinvestment risk since proceeds will be reinvested each year rather than only reinvesting when all of the bonds mature.

Rule 144 allows:

the greater of 1% of the outstanding shares or the weekly average of the preceding 4 weeks trading volume every 90 days. 1% of the share amount

Which of the following choices is NOT a typical characteristic of a 401(k) plan? a. Employee contributions are fully and immediately vested b. Employers must match employee contributions c. An employee's taxable income is reduced by employee contributions d. Employee contributions grow on a tax-deferred basis

b. Employers must match employee contributions Explanation: In a 401(k) plan, an employee can usually make a pretax contribution in the plan and reduce taxable income. Employee contributions and growth in the account are tax-deferred. Employers are not required to match contributions, but may do so.

If a bank holds funds above the amount required by the Federal Reserve Board, this is called: a. The federal funds rate b. Excess reserves c. The repo rate d. Excess margin

b. Excess reserves Explanation: Banks are required to keep a portion of their deposits on reserve with the FRB. By adjusting the amount banks must keep on reserve at the FRB, the Fed can tighten or ease the money supply. If reserve requirements are lowered, the banks are able to extend more credit. Thus, the money supply increases. The opposite effect occurs with an increase in reserve requirements. After meeting its reserve requirement, a bank will look to lend the remaining funds to borrowers. The amount above the reserve requirement is called excess reserves.

In order to implement a portfolio margin program, the firm must obtain approval from: a. The options exchange b. FINRA c. The SEC d. The OCC

b. FINRA Explanation: Firms establishing a portfolio margin program are required to obtain approval from FINRA.

If an issue of commercial paper is rated P-1 by Moody's, it is considered: a. Speculative b. Highest quality c. Intermediate quality d. On credit watch

b. Highest quality Explanation: P-1 (also called Prime 1) is the highest rating that Moody's will assign to commercial paper. Intermediate ratings are P-2 and P-3. Speculative commercial paper would receive a rating of NP (not prime).

According to CAPM, all of the following choices are examples of diversifiable, nonsystematic risk, EXCEPT: a. Credit risk b. Interest-rate risk c. Business risk d. Industry risk

b. Interest-rate risk Explanation: Interest-rate risk is the systematic risk for bonds just as beta measures the systematic risk for stocks. Systematic risk is market risk, which persists despite diversification.

Which of the following statements is TRUE regarding the Interbank market? a. It is the market for fed funds between banks b. It helps establish the spot prices for foreign currencies c. It administers loans to foreign countries d. It is the guarantor of foreign currency options

b. It helps establish the spot prices for foreign currencies Explanation: The Interbank market is the purchase and sale of foreign currencies among large banks. The market helps establish the cash (spot) prices for foreign currencies. Spot prices are often referred to as the spot rate.

Which of the following statements is TRUE about treasury stock? a. It receives dividends b. It is treated as a deduction from outstanding shares c. It has voting power d. It is part of unauthorized stock

b. It is treated as a deduction from outstanding shares Explanation: Treasury stock is issued stock that has been repurchased by the corporation and is retired. It is treated as a deduction from the outstanding shares of a corporation and is no longer part of the capitalization of the corporation. It has no voting rights and does not receive dividends.

JULY 20XX S M T W T F S 1 2 3 4 5 6 7 8 9 10 11 12 13 14 15 16 17 18 19 20 21 22 23 24 25 26 27 28 29 30 31 Use the calendar to answer this question. If an investor bought a stock in a cash account on July 2, when will the settlement date be in a regular-way transaction? a. July 5 b. July 8 c. July 9 d. July 12

b. July 8 Explanation: July 8 is the settlement date. This is three business days after the trade date in a regular-way transaction in a cash or margin account. Thursday, July 4 is not a business day. It is a legal holiday and is not counted. The third business day after the trade date of July 2 is July 8.

Which of the following option orders may be accepted by an order book official? a. Discretionary b. Limit c. Spread d. Not-held

b. Limit Explanation: An order book official on the floor of an options exchange is permitted to accept limit orders only.

If a customer is short RST call options, what other position would be considered when examining position limits? a. Long RST calls b. Long RST puts c. Short RST puts d. Long ABC puts

b. Long RST puts Explanation: If the customer is short RST calls, he anticipates that the market price of RST stock will decline. Since he is bearish on the stock, he could also be long puts on RST. This is considered on the same side of the market.

Which of the following choices is Moody's lowest rating for a municipal note? a. MIG 1 b. MIG 3 c. Aaa d. C

b. MIG 3 Explanation: MIG stands for Moody's Investment Grade and is used to rate municipal notes. There are three MIG ratings, with the best rating being MIG 1 and the lowest rating being MIG 3. Aaa is Moody's best rating for bonds, and C is its lowest rating for bonds.

A customer contacts a registered representative and indicates her risk tolerance is to accept some risk to her initial principal in exchange for higher returns. The RR asks the customer if she understands that the account may lose value but may keep pace with or exceed inflation, and the customer agrees to these conditions. This customer's risk tolerance would BEST be defined as: a. Conservative b. Moderate c. Moderate conservative d. Moderate aggressive

b. Moderate Explanation: An investment risk tolerance in which the customer is willing to accept some risk to her initial principal, with some volatility and a possible loss of the funds invested in exchange for higher returns, is best defined as moderate. Moderate conservative includes low risk with an understanding there may be some volatility in exchange for a small amount of portfolio returns. Moderate or medium aggressive is a situation where the customer is willing to accept high risk and high volatility with a possible loss to her initial principal in exchange for high returns.

What type of options will be used to hedge a portfolio of computer stocks? a. Interest-rate options b. Narrow-based index options c. Broad-based index options d. Yield-based options

b. Narrow-based index options Explanation: A portfolio containing only computer stocks represents just one segment of the market. A narrow-based index also contains stocks from only one segment of the market.

An investor begins a periodic payment deferred variable annuity purchase program. One respect in which this differs from purchasing a mutual fund is that

the investor in the variable annuity contract reports no taxable consequences during the accumulation period.

Which of the following choices is the formula for earnings per share? a. Current market price/ Earning per share of common stock b. Net income less preferred dividends/Number of shares of common stock outstanding c. Earnings before interest and taxes/ Net tangible assets d. Net income plus preferred dividends/ Number of shares of common stock outstanding

b. Net income less preferred dividends/Number of shares of common stock outstanding Explanation: The formula for earnings per share is net income less preferred dividends divided by the number of shares of common stock outstanding.

Before accepting a DVP order from a customer, a broker-dealer must: a. Notify FINRA b. Obtain the name of the customer's agent from the customer c. Receive approval for the trade from the contrabroker d. Notify the appropriate banking regulator

b. Obtain the name of the customer's agent from the customer Explanation: Before accepting a DVP (delivery versus payment) or RVP (receipt versus payment) order from a customer, a broker-dealer must receive the name of the customer's agent and the customer's account number. The order ticket must be marked DVP or RVP.

An investor has been following XYZ Corp. for several years and feels that the company is poised for some very profitable years. Since she wants to purchase a security that offers a consistent annual distribution and one that benefits from XYZ deciding to pay a significant cash dividend to its common stockholders, she should consider purchasing: a. Cumulative preferred stock b. Participating preferred stock c. Collateral secured bond d. Common stock

b. Participating preferred stock Explanation: Participating preferred stock allows the owners to share in the extraordinary earnings of a company. Essentially, participating preferred has a stated dividend, but these shareholders may receive more than that amount based on the profits of the issuing company. Cumulative preferred stock will add all unpaid dividends to a future payment if a cash dividend is to be paid to common shareholders. A collateral secured bond provides the holder with safety due to it being backed by a specific asset of the issuer; however, the issuer will pay no more than the bond's stated rate of interest. Common stock will pay only cash dividends if they are declared by the company s board of directors.

One of your clients anticipates a significant decline in XYZ stock. The client wants to establish a position to take advantage of a large decline, but not expose himself to significant risk. Which of the following actions best satisfies your client's needs? a. Short XYZ stock b. Purchase an XYZ put c. Purchase an XYZ straddle d. Establish an XYZ debit put spread

b. Purchase an XYZ put Explanation: A long put will allow your client to realize a gain determined by the amount the stock falls below the option's strike price, less the premium. The investor is at risk only for the amount paid for the put, i.e., the premium. In selling XYZ short, an investor exposes himself to unlimited risk. When purchasing a straddle, the investor pays a premium greater than when purchasing only one put on the stock. While the debit put spread is bearish, the gain is limited to the difference between the strike price on the long put and the strike price on the short put, less the net premium.

A customer has a nondiscretionary account at a broker-dealer. The customer received a research report and instructs the registered representative to purchase 500 shares of a specific stock on the recommended list. Which of the following actions is MOST appropriate for the registered representative to take? a. Contact the customer and ask her to place a limit order to buy the security b. Purchase the stock no later than the end of that business day c. Purchase the stock any day that you think is best d. Have the order preapproved by a principal and then purchase the stock

b. Purchase the stock no later than the end of that business day Explanation: This is a nondiscretionary account and, therefore, no shares may be purchased unless the customer gives the broker-dealer an order to purchase the security. In some cases, a registered representative may accept the customer's verbal authorization to make certain decisions without it being considered discretionary. If a customer (1) selects the specific security, (2) decides whether to buy or sell the security, and (3) specifies the number of shares, leaving discretion only as to time and/or price, it would not be considered a discretionary order and written authorization would not be required. The customer mentioned all three of these details. This time and price discretion concerning the order is limited to the trading day on the day the order was placed, and must be noted on the order ticket. The client is permitted to give her RR written instructions for a longer period. There is no requirement to have the order preapproved by a principal.

All of the following choices are benefits of a limited partnership, EXCEPT: a. Limited liability b. Recapture c. Flow-through of income and expense d. Tax credits

b. Recapture Explanation: Recapture is a situation where tax benefits previously taken should be paid back to the government. This is obviously not a benefit of a limited partnership.

Which of the following securities would be LEAST suitable for an investor interested in preservation of capital? a. Long-term CDs b. Reverse convertible bonds c. A corporate bond fund d. A floating rate bond maturing in five years

b. Reverse convertible bonds Explanation: Reverse convertible securities would not be suitable for an investor interested in preservation of capital. Reverse convertible securities are short-term notes issued by banks and broker-dealers that usually pay a coupon rate above prevailing market rates. They are considered structured products because, in addition to the coupon rate, the investor may be required to purchase shares of an underlying asset at a fixed price. The underlying asset may be an equity security unrelated to the issuer, or a basket of stock, or an index. The issuer agrees to pay this higher coupon rate since it has an option to sell a security to the investor if the price of the security falls below a specified value known as the knock-in level. If the price of the underlying asset stays above the knock-in level, the investor will receive the high coupon and the full return of his principal (the most beneficial option). If the underlying asset falls below the knock-in level, the investor will be obligated to purchase shares of the underlying asset at a fixed price. The price of this asset may have depreciated below the knock-in level and the investor may receive substantially less than the original principal.

Which of the following items is NOT found on a sell ticket? a. The customer's account number b. The customer's original purchase price of the stock c. The location of the securities d. Solicited or unsolicited

b. The customer's original purchase price of the stock Explanation: All order tickets must contain the customer's account number and whether the registered representative solicited the order or it was unsolicited. A sell ticket must indicate if it is a short sale or a sale of securities owned by the client. The location of the securities must be indicated (long in the customer's account or held by the customer).

FINRA disseminates bond transaction information for all these securities, EXCEPT: a. Non-investment-grade corporate bonds b. Rule 144A securities c. Investment-grade corporate bonds d. GSE bonds

b. Rule 144A securities Explanation: TRACE is a reporting system that was created to provide greater transparency in the corporate bond market. It is not a quotation system or an execution system. Broker-dealers provide quotes and will execute transactions in corporate bonds. There is no regulatory quote or execution system as there is for equity securities. FINRA disseminates bond transaction information for publicly traded, TRACE-eligible securities (which include investment-grade and non-investment-grade bonds, and debt securities issued by a government-sponsored enterprise). Although transactions for securities issued under Rule 144A are reported to TRACE, the information is not disseminated.

According to MSRB rules, a municipal securities representative is NOT permitted to: a. Trade securities b. Supervise a branch office c. Structure new issues of revenue bonds d. Work in the syndicate department

b. Supervise a branch office Explanation: Under MSRB rules, any person in a supervisory position must qualify as a principal.

A limited partner has contributed capital to a direct participation program. Two years later, he extends a loan. Which of the following statements is TRUE if the DPP declares bankruptcy? a. The LP is considered a limited partner for both the capital contribution and the loan b. The LP is considered a limited partner for the capital contribution and a creditor for the loan c. The LP is considered a creditor for the capital contribution and a limited partner for the loan d. The LP is considered a creditor for both the capital contribution and the loan

b. The LP is considered a limited partner for the capital contribution and a creditor for the loan Explanation: A limited partner who has committed capital may also extend a loan to the partnership. If the partnership declares bankruptcy, the LP will be considered a limited partner for the capital contribution and a creditor for the amount of the loan.

Which of the following items is NOT found by reviewing a company's balance sheet? a. The dollar value of the inventory b. The amount of interest paid on the company's bonds outstanding c. The amount of short-term debt d. The value of the treasury stock

b. The amount of interest paid on the company's bonds outstanding Explanation: The amount of interest paid on the company's bonds outstanding (interest expense) is found in a company's income statement. A company's assets (inventory), liabilities (debt or bonds), and shareholders' equity (treasury stock), are found on the balance sheet.

Which of the following indicators is bullish? a. A breakout below a support level b. The bottom of a saucer pattern c. The top of an inverse saucer pattern d. A decrease in the amount of short interest

b. The bottom of a saucer pattern Explanation: A saucer is a chart pattern used by technical analysts that indicates that a stock has formed a bottom in its trading cycle and is ready to rise. The bottom of the saucer pattern is a bullish indicator for the stock. The reverse of the saucer pattern is the inverse saucer, where the stock forms a top in its pattern and is expected to fall. Following the logic used in the saucer, this is a bearish indicator. A breakout below the support level is a bearish signal. The term short interest refers to the amount of a company's shares of common stock that have been sold short and have not yet been covered (closed out). An increase (not decrease) in short interest has historically been considered a bullish indicator by a technical analyst.

Which one of the following events will NOT result in a profit to an uncovered call writer? a. The price of the underlying security falls below and remains below the exercise price of the option b. The call is exercised and the underlying security price is greater than the exercise price plus the premium received c. The price of the option contract declines d. The option contract expires without being exercised

b. The call is exercised and the underlying security price is greater than the exercise price plus the premium received Explanation: An uncovered call writer does not own the underlying stock. If the market price of the underlying stock rises above the exercise price, the stock will be called away. If the market price rises above the exercise price by an amount exceeding the premium, the difference in prices will represent the loss to the writer. For example, if an individual writes 1 XYZ July 50 call for 5 and the market price rises to 60, the stock will be called away. The writer will be required to buy the stock at 60. Since the investor received only 55 (exercise price of 50 plus premium of 5), there will be a 5-point loss.

Which of the following parties is responsible for the safekeeping of the securities owned by a mutual fund? a. The registrar b. The custodian bank c. The sponsor d The transfer agent

b. The custodian bank Explanation: The custodian bank is responsible for the safekeeping of the securities owned by a mutual fund. The custodian bank has no responsibility relating to the management of the fund's portfolio.

A registered representative is permitted to borrow from, or lend funds to, customers in all of the following situations, EXCEPT: a. The customer is an immediate family member of the registered representative b. The customer is an accredited investor c. The loan is based on a business relationship independent from the member firm customer relationship d. The customer has a personal relationship with the registered representative

b. The customer is an accredited investor Explanation: Registered personnel of a member firm are generally prohibited from borrowing money from, or lending money to, any customer. However, if the firm has written rules and procedures that allow borrowing and lending between registered personnel and their customers that meet one of the following conditions, the activities are permissible. - The customer is an immediate family member of the registered person. - The customer is a financial institution or other entity that engages in the business of providing credit, financing, or loans in the course of its business. - The customer and the registered person are both registered with the same member firm. - The customer has a personal relationship with the registered person wherein the loan would not have been solicited, offered, or given if the relationship did not exist. - The loan is based on a business relationship other than that of a member firm customer. There is no exception based on the type of customer, for example, an accredited investor.

A municipal bond is currently trading at 92 and is callable in 10 years at par. What is the effective yield that must be disclosed on a customer's confirmation? a. Yield to call b. Yield to maturity c. Fixed yield d. Current yield

b. Yield to maturity Explanation: The MSRB regulates the yield that must be disclosed on a client's confirmation. The yield disclosed is the lower of the yield to maturity or yield to call. In other words, the yield to worst. If a bond is callable and trading at a discount, the lower of the two would be the yield to maturity.

As it relates to a Nasdaq market maker, the term spread is BEST defined as which of the following? a. The difference between the price that an issuer will receive for its securities and the price that the public will pay for the securities b. The difference between the price at which a firm will buy a security and the price at which it will sell a security c. The amount of profit that a firm will make when it buys a security from or sells a security to a customer d. The amount of the firm's markup or markdown to a customer who buys or sells a security

b. The difference between the price at which a firm will buy a security and the price at which it will sell a security Explanation: When used in reference to a Nasdaq market maker, the spread represents the difference between the price at which the firm is willing to buy (bid) and the price at which the firm is willing to sell (ask or offer) a security. For example, if the bid price is $21.20 per share and the offer price is $21.30 per share, the market maker's spread is $.10. Choice (a) is the spread or profit that an underwriter makes when it sells an IPO to a customer. Choice (c) is a market maker's profit based on its inventory cost for a security (i.e., the price at which it purchased a security from a customer compared to the price at which it was sold to a different customer. The markup or markdown is the difference between the prices the customer paid or received compared to the best bid or offer price of all Nasdaq market makers (the inside market). For example, if the inside market is $25.50 - $25.70 and the customer paid $25.90 to purchase the stock, the markup is $.20

When engaging in a 1035 exchange an individual should be aware that: a. The exchange is a taxable event b. The exchange is not a taxable event but the new annuity may come with additional restrictions c. The exchange is not a taxable event and the policies of the old annuity are remain in place d. The exchange is only permitted if it is unsolicited

b. The exchange is not a taxable event but the new annuity may come with additional restrictions Explanation: The primary benefit of a 1035 exchange is that it is not taxable. However, the new annuity may come with new restrictions making it unsuitable for the investor.

Which of the following situations requires a DK (don't know) notice? a. The firms do not have a clearing agreement b. The firms disagree on the amount of shares in the trade c. One of the firms is not a market maker d. The trade is executed on an ECN

b. The firms disagree on the amount of shares in the trade Explanation: A DK notice is used for a don't know trade. This occurs when one side does not recognize the trade or the firms disagree on the details. Choice (b) is the only situation that would require a DK notice.

Ted Wilson, a registered representative with Ralston Financial, has recommended to his client the purchase of a mutual fund as well as a variable annuity contract underwritten by the same sponsor, Slipstream Growth Fund and Slipstream Variable Annuity. The client has decided to purchase both investments, filling out the applications and forwarding them with one payment for both. Which of the following statements is TRUE regarding the purchases? a. The mutual fund and variable annuity may be purchased without a principal's approval, if suitable b. The growth fund may be purchased immediately but the variable annuity purchase must first be approved by a principal c. The variable annuity may be purchased immediately but the mutual fund must first be approved by a principal d. Both the mutual fund and the variable annuity purchases must be approved by a principal prior to initiating the purchases

b. The growth fund may be purchased immediately but the variable annuity purchase must first be approved by a principal Explanation: A lump-sum payment may be received for both purchases. The money for the variable annuity may be allocated only after a principal has approved the purchase in writing. The money for the mutual fund may be allocated immediately and does not require a principal's approval.

All of the following statements are TRUE concerning marketwide circuit breakers, EXCEPT: a. They are based on the S&P 500 Index b. The levels are calculated on a monthly basis c. A trading halt on one exchange applies to all exchanges d. A 7% decline will halt trading for 15 minutes

b. The levels are calculated on a monthly basis Explanation: Marketwide trading halts are based on the S&P 500 Index and are calculated daily (not monthly). A trading halt on one exchange applies to all exchanges that trade the same security. A Level 1 Market Decline (7%) and a Level 2 Market Decline (13%) will halt trading for 15 minutes. For a Level 3 Market Decline (20%), trading will be halted for the remainder of the day.

Which of the following statements about municipal revenue bonds is NOT TRUE? a. There is no debt limitation set by the issuing municipality b. The maturity of the revenue bond usually coincides with the useful life of the facility being built c. They can be issued by states, political subdivisions, interstate authorities, and intrastate authorities d. The interest and principal are paid from the revenue received from the facility

b. The maturity of the revenue bond usually coincides with the useful life of the facility being built Explanation: Municipal revenue bonds do not have maturity schedules that coincide with the usefulness of the facility being built. They mature prior to the useful life of the facility. Municipal revenue bonds do not have debt limitations as do general obligation bonds. A debt limitation is the statutory or constitutional maximum debt that an issuer may legally incur. Revenue bonds can be issued by states, political subdivisions (such as counties or townships), interstate authorities and intrastate authorities. The interest and principal are paid from the revenue received from the facility.

During a meeting with a customer away from the broker-dealer, a customer hands a registered representative a handwritten note that objects to the price she received when redeeming mutual fund shares. The customer believes she submitted her redemption request early enough to receive the NAV calculated the same day. Instead, she received the NAV calculated the next business day, which was lower. Which of the following statements is TRUE? a. Since the note was received away from the broker-dealer's office, it is not considered official business and it can be ignored b. The note is considered a complaint and the RR must give it to his supervisor c. The note is a complaint about an action in which the mutual fund is responsible and should be forwarded directly to the fund d. The RR is responsible for trying to satisfy the customer and should not attempt to make this the responsibility of the broker-dealer by giving the note to a supervisor

b. The note is considered a complaint and the RR must give it to his supervisor Explanation: FINRA defines a complaint as any written statement of a customer, or any person acting on behalf of a customer, alleging a grievance involving the activities of those persons under the control of the member in connection with the solicitation or execution of any transaction or the disposition of securities or funds of that customer. The note is considered a complaint and the RR must take it to his supervisor. The broker-dealer must keep the complaint in its records and also must record what, if anything, is done about it.

The purchaser of a variable life insurance policy bears which of the following risks? a. The death benefit may fall to zero due to poor market performance b. The policy may have no cash value if the separate account performance is negative c. The insurance company may increase the premiums if the investment performance of the separate account is poor d. The increasing cost of doing business may force the insurance company to raise expense charges against the separate account

b. The policy may have no cash value if the separate account performance is negative Explanation: The cash value of a variable life insurance policy increases or decreases in relation to the performance of the separate account. Poor performance could cause the cash value to decline to zero. Although the death benefit can also increase or decrease, it may never fall below a set minimum. The premiums for variable life policies are fixed for the life of the policy. An expense guarantee clause in life insurance contracts prevents the insurance company from raising expense charges for the administration of the policy.

An investor is short 100 shares of QRS stock at $25 per share, and sells one QRS July 25 put at 2. The investor will make money in all of the following situations, EXCEPT: a. The price of QRS stock remains at $25 per share b. The price of QRS stock rises to $30 per share c. QRS files for bankruptcy and the stock is now worthless d. The July 25 put expires worthless

b. The price of QRS stock rises to $30 per share Explanation: The investor is bearish on the stock, and has taken in additional income by selling a put. By selling the stock and the put, the investor has taken in a total of $27 per share. A profit will be realized as long as the stock price remains below $27 per share. But, since the hedge is limited to the amount of the premium, the investor's maximum loss is still unlimited.

Which of the following factors would be LEAST useful when analyzing the credit risk of an issuer of revenue bonds? a. Engineering reports b. The ratio of the amount of net overall debt to assessed valuation c. Debt service coverage ratio d. Special taxes

b. The ratio of the amount of net overall debt to assessed valuation Explanation: The ratio of the amount of net overall debt (both direct and overlapping) to assessed value is useful in analyzing the credit risk of an issuer of general obligation bonds. A special tax bond is a type of revenue bond and, therefore, the amount of special taxes may be useful in analyzing the credit risk of an issuer of a revenue bond.

Which of the following statements is TRUE concerning the sale of restricted securities? a. If the company is listed on Nasdaq, there is no holding period b. The sale must conform to the provisions of SEC Rule 144 c. A brokerage firm may act only in an agency capacity d. The sale must be at the bid price as determined by the current quote of the outstanding securities

b. The sale must conform to the provisions of SEC Rule 144 Explanation: The sale of restricted securities must conform to the provisions of SEC Rule 144. There is a six-month holding period even if the securities are listed on Nasdaq or the NYSE. A brokerage firm may act in an agency or principal capacity. The sale does not need to be at the bid price as determined by the current quote of the outstanding securities. The sale can be made at whatever price is agreed upon between the buyer and seller.

Which of the following statements is TRUE concerning Rule 144A transactions? a. The securities may be offered only to accredited investors b. The securities may be offered only to qualified institutional buyers c. An investor buying these securities must hold them for six months d. Only domestic issuers may offer securities under this type of offering

b. The securities may be offered only to qualified institutional buyers Explanation: Rule 144A provides an exemption for the purchase of restricted securities by qualified institutions. Qualified institutional buyers (QIBs) are defined as financial institutions that have at least $100 million invested in securities of issuers not affiliated with the entity. These institutions may buy and sell directly with one another without meeting the requirement of Rule 144. The securities offered under Rule 144A may be debt or equity, may be offered by either a domestic or foreign issuer, and may be resold immediately to another QIB. There is no 6-month holding period, as with restricted stock. A private placement under Regulation D may be offered to an unlimited number of accredited investors. An accredited investor is defined as a person with either a net worth of $1,000,000 or annual income of $200,000.

Relative to a convertible bond, which of the following choices will produce a desirable arbitrage situation? a. The stock is at parity with the bond b. The stock is at a premium to parity and the bond is trading at par c. The stock is at a discount to parity and the bond is trading at par d. The yield on the bond equals the yield on the stock

b. The stock is at a premium to parity and the bond is trading at par Explanation: An arbitrage situation occurs when there is a price difference in comparable securities. If the stock is selling above parity, the value of the stock received from converting the bond is more than the value of the bond. An investor could sell the stock short and buy the bond, and then convert the bond and use the stock to cover the short position. For example, a bond convertible into 25 shares is trading at 104 and the stock is selling at $42. If an investor sold 25 shares short at $42 (equaling $1,050), that would be worth more than the value of the bond ($1,040).

The penny stock rules would apply under which of the following circumstances? a. The stock is listed on Nasdaq b. The stock is quoted on the OTC Bulletin Board c. The transaction is not recommended by the broker-dealer d. The customer is an active trader in penny stocks

b. The stock is quoted on the OTC Bulletin Board Explanation: A penny stock, according to SEC rules, is a stock that sells for less than $5.00, that is not listed on Nasdaq or the NYSE. A stock quoted on the OTC Bulletin Board or OTC Pink Market (Pink Sheets) that has a bid price of less than $5.00 is defined as a penny stock. Penny stock rules would not apply under the following conditions. • The customer is defined as an existing customer, which is a person who has maintained an account with a broker-dealer for more than one year, or has previously engaged in 3 or more transactions involving penny stocks (i.e., an active trader of penny stocks) • In nonrecommended or unsolicited transactions • In transactions by a broker-dealer that is not a market maker in that security • In transactions by an institutional accredited investor

Why is the maturity of commercial paper 270 days or less? a. It coincides with the historical 9-month business cycle b. It is an attractive alternative to 6-month Treasury bills c. Short-term corporate debt of 270 days or less is exempt from registration d. It may be purchased by noninstitutional investors

c. Short-term corporate debt of 270 days or less is exempt from registration Explanation: Commercial paper has a maximum maturity of 270 days in order to be exempt from the registration requirements of the Securities Act of 1933. Most commercial paper is purchased by institutional investors.

Which of the following choices makes a financial commitment in the distribution of a new issue of securities? a. The selling group b. The underwriting syndicate c. A customer who provides an indication of interest d. The exchange on which the security will be listed

b. The underwriting syndicate Explanation: The underwriting syndicate makes a commitment to the issuer to purchase the entire offering. If the syndicate cannot resell the offering at the public offering price, it may suffer a loss. While the selling group also participates in the sale of the new issue, it does not run the risk of losses if the securities do not sell. Regarding choice (c), a customer who provides an indication of interest has no obligation of any kind

All of the following statements are TRUE regarding yield curves, EXCEPT: a. In an ascending curve, short-term rates are lower than long-term rates b. They are fixed and may only be changed by the Federal Reserve Board c. In a descending curve, short-term rates are greater than long-term rates d. In a flat yield curve, both short-term and long-term rates are equal

b. They are fixed and may only be changed by the Federal Reserve Board Explanation: Yield curves are ascending (upward sloping from the shorter to longer maturities) when money is easy. When this occurs, short-term rates are lower than long-term rates. A descending yield curve, which is indicative of a tight money situation, shows short-term rates higher than long-term rates. A flat yield curve indicates that short-term and long-term rates are approximately the same. FRB policies may influence yield curves, but they are not fixed and are influenced by a variety of factors.

Which of the following statements is TRUE concerning electronic communication networks (ECNs)? a. They can be used only by retail investors b. They can be used to obtain automatic execution c. They can be used only by institutional investors d. They can be used by clients that do not want to use a broker-dealer

b. They can be used to obtain automatic execution Explanation - Electronic communication networks (ECNs) are trading systems designed to match buyers with sellers of securities. They can be used by both institutional and retail investors. One of the benefits of their use is immediate automatic execution if a matching buy or sell order can be found on the system. ECNs do not allow investors to trade directly with one another, but allow subscribers such as broker-dealers to use these systems to execute the orders sent to them by their clients

Which of the following choices is NOT a factor in secondary-market municipal joint accounts? a. Members may not publish different offering prices b. They require a good faith deposit c. There may be an order period d. There may be a takedown

b. They require a good faith deposit Explanation: A good faith deposit is a sum of money given to the issuer of a new municipal bond issue along with a syndicate's bid and is not a factor in secondary-market transactions. A secondary-market joint account exists when two or more dealers form an account to jointly offer a block of bonds in the secondary market. As with a new issue, there may be an order period as well as a takedown (member's discount). MSRB rules prohibit members of the account from offering the bonds at different prices.

An investor who owns 1,000 shares of ABC informs you that he wants to sell short against the box. Which of the following statements is TRUE? a. This type of transaction is only permitted by institutional investors b. This type of transaction is permitted if the order ticket is marked short c. This type of transaction is permitted if the order ticket is marked long d. This type of transaction is only permitted in a cash account

b. This type of transaction is permitted if the order ticket is marked short Explanation: In certain instances, a client (institutional or retail) that is long a security may want to sell the stock, but not deliver his long position. The client must borrow the security to effect delivery, requiring the order ticket to be marked short. This type of transaction is called selling short against the box. The term box is an old industry term referring to a safe deposit box. Short sales are permitted to be executed only in a margin account.

Which of the following persons establishes positions in secondary market municipal bonds for a broker-dealer? a. Underwriter b. Trader c. Agent d. Principal

b. Trader Explanation: A trader is responsible for positioning (carrying inventory) secondary market municipal bonds. An underwriter is involved in the distribution of new issues.

Gross Domestic Product (GDP) has declined for two consecutive quarters in the U.S. Which of the following industries will most likely be negatively affected by this downturn in the economy? a. Cosmetics b. Transportation c. Food d. Medical

b. Transportation Explanation: Two consecutive quarters of declining GDP figures would be considered recessionary by most economists. Transportation stocks (e.g., railroads, trucking, airlines) are cyclical and the performance of these companies will be affected directly by this event.

As a retirement vehicle, which of the following choices would probably provide the greatest protection of purchasing power? a. Fixed annuities b. Variable annuities c. Corporate bonds d. Mortgage-backed securities

b. Variable annuities Explanation: Variable annuities, theoretically, provide the greatest protection against loss of purchasing power. The payout is based on the securities (mostly equity securities) in the separate account, which historically have increased in inflationary periods. This provides for a larger cash payout to offset the effects of inflation. The other choices given have a fixed payout and do not offer protection against the loss of purchasing power in inflationary periods.

Dividends paid by a corporation to its shareholders are NOT: a. Determined by a corporation's board of directors b. Voted upon by a corporation's shareholders c. In the form of cash or the corporation's stock d. In the form of stock owned in another corporation

b. Voted upon by a corporation's shareholders Explanation: A corporation can pay a dividend to its stockholders in the form of its own stock, stock owned in another corporation, or in cash. The amount of dividend to be paid is determined by the board of directors. Shareholders do not vote on dividend payments.

What information would NOT be found on a municipal bond confirmation? a. Whether the bonds are subject to the alternative minimum tax (AMT) b. Whether the bonds are subject to state income tax c. Whether the bonds were issued as original issue discount securities d. Whether the bonds are subject to federal income tax

b. Whether the bonds are subject to state income tax Explanation: A municipal bond confirmation must disclose certain tax information such as whether the bonds are subject to the alternative minimum tax (AMT), whether the bonds are issued as an original issue discount security, and whether the bonds are subject to federal income tax.

Which of the following choices is NOT considered an established business relationship under the Telephone Consumer Protection Act? a. You have executed two trades for the person called b. You have contacted the person twice but have not yet executed a trade c. The person called has completed an account form to retain your services d. The person has requested information but not yet opened an account

b. You have contacted the person twice but have not yet executed a trade Explanation: According to the Telephone Consumer Protection Act, a business relationship exists if the consumer has made an inquiry, application, purchase, or transaction with respect to the products or services offered by your firm. Prior solicitations do not alone establish a business relationship.

The tranche with the longest maturity and, therefore, the last to receive interest and principal payments within a CMO, is known as the: a. PAC tranche b. Z-tranche c. Supersinker d. Companion tranche

b. Z-tranche Explanation: The separate classes of a CMO are known as tranches. The longest maturity is frequently called the Z-tranche or the accrual bond, and does not receive interest or principal payments until the shorter maturing tranches have been retired.

The requirement for independent verification of a customer's identity when opening an account CANNOT be satisfied by examining a copy of the customer's:

birth certificate

The practice of dollar cost averaging requires the investor to

buy a security in a falling market and buy it in a rising market.

An investor who makes transactions once a month using dollar cost averaging would

buy the same dollar amount of stock.

A technology fund manager concerned about a downturn in the value of his portfolio would hedge by

buying narrow-based index puts.

A registered representative is discussing the investment merits of ABC stock with a customer. The registered representative may say: a. "Let's buy ABC stock because we expect it to go up 4 points in the next two weeks." b. "Our mergers department is working on a leveraged buyout for ABC Corporation, so let's buy it now before it is announced to the public" c. "One of our analysts just issued a favorable research report for public use on ABC stock, which estimates a 10% growth in earnings over the next three years and it appears to be a good situation for you." d. "Let's buy ABC stock because we are in a bull market and all stocks go up in a bull market."

c. "One of our analysts just issued a favorable research report for public use on ABC stock, which estimates a 10% growth in earnings over the next three years and it appears to be a good situation for you." Explanation: This is the only statement that would not be a violation since it is a statement of fact, coupled with an opinion or estimate of what should happen in the future. The other statements are violations because they definitely state an event (the stock will go up) will occur, which cannot be known in advance. Spreading rumors is also a violation.

A customer in her late 40s, who is currently in the 15% tax bracket, has recently inherited $7,000,000. She informs you that she considers herself a conservative investor and wants your advice concerning investing the inheritance. Which of the following choices would be the BEST method of investing the funds? a. 20% in equities, 30% in Treasury bonds, and 50% in tax anticipation notes b. 80% in equities, a 10% mixture of in-state and out-of-state municipal bonds, 10% in corporate bonds c. 30% in equities, 20% in-state municipal bonds, 20% in out-of-state municipal bonds, 15% in Treasury bonds, 10% in revenue anticipation notes, and 5% in tax-exempt money-market funds d. 20% in-state municipal bonds, 20% in out-of-state municipal bonds, 20% in corporate bonds, 20% in Treasury bonds, 10% in revenue anticipation notes, and 10% in tax-exempt money- market funds

c. 30% in equities, 20% in-state municipal bonds, 20% in out-of-state municipal bonds, 15% in Treasury bonds, 10% in revenue anticipation notes, and 5% in tax-exempt money-market funds Explanation: Although this investor is in her late 40s and considers herself a conservative investor, equities should be a part of her asset allocation. Many strategists recommend taking 100% and subtracting the investor's age as a guide to the percentage of the investor's portfolio that should be allocated to equities. As such, a 30% allocation in equities is reasonable (lower than 50%). However, since she is a conservative investor, this is reasonable with the remainder in various fixed-income securities and cash. Prior to inheriting the funds, she would not have been a suitable candidate for tax-exempt or municipal securities due to her low tax rate. After investing in these funds, the income/dividends/potential capital gains would have the effect of increasing her tax rate, so that municipal bonds would be an attractive investment. In-state municipal bonds would offer a higher after-tax return for this investor. Due to the potential of credit risk with municipal bonds, having a portion of the funds in Treasury securities would be a good recommendation. In addition, the investor should invest a portion of the funds in cash or cash alternatives. This is satisfied by allocating a portion of the funds in short-term municipal securities such as tax or revenue anticipation notes and tax-exempt money-market funds. Choice (a) has only a 20% allocation in equities and a 50% allocation of funds in tax anticipation notes, offering no growth potential. Having 100% of the funds in fixed-income investments does not offer the customer a balanced approach and, therefore, the other choices would not be the best method of investing the funds.

Which of the following corporations will be LEAST affected by an increase in interest rates? a. A manufacturing company b. A utility company c. A cosmetics company d. An automotive company

c. A cosmetics company Explanation: When interest rates are rising, industrial corporations that market big-ticket items as well as utilities that are heavy borrowers will be adversely affected. Cosmetic companies, due to the nature of the business and the low cost of their products, are not affected as much by rising interest rates.

Which of the following choices does NOT require additional documentation to transfer stock? a. A partnership account signed by a general partner b. A corporate account signed by an authorized officer c. A custodial account signed by the custodian d. An executor signing for an estate

c. A custodial account signed by the custodian Explanation: The only authorized signature for a custodial account is that of the custodian. There is no further documentation required. In each of the other choices, the transfer agent requires additional documentation showing that the person signing the certificate is authorized to do so.

Which of the following short positions violates SEC rules? a. A customer short stock that he borrowed from the brokerage firm b. A customer short and long the same stock at the same time c. A customer borrowing stock in order to profit from a tender offer d. A customer short stock while owning bonds convertible into that stock

c. A customer borrowing stock in order to profit from a tender offer Explanation: A tender offer takes place when an entity offers to buy a corporation's shares at a premium to the current market price. It is normally done for the purpose of acquiring control of the company. According to SEC rules, a customer may not tender short (borrowed) shares.

For an investor who needs regular income, a GNMA pass-through certificate would be attractive because

the investor would receive a monthly check.

Which of the following choices will qualify for a sales breakpoint on large purchases of mutual fund shares? a. A partnership formed to buy the securities b. A joint account formed between two unrelated individuals c. A husband and wife who are joint tenants with right of survivorship d. An investment club coordinated by a registered representative

c. A husband and wife who are joint tenants with right of survivorship Explanation: Quantity discounts are allowed only for individuals and individual entities such as corporations. Partnerships and investment clubs are not entitled to a quantity discount. Joint accounts normally do not qualify for breakpoints except in cases where there is a dependency relationship in the account (e.g., husband and wife).

Which of the following would increase a partner's basis in a limited partnership? a. Cash distributions b. Losses c. A pro-rata portion of a recourse loan d. Assessments not met by the partner

c. A pro-rata portion of a recourse loan Explanation: A partner's basis in a limited partnership represents the maximum loss that the limited partner may sustain in the program. It is increased by income, additional contributions made by the partner, and the portion of a recourse loan for which the partner is responsible. The basis is reduced by cash distributions and losses. Assessments not met by the partner normally result in a dilution of the partner's ownership interest.

What information would an analyst be MOST concerned with when evaluating a revenue bond? a. The population growth of the municipality b. Debt to assessed valuation c. A rate covenant d. Property taxes

c. A rate covenant Explanation: An analyst would be most concerned with rate covenants. This is an agreement made by the municipal issuer to maintain rates high enough to cover maintenance and operating charges and to meet annual debt service requirements. The other terms are applicable to general obligation bonds.

Cash dividends received from which of the following securities will be taxed as ordinary income? a. Preferred stock issued by a bank b. Common stock issued by an oil company c. A real estate investment trust d. Convertible preferred stock issued by a software company

c. A real estate investment trust Explanation: Currently, dividends paid on both common and preferred stock are taxed at a maximum rate of 20% if the stock is held for more than 60 days. Dividends from a REIT are still taxed at the same rate as ordinary income since a REIT does not pay corporate income tax if it distributes a minimum percentage of its income. The type of company that issued the shares is not relevant to the tax status of the cash dividend.

Which of the following is NOT required to be filed with FINRA? a. A retail communication concerning direct participation programs b. A retail communication concerning collateralized mortgage obligations c. A retail communication that provides information on a broker-dealer d. A retail communication that provides information on variable insurance products

c. A retail communication that provides information on a broker-dealer Explanation: A retail communication concerning direct participation programs (DPPs), collateralized mortgage obligations (CMOs), and investment companies are all required to be filed with FINRA. Investment companies include variable insurance products, mutual funds, closed-end funds, unit investment trusts (UITs), and exchange-traded funds (ETFs). A retail communication that does not make any financial or investment recommendation, or promote a product or service, such as providing information about a broker-dealer, does not need to be filed with FINRA.

Which of the following choices best describes certificates of participation? a. A form of equity financing for a corporation. b. A type of REIT c. A type of bond, typically created through a lease agreement d. A type of bond based on payments from residential mortgages

c. A type of bond, typically created through a lease agreement Explanation: Certificates of participation (COPs) are lease financing agreements, issued typically in the form of a tax-exempt or municipal revenue bond. COPs have been used traditionally as a method of monetizing existing surplus real estate. This financing technique provides long-term funding through a lease that does not legally constitute a loan, thus eliminating the need for a public referendum or vote.

Which of the following direct participation programs has the highest profit potential? a. An oil and gas developmental program b. An equipment leasing program c. A wildcatting program d. A low-income housing program

c. A wildcatting program Explanation: A wildcatting program, also called an exploratory program, searches for oil in unproven areas. Although it is considered the riskiest type of oil and gas program due to the high rate of failure, if oil is found, it has the highest profit potential. This is due to the lower cost of acquiring the land. A development program drills for oil in proven, surveyed sites and the cost for the land is greater. Equipment leasing programs and low-income housing are designed to generate income and have the potential for tax benefits.

An XYZ Corporation convertible bond is selling in the market at $1,248.75. It is convertible at $30. XYZ common stock's market price is 37.50. The bond has been called at 103. Which of the following activities is the LEAST attractive alternative for a holder of the bond? a. Sell the bond b. Convert to common and sell the common c. Allow the bond to be called d. Convert common stock to a bond

c. Allow the bond to be called Explanation: The holder could sell the bond and receive $1,248.75. If he converted, he would receive 33 1/3 shares ($1,000 par divided by $30 per share conversion feature) with a total value of $1,249.88 (33 1/3 x $37.50). The least attractive alternative is to allow the bond to be called and receive $1,030.

When calculating cost depletion for an oil program, all of the following items are necessary, EXCEPT the: a. Adjusted basis of the property b. Recoverable reserves c. Amount extracted and in storage d. Amount extracted and sold

c. Amount extracted and in storage Explanation: To calculate cost depletion, the adjusted basis, amount of recoverable reserves, and the amount extracted and sold are needed. Percentage depletion is based on a percentage of gross income from the property.

All of the following documents must be accompanied or preceded by an OCC risk disclosure document, EXCEPT: a. Options research reports b. A standardized options worksheet discussing straddles c. Options advertising that appears in the newspaper d. Options sales materials discussing projections

c. Options advertising that appears in the newspaper Explanation: Options advertising is a type of retail communication, which must make the offer to send the OCC risk disclosure document upon the customer's request. Options retail communications must be approved by a registered options principal (ROP) prior to use. Options-related retail communications that discuss projections, must be approved by a ROP prior to use and preceded or accompanied by a risk disclosure document.

Which of the following securities is LEAST suitable for an investor seeking income as a primary investment objective? a. A convertible bond b. A high-yield bond fund c. An exchange-traded note (ETN) d. A corporate bond with an adjustable-rate coupon

c. An exchange-traded note (ETN) Explanation: Exchange-traded notes (ETNs) are a type of unsecured debt security. ETNs carry issuer risk that is tied to the creditworthiness of the financial institution backing the note. These securities are not like traditional fixed-income securities since they typically do not make interest payments to investors. The returns are linked to the performance of an index, currency, or commodity and would be suitable for investors who want to speculate on the value of an index. An investor seeking income as a primary investment objective would not be a suitable candidate for an ETN. The other investment choices carry some type of risk but would still typically pay interest.

Who may not trade on the floor of the NYSE? a. An independent broker b. A designated market maker c. An institutional block trader d A floor trader

c. An institutional block trader Explanation: An institutional block trader may forward orders to the NYSE trading floor from the brokerage firm's trading desk but is not physically located and trading on the floor of the NYSE. The designated market maker, an independent broker, and floor traders are permitted to trade on the NYSE floor.

A high net worth investor seeking safety of principal would MOST likely invest in: a. Corporate convertible bonds b. Non-investment-grade corporate bonds c. An investment-grade corporate bond fund d. A variable annuity

c. An investment-grade corporate bond fund Explanation: Safety of principal refers to a customer being able to preserve or retain the initial amount of the investment over its life. Many bonds and bond funds offer investors this feature. The higher the rating, the greater the likelihood the investor will achieve safety of principal. An investment-grade corporate bond fund would offer more safety of principal than non-investment- grade and convertible corporate bonds. A variable annuity may fluctuate in value based on the subaccounts chosen by the investor.

Which of the following investors would be LEAST suitable for an oil and gas direct participation program (DPP)? a. An investor in the highest federal tax bracket b. A retired investor who is in the highest federal tax bracket c. An investor who is concerned about the alternative minimum tax d. An investor who recently inherited $5,000,000

c. An investor who is concerned about the alternative minimum tax Explanation: An investment in an oil and gas limited partnership may have excess depletion and depreciation as well as excess intangible drilling costs. These are tax preference items and may result in an investor being subject to the alternative minimum tax (AMT). The other investors may or may not be suitable for an oil and gas DPP. It would depend on many other factors. However, an investor concerned about the AMT would not want to invest in a security that normally has tax preference items.

MSRB rules require that a municipal securities principal must approve all of the following choices, EXCEPT: a. All municipal transactions b. Municipal advertising c. An official statement sent to customers d. Correspondence sent to customers

c. An official statement sent to customers Explanation: A municipal securities principal does not need to approve an official statement (OS). An OS is prepared by an issuer of municipal securities and issuers are not subject to MSRB rules.

In a new municipal issue, what is a group order? a. An order placed by three or more members b. An institution purchasing bonds from a syndicate c. An order allowing all members to benefit d. A dealer buying for a group of investors

c. An order allowing all members to benefit Explanation: There are four types of orders that can be placed with a syndicate. 1. A presale order is any order placed before the syndicate that actually purchases the issue from the issuer 2. A group order is a situation where all members of the syndicate share in the profit 3. A designated order is usually placed by a large institution that designates two or more members to receive credit for the sale 4. A member order is an order placed by members for their customers

Claire, an attorney, has contributed to her IRA account for many years. Her new employer, Digiphone, has a pension plan. Which of the following statements is TRUE regarding further IRA contributions? a. Claire may contribute, but the growth in the account will be taxable in the year earned b. Whether she may continue to contribute depends on her income level c. Claire may continue to contribute as long as she has earned income d. Due to her participation in a pension plan, Claire may not continue to make IRA contributions

c. Claire may continue to contribute as long as she has earned income Explanation: As long as Claire has earned income, she can fund her IRA. However, based on her income level and tax filing status, she may not be able to deduct her contributions on her tax returns. Claire's participation in a pension plan has no bearing on her ability to contribute to her IRA.

Which of the following securities has prepayment risk? a. Mortgage bonds issued by a utility company b. Bonds issued by Freddie Mac c. Collateralized mortgage obligations d. Commercial paper

c. Collateralized mortgage obligations Explanation: Many homeowners pay off their mortgages early. When interest rates fall, homeowners have an incentive to refinance and pay off their existing mortgages. These prepayments are passed through to the pools holding the old mortgages. The investors then need to reinvest this large amount of principal at a time when interest rates have declined. This is referred to as prepayment risk and it is associated with mortgage-backed securities such as CMOs. Although both Fannie Mae (FNMA) and Freddie Mac (FHLMC) issue mortgage-backed securities, in this question choice (b) covers the bonds of these issuers, which do not have prepayment risk.

Which of the following securities would you LEAST likely recommend to an investor requiring a fixed sum of funds to be received in 10 years? a. A zero-coupon municipal bond b. A high-yield corporate bond c. Collateralized mortgage obligations (CMOs) d. Treasury Inflation-Protected Securities (TIPS)

c. Collateralized mortgage obligations (CMOs) Explanation: The risk that an investor will receive her principal earlier than projected instead of at one time (i.e., prepayment risk) is the most important risk pertaining to mortgage-backed securities such as CMOs. Since the investor wants to receive a fixed amount of funds in 10 years, a CMO would be the least suitable of the securities listed.

Which of the following choices is another way of expressing the earnings multiple? a. Debt-to-equity ratio b. Dividend payout ratio c. Price-earnings ratio d. Operating profit ratio

c. Price-earnings ratio Explanation: The earnings multiple is also called the price-earnings ratio.

A registered representative has sent a preliminary prospectus to various clients who have indicated an interest in a new issue his firm is underwriting. The registered representative is notified that he has been allocated 500 shares of the new issue. The registered representative should: a. Allocate the 500 shares to his most active client b. Allocate 100 shares each to his best clients c. Contact all clients who have received a prospectus asking them if they have made a decision to purchase the new issue that is now available d. Keep the 500 shares for himself

c. Contact all clients who have received a prospectus asking them if they have made a decision to purchase the new issue that is now available Explanation: The most appropriate action for the registered representative to take is to contact all clients who have received a prospectus and ask them if they have made a decision to purchase the new issue that is now available.

The number of times the earnings of a municipal facility exceeds the interest charges and principal payments of a revenue bond for a period is called the: a. Working capital ratio b. EBITDA ratio c. Debt service coverage ratio d. Price-earnings ratio

c. Debt service coverage ratio Explanation: The number of times the earnings of a revenue bond of a municipal facility exceeds the interest charges and principal payments (debt service) for a period is the debt service coverage. Earnings before interest, tax, depreciation, and amortization (EBITDA) is a term associated with corporate bond issuers, not municipal bond issuers.

Of the following broad-based indicators, the one with the narrowest measure of the market is the: a. Standard and Poor's 500 Index b. Wilshire Associates Equity Index c. Dow Jones Composite d. New York Stock Exchange Composite Index

c. Dow Jones Composite Explanation: The Dow Jones Composite contains only 65 stocks. The Wilshire Associates Equity Index shows the dollar value of approximately 7,000 stocks. The S&P 500 Index contains 500 stocks. The NYSE Composite Index consists of all common stocks listed on the NYSE.

Which of the following statements is TRUE concerning a customer who purchases an out-of-state original issue discount (OID) general obligation bond? a. Each year the customer will pay both federal and state income tax b. Each year the customer will pay only federal income tax c. Each year the customer will pay only state and local income tax d. The customer will not pay any tax

c. Each year the customer will pay only state and local income tax Explanation: The upward adjustment in the purchase price of an original issue discount bond is called accretion. The amount accreted each year is considered interest income, which may or may not be taxable depending on the type of security. The interest on an out-of-state municipal security is exempt from federal tax, but subject to state and local income tax. The tax rate is based on the state in which the customer maintains his primary residence.

Which of the following statements is NOT TRUE regarding a SEP IRA? a. An employer makes contributions to an employee's IRA b. An employer is not required to make annual contributions c. Employees are permitted to make contributions to the account d. Employees are immediately vested for any contributions made to the account

c. Employees are permitted to make contributions to the account Explanation: A simplified employee pension plan (SEP IRA) does not allow the employee to make contributions. SEPs are funded by employer contributions only. This is different than for Keogh plans, which do allow for employees to make nondeductible contributions to their own account.

Which of the following organizations enforces municipal securities regulations for broker-dealers? a. The FRB b. The FDIC c. FINRA d. The MSRB

c. FINRA Explanation: Although the MSRB creates rules governing municipal securities broker-dealers, its rules are enforced by other regulatory bodies. The appropriate regulatory agencies are the: - The SEC or FINRA for broker-dealers - Comptroller of the currency for federal banks - The FRB for state banks that are members of the FRB - The FDIC for member banks of the FDIC

A customer may make a single, lump-sum contribution of which of the following amounts to a 529 college savings plan without incurring any taxes? a. An unlimited amount b. The annual gift tax exclusion c. Five times the annual gift tax exclusion d. Ten times the annual gift tax exclusion

c. Five times the annual gift tax exclusion Explanation: States that offer 529 plans determine the specific plan rules such as allowable contributions, investment options (e.g., mutual funds), and deductibility of contributions for state tax purposes. A person may contribute to a 529 college savings plan up to the federal annual gift tax exclusion ($14,000) without paying a gift tax, or the contributor may make a single, lump-sum gift of up to the five-year cumulative limit ($70,000) for tax-free gifting.

Buy-stop orders or sell-stop orders can provide all the following features, EXCEPT: a. Provide price protection for a short position b. Provide price protection for a long position c. Give a broker discretion when the order is activated d. Possibly cause a fluctuation in the market price of a stock

c. Give a broker discretion when the order is activated Explanation: Buy-stop or sell-stop orders do not give a broker discretion when the order is activated. When activated, the order becomes a market order and should be executed immediately. All of the other choices are correct.

A put option may be written in a cash account if the investor: a. Is long the underlying security in the account b. Is short the underlying security in the account c. Has cash in the account equal to the exercise price d. Is long a call option on the same underlying security

c. Has cash in the account equal to the exercise price Explanation: To write a put in a cash account, the customer must have cash in the account equal to the exercise price. If the writer is short the underlying stock, the put is considered covered for margin purposes, but this transaction may not be written in a cash account, only in a margin account.

Briana Corporation, an existing public company, is offering 500,000 shares of common stock to the public through an underwriting syndicate. The prospectus states that 250,000 shares are being offered by selling stockholders and 250,000 shares are being offered by Briana Corporation. Which of the following statements is TRUE regarding this offering? a. It is an initial public offering b. A registration statement is not required c. It is a combined primary and secondary offering d. It is a private placement

c. It is a combined primary and secondary offering Explanation: Of the 500,000 shares being offered, 250,000 shares are being issued for the first time from authorized but unissued shares. This is considered a primary offering. The 250,000 shares being offered by the selling stockholders have already been issued by the corporation. This makes them a secondary offering. The offering is, therefore, a combined primary and secondary offering. A registration statement is required because the securities are being offered to the public.

On June 21, 2011, a municipal securities salesperson decided to leave his job and try his hand at real estate sales. Although able to earn a good living, he decided that he liked municipal sales and rejoined the municipal securities firm on July 15, 2013. Which of the following statements is TRUE? a. He may resume selling municipal securities immediately b. He must wait 90 calendar days before he may sell municipal securities c. He must wait until he passes the MSRB examination for municipal representatives before he may sell municipal securities d. He must pass the MSRB examination for municipal securities representative and must wait a minimum of 90 calendar days

c. He must wait until he passes the MSRB examination for municipal representatives before he may sell municipal securities Explanation: His registration expired two years after he left the municipal securities firm. To requalify as a municipal representative, he must pass the qualifying exam. He will not need to wait 90 days, because once the apprentice period has been served, it does not need to be repeated.

A 60-year-old individual has been putting money in an annuity for 15 years and has been informed of another variable annuity that is offering higher returns. He is not in need of income at this time and is looking to defer income for several more years. Which of the following suggestions is most suitable in light of this individual's circumstances? a. He should use the provisions of a 1035 exchange to move the money from his current annuity to the new annuity offering higher returns b. An exchange would be suitable if he hasn't made an exchange within the last 36 months c. He should maintain his existing variable annuity and not begin taking distributions until such time as he needs them d. The individual can begin taking tax-free distributions from his existing annuity

c. He should maintain his existing variable annuity and not begin taking distributions until such time as he needs them Explanation: In most situations, senior citizens should not be starting, or exchanging into, a new variable annuity since they are designed primarily as long-term investments. Contributions grow tax-deferred and the earnings are taxable only when the annuitant starts taking distributions. While a 1035 exchange allows a person to move from one annuity to another without the exchange being taxable, the new annuity is subject to its own sales charges and the benefits of the original annuity contract will be lost. Exchanges done within 36 months of a previous exchange can be viewed as churning and being unsuitable.

A customer is considering an investment in a hedge fund since many of his business associates have been receiving high returns over the last few years. A registered representative may make which of the following statements? a. Mutual funds are subject to less regulatory oversight than hedge funds b. Mutual funds pool investors' money and manage the portfolio, whereas hedge funds manage each investor's assets separately c. Hedge funds often use higher degrees of leverage than mutual funds d. Hedge funds may be suitable for many customers, whereas mutual funds are generally suitable for sophisticated, wealthy investors only

c. Hedge funds often use higher degrees of leverage than mutual funds Explanation: Mutual funds and hedge funds both pool investors' money to manage assets. Unlike mutual funds, hedge funds are often exempt from regulatory oversight, use leverage, and often employ aggressive financial strategies such as short selling and placing large bets on individual companies or sectors of the market. Hedge funds typically have high minimum investment requirements that make them suitable only for professional and wealthy investors.

Dedicated Securities has been invited to join a syndicate selling a new offering of common stock. The head of the firm's syndicate department notices that the agreement among underwriters mentions a penalty bid. Which of the following choices is an example of a penalty bid? a. If Dedicated fails to sell its allotment, it will be liable for twice its normal commitment b. If Dedicated fails to solicit a certain number of indications of interest, it will be required to pay a fee to the syndicate manager c. If Dedicated sells some of the issue to a customer, who later sells the stock back to the syndicate at the stabilizing bid, Dedicated will forfeit the concession on those shares d. If Dedicated sells some of the issue to a customer, who later sells the stock back to the syndicate at the stabilizing bid, Dedicated could be penalized for failure to maintain the public offering price

c. If Dedicated sells some of the issue to a customer, who later sells the stock back to the syndicate at the stabilizing bid, Dedicated will forfeit the concession on those shares Explanation: A penalty bid is an arrangement that permits the managing underwriter to reclaim a selling concession from a syndicate member when securities originally sold are repurchased by the syndicate in stabilizing transactions.

Which of the following statements is NOT TRUE regarding the characteristics of options and warrants? a. Warrants are created by the corporation whose stock underlies the instrument, and options are created by contract between an option buyer and an option writer b. Both options and warrants can expire worthless if they are not exercised c. If options are exercised, a set price must be paid for the underlying security and, if warrants are exercised, the securities are received at no additional cost d. Both options and warrants can be bought and sold in the secondary market

c. If options are exercised, a set price must be paid for the underlying security and, if warrants are exercised, the securities are received at no additional cost Explanation: Both options and warrants have a strike price. If exercised, the transactions for the underlying security will occur at that set price. It is in the case of convertible bonds or convertible preferred stock that investors can convert the security into the underlying stock with no additional payment of money.

Regulation AC (Analyst Certification) does NOT apply to: a. Brokers b. Dealers c. Issuers d. Research analysts

c. Issuers Explanation: Under Regulation AC, a research analyst must certify that her research report accurately reflects her personal views. The analyst must also disclose whether she received compensation for her specific recommendation. Under this rule, broker-dealers are also required to obtain periodic certifications by research analysts regarding public appearances (attesting that a statement was made regarding the views she expressed being her own and a written certification documenting that she did not receive compensation to make positive comments). Issuers are not regulated under Regulation AC.

The NYSE Composite Index is composed of: a. The common and preferred stocks listed on the NYSE b. The common stocks, preferred stocks, and bonds listed on the NYSE c. Only the common stocks listed on the NYSE d. Preferred stocks and bonds listed on the NYSE

c. Only the common stocks listed on the NYSE Explanation: The NYSE Composite Index is composed of only the common stocks listed on the NYSE.

A generic ad for an investment company placed by a broker-dealer would contain

the name of the broker-dealer, but not the name of the investment company.

The economic theory stating that government intervention in the marketplace is necessary for controlling economic growth is known as: a. Supply-side economics b. Monetary theory c. Keynesian theory d. The Dow Theory

c. Keynesian theory Explanation: Keynesian economic theory looks at the demand side of the marketplace. It states that government intervention in the marketplace (by using such measures as expenditure programs) is necessary for controlling the economy. Supply-side economics states that the government should reduce marginal tax rates and the size of government to promote economic growth. Monetary theory looks to increase or decrease the money supply in order to control the economy. The Dow Theory, which is followed by some technical analysts, states that a major reversal in the market has occurred when both the Dow Jones Industrial Average (DJIA) and the Dow Jones Transportation Average (DJTA) break their trends.

Which of the following formulas is used to determine the total equity in a combined margin account? a. LMV + DR - CR - SMV b. LMV - DR + SMV - CR c. LMV + CR - DR - SMV d. LMV - CR - DR + SMV

c. LMV + CR - DR - SMV Explanation: To determine the equity in a combined margin account, take the long market value (LMV) plus the credit balance (CR), then subtract the debit balance (DR) and the short market value (SMV).

For variable annuities, which of the following payout options provide the highest payout? a. Joint and last survivor life annuity b. Life annuity with period certain c. Life annuity d. Unit refund life annuity

c. Life annuity Explanation: Annuitants will receive the greatest cash flow from the life annuity payout option. This option allows an annuitant to receive payments for his lifetime. At death, the payments cease since no beneficiary is designated and, therefore, the insurance company is relieved of its obligation to make payments. The annuitant assumes the greatest degree of risk with this type of payout. (89732)

Mrs. Smith is short 100 shares of DEF stock. She is concerned that the stock is going to increase in price temporarily, but does not want to cover the short position. Which option position gives Mrs. Smith the BEST protection? a. Long 1 DEF put b. Short 1 DEF put c. Long 1 DEF call d. Short 1 DEF call

c. Long 1 DEF call Explanation: The best possible upside protection can be accomplished with the purchase of 1 DEF call. If Mrs. Smith is long a call, this allows her to buy the stock from the writer if the stock goes up, thus protecting the short position.

Which of the following positions best enables an investor to take advantage of a significant appreciation in DEF stock? a. A debit DEF call spread b. A credit DEF put spread c. Long a DEF straddle d. Short a DEF straddle

c. Long a DEF straddle Explanation: The long straddle offers an investor the ability to realize unlimited gains since the client is long a call option. The gains are determined by the amount the stock appreciates. While a debit call spread is bullish, the gain is limited to the difference between the strike price on the long call and the strike price on the short call. The credit put spread is also bullish, but the gain is limited to the net premium received. The short straddle exposes an investor to unlimited risk if the stock rises.

When comparing long-term bonds and short-term bonds, all of the following statements are TRUE, EXCEPT: a. Long-term bonds generally have higher yields b. Fluctuations in the dollar price of long-term bonds are usually greater than for short-term bonds when the general level of interest rates change c. Long-term bonds generally provide greater liquidity than short-term bonds d. There is more purchasing power risk with long-term bonds when compared to short-term bonds

c. Long-term bonds generally provide greater liquidity than short-term bonds Explanation: When comparing long-term bonds and short-term bonds, all of the choices listed are true except long-term bonds generally provide greater liquidity than short-term bonds. Short-term bonds do not suffer from as large a price movement as long-term bonds when interest rates are changing. Long-term bonds are open to greater market risk, interest-rate risk, and purchasing-power risk. Both individual and institutional investors alike are more willing to accept a lower return (yield) in favor of more stable principal (less severe price swings).

When a client buys a bond above par, the confirmations must indicate the: a. Rating b. Contraparty c. Lower of yield to call or yield to maturity d. Catastrophe call provisions

c. Lower of yield to call or yield to maturity Explanation: A bond's confirmation must disclose the lower of the yield to maturity or the yield to call. This is sometimes referred to as the yield to worst.

For tax purposes, which of the following is NOT deducted from rental income in a real estate program? a. Depreciation b. Maintenance c. Mortgage amortization d. Property tax

c. Mortgage amortization Explanation: Expenses that are deducted from rental income in a real estate program include maintenance, property tax, depreciation, and mortgage interest. Paying off (amortizing) the principal of a mortgage is not an expense and may not be deducted from rental income for tax purposes.

Which of the following terms are synonymous? a. Net asset value and offering price b. Selling price and bid price c. Net asset value and redemption price d. Bid price and management fee

c. Net asset value and redemption price Explanation: The net asset value and redemption price are synonymous. An investor who owns a mutual fund may redeem (sell) the security back to the fund at the net asset value (NAV).

A registered representative finds that most of his customers are reluctant to buy stocks due to extreme market volatility. Until the volatility subsides, which of the following actions is NOT a suitable strategy to recommend to these nervous clients? a. Invest in money-market funds b. Invest in Treasury securities c. Offer to reimburse customers who lose money in stocks d. Invest in CDs

c. Offer to reimburse customers who lose money in stocks Explanation: Industry rules prohibit registered representatives from guaranteeing customers against loss. During times of uncertainty and extreme volatility, high quality and short-term investments are normally good recommendations for investors who are reluctant to buy stocks.

A narrow-based index option may be used to hedge a portfolio of: a. Treasury bonds b. Money-market securities c. Oil company stocks d. Diversified blue-chip stocks

c. Oil company stocks Explanation: A portfolio consisting of stocks from the same industry may be protected (hedged) against adverse market movements by using narrow-based index options. A narrow-based index gives a measurement of stocks in a particular industry or sector of the economy. A broad-based index option would be used to hedge a diversified stock portfolio.

An investor wishes to buy a limited partnership investment that has the goal of capital appreciation without producing currently taxable cash flow. Which of the following choices BEST suits the investor's needs? a. Low income housing b. Oil and gas income program c. Raw land d. Equipment leasing

c. Raw land Explanation: Raw land will satisfy an investor's need for an investment that has the potential for capital appreciation without producing currently taxable income. However, raw land is not eligible for depreciation deductions or tax credits. Due to the limited benefits, an investment in raw land is considered speculative.

In a discussion with a client, a registered representative refers to a bond yield that has been reduced by the inflation rate. This yield is known as the: a. After-tax yield b. Discount rate c. Real interest rate d. LIBOR

c. Real interest rate Explanation: The real interest rate is the yield of a security reduced by the inflation rate. While it represents earnings remaining once inflation is taken into account, the real interest rate does not factor in the tax consequences. The discount rate is the rate of interest that the Federal Reserve charges member banks for loans. LIBOR (the London Interbank Offered Rate) is the rate of interest that banks in London charge each other for short-term loans.

Taxable income normally includes: a. The interest on municipal bonds issued in the state in which the taxpayer lives b. The taxpayer's annual 401(k) contributions c. Reinvested dividends paid on a mutual fund investment d. Any unrealized capital appreciation on stocks that the taxpayer owns

c. Reinvested dividends paid on a mutual fund investment Explanation: Taxable income includes income from all sources after all applicable deductions and adjustments are made. Reinvested dividends must be declared as income and are thus taxable. Interest on municipal bonds issued in the state in which the owner resides is usually exempt from both federal and state income taxes. 401(k) contributions are made on a pretax basis and are not included in taxable income until the taxpayer begins taking distributions. Unrealized capital gains on stocks are not included in taxable income.

According to the Securities Exchange Act of 1934, a person who is on the board of directors of a public corporation and owns 3% of the company stock is: a. Required to sell shares of the company stock currently owned every three months b. Not permitted to purchase additional shares of the company stock c. Required to register as an insider of the corporation d. Not required to register as an insider of the corporation

c. Required to register as an insider of the corporation Explanation: An insider, as defined by the Securities Exchange Act of 1934, is a director, officer, or owner of more than 10% of the voting stock of a corporation and his immediate family members. Individuals who become insiders are required to report to the SEC within 10 days of becoming insiders. An officer or director is required to register regardless of the number of shares he owns of the public corporation. Insiders are not permitted to make short-swing profits in the stock of the corporation in which they are insiders. If an insider sells the stock at a profit within six months of its acquisition, or sells stock for a profit that was held six months or longer and then repurchases it within six months of the sale, the corporation may sue for recovery of the profit. Insiders are also not permitted to short the stock of the company in which they are shareholders. Insiders are never required to sell shares, but are permitted to buy additional shares as long as it is reported to the SEC.

A double-barreled municipal bond is backed by the: a. Revenues of a project b. Taxes of a municipality c. Revenues of a project and taxes of a municipality d. Revenues of the U.S. government

c. Revenues of a project and taxes of a municipality Explanation: A double-barreled municipal bond is backed by two sources of income, which would be the revenues of a project and the taxes of a municipality.

All of the following government agencies are involved in the housing market, EXCEPT: a. FNMA b. FHLMC c. SBA d. GNMA

c. SBA Explanation: The SBA is the Small Business Administration and is not involved in the housing market. The SBA is a federal agency involved in providing financial assistance to small businesses.

The 5% Markup Policy applies to: a. A primary distribution (new issue) b. A registered secondary distribution requiring a prospectus c. Securities quoted on Nasdaq d. Municipal securities

c. Securities quoted on Nasdaq Explanation: The 5% Markup Policy does not apply when a security is being issued with a prospectus or for municipal securities. In this example, a prospectus would be required for a primary distribution as well as a registered secondary distribution. Securities quoted on Nasdaq would be the only choice given for which the 5% guideline would apply.

Lyle, Molly, and Seena have a joint account registered as Tenants in Common. In the event that Seena dies, which of the following statements is TRUE? a. The account would be liquidated as soon as the brokerage firm learns of Seena's death b. Lyle and Molly must change the arrangement to a Joint Tenants with Right of Survivorship c. Seena's estate has a claim on her portion of the account's assets d. Seena's share of the assets in the account are automatically transferred to Lyle and Molly

c. Seena's estate has a claim on her portion of the account's assets Explanation: Upon learning of Seena's death, the brokerage firm will freeze the account. Seena's executor will then provide documentation to establish authority to act on behalf of the estate. Typically, Seena's estate will become the third joint owner in the existing Tenants in Common

Under the partnership democracy provisions of a limited partnership, the limited partners are NOT permitted to: a. Petition the court to have the general partner removed b. Petition the court to have the partnership dissolved c. Sell assets owned by the partnership d. Sue the general partner

c. Sell assets owned by the partnership Explanation: Selling assets is a management decision and is, therefore, not the role of the limited partners. Democracy provisions would permit the other choices.

A customer wants preservation of capital and safety of income. Which of the following securities would best meet the customer's objectives? a. Income bonds b. Debentures c. Several municipal bonds rated AA or better d. One AAA-rated municipal bond

c. Several municipal bonds rated AA or better Explanation: Purchasing several AA- or better-rated municipal bonds with various maturity dates is more advantageous than purchasing just one municipal bond. The investor is diversifying his portfolio. If the investor purchased one bond and it were to default, he would no longer receive interest payments. The market value of the bond would decline considerably, causing a severe loss for the investor. Income bonds and debentures are not as safe as municipal bonds.

Which of the following activities does NOT take place during the cooling-off period? a. The due diligence meeting b. Issuing a red herring c. Stabilizing the issue d. Blue-Skying the issue

c. Stabilizing the issue Explanation: When a new stock issue is going to be offered, a registration statement must be filed with the SEC. After the filing, there is a period when the SEC reviews the information to ensure full disclosure. During the cooling-off period, a preliminary prospectus (red herring) is prepared to be used to receive indications of interest from the public. The issue must be registered in each state in which it will be sold according to state (Blue-Sky) laws. Prior to the completion of the final prospectus, a due diligence meeting is held where all concerned parties (issuer and underwriter) meet to insure that everything has been done properly. Stabilization of the issue takes place after the new security is selling in the market.

In a rights offering, an underwriter offers to purchase all the shares the issuing corporation may not be able to sell. This is known as a(n): a. Firm-commitment underwriting b. Best-efforts underwriting c. Standby underwriting d. All-or-none underwriting

c. Standby underwriting Explanation: A type of underwriting in which the underwriter agrees to buy all the shares not subscribed to in a rights offering is a standby underwriting. The issuing corporation realizes that many shareholders will not participate in the rights offering. This may amount to a large number of the shares being offered. The corporation will not receive the money for the shares that are not subscribed to. The underwriter that is standing by to buy all the unsubscribed shares will either buy them at a discount or will receive a fee. This type of an arrangement assures the issuing corporation it will be able to raise the amount of capital it requires.

A type of order that becomes a market order when a round-lot trades at or through a particular price is called a: a. Market order b. Limit order c. Stop order d. Stop-limit order

c. Stop order Explanation: A type of order that becomes a market order when a round-lot trades at or through a particular price is called a stop order. A variation of a stop order is a stop-limit order, which is activated when a round-lot trades at or through a particular price, along with the requirement that the limit price be satisfied.

Which of the following statements is NOT considered misleading regarding a variable annuity communication? a. Telling a client that a variable annuity is a mutual fund b. Representing that a variable annuity will meet short-term liquidity needs c. Telling a client about the negative impact of an early redemption d. Making a representation that a death benefit guarantee applies to the investment return of the annuity

c. Telling a client about the negative impact of an early redemption Explanation: FINRA is concerned about misleading communications regarding product identification, liquidity, and claims regarding guarantees. A firm should not imply that the underlying account is a mutual fund. Annuities should be purchased with long-term goals in mind, not short-term liquidity needs. Death benefits may be guaranteed, but investment results may not. It would be advisable to inform a potential investor of surrender charges incurred as a result of early redemption.

XYZ Corporation borrows money at a rate of interest that is one point above LIBOR. Therefore, the rate is based on: a. A long-term bond index b. The U.S. prime rate c. The London Interbank Offered Rate d. A rate established by the Federal Reserve

c. The London Interbank Offered Rate Explanation: LIBOR is the London Interbank Offered Rate. It is the average rate that banks charge each other on loans for London deposits of Eurodollars.

A research analyst at a broker-dealer is preparing a research report recommending ABC common stock. Which of the following situations need not be disclosed? a. ABC Corp is an investment banking client of the broker-dealer b. The broker-dealer has a 1% or greater beneficial ownership in ABC common stock c. The broker-dealer has a 1% or greater beneficial ownership in ABC nonconvertible bonds d. The broker-dealer makes a market in ABC common stock

c. The broker-dealer has a 1% or greater beneficial ownership in ABC nonconvertible bonds Explanation: The broker-dealer is required to make certain disclosures in its research reports, such as whether the firm has an investment banking relationship or makes a market in the common stock of ABC. It must also disclose its ownership in a subject security if the ownership is equal to or greater than 1% beneficial ownership in common equity. Since nonconvertible debt is not considered common equity, disclosure is not required.

The CEO of a publicly traded company has inadvertently disclosed material, nonpublic information in a conference call with securities analysts. According to Reg FD, which of the following statements is TRUE? a. The CEO is guilty of insider trading b. The CEO and analysts are guilty of insider trading c. The company must make a public disclosure of this information within 24 hours d. The CEO has violated Rule 8K

c. The company must make a public disclosure of this information within 24 hours Explanation: Reg FD requires that material, nonpublic information disclosed to analysts or other investors must be made public. If the disclosure was intentional, the information must be simultaneously disclosed to the public. If unintentional, the public disclosure must be made within 24 hours. A Form 8K, filed with the SEC, is one method of meeting the public disclosure requirement.

Which of the following statements is NOT TRUE regarding accounts established under the Uniform Gifts to Minors Act? a. Taxes are the responsibility of the minor b. The custodian makes all investment decisions in the account c. The custodian may use account positions to cover short options positions in his own personal account d. The account must reflect the minor's Social Security number

c. The custodian may use account positions to cover short options positions in his own personal account Explanation: The custodian may not use securities in the custodian account to cover an options position in his own personal account. All securities in the custodian account must be used only for the benefit of the minor.

When selling a security for a customer, all of the following items MUST be included on the sell ticket, EXCEPT: a. The number of shares or par value b. The location of the security c. The customer's Social Security number d. The customer's account number

c. The customer's Social Security number Explanation: An order ticket must include the customer's account number, number of shares or par value (for a bond), name of the security, limitations (limit, stop, etc.), and whether it is buy or sell. If it is a sell order, the location of the security (long or short) must be indicated. The customer's Social Security number is not needed on the order ticket.

Marking-to-the-market is

the revaluing of securities held long or short in the account based on the actual CMV of the securities.

A variable life insurance policy has an AIR of 5%. The separate account recently performed at a 4% rate of return. Which of the following statements BEST describes the effect of this rate of return on the death benefit of the policy? a. The death benefit will increase as long as the rate of return is positive b. The death benefit will decrease to a determined level c. The death benefit will decrease, but not below the guaranteed minimum d. The rate of return of the separate account affects only the cash value, not the death benefit

c. The death benefit will decrease, but not below the guaranteed minimum Explanation: If the return of the separate account exceeds the AIR, the death benefit will increase. If the return of the separate account is less than the AIR, the death benefit will decrease. However, the death benefit may not decrease below the initial face value of the policy.

Which of the following rates is set by the Federal Reserve Board? a. The broker loan rate b. The federal funds rate c. The discount rate d. The prime rate

c. The discount rate Explanation: Of the choices given, only the discount rate is set by the Federal Reserve Board. The prime rate is the rate of interest that commercial banks charge their best-rated customers and is established by each bank. The broker loan rate, which is the rate of interest charged to brokerage firms for margin loans, is set by each bank. The federal funds rate is the charge for overnight loans between banks and is set by each bank.

Which of the following factors would be LEAST useful when analyzing the credit risk of an issuer of revenue bonds? a. User charges b. Rental and lease payments c. The federal funds rate d. Concessions and fees

c. The federal funds rate Explanation: Current interest rates are factors that will affect all bond issuers and would be least useful when analyzing a specific issuer of revenue bonds. The other choices are all sources of revenue to be used to pay the interest and principal on a municipal revenue bond.

An article in The Bond Buyer states that the placement ratio rose to 91.5%, up from 78.7% a week ago. Based on this information, which of the following statements BEST describes the placement ratio? a. The ratio between deals that have been announced previously and those that have already come to market b. The ratio between deals that have come to market and those that still remain on the calendar c. The percentage of bonds brought to market and placed with clients as compared to the total amount that was available for sale d. The percentage of bonds awaiting sale to the public compared to those that will be underwritten in the next 30 days

c. The percentage of bonds brought to market and placed with clients as compared to the total amount that was available for sale Explanation: The placement ratio is a means of gauging the amount of bonds that have been underwritten recently on a new-issue basis that have moved into the hands of the ultimate investor. In other words, it is the percentage of new bonds that were sold compared to those that were originally available for sale. The higher the placement ratio, the more bonds there are moving into the hands of investors.

All of the following statements are TRUE of covered call option writing, EXCEPT: a. The writer can increase the overall yield on his portfolio b. It is considered a conservative option strategy c. The premium received guarantees the writer cannot have a loss on the underlying security d. The writer will have a short-term capital gain if the option expires unexercised

c. The premium received guarantees the writer cannot have a loss on the underlying security Explanation: All of the choices listed are true except the premium received guarantees the writer cannot have a loss on the underlying security. The security can decline in price below the breakeven point (cost price of the stock minus the premium), causing the writer to have a loss on the stock. If the option expires, the writer will always have a short-term capital gain from the premium received.

When analyzing a general obligation bond, which of the following factors is NOT a positive indicator of the bond's quality? a. Voter registrations have increased over the last 18 months b. The department of motor vehicles reports that out-of-state drivers have been registering their cars in your state at an increasing rate c. The state increased the toll for the use of the turnpike d. A multiplex cinema, do-it-yourself store, and book-selling chain have all announced new franchises in your community

c. The state increased the toll for the use of the turnpike Explanation: An increasing population trend and a mixture of diverse businesses (both new and established) are positive demographic indicators that reinforce the quality of general obligation issues. User fees are generally associated with revenue bond issuers.

Fred Brick recently requested annuitization after contributing to a variable annuity for 12 years. The actuary applied an assumed interest rate of 3% and determined his first payment. The performance of the separate account on an annualized basis for the three months following annuitization is: 5%, 3%, and 4%, respectively. Which of the following statements can be made about Mr. Brick's payments? a. Payments rose in each of the three months b. Payments remained the same in each month c. The third payment was higher than the previous month's payment d. His total payments have increased by 12%

c. The third payment was higher than the previous month's payment Explanation: If the performance in the separate account is greater than the assumed interest rate (AIR), payments will increase. If performance is less than the AIR, payments will decline, and if performance equals the AIR, payments will remain the same. Comparisons are made to the previous month and not to the original month. In the third month, the performance of 4% was higher than the AIR, resulting in a payment that was greater than the previous month's payment.

Which of the following statements is NOT TRUE of industrial development revenue bonds? a. They are issued by local municipal governments b. They may be used to finance the construction of commercial property that will be used by private corporations c. Their credit rating is determined by an analysis of the municipal government issuing the bonds d. Interest is paid from rents received from private corporations

c. Their credit rating is determined by an analysis of the municipal government issuing the bonds Explanation: Industrial development revenue bonds are issued by local municipal governments to build factories or other commercial properties. The plant or property is leased by the municipality to a corporation. The interest on the bonds is paid from the lease rental payments made by the corporation. The credit rating of the bond is based on the credit rating of the corporation and not on an analysis of the credit rating of the municipal government issuing the bonds.

The difference between the syndicate bid and the reoffering price on a competitive bid of a new municipal underwriting is

the spread.

Which of the following statements is NOT TRUE about defined benefit plans? a. Contributions are based on a predetermined distribution amount b. The employee does not know the amount her employer will contribute each year c. These plans provide tax-free distributions to participants d. These plans are ERISA-qualified retirement plans

c. These plans provide tax-free distributions to participants Explanation: An ERISA-qualified retirement plan is generally established as either a defined contribution or a defined benefit plan. In a defined contribution plan, a specific contribution is made each year and benefits are equal to the amounts provided by the total of contributions and earnings in the plan. A defined benefit plan promises specific benefits at retirement. Contributions to the plan are calculated to provide the promised benefits upon retirement and, therefore, the employee does not know the amount her employer will contribute each year. Distributions from a pension plan are not tax-free and are typically considered ordinary income.

Which of the following descriptions characterizes inverse exchange-traded funds (ETFs)? a. They are designed to deliver the same performance as an index or other benchmark b. They are designed to deliver a multiple of the performance of an index or other benchmark c. They are designed to deliver the opposite of the performance of an index or other benchmark d. They are designed to deliver a multiple of the opposite performance of an index or other benchmark

c. They are designed to deliver the opposite of the performance of an index or other benchmark Explanation: An inverse ETF is designed to deliver the opposite of the performance of an index or other benchmark. For example, an inverse ETF based on the DJIA seeks to deliver opposite performance of that index. So, if the DJIA rises by 1%, an inverse ETF would decrease by 1%, and if the DJIA falls by 1%, the inverse ETF would increase by 1% before fees and expenses. Choice (a) is a regular ETF, choice (b) a leveraged ETF that seeks to deliver a multiple of the performance of an index or other benchmark, and choice (d) is a leveraged inverse ETF.

Long-term certificates of deposit (CDs) have which of the following characteristics? a. They may only be sold by broker-dealers that are subsidiaries of banks b. They are considered risk-free investments c. They may be sold prior to maturity at a price that is different from the client's original cost d. They are not subject to interest-rate risk since the principal is insured by the FDIC

c. They may be sold prior to maturity at a price that is different from the client's original cost Explanation: Long-term CDs have a maturity of more than one year. Since the securities are traded in the secondary market, changes in interest rates will cause price fluctuations. If sold prior to maturity, a CD investor may have a loss or gain. Long-term CDs are issued by banks, but may be sold by any type of broker-dealer. The FDIC provides protection up to $250,000.

If interest rates decline, which of the following securities will probably have the greatest increase in market value? a. Treasury bills b. Commercial paper c. Treasury bonds d. Treasury notes

c. Treasury bonds Explanation: When interest rates decline, the securities with the longest maturities will most likely have the greatest price increase.

All of the following documents are needed to open a new discretionary margin account, EXCEPT a: a. New account form b. Basic customer margin agreement c. Trust agreement d. Power of attorney

c. Trust agreement Explanation: A new account form, a basic customer margin agreement, and a power of attorney are needed to open a new discretionary account. The basic customer margin agreement includes the hypothecation, loan consent, and credit agreements. A trust agreement is needed to open a trust account.

Which of the following choices BEST describes Eurodollars? a. U.S. dollars on deposit in U.S. banks b. U.S. dollars on deposit in European banks c. U.S. dollars on deposit in foreign banks d. European currency on deposit in U.S. banks

c. U.S. dollars on deposit in foreign banks Explanation: Eurodollars are defined as U.S. dollars on deposit in foreign banks, not just in Europe.

Supplemental documentation would NOT be required when opening which of the following types of accounts? a. Guardian b. Partnership c. Uniform Transfers to Minors d. Account for an estate

c. Uniform Transfers to Minors Explanation: A copy of the court appointment of the guardian is necessary for choice (a). To open a partnership account, a copy of the partnership articles should be obtained. In the case of an account for an estate, documentation should be obtained that shows the executor or administrator is properly authorized. Many new account forms contain UTMA/UGMA as one of the standard ownership choices, making additional documentation unnecessary.

The security with the longest expiration date would normally be a: a. Put b. Call c. Warrant d. Right

c. Warrant Explanation: A warrant generally has an expiration date longer than a put, call, or right. There are some warrants which never expire.

To diversify a corporate bond portfolio, which of the following factors is NOT a concern? a. Coupon b. Maturity c. Price d. Geographic location

c. price Explanation: A corporate bond portfolio may be diversified with different issuers, coupons, maturities, and geographic locations. The price of the bonds does not help to diversify.

As the price of the volatility market index (VIX) rises, investors should expect

call and put option premiums to rise.

An initial Regulation T margin call may be met by depositing

cash equal to the call. listed marginable securities with a loan value equal to the call.

NASDAQ Regular Market trading sessions:

coincides with the NYSE trading hours

A "blue chip" corporation experiencing a short term cash flow shortage could issue:

commercial paper

A special situations fund invests in:

companies undergoing a takeover or bankruptcy

Under SEC Rule 10b-10, customer confirmations must be sent at or before

completion of a transaction.

Four municipal bonds maturing in 2039 are all selling at a 7.00 basis. Which of the following bonds is most likely to be refunded? a. 5 1/2% callable in 2024 @ 103 b. 6 1/2% callable in 2023 @ 100 c. 7% callable in 2024 @ 103 d. 7 1/2% callable in 2023 @ 100

d. 7 1/2% callable in 2023 @ 100 Explanation: The most common reason for a municipality to refund an outstanding issue is to save interest costs. If a municipality can borrow money at a lower rate than the outstanding issue, it can use this money to refund the outstanding issue and thus save interest cost. The bonds are selling at a 7.00% yield. The municipality can then expect to borrow new monies at a 7.00% interest rate. The municipality can only save money by refunding an issue with a higher interest rate, 7 1/2%

A customer has a federal tax rate of 35% and a state tax rate of 5%. Which of the following investments would afford him the BEST after-tax yield? a. A 6.40% in-state municipal bond b. A 7.05% out-of-state municipal bond c. A 10.85% investment-grade corporate bond d. A 11.45% real estate investment trust (REIT)

d. A 11.45% real estate investment trust (REIT) Explanation: The major advantage of municipal bonds for most investors is that the interest received from the bond is exempt from federal taxes. In addition, most states also exempt interest from bonds issued within their state from a resident's state and local income taxes. However, if a state resident earns interest from an out-of-state municipal security, that interest is usually subject to state and local taxation. If an investor in a particular tax bracket would like to compare the benefit of tax-free interest income to after-tax income of a taxable bond, it is necessary to find the equivalent taxable yield. The investment grade corporate bond and REIT are fully taxable. Since the investor can purchase an in-state municipal bond and out-of-state municipal bond, we use the combined rate of 40% for the in-state bond and the federal rate of 35% for the out-of- state bond. The formula is: Municipal Bond Yield / (100% - Investor's Tax Bracket) = Equivalent Taxable Yield The customer is in the 40% combined tax rate. The municipal bond has a yield of 6.40%. 6.40% (Municipal Bond Yield) / 60% (100% - 40%) = 10.67% Equivalent Taxable Yield The out-of-state municipal bond has a yield of 7.05% and the equivalent taxable yield is 10.85% (7.05% / 65%). The REIT has the best or highest after-tax yield.

A customer has a federal tax rate of 35% and a state tax rate of 7%. Which of the following investments would afford him the BEST after-tax yield? a. A 6.40% in-state municipal bond b. A 7.10% out-of-state municipal bond c. A 10.95% investment-grade corporate bond d. A 11.50% mortgage bond

d. A 11.50% mortgage bond Explanation: The major advantage of municipal bonds for most investors is that the interest received from the bond is exempt from federal taxes. In addition, most states also exempt interest from bonds issued within their state from a resident's state and local income taxes. However, if a state resident earns interest from an out-of-state municipal security, that interest is usually subject to state and local taxation. If an investor in a particular tax bracket would like to compare the benefit of tax-free interest income to after-tax income of a taxable bond, it is necessary to find the equivalent taxable yield. The mortgage bond is a type of corporate bond and both are fully taxable. Since the investor can purchase an in-state municipal bond and out-of-state municipal bond, we use the combined rate of 42% for the in-state bond and the federal rate of 35% for the out-of-state bond. The formula is: Municipal Bond Yield / (100% - Investor's Tax Bracket) = Equivalent Taxable Yield The customer is in the 42% combined tax rate. The municipal bond has a yield of 6.40%. 6.40% (Municipal Bond Yield) / 58% (100% - 42%) = 11.03% Equivalent Taxable Yield The out-of-state municipal bond has a yield of 7.10% and the equivalent taxable yield is 10.92% (7.10% / 65%). The mortgage bond has the best or highest after-tax yield.

An investor is in the 35% tax bracket. Which of the following investments would afford him the BEST after-tax yield? a. A 3.50% general obligation bond b. A 4.10% Treasury bond c. A 5.25% investment-grade corporate bond d. A 5.75% non-investment-grade corporate bond

d. A 5.75% non-investment-grade corporate bond Explanation: The 3.50% general obligation bond (municipal bond) is exempt from federal income taxes. The other investments are subject to federal income taxes and 35% of the income received would be taxable. The taxable equivalent yield of the 3.50% municipal bond is 5.38%. This is calculated by dividing the 3.50% municipal yield by the complement of the tax bracket which is 65%. The highest (best after-tax yield) would be found in the 5.75% non-investment-grade corporate bond.

Ralph, a New York City resident, sold his apartment for $250,000. He is contemplating purchasing another property within the next 2 to 6 months, but wants to keep the proceeds invested while he is looking. Ralph's primary goals are preservation of capital, liquidity, and limiting his tax liability. Which of the following securities best meets his objectives? a. A corporate bond fund rated AA b. MBIA-insured revenue bonds c. High-grade preferred stock d. A U.S. government money-market fund

d. A U.S. government money-market fund Explanation: A U.S. government money-market fund is not only safe, but the income received by Ralph is exempt from state and local taxes. This is not to be confused with a U.S. government bond fund, which may experience loss of capital if interests rates were to rise sharply. The other investments can result in a loss of capital if interest rates rise.

A limited partnership would be LEAST suitable for which of the following accounts? a. An institutional account b. A trust account c. A corporate account d. A UTMA account

d. A UTMA account Explanation: Of the choices listed, a UTMA (custodian or minor's) account would be least suitable for a limited partnership. A limited partnership generally has limited marketability and a lack of liquidity. In addition, most custodian accounts would not be in a position to benefit from the tax advantages of a limited partnership or a DPP.

A broker-dealer has failed because it has a net capital deficiency. Which of the following parties is NOT covered by SIPC? a. A margin account with $300,000 of securities being held in street name b. A customer account with a $70,000 cash balance c. An IRA with securities valued at $400,000 and $100,000 of cash d. A creditor of the broker-dealer who is owed $40,000

d. A creditor of the broker-dealer who is owed $40,000 Explanation: SIPC provides protection for customer accounts in the event of a broker-dealer's failure. Each account is covered for up to $500,000, of which $250,000 may be cash. SIPC does not insure creditors of the broker-dealer or the failed firm's own inventory account.

A registered representative is reading an article in a popular magazine about the advantages of tax deferral in retirement planning. There is a note that reprints of the article are available. In order to send these reprints to existing and prospective customers,

member alterations to the contents are only to make it consistent with applicable regulatory standards or to correct factual errors.

Which of the following choices does NOT delegate power of attorney to a third party for the purpose of making securities transactions? a. A husband b. A wife c. A corporation d. A custodian

d. A custodian Explanation: Of the choices given, the only one that does not delegate power of attorney to a third party for the purpose of making securities transactions is a custodian for a minor. The custodian acts as the fiduciary for a minor's account and does not delegate a power of attorney.

Which of the following persons may not delegate power of attorney to a third party for the purpose of making securities transactions? a. A husband b. A wife c. A corporation d. A custodian for a minor

d. A custodian for a minor Explanation: Of the choices given, the only one that may not delegate power of attorney to a third party for the purpose of making securities transactions is a custodian for a minor.

An investor who is in his late 80s wants tax-free income and the ability to provide funds for his children after his death. Which of the following choices should an RR recommend? a. An asset allocation mutual fund b. A portfolio of long-term municipal bonds c. A laddered corporate bond portfolio d. A laddered municipal bond portfolio

d. A laddered municipal bond portfolio Explanation: Through municipal bond purchases, investors receive tax-free income. A laddered portfolio is one that invests in the same type of bonds with different maturity dates. When the first bond matures, the proceeds are then reinvested in the long side (i.e., longest maturity) of the ladder. By laddering the portfolio, investors will have greater access to money if the need arises. For instance, if short-term funds are needed (e.g., a person wants to provide funds for his children), the investor may use the amount that is maturing rather than being forced to sell the longer term bonds in the secondary market.

For the following size transactions, ABC mutual fund has a bid price of $8.50 and an asked price of $9.26. Which of the following sales is allowed under the Conduct Rules? a. A member sells 250 shares of ABC fund at $9.10 to a nonmember firm b. A member sells 250 shares of ABC fund at $9.10 to another firm through a nonmember firm c. A member sells 250 shares of ABC fund at $9.10 to one of the firm's customers d. A member sells 250 shares of ABC fund at $9.26 to one of the firm's clients

d. A member sells 250 shares of ABC fund at $9.26 to one of the firm's clients Explanation: A FINRA member firm may not sell the fund at a discount ($9.10) to a nonmember firm or to one of the firm's customers (the public). According to the Conduct Rules, a member firm can only give a discount from the public offering price to another member firm. The discounts and selling concessions member firms give to each other are inducements for firms to be members of FINRA (the self-regulatory organization of over-the-counter broker-dealers) and to abide by its rules and regulations.

Which of the following companies is LEAST affected by changes in the business cycle? a. A construction company b. A machine tool company c. An automobile manufacturer d. A pharmaceutical company

d. A pharmaceutical company Explanation: Changes in the business cycle will have the least effect on a defensive company (such as pharmaceuticals or utilities). The demand for the products produced by defensive companies will not be hurt by a downturn in the economy. The other choices are examples of cyclical companies. These companies tend to parallel the economy of the country. If the economy is expected to prosper, then you can expect a cyclical company to prosper. If the economy is expected to decline, then you can expect a cyclical company to be less prosperous.

Approval by a principal is NOT required when sending a customer which of the following documents? a. An abstract from an Official Statement b. A form letter c. A research report d. A red herring

d. A red herring Explanation: An abstract from an Official Statement, a form letter, and a research report are considered advertising or sales literature and must be approved. A red herring (preliminary prospectus) is used to provide a potential investor with information and is regulated by the SEC.

A type of money-market security usually collateralized by U.S. Treasury securities, in which an investor agrees to lend funds to a broker-dealer for a specified time and rate, is called: a. Federal funds b. A reverse repurchase agreement c. LIBOR d. A repurchase agreement

d. A repurchase agreement Explanation: In a repurchase agreement (repo), a dealer sells securities (usually T-bills) to an investor and agrees to repurchase them at a specific time, at a specified price. In effect, the dealer is borrowing funds from an investor and securing the loan with securities (a collateralized loan). The investor (the lender) receives the difference between the purchase price and the resale price of the securities in return for making the loan. If a dealer purchases securities and agrees to sell them back to an investor at a specific date and price, this is known as a reverse repo or matched sale. In this situation, the dealer lends funds (with securities as collateral) to the investor and earns the difference in sales prices. Many corporations and financial institutions, as well as dealers, engage in repos and reverse repos. Repos and reverse repos are typically short-term, with most being overnight transactions. In most cases the transaction is determined from the dealer's point of view. In other words, if the dealer is borrowing funds and the customer is lending, it is a repo. If the customer is borrowing funds and the dealer is lending it is a reverse repo. If the transaction is between two dealers, determination of whether the transaction is a repo or reverse repo depends on which dealer initiates the transaction.

A customer's account is currently frozen. A registered representative is NOT permitted to accept: a. An order in a cash account if all the money is in the account before the order is entered b. An order in a margin account if the total dollar amount of the purchase is in the account before the order is entered c. A sell order for a security in a cash account if the security is in the cash account before the order is entered d. A sell order for a security in a cash account if the security is not in the account before the order is entered

d. A sell order for a security in a cash account if the security is not in the account before the order is entered Explanation: When an account is frozen, the customer must have in the account what is required to complete the trade before an order may be accepted. This means that the required monies or securities must be in the account prior to accepting any purchase or sale orders. Choice (d) is not permitted as the securities are not held in the account at the time of accepting the sell order.

XYZ Corporation will need to borrow funds in the bond market soon. While current interest rates are not attractive from its viewpoint, the company knows that interest rates could drop suddenly. The company would like to be ready to sell the bonds quickly. It would also like the bonds to be as liquid as possible in order to attract investors. Which of the following choices is most appropriate for its needs? a. A private placement under Regulation D b. An intrastate offering under Rule 147 c. A traditional registration statement d. A shelf registration under Rule 415

d. A shelf registration under Rule 415 Explanation: While the sales described in choices (a), (b), and (d) will usually be faster than a full registration, both Regulation D and Rule 147 place various restrictions on resales, reducing the liquidity of the issue. A shelf registration under Rule 415 will satisfy all of XYZ Corporation's needs.

Which of the following positions exposes an investor to the most risk? a. A bullish call spread b. A bullish put spread c. Owning put options d. A short straddle

d. A short straddle Explanation: A short straddle consists of a short call and a short put, on the same underlying stock, with the same strike price and expiration month. The investor has an unlimited loss potential on the short call leg of the straddle. Spread positions limit the potential loss to the investor. For debit spreads (i.e., bullish call spreads and bearish put spreads), the loss is limited to the difference between the premiums. For credit spreads (i.e., bearish call spreads and bullish put spreads), the loss is limited to the difference between the strike prices minus the credit. The owner of a put option is only at risk for the premium paid to purchase the option.

Which of the following municipal entities would NOT issue overlapping debt? a. A park district b. A library district c. A school district d. A turnpike authority

d. A turnpike authority Explanation: Overlapping debt involves only general obligation borrowing. A turnpike authority would typically issue only revenue bonds.

Which of the following services does NOT rate fixed-income securities? a. Moody's b. Standard & Poor's c. Fitch d. AMBAC

d. AMBAC Explanation: AMBAC insures new municipal bond issues. Moody's, Standard & Poor's, and Fitch are credit rating services.

When a broker-dealer sells a security to a client and charges a commission on the transaction, it is acting as the client's: a. Market maker b. Principal c. Designated market maker d. Agent

d. Agent Explanation: A broker-dealer that buys securities from or sells securities to a client without owning the securities is acting as the client's agent or broker. The broker-dealer does not have any risk and the client pays a commission on this type of transaction. When acting in a principal capacity, the client is charged a markup or markdown.

Which of the following choices is NOT approved by a municipal securities principal? a. A research report sent to a customer regarding municipal securities b. Each account engaging in municipal securities transactions c. All advertising relating to municipal securities d. All orders to buy municipal bond funds

d. All orders to buy municipal bond funds Explanation: A municipal securities principal must promptly approve (i.e., initial) all transactions for municipal securities but not municipal security bond funds, correspondence, and advertising. Research reports are considered advertising.

An investor purchases a two-year ABC call. Which of the following designations accurately describes the exercise of the option? a. European style, next business day settlement b. European style, three business days' settlement c. American style, next business day settlement d. American style, three business days' settlement

d. American style, three business days' settlement Explanation: Long-term anticipation securities (LEAPS) may be exercised on any day prior to expiration (American style). Exercise settlement is in the underlying stock, in three business days.

Pennsylvania Power Company has announced that it will refund $800 million of its outstanding 6 1/4% bonds that were to mature in 2040. The bonds will be refunded at 106.75% of par value using the proceeds of an $800 million refunding issue. The refunding issue has a 4 1/2% coupon rate and matures in 2030. The refunding will reduce all of the following items, EXCEPT the: a. Interest cost to the issuer b. Company's maturity schedule c. Company's annual debt service obligation d. Amount of outstanding debt

d. Amount of outstanding debt Explanation: Pennsylvania Power has reduced the interest charges (from 6 1/4% to 4 1/2%) by advancing the repayment of its existing debt. The company is paying off the 6 1/4% bonds, due in 2040, with the issuance of another bond. It is also improving its debt service by reducing the interest rate from 6 1/4% to 4 1/2%, thereby reducing its fixed charges each year. However, the amount of debt outstanding will remain the same at $800 million.

If a customer's objectives are safety of principal and income, you as the registered representative would NOT suggest: a. AA-rated corporate bonds b. High-grade preferred stocks c. A bond fund which invests in investment-grade municipal bonds d. An Exchange-Traded Fund that tracks the S&P 500 Index

d. An Exchange-Traded Fund that tracks the S&P 500 Index Explanation: An ETF that tracks the S&P 500 Index invests in common stocks that will not pay a high dividend and will fluctuate in value with the general equity market. This customer wants income and safety of principal, which may be found in the other three investment choices.

Which of the following securities are based on the credit rating of the issuer? a. An exchange-traded fund (ETF) b. A mutual fund that contains only non-investment-grade securities c. A closed-end bond fund d. An exchange-traded note (ETN)

d. An exchange-traded note (ETN) Explanation: Exchange-traded notes (ETNs) are a type of unsecured debt security. ETNs carry issuer risk that is tied to the creditworthiness of the financial institution backing the note. If the issuer's financial condition deteriorates, it can impact the value of the ETN negatively, regardless of how its underlying index performs. The credit rating of the securities included in a mutual fund, closed-end bond fund company, or ETF has an impact on these types of securities. These securities are not affected by the ratings of the company that is issuing the fund or ETF

A registered representative has established a discretionary municipal bond account for Mr. Smith. Which of the following securities may the registered representative purchase for Mr. Smith's account? a. General obligation bonds only b. Bonds rated Baa or higher only c. Double-barreled bonds only d. Any suitable bonds

d. Any suitable bonds Explanation: A registered representative must be sure that the bonds purchased are suitable.

An increase in which of the following factors does NOT indicate credit conditions are deteriorating for a municipality? a. Bankruptcies b. Consumer debt c. Bond defaults d. Assessed valuations

d. Assessed valuations Explanation: All of the items mentioned would indicate credit conditions are deteriorating for a municipality except an increase in assessed valuations. This is the value placed on property by the municipality for purposes of taxation. An increase in assessed valuations would indicate that homes within the municipality are increasing in value, which will improve the municipality's credit.

Which of the following Moody's ratings is the most speculative? a. Aa b. A c. Baa d. Ba

d. Ba Explanation: Of the choices given, Ba is the most speculative. The highest Moody's rating is Aaa.

An established customer has purchased penny stocks through a broker-dealer on five occasions. When making future recommendations to the customer regarding these securities, the broker-dealer must: a. Obtain a written statement from the customer for each trade b. Have the customer sign a suitability statement for each trade c. Have the trades preapproved by a principal d. Be sure that the recommendations take into account the customer's investment objectives

d. Be sure that the recommendations take into account the customer's investment objectives Explanation: The account approval requirements for penny stocks under SEC Rule 15g-9 do not apply to existing customers who have maintained an account with a broker-dealer for more than one year or have previously engaged in three or more transactions involving penny stocks. All recommendations to a customer should take into account the customer's investment objectives.

Which of the following option positions is an example of a combination? a. Buy an XYZ June 60 call and sell an XYZ June 65 call b. Buy an XYZ June 60 call and buy an XYZ June 60 put c. Sell an XYZ June 60 call and sell an XYZ June 60 put d. Buy an XYZ June 60 call and buy an XYZ June 65 put

d. Buy an XYZ June 60 call and buy an XYZ June 65 put Explanation: A long straddle is defined as the simultaneous purchase of two options that have the same expiration and strike price, but consist of one call and one put. A short straddle is defined as the simultaneous sale of two options that have the same expiration and strike price, but consist of one call and one put. Choice (b) is a long straddle and choice (d) is a short straddle. A combination is similar to a straddle; however, the strike prices and/or expirations must be different. Choice (c) is a long combination. A spread is defined as the simultaneous sale and purchase of two options of the same class (same stock and same type of option), but it will have different strike prices and/or expirations. Choice (a) is a spread.

Who enacts fiscal policy? a. The Federal Reserve Board b. The Comptroller of the Currency c. The FDIC d. Congress

d. Congress Explanation: Fiscal policy is enacted by Congress. Fiscal policy is the use of the government's power to tax and spend. Control of the economy by changing the levels of government spending and taxation can either put money into the economy, or take money out of the economy. Monetary policy is carried out by the Federal Reserve Board's use of its available options for increasing or decreasing the supply of money and credit in the economy.

A corporation with an excellent earnings record has several issues of bonds outstanding. During a period of stable interest rates, which of the following securities are expected to fluctuate the most? a. Mortgage bonds b. Commercial paper c. Debenture bonds d. Convertible bonds

d. Convertible bonds Explanation: The convertible bonds will fluctuate the most because they are convertible into common stock. The price fluctuates with the price movements of the common stock. The fact that interest rates are stable is another reason why convertible bonds is the best answer. If the question had stated that interest rates were moving sharply upward or downward, then all other bonds would fluctuate sharply in price to bring yields in line with interest rates. However, the question asks what will happen in a period of stable interest rates. Given that statement, the best answer is that convertible bonds will fluctuate the most.

All of the following information should be obtained by a registered representative when opening a new account for a customer, EXCEPT the: a. Street address b. Tax identification number c. Occupation d. Education

d. Education

When a corporation goes bankrupt, which of the following would be the last to be paid? a. Internal Revenue Service b. Debenture holders c. Preferred stockholders d. Holders of warrants

d. Holders of warrants Explanation: A holder of common stock is usually the last to be paid in a bankruptcy. A holder of a warrant has the right to purchase common stock and is paid after a holder of common stock. In practical terms, the common stock is worthless if a corporation declares bankruptcy.

All of the following statements are TRUE of Blue-Sky laws, EXCEPT: a. Broker-dealers are required to register in each state in which they do business b. Registered representatives are required to be registered in each state in which they do business c. When offering its shares to the public, an issuer is required to register in the states where it sells its securities d. If securities are exempt from state securities registration, they are also exempt from federal securities registration

d. If securities are exempt from state securities registration, they are also exempt from federal securities registration Explanation: All of the choices listed regarding Blue-Sky laws are true except the statement that if securities are exempt from state securities registration, they are also exempt from federal securities registration. The opposite is true. For example, securities listed on the NYSE are subject to federal securities registration, but exempt from state securities registration. Blue-Sky laws are state securities laws that are not part of the Securities Act of 1933. Blue-Sky laws regulate the issuance and trading of securities within the states (intrastate). The 1933 act is a federal act that regulates new securities issues that will be sold interstate. Issuers raising capital in a state, broker-dealers and RRs conducting business in a state must register in that state.

A registered representative tells a municipal bond fund manager that, in return for purchasing bonds for the fund, the representative will sell that bond fund exclusively to clients. This arrangement is: a. Permitted provided that it is disclosed to customers b. Permitted without restriction c. Permitted if approved by a principal d. Prohibited according to MSRB rules

d. Prohibited according to MSRB rules Explanation: MSRB rules prohibit a broker-dealer or registered representative from having reciprocal dealings with investment companies. A registered representative may promote the services of the firm (such as timely executions and quality research) to solicit business from a municipal bond fund manager, but may not offer to sell fund shares in return for brokerage business from the fund.

The Barge Towing Corporation has announced in a tombstone ad that it will issue $5,000,000 of 10% convertible subordinated debenture bonds convertible into common stock at $10.50. The bonds will mature in November 2030 and are being issued at their $1,000 par value. Which of the following statements is TRUE? a. The bonds are being issued at a premium b. The bonds are being issued at a discount c. The bonds can be redeemed by holders before November 2030 d. If the company should go bankrupt, the subordinated debenture holders would be paid after all other bondholders and general creditors but before common stockholders

d. If the company should go bankrupt, the subordinated debenture holders would be paid after all other bondholders and general creditors but before common stockholders Explanation: The bonds are subordinated debenture bonds and are issued at par value. If the company should go bankrupt, the subordinated debenture holders will be paid after all other bondholders and general creditors, but before common stockholders.

A client is interested in obtaining the expense ratio of a mutual fund recommended by the RR. Which of the following actions would be BEST for the RR to take? a. Instruct the client to obtain the information from FINRA b. Refer the client to the fund's sponsor since the RR may not be authorized to release this information c. Instruct the client to obtain that information from the SEC database of mutual fund prospectuses d. Inform the client that this information may be obtained by reviewing the front of the fund's prospectus

d. Inform the client that this information may be obtained by reviewing the front of the fund's prospectus Explanation: Mutual funds are required to disclose in the front of a prospectus a standardized fee table of all its fees. The fee table must include the expense ratio, which is the percentage of a fund's assets that is used to pay its operating costs. It is determined by dividing total expenses by the average net assets in the portfolio.

A customer wants safety of income and preservation of capital. Which of the following securities is MOST suitable? a. High-yield corporate bonds b. Collateralized mortgage obligations c. Convertible bonds d. Investment-grade corporate bonds

d. Investment-grade corporate bonds Explanation: Investment-grade (highly rated) corporate bonds offer an investor safety of income and preservation of capital. The risk of default is minimal. The investor realizes income as well as preservation of capital. The other choices offer income, but have a higher degree of capital risk and, therefore, less preservation of capital.

A husband and wife with children going to college in 2, 11, and 16 years are planning to set up an account to pay for their children's college education. Which of the following investments are most suitable for this purpose? a. Money-market funds b. Certificates of deposit maturing every 12 months c. Junk bonds with serial maturities coinciding with the children's college attendance d. Investment-grade corporate bonds with maturities coinciding with the children's college attendance

d. Investment-grade corporate bonds with maturities coinciding with the children's college attendance Explanation: Given these choices, the investment-grade bonds with serial maturities of 2, 11, and 16 years appear to be the most suitable investment. Money-market funds are used more as a parking place for funds until an investment decision can be made. CDs may be used, but are not as attractive as choice (d) since the CDs mature in 12 months. Junk bonds carry too much risk for their intended purpose.

A high net worth investor seeking safety of principal would MOST likely invest in: a. Non-investment-grade municipal revenue bonds b. Non-investment-grade corporate bonds c. The maximum amount allowable in a 529 plan d. Investment-grade municipal revenue bonds

d. Investment-grade municipal revenue bonds Explanation: Safety of principal refers to a customer being able to preserve or retain the initial amount of the investment over its life. Many bonds will offer investors this feature. The higher the rating, the greater the likelihood the investor will achieve safety of principal. Investment- grade municipal revenue bonds will offer safety of principal and will also offer a high net worth investor tax-exempt income. A 529 plan would be beneficial if the investor's objective were tax- advantaged funding for a child's college education.

Which of the following choices gives the best indication of current interest rates on revenue bonds? a. Visible supply b. Placement ratio c. List of 20 bonds d. List of bonds with 30-year maturities

d. List of bonds with 30-year maturities Explanation: The Bond Buyer computes the Revenue Bond Index which is the average yield of 25 revenue bonds with 30-year maturities.

Which of the following transactions may be executed in a cash account? a. Short 100 shares of XYZ stock b. Long 500 shares of ABC stock and short 500 shares of ABC stock c. Short 10 DEF May 50 call options d. Long 5 GHI Jan 15 put options

d. Long 5 GHI Jan 15 put options Explanation: A purchase of options may be transacted in a cash or margin account. Selling short stock and short option positions must be executed in a margin account.

A brochure from a broker-dealer sent to customers concerning collateralized mortgage obligations (CMOs) is permitted to contain: a. A comparison between CMOs and Treasury securities b. A comparison between CMOs and bank certificates of deposit c. A comparison between CMOs and municipal securities d. No comparison to other securities

d. No comparison to other securities Explanation: Any type of promotional communication made available to customers may not compare CMOs to any other security. This is due to the uniqueness of this product.

Which of the following choices is NOT a characteristic of a HOLDR? a. Diversification b. The right to vote c. The ability to control when you are taxed d. Once-a-day pricing

d. Once-a-day pricing Explanation: Holding Company Depository Receipts (HOLDRs) are created by depositing securities of a certain sector (e.g., Biotech, Internet, Retail) into a trust and selling interests in the trust to investors. HOLDRs offer investors diversification similar to an Exchange-Traded Fund (ETF). Unlike ETFs, the owner of a HOLDR has an ownership interest in the shares of the companies that the HOLDR is invested in and would retain the right to vote. Once the portfolio has been created, the makeup of the portfolio will typically not change, although if a company included in the portfolio goes bankrupt or merges with another company, the makeup of the HOLDR may be altered. Investors have the ability to control when they are taxed, since they determine when to hold or sell the HOLDR. An investor in a mutual fund does not have that benefit since the portfolio manager would determine when to hold or sell the securities in the fund. Benefits also include liquidity and pricing throughout the day (i.e., they are exchange-traded) as compared to an index or sector mutual fund, which has daily pricing and is purchased directly from the fund. HOLDRs have no management fees and are considered low-cost since there are only small transaction costs and custodian fees.

Which of the following advantages is NOT a benefit of owning a real estate investment trust? a. Stable dividend income b. The ability to buy and sell shares easily c. Diversification d. Protection against rising interest rates

d. Protection against rising interest rates Explanation: Real estate investment trusts (REITs) offer investors a stable dividend based on the income produced by owning a diversified portfolio of properties and/or mortgages. Most REITs trade on an exchange, offering investors liquidity. Since investors usually purchase REITs for their high dividend yield, if interest rates increase, the value of their shares will usually decrease as other newly issued income earnings securities become more attractive.

The MSRB performs all of the following functions, EXCEPT: a. Regulate municipal securities dealers b. Regulate municipal securities representatives c. Regulate municipal securities advertising d. Set fixed commissions for municipal dealer agency transactions

d. Set fixed commissions for municipal dealer agency transactions Explanation: The MSRB does not set fixed commissions for municipal dealer agency transactions. MSRB rules regarding commissions state that they shall be fair and reasonable and negotiated between buyer and seller.

Which of the following statements is NOT TRUE concerning a structured product offered by an RR? a. They are usually registered with the SEC b. The principal that the investor would receive may be based on the value of a stock traded on an exchange c. The principal the investor would receive may be based on the value of a foreign currency d. Since this product is usually sold by a bank, the principal will be protected by the FDIC

d. Since this product is usually sold by a bank, the principal will be protected by the FDIC Explanation: Structured products may be linked to individual securities, commodities, foreign currencies, or indexes. These products are underwritten by most major financial services institutions and are usually registered as securities with the SEC. Structured products are not bank deposits and are not insured by the Federal Deposit Insurance Corporation (FDIC). This fact should be disclosed by an RR when offering this product to clients.

In which of the following situations does an investor have unlimited risk? a. Sold a call and is long the stock b. Sold a put and is long the stock c. Bought a call and is short the stock d. Sold a put and is short the stock

d. Sold a put and is short the stock Explanation: Selling a put and being short stock would be the only choice given where an investor would have unlimited risk. The short position would be the unlimited risk situation if the stock were to increase in value. If the market value of the stock is increasing, the purchaser of the put will not exercise the option. The short seller will lose money on the increase of the stock price. In choice (c), the short seller is protected against a rise in the stock by owning a call. In choice (a), the investor will have a loss if the price of his stock declined. However, the potential loss is limited since the stock's price can only decline to zero, creating a loss equal to the stock's cost minus the premium received for selling the call. In choice (b), the loss will again be limited to the stock's value declining to zero.

Which of the following securities will MOST likely be subject to a withholding tax? a. An initial public offering (IPO) b. A real estate investment trust (REIT) c. A bond issued by a U.S. company that earns income overseas d. Stock issued by a foreign company that earns income in the U.S.

d. Stock issued by a foreign company that earns income in the U.S. Explanation: Choice (d) is an example of an ADR, representing stock issued by a foreign corporation that is traded in the U.S. Dividends paid to a U.S. investor on foreign securities, such as an ADR, may be subject to a withholding tax by the country from which they were paid. If the investor has securities that paid dividends that were subject to a foreign tax, the broker-dealer will send the investor a form that will report the gross amount of the dividends or interest and the amount of tax withheld by the foreign government. The fact that the company earns income in the U.S. is not relevant.

Which of the following choices represents the percentage of new municipal issues brought to market during a particular week that has already been sold? a. The Bond Buyer Index b. The Blue List c. The Visible Supply d. The Placement Ratio

d. The Placement Ratio Explanation: The placement ratio represents the percentage of new municipal bond issues of $5,000,000 or more that has been sold in a particular week. The Bond Buyer compiles this ratio.

Which of the following indexes or averages is made up of the largest number of stocks? a. The Dow Jones Composite Index b. The S&P 500 Index c. The NYSE Index d. The Wilshire Associates Equity Index

d. The Wilshire Associates Equity Index Explanation: The Wilshire Associates Equity Index shows the market value in dollars of roughly 7,000 NYSE, NYSE MKT (formerly NYSE Amex), and Nasdaq stocks. It contains the most stocks of the choices listed.

To determine what would happen to the coverage of revenue bonds when more bonds are going to be issued in the future, one should examine: a. The rate covenants of the bond b. Feasibility studies c. The refunding procedure of the bond d. The additional bonds test

d. The additional bonds test Explanation: The additional bonds test sets a minimum level of coverage of debt service for interest and principal for all outstanding bonds and for future debt. The additional bonds test protects original bondholders against the dilution of the debt service coverage. Rate covenants insure that rates will increase in line with costs to insure proper revenues for the maintenance of the facility or project and payment of the debt service. Feasibility studies are conducted to insure the proper need of the project being developed. Refunding is used to lower interest expense on bonds through the issuance of new bonds at lower coupon rates. The proceeds of the new bond sale would be used to repurchase the already outstanding high-coupon bonds.

What information would NOT need to be disclosed by a broker-dealer in a research report? a. The broker-dealer received compensation for assisting the company in an acquisition b. The analyst provided a target price for the company c. The analyst is a director of the company d. The analyst had owned shares in the company one year before writing the report

d. The analyst had owned shares in the company one year before writing the report Explanation: A broker-dealer is required to make certain disclosures in its research reports. Any investment banking compensation paid during the last 12 months, the anticipated price target, and the fact that the analyst is a director of the company are all required disclosures. In addition, any ownership in the company held by the analyst or a member of the analyst's immediate family at the time the report is issued must be disclosed. The fact that the analyst formerly owned shares that were sold does not need to be disclosed.

According to MSRB rules, which of the following situations will cause a broker-dealer to reject the delivery of municipal bonds? a. A legal opinion is attached to, rather than imprinted on, the bonds b. Registered bonds are expected and delivered with proper endorsement c. Bearer bonds are delivered in $5,000 denominations d. The bonds have been called by the issuer

d. The bonds have been called by the issuer Explanation: Good delivery for municipal bonds, unless otherwise specified, requires delivery of bonds that have not been called and have an imprinted or attached legal opinion. The bonds may be in bearer or registered form. Bearer bonds may be delivered in $1,000 or $5,000 denominations. If delivery of registered bonds is expected, they must be endorsed properly and in any denomination from $1,000 to a maximum of $100,000, in $1,000 increments.

A call option would be considered covered if it was written against all of the following choices, EXCEPT: a. The underlying common stock held in a cash account b. The underlying common stock held in a bank c. The underlying common stock held in a trust company d. The convertible bonds or convertible preferred stock of another corporation

d. The convertible bonds or convertible preferred stock of another corporation Explanation: The option would be considered covered if written against all of the choices listed except the convertible preferred stock or convertible bonds of another corporation. A security that is convertible into common stock is acceptable, but it must be of the same corporation and be convertible immediately into at least the same number of shares represented by the options written.

The separate account of a variable life policy has performed poorly for some time. A client is concerned about her cash value and death benefit. Which of the following statements is TRUE? a. The cash value is guaranteed b. The cash value may decline, but not below a contractual minimum c. The death benefit is the lesser value of the contract or the amount of the investment d. The death benefit may decline, but not below a contractual minimum

d. The death benefit may decline, but not below a contractual minimum Explanation: If the performance of the separate account of a variable life insurance policy is less than the assumed interest rate (AIR), the death benefit will decline. However, the death benefit can never drop below the face value of the policy. The cash value may also decline. However, there is no guaranteed minimum.

Which of the following ratios would be used by an analyst examining the capital structure of an industrial corporation? a. The current ratio b. The dividend payout ratio c. The price/earnings ratio d. The debt-to-equity ratio

d. The debt-to-equity ratio Explanation: The capital structure of a corporation is the dollar amount of the corporation's capitalization (equity and debt securities). An analyst will, therefore, be interested in the debt-to-equity ratio. This is actually the ratio of those securities creating fixed charges (bonds plus preferred stock) to common stock.

Which of the following rates is set by the Federal Reserve Board? a. The broker call rate b. The federal funds rate c. The prime rate d. The discount rate

d. The discount rate Explanation: Of the choices given, only the discount rate is set by the Federal Reserve Board. The prime rate is the rate of interest that commercial banks charge their best-rated customers and is established by each bank. The call rate, which is the rate of interest charged to brokerage firms for margin loans, is set by each bank. The federal funds rate is the charge for overnight loans between banks and is set by each bank.

An employee of a municipal securities firm would like to open an account with another municipal securities firm. Which of the following statements regarding the employee and the account is NOT TRUE? a. The employer must receive duplicate copies of all transactions made in the account b. The employee must abide by all trading instructions from the employer regarding the account c. The employer must be notified in writing of the employee's intention to open the account d. The employee may not purchase new issues offered on a negotiated basis

d. The employee may not purchase new issues offered on a negotiated basis Explanation: MSRB member firm employees may open an account with a competing firm, provided they abide by the requirements addressed in choices (a), (b), and (c). The employee may purchase new issues or any other type of municipal security.

A person maintains an IRA account and has contributed only pretax dollars. Withdrawals from this account will be treated as which of the following? a. A return of capital b. A portion will be taxable as ordinary income and a portion will be tax-free c. A portion will be taxable as a capital gain and a portion will be tax-free d. The entire withdrawal will be taxable as ordinary income

d. The entire withdrawal will be taxable as ordinary income Explanation: The earnings on an IRA account grow tax-deferred. If only pretax contributions are made, the entire withdrawal is taxable as ordinary income. Withdrawals from a tax-deferred account are never taxable as a capital gain. If an investor maintains an IRA account that has pretax and after-tax contributions and makes withdrawals, the IRS considers withdrawals to come from both sources. Therefore, a portion of the withdrawal is taxable and the other portion is tax-free.

All of the following choices are characteristics of a Health Savings Account (HSA), EXCEPT: a. The amount a person may contribute each year is limited b. It is not open to individuals who are enrolled in their company's health insurance plan c. The funds may be invested in mutual funds d. The funds must be used each year and may not be carried over

d. The funds must be used each year and may not be carried over Explanation: A Health Savings Account (HSA) is a tax-advantaged account that can be used by individuals to pay for qualified medical expenses. An HSA is not open to all individuals. It is generally open only to persons who are not enrolled in any type of health plan other than a qualified, high-deductible health plan. Contributions are made in pretax dollars (which are limited under IRS guidelines), grow tax-free, and withdrawals are tax-free if used to pay qualified medical expenses. The funds may be invested in mutual funds, although the types of funds may be limited by an HSA trustee (a financial institution). The funds do not need to be used each year and may be carried over to be used in the future. Once you reach age 65, your funds can be withdrawn at any time and are subject only to ordinary income tax. (You avoid the 20% IRS penalty.) However, you may avoid any tax by continuing to use the funds for qualified medical expenses.

listing of coincident economic indicators includes: a. Expenditures for capital goods b. The S&P Index c. Housing starts d. The industrial production index

d. The industrial production index Explanation: The industrial production index is a coincident indicator. Coincident indicators tend to move directly with the business cycle and show what is currently happening in the economy.

Which of the following statements is TRUE concerning reverse convertible securities? a. An investor will receive a coupon rate below prevailing market rates b. An investor is anticipating a decrease in the value of the underlying asset c. They would be suitable for an investor who wants to own shares of the underlying asset d. The investor is anticipating that the price of the underlying asset would be above the knock-in value

d. The investor is anticipating that the price of the underlying asset would be above the knock-in value Explanation: Reverse convertible securities are short-term notes issued by banks and broker-dealers that usually pay a coupon rate above prevailing market rates. They are considered structured products because, in addition to the coupon rate, the investor may be required to purchase shares of an underlying asset at a fixed price. The underlying asset may be an equity security unrelated to the issuer, or a basket of stock, or an index. The issuer agrees to pay this higher coupon rate since it has an option to sell a security to the investor if the price of the security falls below a specified value known as the knock-in level. If the price of the underlying asset stays above the knock-in level, the investor will receive the high coupon and the full return of her principal (the most beneficial option). If the underlying asset falls below the knock-in level, the investor will be obligated to purchase shares of the underlying asset at a fixed price. The price of this asset may have depreciated below the knock-in level and the investor may receive substantially less than the original principal. The investor is anticipating a stable price for the underlying asset and is not able to participate in any increase in the value of the underlying asset. Choice (c) is incorrect since the investor does not want to own the underlying asset.

If a registered representative opens an account for an investor, the registered representative would need to know all of the following information, EXCEPT: a. The investor's tax identification number b. If the investor is a U.S. citizen c. The investor's investment objectives d. The investor's educational background

d. The investor's educational background Explanation: A registered representative does not need to know the customer's educational background to open an account. A reasonable attempt should be made to obtain all the other information when opening an account for a customer.

A 4.65% New York City GO bond matures in 20 years. The bond is callable in 8 years at 103. Which of the following statements is TRUE? a. The investor has 3 years of call protection b. The issuer must pay investors an 8-point call premium to exercise the call privilege on the bonds c. The investor will receive less for the bond if it is called versus holding the bond to maturity d. The issuer may exercise the call provision anytime after the 8th year

d. The issuer may exercise the call provision anytime after the 8th year Explanation: The call premium of 3 points ($30 per bond) refers to the amount above par value which the issuer must pay the owner of the bond when the bond is called. Issuers usually call outstanding bonds when interest rates decline, and they are able to issue new bonds at lower rates of interest. The bond has 8 years of call protection. The issuer would need to make an outlay of cash to call back the bonds, but would save money because of the lower rate of interest the issuer would pay on the new bonds. A call provision is exercised by an issuer and not the bondholder.

When making a presentation on 529 plans, what information is NOT required? a. Discussing the risks and costs involved with the different types of plans b. A disclaimer stating that, prior to investing in a plan, you should read the official statement c. A disclaimer that the client should check with her home state to learn if it offers tax benefits to those clients who invest in its plan d. The name and contact information for the municipal securities principal who will approve the customer's investment in the plan

d. The name and contact information for the municipal securities principal who will approve the customer's investment in the plan Explanation: Under MSRB rules, an RR is required to disclose certain information when promoting 529 plans. The RR must discuss the risks and costs involved with the different types of plans, must provide a disclaimer stating that, prior to investing in a plan, the customer should read the official statement, and must provide a disclaimer that the client should check with her home state to learn if it offers tax benefits to those who invest in its plan. There is no requirement to provide the name and contact information for the municipal securities principal who will approve the customer's investment in the plan.

When pricing a bond, what information is NOT required? a. The coupon rate b. The maturity date c. The settlement date d. The number of bond years

d. The number of bond years Explanation: When pricing a bond (determining the yield when price is known or determining the price when yield is known), the coupon, settlement date, and maturity are required. The number of bond years is used to determine the net interest cost when an underwriting is bidding on a new issue of municipal bonds.

Which of the following statements is NOT TRUE regarding the purchaser of a put option? a. The purchaser has a right to sell stock b. The purchaser limits the amount of money he could lose if the value of the underlying stock increases c. The purchaser benefits if the value of the underlying stock declines d. The only way to realize a profit is to exercise the option

d. The only way to realize a profit is to exercise the option Explanation: Choice (d) is not true. The investor could profit by either exercising or liquidating the put. The other choices are true statements. The purchaser of a put has a right to sell stock. The maximum loss that a purchaser of an option (put or call) can sustain is the amount of the premium paid. The purchaser of a put can profit if the underlying stock declines in value.

Which of the following BEST describes the term, payment for order flow? a. The concession paid to a member of a selling group b. The prohibited practice of mutual fund distributors paying kickbacks to registered representatives c. The portion of a mutual fund's 12b-1 fee that is paid to a third-party salesperson d. The payment to a broker-dealer by a market maker in return for routing orders to that market maker

d. The payment to a broker-dealer by a market maker in return for routing orders to that market maker Explanation: It is permissible for broker-dealers to receive payments from a market maker in return for executing orders through that market maker. Any payments for order flow must be disclosed to customers.

Under NYSE rules, a not-held order

is good for the day only.

High-tax-bracket investors are likely to receive the most favorable tax treatment from investing in

municipal bonds issued by a political subdivision of their state.

All of the following municipal bond transactions take place in the secondary market, EXCEPT: a. Submitting an offer to sell bonds to a sinking fund b. A tax swap c. Two municipal securities broker-dealers purchasing a block of bonds jointly d. The placing of a designated order

d. The placing of a designated order Explanation: All of the choices listed take place in the secondary market except the placing of a designated order. A designated order is an order directed to a syndicate manager by an institutional account designating two or more members of the underwriting account to receive credit for that order. This order is placed directly with the syndicate manager during the order period prior to the release of the bonds for secondary trading.

A customer does not have a discretionary account with his brokerage firm. The firm may decide which of the following factors for the customer? a. Whether to buy or sell b. The quantity if the customer specifies the security and price c. The security if the customer specifies the quantity and price d. The price if the customer specifies the security and quantity

d. The price if the customer specifies the security and quantity Explanation: If a customer indicates the specific stock and amount that he wishes to buy or sell, the registered representative is permitted to choose the time and price of execution. This would not be considered a discretionary order and written trading authorization would not be necessary.

Volume and holding-period restrictions do NOT apply to the resale of private placements when: a. Purchasers' representatives assist investors b. Both parties are accredited investors c. The transaction is initiated by a registered principal d. The purchaser is a qualified institutional buyer

d. The purchaser is a qualified institutional buyer Explanation: Under Rule 144A of the Securities Act of 1933, the owner of securities obtained through a private placement may resell those securities to a qualified institutional buyer (QIB) without the volume and holding-period restrictions of Rule 144. Qualified institutional buyers must have at least $100 million dollars of investable assets.

All of the following statements are TRUE concerning private activity bonds, EXCEPT: a. The interest on these bonds might not be tax-exempt for some investors b. The interest on these bonds might be subject to the alternative minimum tax c. The possibility that the bonds might be subject to taxation would be reflected in the yield at which the bond trades d. These types of municipal bonds are generally GOs

d. These types of municipal bonds are generally GOs Explanation: Private activity bonds are issued to finance the construction of a facility that will be used by a private corporation. Interest earned on such bonds is often subject to the Alternative Minimum Tax (AMT). The AMT is a second method of calculating federal income tax liability. Taxpayers must pay the larger of the AMT or the result of the regular (Form 1040) income tax calculation. In theory, this is true for all taxpayers but, in reality, the AMT is only an issue for higher income taxpayers or those with special tax preference items on their returns. When calculating the alternative minimum tax, certain items may need to be included in taxable income that normally are not. One of these items is the interest on many private activity bonds. Therefore, a taxpayer subject to the AMT may lose the tax exemption on these bonds. Since this is a disadvantage, these bonds generally trade with higher yields than regular municipal bonds to reflect that the interest received might be taxable. Choice (d) is correct because private activity bonds are generally revenue bonds, not general obligation bonds.

Which of the following statements is TRUE regarding dividend and capital gain distributions of mutual funds? a. They may be combined to determine the total yield b. The taxes may be deferred if they are invested in additional mutual fund shares c. Dividends are taxed as long-term capital gains d. They can be reinvested automatically in additional shares if the shareholder chooses to do so

d. They can be reinvested automatically in additional shares if the shareholder chooses to do so Explanation: Of the choices listed, the only true statement regarding dividend and capital gain distributions from an open-end investment company (mutual fund) is that they can automatically be reinvested in additional shares if the fundholder chooses to do so. They may not be combined to determine yield. The taxes must be paid in the year that the distribution is realized, even if the distribution is reinvested. Qualified dividends are taxable at a maximum rate of 20% regardless of how long the fund is held and nonqualified dividends are taxed as ordinary income. Long-term capital gain distributions are taxable as long-term capital gains regardless of how long the fund shares are held.

All of the following statements are TRUE concerning both auction rate securities (ARSs) and variable-rate demand obligations (VRDOs), EXCEPT: a. Interest rates are set at specified intervals b. They are often issued by municipalities c. They are long-term securities with short-term trading features d. They have a put feature allowing the holder to redeem the security at par

d. They have a put feature allowing the holder to redeem the security at par Explanation: Although they are both long-term securities with short-term trading features, only VRDOs have a put feature that permits the holder to sell the securities back to the issuer or third party. Auction rate securities (ARSs) do not have this feature and, if the auction fails, the investor may not have immediate access to her funds. In addition, ARSs use an auction process to reset the interest rate on the securities, whereas the interest rate on a VRDO is reset by the dealer at a rate that allows the securities to be sold at par value.

A registered representative is sending an e-mail to all her customers in anticipation of a new product being offered by the firm. If her customers consist of a large group of individual investors with assets from $100,000 to $100,000,000, which of the following statements is TRUE? a. This would be defined as a correspondence b. This would be defined as an institutional communication c. This would be defined as both a retail and an institutional communication d. This would be defined only as a retail communication

d. This would be defined only as a retail communication Explanation: FINRA's Communications with the Public Rule defines different types of communication. - Correspondence, which is defined as any written or electronic communication that is distributed or made available to 25 or fewer retail investors within any 30 calendar-day period. - Institutional communication, which is defined as any written or electronic communication that is distributed or made available only to institutional investors. This would not include any internal communication by the broker-dealer. - Retail communication, which is defined as any written or electronic communication that is distributed or made available to more than 25 retail investors within a 30 calendar-day period. - Public appearances are situations where employees associated with a broker-dealer or sponsor participate in a television or radio interview, seminar, or forum, or make a public appearance, or engage in speaking activities that are unscripted and are not otherwise considered retail communication. Social media sites, which permit real-time communication or interactive, electronic forums, fall under the guidelines of a public appearance (e.g., Facebook, Twitter, and LinkedIn). An institutional investor under this rule is any bank, S&L, insurance company, registered investment adviser or investment company (mutual fund), any person with total assets of at least $50 million, government entity, employee benefit plan, any member firm or registered person of the firm, and any person acting solely for any institutional investor. Since the e-mail is being sent to both retail and institutional investors, it is defined as a retail communication. If the communication is made available only to customers with total assets of at least $50 million, then it would be defined as an institutional communication.

Which of the following sources of revenue is NOT used to pay the debt service on general obligation bonds? a. Income taxes b. Property taxes c. Licensing fees and traffic fines d. Tolls collected at a tunnel located in the municipality

d. Tolls collected at a tunnel located in the municipality Explanation: A general obligation (GO) bond is backed by the full faith and credit of the municipality. Items that may be used to pay the debt service on GO bonds include fines, sales taxes, property taxes, income taxes, and licensing fees. Items such as tolls, concessions, and lease rental payments would be used to back a revenue bond.

Which of the following securities will probably have the greatest fluctuation in price when interest rates move up or down? a. Commercial paper b. Treasury bills c. Treasury notes d. Treasury bonds

d. Treasury bonds Explanation: Treasury bonds have the longest maturity of the choices listed and will have the greatest fluctuation in price. Since they have the longest maturity, they will be exposed to the risks of the marketplace for the longest period.

Which of the following securities is exempt from state taxes? a. Corporate stock b. Convertible bonds c. Federal National Mortgage Association (FNMA) bonds d. Treasury notes

d. Treasury notes Explanation: All of the choices listed are subject to state taxes except Treasury notes, which are U.S. government obligations and are subject to federal taxes, but exempt from state taxes.

Which of the following statements is NOT TRUE of treasury stock? a. It is listed on the company's balance sheet b. Treasury stock has no voting rights and does not receive dividends c. It is outstanding stock that has been repurchased by the corporation d. Treasury stock has been issued by the U.S. Treasury and was purchased by a corporation

d. Treasury stock has been issued by the U.S. Treasury and was purchased by a corporation Explanation: Treasury stock is stock that has been issued and was outstanding but has been repurchased by the company. Treasury stock does not have voting rights nor the right to receive dividends.

Alan and Marie Johnson have one child and have just purchased a home. The Johnsons count on Marie's regular income from her medical practice to pay their mortgage and other regular bills. Alan's irregular income from the sale of his sculptures provides investment and discretionary income. The Johnsons want to purchase life insurance that will provide the potential for appreciation of future benefits, but are uncertain how much to purchase due to the unpredictable nature of Alan's income. Which of the following types of insurance is MOST appropriate for the Johnsons? a. Whole life insurance b. Variable life insurance c. Universal life insurance d. Variable universal life insurance

d. Variable universal life insurance Explanation: Universal life insurance will allow the Johnsons to vary their premiums based on current income levels and insurance requirements, while variable life insurance will provide returns based on the performance of a separate account. A combination of the two types, called variable universal life or flexible premium variable life, is most appropriate.

Which of the following positions/strategies is NOT bullish? a. A married put b. A short put c. A long 40 call and a short 50 call d. Writing a straddle

d. Writing a straddle Explanation: Straddle writers expect a neutral market and obtain the maximum gain if each option expires. Each of the other choices has an opportunity for a profit if the underlying security rises in value.

Upon exercise of a British Pound World Currency call option, the writer will:

deliver U.S. dollars

All of the following characteristics are advantages of a real estate investment trust (REIT)

diversification. liquidity. professional management.

On exercise of the option, the holder of a long call will realize a profit if the price of the underlying stock

exceeds the exercise price plus the premium paid.

Reinvestment risk is the chance that, after purchasing a bond, interest rates

fall.

One of the distinguishing differences between variable annuities and mutual funds is that variable annuities

have a separate account.

As a registered representative, you have a customer that is concerned about losing principal when investing in a mutual fund. Because the customer is risk averse, you recommend a principal-protected fund. Which of the following statements is correct?

he principal is guaranteed, less any front-end load charges. There is a lockup period before shares may be redeemed. The portfolio may contain both stocks and bonds

In the case of a real estate direct participation limited partnership program, nonrecourse financing will

increase a limited partner's original cost basis.

The Fourth Market is direct trading of securities between:

institutions

The terminology guaranteed full faith and credit is most applicable to

interest and principal on a U.S. government-issued bond.

The Securities and Exchange Commission regulates all of the following

investment adviser and client relationships. initial public stock offerings. the secondary market.

An investor whose primary objective is a steady income flow would probably be best served by building a portfolio of

investment-grade debentures.

Your client wishes to invest $50,000 into shares of the ACE Mutual Fund. This morning's financial news indicated that the POP for ACE was $10.86, while the NAV was $10 per share. The client's order is placed at 2:00 pm Eastern time. Based on this information, you could confirm to the client a purchase of

nothing yet, as you must wait for the POP to be computed based on the day's close.

All of the following deal with the secondary market except

notice of sale. A notice of sale is published to provide syndicates with information on proposed new (primary market) issues.

All of the following are minimum requirements for listing on the NYSE

number of publicly held shares. number of shareholders. market value of publicly held shares.

For a customer who has purchased stock and wants to write a call option, the option ticket would be marked

opening sale. It is an opening sale if your client is writing the option.

An investor in a limited partnership generating passive losses can offset these against

passive income from other partnerships. rental income from direct investments in real estate.

Risk-less and simultaneous transactions by a broker-dealer are

permissible if they comply with the 5% policy

Bob and Tori are a married couple in their 40s filing a joint tax return. Both contribute to their employer's qualified retirement plan and will have a combined adjusted gross income of $4,000,000 this year. Bob and Tori are

permitted to make nondeductible contributions to their traditional IRAs.

Progressive taxes would include

personal income tax. gift taxes. estate taxes.

An analyst is reviewing the demographics of a municipality as part of a credit review for a G.O. bond issue. The analyst is reviewing:

population trends

When an individual associated with another FINRA member firm wishes to open up an investment account at another member firm, the executing member must

provide duplicate statements and confirmations if requested by the employer member

All of the following are conflicts of interest in a limited partnership EXCEPT a general partner:

receiving disproportionate share of partnership income

An investor who purchases 20-year Aaa rated corporate zero-coupon bonds would be least concerned with

reinvestment risk because there are no interest payments to reinvest.

AMZ Corporation has declared a cash dividend of $1.10. On the ex-dividend date, an open order to buy AMZ at 52 stop would

remain at 52 stop. This is a buy stop order and those are placed above the market so there is no adjustment to the price.

On the order book, all of the following orders are reduced on the ex-date for a cash dividend

sell stop buy limit. sell stop limit.

In determining whether there has been a violation of position limits, long puts will be aggregated with:

short calls

FINRA Rule 2210, communications with the public, has a number of filing requirements. Some communications are prefiled, others are postfiled, and some are excluded from filing with FINRA. Included in the list of exclusions would be retail communications

that do no more than identify a national securities exchange symbol of the member or identify a security for which the member is a registered market maker.

All of the following statements regarding a municipality's debt limit are true except

that the debt limit is the maximum amount a municipality can borrow in any one year (it is the max limit they can HAVE)

J.B. Collingsworth is the CEO and largest single shareholder in Collingsworth Industries, Inc. (CII). Three months ago, J.B. purchased 2,000 shares of CII in the secondary market at $6 per share. Yesterday, J.B. sold those shares for $8 per share. Under federal law,

the $4,000 profit on the sale must be returned to CII. J.B. has violated the "short swing" profit rule of the SEC. That rule states that any affiliate of a public company realizing a profit on a sale of that company's stock within a six-month period of the purchase, must return that profit to the company. The term used is "disgorged."

A customer buys a 20-year, 7% bond on a 7.35 basis. The bond is callable in six years at 103, in eight years at 102, in 10 years at 101, and at par beginning in the 12th year. The customer's confirmation will show yield to

the 20-year maturity.

Popular Investment Securities, a FINRA member firm, produces short videos describing the general characteristics of different types of securities. Periodically, an interstitial appears during the video. Under FINRA's rules on communications with the public,

the appearance of the interstitial defines the video as retail communication requiring filing.

An important feature of scheduled premium variable life insurance policies is that

the death benefit can never fall below the guaranteed minimum amount.

If a customer is long 1 GGZ Oct 50 call at 11 and short 2 GGZ Oct 60 calls at 5, the maximum loss potential is

unlimited Because the customer is short 2 calls and long 1 call, one of the short calls is uncovered. The loss potential for a naked call writer is unlimited on the upside.

One of your clients purchased shares of the Ajax Mutual Fund several months ago. At that time, the net asset value (NAV) of the fund was $17.20. Today, the NAV is $17.56, and your client wants to know what accounts for the difference. You should advise her that the difference likely represents

unrealized appreciation.

If a customer purchases a new issue of stock from a syndicate member, the customer will pay the public offering price

with no markup or commission. New issues are sold at the public offering price without a commission or markup.


Ensembles d'études connexes

ASSIGNMENT 1 - SECTION A - CA INSURANCE ETHICS & CODE

View Set

Price Index and Real Interest Rate

View Set

PrQ M18: Practice Quiz - Module 18: The Costs of Inflation

View Set